You are on page 1of 411

Economic academic year 2006-2007, for the children of Persons of Indian Origin (PIOs) and

Non-Resident Indians (NRIs) to assist them in pursuing Under Graduate courses in Indian
Universities/Institutes. It is being implemented by Ministry of External Affairs. Under it,
financial assistance in the form of scholarship is provided for specific undergraduate courses
in Professional and Non-Professional courses. Only fresh year students (1st year) students
are eligible to get the scholarship.
Q.1)
Identify the scheme mentioned in the above passage
a. Scholarship Program for Diaspora Children
b. Scholarship Program for Empowering NRI Children
c. Scholarship Program for Overseas Children
d. Scholarship Program for PIO/NRI Children
e. Scholarship Program for Diaspora Children
Answer- (e)
Q.2)
Which of the following educational discipline is not covered under the scheme mentioned in
the above passage?
a. Engineering
b. Medical
c. MBA
d. PhD Programmes
e. None of the above
Answer- (b)
Q.3)
What is the total number of scholarships seats earmarked for Emigration Check Required
(ECR) countries subject to fulfilling eligibility conditions?
a. 50
b. 70
c. 60
d. 100
e. 40
Answer- (a)
Q.4)
What is the name of the portal to submit the online application for obtaining scholarships
under the scheme mentioned in the above passage?
a. SPDC portal
b. Shiksha portal
c. Padhai portal
d. NRI portal
e. Scholarship portal
Answer- (a)
Q.5)
Identify incorrect statement about the scheme mentioned in the above passage
a. Children of Indian Workers in ECR countries studying in India are eligible to apply for
scholarship
b. Scholarship is provided to 150 selected students, without earmarking NRIs/ PIOs.
c. Under it, financial assistance is provided for food facilities along with tuition fee,
admission fee and post admission services.
d. Candidates from all the categories must have passed grade 12th from abroad.
e. None of the above
Answer- (c)
Explanation –
Under the scheme financial assistance for specific undergraduate courses in Professional
and Non-Professional courses is provided towards tuition fee, admission fee and post
admission services.
This scheme is being implemented by the ministry of external affairs and every year the
ministry publishes a notification explaining the details of the scheme and inviting applications
from the diaspora children. Similarly, the official notification for this scheme came up a few
months before the 2018 phase 2 examination with 30th September as the last date for
application. And do you know, when the 2018 phase 2 examination happened? Yes, it was
on 15th September 2018. So, the examiner had all the right to ask questions from this
scheme.
Now, talking about the questions, all the questions were direct and straightforward. You can
easily attempt all questions if you go through the first 3-4 pages of the official document of
the scheme. Again, you don’t have to worry about such schemes, as long as PIB 247
sessions are going on. All the schemes in news are covered in these classes.
I.6-9) Read the passage and answer the questions that follow
In view of the difficulty being faced by the pregnant women and parents of sick new- born
along-with high expenditure on delivery and treatment of sick- new-born, Ministry of health
and Family Welfare has taken a major initiative to ensure better facilities for women and
child health services. In 2011, it had launched this scheme to motivate those who still choose
to deliver at their homes to opt for institutional deliveries. It provides completely free and
cashless services to pregnant women including normal deliveries and caesarean operations
and sick new born. It is estimated to benefit more than 12 million pregnant women who
access Government health facilities for their delivery. All the States/UTs have initiated
implementation of the scheme.
Q.6)
Identify the scheme mentioned in the above passage
a. Pradhan Mantri Matru Vandana Yojana
b. Janani Shishu Suraksha Karyakaram
c. Janani Suvidha Yojana
d. Ayushmati Scheme
e. Mamta Friendly Hospital Scheme
Answer- (b)
Q.7)
For how many days there is a provision of hospitalization for sick new born children under
the scheme mentioned in the above passage?
a. 15 days
b. 10 days
c. 20 days
d. 30 days
e. 45 days
Answer- (d)
Q.8)
For how many days the free diet is provided for Normal and C-section Delivery respectively,
under the scheme mentioned in the above passage
a. 3 and 7 days
b. 4 and 8 days
c. 5 and 10 days
d. 6 and 11 days
e. 2 and 5 days
Answer- (a)
Q.9)
From which district the scheme mentioned in the above passage was launched?
a. Chikkaballapur
b. Vapi
c. Shimla
d. Bhiwadi
e. Mewat
Answer- (a)
Explanation–
Questions number 6 to 9 are also from a government scheme which is one of the important
schemes of the ministry of health and family welfare. Now, here I am not going to explain
why this question was asked because I have explained this many times before in other past
year papers.
What I want to explain here is – How to identify a scheme from the passage? Because if you
would not able to identify the scheme, it costs you 3-4 questions which means 6-8 marks.
And you better know that there is a fight for even a single mark.
In the process of identifying the scheme, you have to focus on the keywords given in the
passage. If I talk about the passage given above, here are some keywords –
“Ministry of health and family welfare”, “launched in 2011”, “free and cashless services to
pregnant women including normal deliveries and caesarean operations and sick newborn”,
“12 million pregnant women”.
Now after looking at these keywords, you would ask your brain to recollect the information
which you have given to it while studying. If you have ever gone through this scheme, your
brain will give you the answer – “Oh! It is Janani Shishu Suraksha Karyakaram”. And you will
mark the correct answer.
I hope you have the understood the most important part of solving questions of government
schemes. Cheers!
I.10-12) Read the passage and answer the questions that follow [2 Marks]
The Ministry of Rural Development had launched this scheme in 1999 in rural areas of the
country. It is a holistic Scheme covering all aspects of self-employment such as organisation
of the poor into Self Help Groups, training, credit, technology, infrastructure and marketing. It
is funded by the Centre and the States in the ratio of 75:25. It aims is to bring every assisted
family above the poverty line within three years by providing them income generating assets
through a mix of bank credit and Government subsidy. The rural poor such as those with
land, landless labour, educated unemployed, rural artisans and disabled are covered under
the scheme.
Q.10)
Identify the scheme mentioned in the above passage
a. Swarnajayanti Gram Swarojgar Yojana
b. Deen Dayal Upadhyaya Grameen Kaushalya Yojana
c. Skill development through Rural Self Employment and Training Institutes
d. Mahatma Gandhi National Rural Employment Guarantee Scheme
e. Prime Minister’s Employment Generation Programme
Answer- (a)
Q.11)
Identify the incorrect statement about the scheme mentioned in the above passage
a. It provides maximum Rs. 7500 for capacity building, trading and skill development,
credit training etc
b. It provides ceiling of Rs. 10000 for minor Irrigation project
c. The size of loan under this scheme will be maximum Rs. 25 Lakhs
d. It provides for formation of Self-Help Groups (SHGs), nurturing and their linkages
with banks.
e. None of the above
Answer- (d)
Q.12) Which of the following statement/s about the scheme mentioned in the above passage
is incorrect
1. It focuses on vulnerable sections of the rural poor.
2. There is no Involvement of Panchayat Raj Institutions in its implementation
3. It aims at establishing MACRO enterprise

a. Only 3
b. Only 2
c. Only 1 and 3
d. All of the above
e. Only 2 and 3
Answer- (e)
Explanation –
This question is important to be discussed. Why? Let me tell you –
Every year, many students ask me these questions – “Do we have to cover schemes which
are not in news? Do we have to cover old schemes? The answer to such questions was
given by the RBI itself in 2018 examination when it asked questions from a scheme of 1999
i.e Swarnajayanti Gram Swarojgar Yojana.
So, you never know, from which scheme the questions are going to asked. So, be prepared
for the worst.
Now, most of you will say, - “That means we should cover all the schemes which are being
implemented since independence” How is it possible?
No, I am not saying that. I am just asking you to cover the LANDMARK SCHEMES of the
government of India and not from 1947 because there is no scheme which is being
implemented since 1947. But if there is any scheme which is being implemented even after
30-40 years since its launch, that scheme is important. For ex – Integrated Child
Development Scheme which was launched in 1975 but even today it is being implemented
by the ministry.
Got my point?
I.13-17) Read the passage and answer the questions that follow
This scheme aims to involve the voluntary sector and training institutions of repute to
improve educational and socioeconomic conditions of the target group i.e. Scheduled Castes
with a view to upgrade skill to enable them to start income generating activities on their own
or get gainfully employed in some sector or the other. The guiding spirit behind the
formulation of this scheme is that good voluntary organizations should not only be assisted
but also consciously built up.
Q.13)
Identify the scheme mentioned in the above passage
a. National Scheduled Castes Finance and Development Corporation
b. Scheme for funding the Voluntary Organizations
c. Central Sector Scholarship Scheme for SC Students
d. Babu Jagjivan Ram Chhatrawas Yojana
e. Scheme of Grant-in-aid to Voluntary and other Organizations Working for Scheduled
Castes
Answer- (e)
Q.14)
Which Ministry is implementing scheme mentioned in the above passage?
a. Ministry of Corporate Affairs
b. Ministry of Micro, Small and Medium Enterprises
c. Ministry of Social Justice and Empowerment
d. Ministry of Tribal Affairs
e. Ministry of Agriculture and Famers Welfare
Answer- (c)
Q.15)
What is the eligibility criteria for an organisation coming under this scheme?
a. Registered under Society Act, 1960
b. Indian Red Cross society and its Branches
c. Section 25 of Companies Act
d. A and B
e. A, B and C
Answer- (e)
Q.16)
Which of the following provisions of assistance is/are covered under the scheme mentioned
in the above passage?
1. Rent for building in which project is being run.
2. Scholarship only for orphan children for foreign studies.
3. Stipend to the students and trainees

a. 1 only
b. 2 Only
c. 1 and 3 Only
d. 2 and 3 Only
e. 1, 2 and 3
Answer- (c)
Q.17)
Which of the following was the objective of the scheme mentioned in the above passage?
a. To provide educational facilities to only primary school students form Scheduled
Caste.
b. To provide basic government health facilities to Scheduled Caste.
c. To build basic infrastructure in deficient Scheduled Caste dominant areas.
d. To enhance the reach of development interventions of the Government and fill the
gaps in service deficient Scheduled Caste dominant areas,
e. None of the above
Answer- (d)
Explanation –
Again, questions from a government scheme. So, here I am not going to explain anything
except reminding you once again that – You have to cover all the schemes which have been
in news one year before the exam and the LANDMARK schemes of the government of India.
The LANDMARK schemes mean those schemes which remain in news in every parliament
session. These are the schemes which are a part of the answers submitted by the ministry in
parliament in every session. For ex – MGNREGA, PM Gram Sadak Yojana, PM Awas
Yojana etc.
I.18-20) Read the passage and answer the questions that follow
The GNI at current prices is estimated at Rs. 165.87 lakh crore during 2017-18, as
compared to Rs. 150.77 lakh crore during 2016-17, showing a rise of 10 percent.
The per capita income at current prices during 2017-18 is estimated to have attained a level
of Rs. 112835 as compared to the estimates for the year 2016-17 of Rs. 103870 showing a
rise of 8.6 percent.
The Gross National Income (GNI) at 2011-12 prices is now estimated at Rs. 128.64 lakh
crore during 2017-18. In terms of growth rates, the Gross National Income is estimated to
have risen by 6.7 percent during 2017-18, in comparison to the growth rate of 7.1 percent in
2016-17.
Q. 18)
Which of the following Ministry releases these estimates mentioned in the above passage?
a. Ministry of Finance
b. Ministry of Commerce and Industry
c. Ministry of Education
d. Ministry of Statistics and Programme Implementation
e. Ministry of Home Affairs
Answer- (d)
Q.19)
Gross National Income (GNI) at 2011-12 prices is now estimated at Rs. 128.64 lakh crore
during 2017-18, as against the previous year’s estimate of _______
a. Rs. 130 lakh crore
b. Rs. 120 lakh crore
c. Rs. 124 lakh crore
d. Rs. 127 lakh crore
e. Rs. 129 lakh crore
Answer- (b)

Q.20)
How is per capita income computed?
a. GDP/Working Population
b. GNI/Working Population
c. GNI/ Population
d. GDP/Population
e. None of the above
Answer- (c)
Explanation –
Question numbers 18 to 20 are from Provisional Estimates of Gross National Income
released by the ministry of statistics and program implementation.
Now, the significance of questions from this topic can be determined from the fact that we
are studying “ECONOMIC & SOCIAL ISSUES” and don’t you think that National Income is
an integral part of Economic Issues.
Also remember, every year questions from reports and indices are asked. So, you cannot
ignore such important reports. Try to cover all reports which are being released by the
government through PIB.
I.21-24) Read the passage and answer the questions that follow
Executive Board of the International Monetary Fund (IMF) conducted Article IV consultation
with Reserve Bank of India. It found that stability-oriented macroeconomic policies and
progress on structural reforms continue to bear fruit. Following disruptions related to the
November 2016 currency exchange initiative and the July 2017 goods and service tax (GST)
rollout, growth slowed to 6.7 percent in FY2017/18, but a recovery is underway led by an
investment pickup. Headline inflation averaged 3.6 percent in FY2017/18, a 17-year low,
reflecting low food prices on a return to normal monsoon rainfall, agriculture sector reforms,
subdued domestic demand, and currency appreciation. With demand recovering and rising
oil prices, medium-term headline inflation has risen to 4.9 percent in May 2018, above the
mid-point of the Reserve Bank of India (RBI)’s headline inflation target band of 4 percent ± 2
percent. External vulnerabilities remain contained but have risen. The current account deficit
(CAD) widened to 1.9 percent of GDP in FY2017/18, on rising imports and oil prices. Gross
international reserves rose to US$424.5 billion (about 8 months of prospective imports of
goods and services) at the end of March 2018, but declined to US$407.8 billion in the third
week of June 2018. Persistently-high household inflation expectations and large general
government fiscal deficits and debt remain key macroeconomic challenges. Systemic macro
financial risks persist, as the weak credit cycle could impair growth and the sovereign-bank
nexus has created vulnerabilities.
Q.21)
As a result of the conditions mentioned in the above passage, which policy rate corridor was
narrowed by RBI to +/-25 basis points from +/- 50 basis points effective April 6, 2017 to
ensure finer alignment of the operating target with the policy rate.
a. Reverse Repo rate
b. Statutory Liquidity Ratio
c. Marginal Standing Facility rate
d. Repo rate
e. None of the above
Answer- (d)
Q.22)
The policy rate mentioned in above question was increased by RBI to
a. 6.25%
b. 5.75%
c. 6.0%
d. 5.5%
e. 6.5%
Answer- (a)
Q.23)
What is headline inflation rate referred in the passage above?
a. Wholesale Price Index
b. Core inflation Index
c. Consumer Price Index
d. Non-core inflation Index
e. None of the above
Answer- (c)
Q.24)
According to above passage, unlike some other emerging market economies, India‘s
manufacturing sector remains relatively small, reflecting major rigidities in
1. Land Sector
2. Labour Sector
3. Product Marks
4. Government ownership of PSBs

a. 1 and 3 Only
b. 2, 3 and 4 Only
c. 3 and 4 Only
d. 1, 2 and 3 Only
e. All of the above
Answer- (d)
Explanation –
The passage given for questions 21 to 24 was picked up by the examiner from a publication
of IMF. This publication was about the discussions happened between India and IMF
regarding the status of economy in 2017-18.
These discussions reflected in the RBI monetary policy which was released just before the
2018 phase 2 examination. Hence, the questions were expected.
Hence, I would like to add one more topic to the list of important topics – “RBI MONETARY
POLICY”
I.25) Read the excerpt from report below and answer the questions that follow
However persistently-high household inflation expectations and large general government
fiscal deficits and debt remain key macroeconomic challenges. Systemic macro-financial
risks persist, as the weak credit cycle could impair growth and the sovereign bank nexus has
created vulnerabilities. Economic risks are tilted to the downside, the report said, adding that
on the external side, risks include a further increase in international oil prices, tighter global
financial conditions, a retreat from cross border integration including spill over risks from a
global trade conflict, and rising regional geopolitical tensions. Domestic risks pertain to tax
revenue shortfalls related to continued GST implementation issues and delays in addressing
the twin balance sheet problems and other structural reforms. It also underscored the need
for continued prudent macroeconomic policies and renewed emphasis on macro-financial
and structural reforms.
Q. 25)
Which organisation had released this report?
a. World Bank
b. International Monetary Fund
c. World Economic Forum
d. Asian Development Bank
e. None of the above
Answer- (b)
Explanation –
This report was released by IMF in August 2018 after executive board meeting of IMF with
India. Now, this question was quite difficult because one cannot go through the entire report
word to word. So, how can one attempt this question? Let me explain.
Read the passage carefully and recall the objectives of IMF. The passage talks about key
macroeconomic challenges, economic growth, financial conditions etc. Now, have a look at
the objectives of IMF which are furthering monetary cooperation, encouraging economic
growth etc.
Don’t you think that the key words mentioned in the passage are in consonance with the
objectives of IMF?
I think they are.
Hence, the answer is IMF.
I.26-29) Read the passage and answer the questions that follow
Government had constituted a Committee in May, 2016 to review the Fiscal Responsibility
and Budget Management (FRBM) Act, 2003. It had wide ranging Terms of Reference (ToR)
to comprehensively review the existing FRBM Act in the light of contemporary changes, past
outcomes, global economic developments, best international practices and to recommend
the future fiscal framework and roadmap for the country. Subsequently, TOR were enlarged
to seek the Committee’s views on certain recommendations of the Fourteenth Finance
Commission and the Expenditure Management Commission. These primarily related to
strengthening the institutional framework on fiscal matters as well as certain issues
connected with new capital expenditures in the budget.
Q.26)
Who is the head of this committee mentioned in the above passage?
a. Shyam Benegal
b. PJ Nayak
c. N K Singh
d. Hari Gautam
e. P Malimath
Answer- (c)
Q.27) How much debt-to-GDP ratio has been recommended by the committee mentioned in
the above passage for the Central Government?
a. 20%
b. 30%
c. 35%
d. 40%
e. 45%
Answer- (a)
Q.28)
Which of the following is/are not the objectives of FRBM act?
1. Efficient management of expenditure, revenue and debt
2. Macroeconomic stability
3. Achieving a balanced budget
4. Better coordination between fiscal and monetary policy

a. Only 1 and 2
b. Only 2, 3 and 4
c. Only 4
d. Only 2
e. All are the objectives
Answer – (e)
Q.29)
What does Fiscal Deficit refer to?
a. All Government Expenditure – All Government Receipts except borrowings
b. All Government Expenditure – All Government Receipts including borrowings
c. All Government Expenditure – Domestic Government Receipts Only
d. Government Revenue Expenditure Only – Government Revenue Receipts Only
e. None of the above
Answer- (a)
Explanation –
I hope you know, that FRBM is a part of the syllabus of ESI. Now, if any topic is a part of the
syllabus, you have to be updated about that topic. Anything in news related to that topic is
significant for the exam.
Even if FRBM would not have been in the syllabus, The NK Singh committee on FRBM
holds importance. So, you need to be aware of such important committees and their
recommendations. And to be aware, you have to be awake.
The idea is – “Jaago Students Jaago”
The questions were very basic in nature. All the questions could have been attempted with
just a glimpse of FRBM act and this committee and its recommendations.
I.30-34) Read the passage and answer the questions that follow
The Department for Promotion of Industry and Internal Trade (DPIIT) has developed this
BRAP framework in 2016 and circulated it with States/UTs for its implementation. It seeks to
bring ease to regulatory compliance for businesses. It is ambitious program for reforms in
partnership with State Governments to make it easier, simpler and quicker for businesses to
operate. These reforms cover the entire lifecycle of the business and aim at reducing
compliance burden on industry significantly. States/ UTs have been actively engaged in
envisioning, designing and implementing these reforms that are to be undertaken each year.
This framework is designed keeping in mind 2 factors viz. Measurability and Comparability
across States. It is the first such platform for knowledge on regulatory compliance
requirements across 36 States/UTs in the country.
Q.30)
What is the full form of BRAP mentioned in the above passage?
a. Boosting Reform Actionable Plan
b. Boosting Reform Action for Performance
c. Business Reform Activity Performance
d. Business Reform Action Plan
e. None of the above
Answer- (d)
Q.31)
Which state has been ranked last as per BRAP mentioned in the above passage?
a. Assam
b. Bihar
c. Meghalaya
d. Madhya Pradesh
e. Nagaland
Answer- (c)
Q.32) With which institution has DIPP collaborated for developing BRAP?
a. International Monetary Fund
b. World Bank
c. World Economic Forum
d. United Nations Development Programme
e. None of the above
Answer- (b)
Q.33) How many action points have been included in BRAP?
a. 108
b. 217
c. 372
d. 469
e. 512
Answer- (c)
Q.34) From which of the following, feedback was taken for BRAP?
1. Existing businesses
2. New Businesses
3. Architects
4. Lawyers

a. 1, 2 and 3
b. 2, 3 and 4
c. 1, 3 and 4
d. 1, 2 and 4
e. All of the above
Answer- (e)
Explanation –
On 10th July 2018, the ministry of commerce and industry released the Ease of Doing
Business Ranking under the Business Reforms Action Plan, a few months before the phase
2 exam. Now should I tell you that it was an important ranking? I think I should not because
you are quite smart to understand this.
Now, talking about the questions – Questions were asked from the PIB release directly. This
happens with every index and report. The questions are asked from those points only which
are in news. The examiner will never examine the entire report to frame a question. Only the
facts which are in the news and the executive summary of any index or report are enough for
the examination.
Q.35)
Which of the following is/are objectives of SEZ Act 2006?
a. Generation of additional economic activity.
b. Promotion of exports of goods and services.
c. Promotion of investment from domestic and foreign sources
d. Development of infrastructure facilities.
e. All of the above
Answer- (e)
Explanation –
This question was asked from the static syllabus of ESI which indicates that that static part
of ESI is equally important. These days many students are asking this question that whether
we should cover static part or not because most of the questions are being asked from the
current affairs.
I agree that these days most of the questions are being asked from the current affairs but not
all. And who knows in the next exam, the examiner changes the pattern of the questions and
asks question from the static part. Moreover, today we have descriptive part in ESI where
the questions cannot be attempted if a candidate does not cover the static part of ESI.
Hence, you cannot ignore the static part.
Do not do budgeting while preparing for an exam, try to cover as much as possible. Because
it is not just an exam, it can decide your future.
One Markers
Q.36)
The ratio by which the mean income of the poor falls below the poverty line is referred as
a. Population below poverty
b. Poverty Count
c. Poverty Gap
d. Head Count ratio
e. Poverty Trend
Answer- (c)
Explanation –
Straight forward and direct question from a very basic concept. Even if you don’t know the
answer, you could have easily attempted this question by using logics. The statement itself is
indicating that only possible answer is Poverty gap.
So, don’t be nervous if you don’t know the answer to a question. Use your brain and give it a
try using smart logics and guesses.
Q.37)
Sustainable Development Goals (SDGs) which were made more "actionable" by UNGA
reAnswer that identifies specific targets for each goal and provides indicators to measure
progress. When are these SDG targets are to be achieved?
a. 2025
b. 2030
c. 2035
d. 2040
e. 2045
Answer- (b)
Explanation –
Ohoho.. You can’t expect easier question than this. This question could have been
attempted by even a layman who watches the great News channels of the country. Then
why can’t you? I am sure you can easily mark this question correct.
Q.38)
Which is the first multilateral organization to have its secretariat in India with United Nations
as Strategic Partner?
a. South Asian Association for Regional Cooperation
b. Bay of Bengal Initiative for Multi-Sectoral Technical and Economic Cooperation
c. International Solar Alliance
d. Shanghai Cooperation Organisation
e. BRICS
Answer- (c)
Explanation –
Again, an easy question. There is only one such organization which has its headquarters in
India. Considering this fact, I believe the examiner has all the right to ask a question from
this organization in any exam. Also, international and regional organizations are a part of the
static syllabus of ESI which makes this question relevant.
Q.39)
Which of the following poverty is caused due to sudden health issue or calamity and is
temporary in nature?
a. Situational Poverty
b. Generational Poverty
c. Relative Poverty
d. Absolute Poverty
e. None of the above
Answer- (a)
Explanation –
Look at the syllabus of ESI and ask yourself – “Is it a difficult question?” I believe NO.
Because poverty is explicitly mentioned in the syllabus and that’s why question from this
topic can be asked.
Also, talking about the question, it is an easy form of question where you just have to identify
the type of poverty. And that too could have been done very easily during the exam because
in the question, you are required to tell the name of the poverty “caused due to sudden
health issue or calamity”, which is a situation. Hence, the best suitable answer is Situational
Poverty.
Q.40)
In Union Budget 2018-19, a new Scheme “Operation Greens” was announced on the line of
“Operation Flood” to promote Farmer Producers Organizations, agri-logistics, processing
facilities and professional management. What is the outlay of this scheme?
a. Rs. 300 crores
b. Rs. 100 crores
c. Rs. 400 crores
d. Rs. 600 crores
e. Rs. 500 crores
Answer- (e)
Q.41)
How much cumulative deduction was announced in Union Budget 2018-19 to salaried class
in lieu of the present exemption in respect of transport allowance and reimbursement of
miscellaneous medical expenses?
a. Rs. 20000
b. Rs. 35000
c. Rs. 40000
d. Rs. 65000
e. Rs. 80000
Answer- (c)
Q.42)
How much amount was allocated for infrastructure spending in Union Budget 2018-19?
a. Rs. 5.97 lakh crore
b. Rs. 3.21 lakh crore
c. Rs. 6.89 lakh crore
d. Rs. 4.72 lakh crore
e. Rs. 7.10 lakh crore
Answer- (a)
Explanation –
Question numbers 40 to 42 are from union budget. The questions were of very basic nature
and asked directly from the budget. Almost every year, questions from Union Budget and
Economic Survey are asked in the exam. So, you must prepare both the topics.
Now, the question is how to prepare these topics. Answer is very simple. Every year on the
day of Economic Survey and Budget, I conduct a session explaining every aspect of the
survey and the budget. So, take a chill pill.
Q.43)
Consumer Goods in the economics are also referred as?
a. Public Goods
b. Private Goods
c. Club Goods
d. Capital Goods
e. Final Goods
Answer- (e)
Explanation –
This was a static question asked from the static syllabus of ESI. Here, again, if you do not
know the answer, you can easily mark it correct. But, how?
Consumers are those who consumes the good. And a good can only be consumed when it
is at its final stage. Hence, very conveniently the answer is final goods.
Cheers!
Q.44)
The Human Development Report (HDR) is an annual report published by the Human
Development Report Office. This office functions under which organisation?
a. United Nations Economic and Social Council
b. United Nations International Children's Emergency Fun
c. United Nations Development Programme
d. United Nations Environment Programme
e. None of the above
Answer- (c)
Explanation –
When I say report and indices are important for this exam, you should take it seriously.
Because, see, one more question from Report. Although the question was simple and was
asked from a prominent report i.e Human Development Report yet you must be prepared for
a bouncer. You never know, from which report the examiner is going to ask a question in the
next exam. So, try to cover all the important reports and indices out there.
Q.45)
According to the Union Budget 2018-19 presented by the Union Finance Minister, the
agriculture exports from India was _________ per year, against the potential of $100 billion?
a. $60 Billion
b. $40 Billion
c. $20 Billion
d. $30 Billion
e. $50 Billion
Answer- (d)
Explanation –
A question from Union Budget again. I am not going to tell you again and again the same
stuffs. So, just one thing – “PREPARE THE BUDGET THOROUGHLY”
Q.46)
What does the first P in PPP stand for?
a. PrivateCc
b. Purchasing
c. Parity
d. Power
e. Public
Answer- (b)
Explanation –
I can’t expect easier question than this. This was the easiest question asked in this paper.
The term PPP can be seen at one or the other page of a daily newspaper. So, I don’t think
that to attempt this question, you need any kind of preparation.
Q.47)
Impact of Globalization on Family, Work and Culture etc can termed as?
a. Political Globalization
b. Economic Globalization
c. Individualistic Globalisation
d. Social Globalisation
e. None of the above
Answer- (d)
Explanation –
This question was quite logical in nature. Whenever we see term “Social”, the first thing that
comes to our mind is society. And the society consists of family, work, culture, people etc.
Hence, the answer to this question is social globalization.
Q.48)
The TRIPS Agreement is part of the “single undertaking” resulting from the Uruguay Round
negotiation of which of the following organisation?
a. World Bank
b. International Monetary Fund
c. World Trade Organisation
d. UN Conference on Trade and Development
e. None of the above
Answer- (c)
Explanation –
Have you ever read about World Trade Organization? If yes, this question is a cakewalk for
you. Because when you read about WTO, first you read about GATT and the eight rounds of
negotiations which happened before the formation of WTO. The Uruguay round was the
eighth and the last round which led to the formation of WTO. And even if you did not know
about the TRIPS agreement, you could have easily attempted this question with the help of
“Uruguay round”
Q.49)
Which became the first Indian state to conduct comprehensive survey to ascertain the extent
of Multidimensional Poverty in the state?
a. Maharashtra
b. Tamil Nadu
c. Karnataka
d. Gujarat
e. Andhra Pradesh
Answer- (e)
Explanation –
In the year 2018, Andhra Pradesh became the first state to launch Multidimensional Poverty
index. So, this question was from current affairs and you can attempt such questions if you
are following any monthly magazine of current affairs. This was covered by every magazine
in the market.
Q.50)
Which of the following is not involved as part of the macro-economic theory?
a. Analysis of specific company
b. Inflation rate
c. Supply of money
d. Aggregate demand
e. Foreign exchange rate
Answer- (a)
Explanation –
When we talk about Macro Economy, we talk about the economy of a country as a whole. If
you understand the meaning of macro economy, nothing is left after that. Mark your answer
as option (a) and move ahead.
Q.51)
GDP is measured from the perspective of?
a. Producers
b. Consumers
c. Intermediaries
d. A and B
e. A, B and C
Answer- (b)
Explanation –
If we go by the definition by the definition of GDP, we can mark this question right. What is
the definition of GDP? GDP is the total value of all the goods and services produced within
the boundaries of a country in a year.
Now, when you know the definition, let’s come back to the question.
The definition of GDP implies that GDP measures monetary value of final good and services
i.e. those goods and services that are bought by the final user i.e. – CONSUMER.
Hence, the answer would be option (b) – Consumers.

Q.52)
Which amongst the following is Gross National Income?
a. Net Domestic Product + Indirect Taxes – Subsidies
b. Gross domestic product – Indirect Taxes + Subsidies
c. Net domestic product + Receivable from abroad
d. Gross domestic product – Receipts from abroad
e. Final Goods Produced + Receivable from abroad
Answer- (e)
Explanation –
This was a direct formula-based question from the chapter National Income. Such questions
are asked in the exam to test your retention ability. The more you retain, the more suitable
you are for this post (as per the examiner). So, while reading the chapter of National income,
go through all the formulas viz; GDP, NDP, GNI, NI etc.
Q.53)
Which of the following statement is true about Fiscal Policy?
a. It includes revising interest rates and changing bank reserve requirements.
b. It is classified as either expansionary or contractionary.
c. It controls overall money supply and achieve economic growth.
d. It is for maintaining price stability
e. None of the above
Answer- (e)
Explanation –
Fiscal policy and monetary policy are among the important topics for this exam. You need to
cover both topics thoroughly.
The question above talks about the features of fiscal policy and monetary policy. Option (a),
(c) and (d) are the exclusive features of monetary policy while option (b) is the feature of
both. Hence, the correct answer should be option (b) i.e. “It is classified as either
expansionary or contractionary”
Q.54)
This is core Centrally Sponsored Scheme (CSS) aimed at quickly and effectively
transforming selected aspirational districts. These districts were selected on parameters like
poverty, public health, nutrition, education, gender, sanitation, drinking water, livelihood
generation which are in sync with SDGs and fall within the realm of Panchayats. It will
enable Panchayats to function effectively to achieve SDGs and other development
objectives that require significant Capacity building efforts. What is name of this scheme?
a. Rashtriya Gram Swaraj Abhiyan
b. Pradhan Mantri Awaas Yojana Gramin
c. Deen Dayal Upadhyaya Grameen Kaushalya Yojana
d. Sansad Adarsh Gram Yojana
e. None of the above
Answer- (a)
Explanation –
I believe till now; you have become an expert in “How to identify a scheme from a passage”.
Yes, because I have told you several times how to do it. Various hints are given in the above
passage which leads to the answer i.e. Rashtriya Gram Swaraj Abhiyan
Q.55)
Which of the following is used as policy tool used to protect domestic industries by changing
the conditions under which goods compete in such a way that competitive imports are
placed at a disadvantage?
a. Tariffs
b. Wealth tax
c. GST
d. Direct Taxes
e. None of the above
Answer- (a)
Explanation –
If you look at the options carefully, you can answer this question conveniently. All the given
taxes except tariffs are imposed on the goods and services produced within the country.
However, the tariffs are imposed on imported goods to protect domestic industries from the
negatives of imported goods.
Q.56)
Government policies and trade measures restrict international trade to help domestic
industries is called as
a. Trade wars
b. Protectionism
c. Trade restrictions
d. Import bans
e. None of the above
Answer- (b)
Explanation –
Let’s do this question using elimination technique. I would outrightly reject options (a) and
(d). This is because protecting domestic industries does not mean trade wars. Also, while
restricting international trade, a country does not completely put a ban on imports.
Now consider option (c). I would think twice before marking this option because the
statement given in the question itself declares that the government is restricting international
trade. So, when something is already given in the statement, how can it be the answer.
Hence, the correct answer would be protectionism.

Q. 57)
Which of the following term refers to the way that changes in the price of one good can affect
the quantity demanded of another good?
a. Price elasticity
b. Demand elasticity
c. Income elasticity
d. Functional elasticity
e. Cross elasticity
Answer- (e)
Explanation –
This was a complete static question which was given to test your conceptual knowledge.
Such questions remind us the fact that one cannot clear this examination by just cramming
everything. You have to be conceptually strong to clear this exam. And I believe that you are
not among those who ignore such important topics.
Q. 58)
Which institution was established in 1945 for securing stability, monetary co-operation etc?
a. UN Conference on Trade and Development
b. World Trade Organization
c. World Bank
d. International Monetary Fund
e. None of the above
Answer- (d)
Explanation –
Do you remember Bretton Woods Conference? If yes, Let me ask you another question –
Which two organizations were established as a result of this conference?
Yes, they were IMF and World Bank.
Now, I want to ask one more question – What are the objectives of these organizations?
Yeah, you are right, World Bank is committed towards reducing poverty, increasing shared
prosperity and promoting sustainable development while IMF is committed towards
furthering international monetary cooperation encouraging economic growth.
Now, I hope you get the answer to the above question.
Q. 59)
Population that maximizes the country's Growth and Development?
a. Under Population
b. Over Population
c. Optimal Population
d. Fixed Population
e. None of the above
Answer- (c)
Explanation –
Everything in this world is at its best when it is at its optimum level and so is the population.
Hence, without any doubt, the answer to this question is Optimal population. Overpopulation
creates a burden on the resources of the nation. Due to underpopulation, a country lacks
human capital which is required for the country’s growth and development.
Q.60)
Which of the following is not included in GDP calculation?
a. Investment
b. Net Exports
c. Government Spending
d. Investment Expenditure
e. Wellbeing of people
Answer- (e)
Explanation –
Before discussing this question, let me ask you a question – Why would “well-being of
people” be included while calculating GDP? Do you have an answer to this question? I hope
you have.
GDP is the monetary value of all goods and services produced within the boundaries of a
nation in a year. When we are talking about monetary value, we cannot include moral values
like well-being of people. Hence, the answer is option (e).
Q.61)
Which of the following is the objective of the HRIDAY Scheme?
a. To establish infrastructure for ensuring adequate sewage networks and water supply
in the urban areas.
b. To facilitate inclusive urban development.
c. To identify and develop the Heritage cities
d. To addresses urban housing shortage among the EWS/LIG and MIG categories
including the slum dwellers.
e. None of the above
Answer- (b)
Explanation –
To mark this question correct, you just have to aware of the full form of HRIDAY. The full form
is – Heritage City Development and Augmentation Yojana. Have you got my point?
I always say, that the objective of almost every scheme can be derived from its name itself.
And the above question is an example for this.

Q.62)
Which of the following inflation theory combines both the Pull Back and Push Back to explain
the inflation?
a. Market power theory
b. Bottle neck theory
c. Monetary theory
d. Markup theory
e. None of the above
Answer- (d)
Explanation –
This was again a static question based on the conceptual understanding. If you are
conceptually strong, you can easily mark this question correct. The question is based on
various theories of inflation which are a part of static ESI syllabus. So, the idea is to give
equal preference to static topics
Q.63)
What is the revised Fiscal Deficit Target against the earlier target of 3%?
a. 4%
b. 4.2%
c. 3.7%
d. 3.5%
e. 3.3%
Answer- (e)
Explanation –
Various deficit targets along with Fiscal Deficit target are announced every year in the Union
Budget. So, you need to cover all the targets and their revision done by the government, if
any.
Q.64)
Who was the chairperson of committee on FRBM act formed in 2016, to review the FRBM
act?
a. N K Singh
b. JB Mohapatra
c. Nandan Nilekani
d. RK Singh
e. PS Ahluwalia
Answer – (a)
Explanation –
I hope you all know that the FRBM act is mentioned in the syllabus of the ESI. When it is
mentioned in the syllabus, you have to cover every aspect of it. By every aspect, I mean all
the amendments, any committee formed for it etc. This question was based on a very famous
committee set up by the government to review the act. The committee was chaired by NK
Singh which is the answer to this question.
Q.65)
According to Rangarajan committee for poverty estimation, persons spending below how
much amount a day in cities are considered poor?
a. Rs.32
b. Rs.47
c. Rs.30
d. Rs.41
e. Rs.20

Answer – (b)

Explanation –

For poverty estimation, various committees have been formed by the government of India. All
the committees are important for the exam. So, you must cover the recommendations of all
those committees formed for estimating poverty. Rangarajan committee was one such
committee formed for the same purpose. It recommended that persons spending below Rs.47
a day in cities and Rs.32 a day in village are considered poor.

Finance Management
2 Markers
Q.1)
Depending on the time at which corrective action is taken, controls are of three kinds. Which
one of the following controls involves the evaluation of inputs and taking corrective measures
before a particular sequence of operations is completed?

a. Feedback control
b. Reactive control
c. Concurrent control
d. Close loop control
e. Feedforward control

Answer – (e)
Explanation –
Let us talk about this question now. This question was asked from the topic
“CONTROLLING”. Controlling is one of the imperative functions of Management, albeit it is a
whole branch of its own. Controlling consists of verifying whether everything occurs in
conformities with the plans adopted, instructions issued, and principles established. And to
do that controlling involves different techniques of controlling.
Although the examiner will not know what is controlling you, it can ask about the types of
controlling, like this examiner.
So, make sure to do in-depth study if you want to score good marks. J

Q.2)
Power involves the ability to influence the behaviour of others. Which of the following
power is depends on the fear of negative results from failing to comply?
a. Coercive Power
b. Reward Power
c. Legitimate Power
d. Expert Power
e. Referent Power

Answer – (a)
Explanation –
Here again comes the easy question. This question is from the topic “POWER”. The
topic “POWER” in general is easy to understand. We have different kinds of power
but in this question particularly they are asking about coercive power. It is important
to understand certain keywords with respect to each option, for instance, if I talk
about ‘expert power’, here we can look for keywords like ‘expertise’ or ‘special
knowledge’.

Q.3)
Fill in the blank with an appropriate option below.
¾¾¾¾ is the size of the order for which the total ordering and carrying costs are at a
minimum.

a. Economic Utilization Quantity


b. Economic Order Quantity
c. Economic Manufactured Quantity
d. Economic Supply Quantity
e. None of the Above

Answer – (b)
Explanation –
This question is from the miscellaneous topic of RBI. However, this topic is not
included in the syllabus now.

Q.4)
In which type of inventory management system, the supply of components is
regulated through the use of instruction cards sent along the production line?

a. EOQ
b. Kaizen Technique
c. Total Quality Management
d. LIFO/FIFO
e. Kanban Technique

Answer – (e)
Explanation –
This is a part of the “CONTROLLING” topic. If someone has not studied either of
these techniques, then it would become challenging to answer such questions.
Hence, conceptual clarity is paramount.

Q.5)
A leadership theory which is based on two behavioural dimensions, the concern for
people and the concern for production is known as:

a. Managerial Grid Model


b. Fiedler’s Contingency Model
c. Hersey and Blanchard’s Situational Theory
d.
d. House’s Path Goal Theory
e. Charismatic Leadership

Answer – (a)
Explanation –
Again, same chapter “LEADERSHIP”. You must agree with me know it’s RBI’s favourite
topic. You should consider this a basic question. If you start reading about the “Managerial
Grid Model” this statement would be the first to read. Not so many technicalities in this
question. A simple reading would be sufficient to answer such questions.

Q.6)
When a manager gives specific advice, clarifies expectations, assigns tasks and works with
subordinates to identify behaviour to accomplish a task, he is using which leadership style:

a. Consultative leadership
b. Participative leadership
c. Delegative leadership
d.
e.
d. Transactional leadership
e. Transformational leadership

Answer – (b)
Explanation –
Again, same chapter “LEADERSHIP”. Question on the style of leadership again.
Follow the tip which we discussed above J.

Q.7)
Under which style of leadership does a leader act as a paternal or fatherly figure?

a. Autocratic or Authoritarian Leadership


b. Participative or Democratic Leadership
c. Free Rein or Laissez Faire Leader
d. Paternalistic Leadership
e. None of the Above

Answer – (d)
Explanation –
Again, same chapter “LEADERSHIP”. The question is about the styles of leadership.
With the help of keywords, for instance, paternal or fatherly figure one can easily
eliminate the options and can choose the correct answer.

Q.8)
Which of the following organisation structure is a combination of functional and
project organisation structure?

a. Network organisation
b. Learning organisation
c. Matrix organisation
d. Boundaryless organisation
e. Line and staff organisation

Answer – (c)
Explanation –
This question was part of the chapter “Organisational Structure”. I already gave you
an explanation of how to attempt these questions. Kindly read it again.

Q.9)
Which of the following decision-making metric under which the worst outcome is
selected among many and the worst payoff related to the outcome used in
decision-making?

a. Maximin
b. MiniMin
c. Maximax
d. Minimax
e. Expected Value

Answer – (a)
Explanation –
This question is asked from the topic “DECISION MAKING” which is a subpart of the
TOPIC “PLANNING”. Techniques of decision making is important for the examination
purpose.

Q.10)
In which theory of James MacGregor Burns leaders and followers help each other to
move to a higher level of motivation and morale?
a. Trait Theories
b. Contingency Theories
c. Behavioural Theories
d. Transformational Theories
e. Participative Theories

Answer – (d)
Explanation –
This question is from the topic called “LEADERSHIP”. This always has been one of
the favourite topics of the RBI examination. You can expect analytical or case study
questions from this question. Questions on theories of leadership can be confusing if
one does not have conceptual clarity. Here one cannot understand from the
statement which specific theory it is talking about.

Q.11)
Select from the option below, what are the different types of trade that are currently
settled through CCIL?

1. Forex Interbank Cash


2. Spot and Forward Transactions
3. Future Settlements
4. Option Settlements

a. 1 and 2
b. 3 and 4
c. 1 and 4
d. All the above
e. None of the above

Answer - (a)
Explanation –
This question is from an important topic called “FINANCIAL MARKET”. This is a
static-based question. CCIL stands for Clearing Corporation of India Limited. It was
set up in 2001 to provide exclusive clearing and settlement for transactions in Money
and Foreign Exchange. To be honest there is no trick to solving these questions you
will have to read and revise. The aim should be to cover the syllabus in detail and
revise frequently with the help of objective questions and keep yourself aware of
financial news.

Q.12)
The rupee has been weakening lately and has reached a record low of 70. What
suitable actions can be taken by RBI to further stabilize it?
a. Sell dollars
b. Buy dollars
c. Printing more currency
d. Sterilization
e. Market forces will decide itself

Answer – (a)
Explanation–
This is a current affairs-based question. Of course, to solve these kinds of questions
you need to have conceptual clarity. For instance, if I specifically talk about this
question, one cannot answer what steps should be taken by RBI unless one is aware
of the concept of depreciation, devaluation etc. That is why students should read the
static part in greater detail.

Q.13)
XYZ company issued bonus shares in the ratio of 1:1, what will be its effect on
Tangible worth?

a. Increase
b. Decrease
c. No change
d. None of the above
e. Not enough information

Answer – (c)
Example –
This question is from the topic called “INDIAN FINANCIAL SYSTEM”. This is a
conceptual question. You should know the concept of bonus shares in detail in order
to answer this question.

Q.14)
What is the correct order of claims (priority of liquidation) in the waterfall structure of
the Insolvency and Bankruptcy Code, 2016?

a. Insolvency costs, secured creditors, Government dues, Preference


Shareholders
b. Unsecured creditors, Insolvency costs, Government dues, Preference
Shareholders
c. Insolvency costs, Government dues, Preference shareholders, Unsecured
creditors
d. Government dues, Preference shareholders, secured creditors, Insolvency
costs
e. Unsecured creditors, Insolvency costs, Government dues, wages and unpaid
dues owed to employees
Answer – (a)
Explanation –
This question is from the topic called “Insolvency and Bankruptcy Code, 2016” which
comes under the sub-topic “Risk Management”. It provides a time-bound process for
resolving insolvency in companies and among individuals.
This is a static-based question. Basic knowledge of accountancy or how the claims
are settled of a company can also help in solving these kinds of questions.

Q.15)
Which of the following below must abide by the provision of IBC, 2016?

a. Individuals, Unlimited Partnerships, Limited Liability Partnerships and


Companies
b. Unlimited Partnership Firms, Limited Liability Partnerships and Companies
c. Individuals, Limited Liability Partnerships and Companies
d. Individuals and Companies
e. Only corporates

Answer – (a)
Explanation –
This question is from the topic called “Insolvency and Bankruptcy Code, 2016” which
comes under the sub-topic “Risk Management”. It provides a time-bound process for
resolving insolvency in companies and among individuals.
This is a static-based question which can only be solved by reading the provisions of
IBC in detail.

Q.16)
Basel Norms are the international banking regulations issued by the Basel
Committee on Banking Supervision. For now, three guidelines have been released
Basel I, Basel II and Basel III. Which of the following is/are the parameters of Basel
III?

1. Minimum capital requirement


2. Supervisory Review Process
3. Market Discipline

a. Only 1
b. Only 2
c. Only 3
d. Only 1 and 2
e. only 1, 2 and 3

Answer – (e)
Explanation –
This question is from an important topic called “Basel Norms” which is a sub-topic of
“RISK MANAGEMENT”. Basel norms or Basel accords are the international banking
regulations issued by the Basel Committee on Banking Supervision. It is an effort to
coordinate banking regulations across the globe, with the goal of strengthening the
international banking system.
This is also a static question. To be honest this is a very basic question to answer.
Minimum capital requirement, supervisory review process and market discipline are
three strong pillars of Basel 3.

Q.17)
which of the following is/are considered as risks under Basel 3 Norms issued by
BCBS?

a. Market Credit
b. Credit Risk
c. Liquidity Risk
d. Only 1 and 2
e. 1, 2 and 3

Answer – (e)
Explanation –
This question is from an important topic called “Basel Norms” which is a sub-topic of
“RISK MANAGEMENT”. Basel norms or Basel accords are the international banking
regulations issued by the Basel Committee on Banking Supervision. It is an effort to
coordinate banking regulations across the globe, with the goal of strengthening the
international banking system.
This is again a static question to answer. There are no tips and tricks to solve a static
question you will have to study. One thing is sure “BASEL ACCORDS” is an
important topic for the examination which you shouldn’t miss.

Q.18)
Keeping in Basel III norms, answer the following question.
Which of the following is/are the components of Pillar 1 capital?

1. Common Equity Tier Capital


2. Additional Tier 1
3. Tier 2 Capital
4. Capital Conservation buffer
5. Only 1 and 2

a. 1), 2) and 3
b. 1) and 4)
c. 2) and 4)
d. 1), 3) and 4)
e. All the mentioned.

Answer – (e)
Explanation –
This question is from an important topic called “BASEL NORMS” which is a sub-topic
of “RISK MANAGEMENT”. Basel norms or Basel accords are the international
banking regulations issued by the Basel Committee on Banking Supervision. It is an
effort to coordinate banking regulations across the globe, with the goal of
strengthening the international banking system.
This is again a static question to answer. There are no tips and tricks to solve a static
question you will have to study. One thing is sure “BASEL ACCORDS” is an
important topic for the examination which you shouldn’t miss.

Q.19)
Fill in the blank with an appropriate word below.

With an objective to encourage corporates to tap the debt market, the Government of
India, in the Union Budget for 2018-19 announced large corporates to meet about
¾¾¾¾ of their financing needs from the debt market.

a. 20%
b. 22%
c. 51%
d. 25%
e. 10%

Answer – (d)
Explanation – This question is a part of the topic called “Union Budget”. This is a
current affairs-based question. I think I have emphasized enough on Budget, let’s
proceed with the next question now.

Q.20)
Priority Sector Lending (PSL) emerged as an important idea to ensure the availability
of credit to all the neglected sectors of society. One of these sectors is education.
What is the current loan limit for education for studying in India and abroad
respectively?

a. Rs.20 lakhs and Rs.10 lakhs


b. Rs.10 lakhs and Rs.20 lakhs
c. Rs.25 lakhs and Rs.25 lakhs
d. Rs.20 lakhs and Rs.25 lakhs
e. Rs.10 lakhs and Rs.15 lakhs
Answer – (b)
Explanation –
This question is a part of the topic called “INDIAN BANKING SYSTEM”. This is a
static-based question. PSL is an important topic for the examination. You must be
wondering WHY? Because the goal of its to provide credit to the weaker sections of
society. For instance, (a) Agriculture (b) Micro, Small and Medium Enterprises (c)
Export credit (d) Housing (e) Social infrastructure (f) renewable Energy (g) Others.
RBI has mandated fixed targets for each of these sectors so that banks do not
neglect any of these sections. As I already mentioned this is a static-based question
and as such any tricks do not help in solving questions. The best trick is to
remember it by repetitive revision.

Q.21)
Under the revised guidelines for Priority Sector Lending (PSL), for housing loans in
metropolitan areas the loan limit has been raised from Rs.25 lakh to which of the
following?

a. 27
b. 30
c. 35
d. 4
e. 28

Answer – (b)
Explanation –
This question is a part of the topic called “INDIAN BANKING BANKING”. This is a
current affairs-based question. PSL is an important topic for the examination. You
must be wondering WHY? Because the goal of its to provide credit to the weaker
sections of society. For instance, (a) Agriculture (b) Micro, Small and Medium
Enterprises (c) Export credit (d) Housing (e) Social infrastructure (f) renewable
Energy (g) Others.
RBI has mandated fixed targets for each of these sectors so that banks do not
neglect any of these sections. Keep yourself updated with important financial news
articles and good videos so that you do not miss these important updates.

Q.22)
Fill in the blank with an appropriate option below.
As per RBI guidelines for priority sector lending ¾¾¾¾¾ % of the total net bank
credit should go to priority sector advances and ¾¾¾¾¾ % of adjusted net bank
credit should go to the agriculture sector.

a. 40% and 18%


b. 35% and 20%
c. 45% and 18%
d. 20% and 10%
e. 32% and 18%

Answer – (a)
Explanation –
This question is a part of the topic called “INDIAN BANKING SYSTEM”. This is a
current affairs-based question. PSL is an important topic for the examination. You
must be wondering WHY? Because the goal of its to provide credit to the weaker
sections of society. For instance, (a) Agriculture (b) Micro, Small and Medium
Enterprises (c) Export credit (d) Housing (e) Social infrastructure (f) renewable
Energy (g) Others.
RBI has mandated fixed targets for each of these sectors so that banks do not
neglect any of these sections. Keep yourself updated with important financial news
articles and good videos so that you do not miss these important updates.

Q.23)
Return on investment is the ratio of net profit and cost of investment. ROI provides
the measure of which of the following?

a. Profitability
b. Cash flow
c. Production
d. Time Value
e. Liquidity

Answer – (a)
Explanation –
This question is from the topic called “RISK MANAGEMENT”. This is a factual
question to answer. Return on investment is a metrix used to understand the
profitability of an investment. It compares how you paid for an investment to how
much you earned to evaluate its efficiency.
Basic understanding of ROI will be helpful to solve this question.

Q.24)
Which of the following is the most important dimension in driving the price of a stock
in the market?

a. Net Sales
b. Operating costs
c. P/E Ratio
d. EPS
e. Cash Flow

Answer – (d)
Explanation –
Earnings per share or EPS is an important financial measure, which indicates the
profitability of a company. This question is specifically part of the financial statements
of a company. This is a conceptual question to answer.

Q.25)
Introduction of Fiscal Responsibility and Budget Management (FRBM) Bill 2000 has
paved the way for the removal of –

a. Ad Hoc Bill
b. Ways and Means limit
c. Short Term Advance
d. Long Term Advance
e. None of the above

Answer – (d)
Explanation –
This question is a part of the topic called the FRBM Act. This topic is RBI’s all-time
favourite topic. This is a factual-based question.
It establishes financial discipline to reduce the fiscal deficit. The FRBM Act aims to
introduce transparency in India's fiscal management systems. The Act’s long-term
objective is for India to achieve fiscal stability and to give the Reserve Bank of India
(RBI) flexibility to deal with inflation in India. The FRBM Act was enacted to introduce
a more equitable distribution of India's debt over the years.

Q.26)
As per Union Budget 2018-19, which of the following schemes is related to price
fixation to ensure farmers are given the right price for their produce?

a. Operation Food
b. Operation Green
c. Operation Fasal
d. Operation Kisan
e. Operation Ghas

Answer – (b)
Explanation –
This question is a part of the topic called “BUDGETING”. This is again a current
affairs-based question.

Q.27)
Which of the following statements correctly demonstrated the difference between
Debt Recovery Tribunal (DRT) and NCLT (National Company Law Tribunal)?
a. The Debt Recovery Tribunal (“DRT”) shall be the adjudicating authority
(“Adjudication Authority”) with jurisdiction over individuals and partnerships.
The National Company Law Tribunal (“NCLT”) shall be the Adjudicating
Authority with jurisdiction over companies, and other limited liability entities
(excluding LLPs.)
b. The Debt Recovery Tribunal (“DRT”) shall be the adjudicating authority
(“Adjudication Authority”) with jurisdiction over individuals and partnership
firms other than Limited Liability Partnerships (“LLPs”). The National
Company Law Tribunal (“NCLT”) shall be the Adjudicating Authority with
jurisdiction over companies, and other limited liability entities (including LLPs.)
c. The Debt Recovery Tribunal (“DRT”) shall be the adjudicating authority
(“Adjudication Authority”) with jurisdiction over partnership firms other than
Limited Liability Partnerships (“LLPs”). The National Company Law Tribunal
(“NCLT”) shall be the Adjudicating Authority with jurisdiction over companies,
and other limited liability entities (excluding LLPs.)
d. The Debt Recovery Tribunal (“DRT”) shall be the adjudicating authority
(“Adjudication Authority”) with jurisdiction over individuals only. The National
Company Law Tribunal (“NCLT”) shall be the Adjudicating Authority with
jurisdiction over companies, and other limited liability entities (including LLPs.)
e. The Debt Recovery Tribunal (“DRT”) shall be the adjudicating authority
(“Adjudication Authority”) with jurisdiction over partnership firms other than
Limited Liability Partnerships (“LLPs”) only. The National Company Law
Tribunal (“NCLT”) shall be the Adjudicating Authority with jurisdiction over
companies, and other limited liability entities (including LLPs.)

Answer – (b)
Explanation –
This question is a part of the topic called “RISK MANAGEMENT”. This is a
conceptual-based question. Students should be clear with DRT and NCLT to answer
this question.

Q.28)
Fill in the blank with an appropriate option below.
¾¾¾¾¾ is used to regulate liquidity in the economy, ¾¾¾¾¾ is used to control
cash flow in the market.

a. repo rate, reverse repo rate


b. reverse repo rate, repo rate
c. cash reserve ratio, repo rate
d. Repo rate, cash reserve ratio
e. Statutory liquidity ratio, cash reserve ratio

Answer – (a)
Explanation –
This question is from the topic called “INDIAN BANKING SYSTEM”. This is a
static-based question. As I mentioned earlier you should be well-versed with these
terminologies before you answer these kinds of questions.

Q.29)
Which of the following banking terminologies is being discussed in the following
paragraph?

It is a financial benchmark rate for interest rate swaps, overnight call money,
collateralised borrowing and lending obligations (CBLO), floating rate bonds, and
short-term corporate loans in India. It is the rate at which Indian banks borrow from
one another on an overnight basis. It was first introduced in India in 1998 after the
formation of the Fixed Income Money Market and Derivatives Association of India
(FIMMDA).

a. MCLR
b. Bank Rate
c. MIBOR
d. REPO Rate
e. Reverse Repo rate

Answer – (c)
Explanation –
This question is from the topic called “INDIAN BANKING SYSTEM”. This is a
static-based question. As I mentioned earlier you should be well-versed with these
terminologies before you answer these kinds of questions.

Q.30)
Which of the following statements is associated with Fiedler’s Contingency Model?

a. There is the best style of leadership which is universally applicable to all


situations.
b. A leadership style may be effective under one situation and ineffective under
the other.
c. Fiedler identified three basic styles of leadership
d. a) and b)
e. b) and c)

Answer – (b)
Explanation –
Again same chapter “LEADERSHIP”. Only in-depth study can help you answer these
types of questions as the question is conceptually based. Here your vocabulary will
also help if you know the meaning of contingency despite not studying the topic can
help attempt the question.
Q.31)
In Herzberg’s Two-factor theory, Frederick Herzberg has given a distinction between
Motivating factors and Hygiene factors. Read the following options carefully and
mark the one which is not one of the hygiene factors.

a. Pay
b. Fringe Benefits
c. Physical working conditions
d. Recognition
e. Interpersonal Relations

Answer – (d)
Explanation -

This question was asked in the chapter “MOTIVATION”. You can say there are
certain favourite topics of RBI and motivation is one of them. Here the question is
asking about the distinction between motivating factors and hygiene factors. This is a
conceptual-based question and your concepts should be clear before you make a
distinction between these two factors.

Q.32)
In which of the following techniques of performance appraisal feedback is get from
supervisors, subordinates, peers, and self?

a. Work oriented Appraisal


b. 360-degree Appraisal
c. Informal Appraisal
d. Interview Appraisal
e. Self-Appraisal

Answer – (b)
Explanation –
This question has been asked several times in the examination now. The question is
from the topic “PERFORMANCE APPRAISAL”. The Question is conceptual here.
Techniques of performance appraisal are very important from the exam point of view.
We count this topic as one of the easiest and scoring. However, our bad luck this
topic is not the part of RBI syllabus now.

Q.33)
Individuals have the inherent behavior of comparing their performance with others
and this inadvertently influences their behavior in some form of the other. In the light
of the above statements which of the below mentioned theories shall best describe
the comparing attitude of the individuals.
a. Valence Theory
b. Equity Theory
c. Theory X and Y
d. Self-Efficacy Theory
e. Expectancy Theory

Answer – (b)
Explanation –
Another question from the theories of motivation students. Never ever skip the topic
of motivation. It carries high marks. This is the equity theory given by John Stacey
Adams. This theory is memorised based, easy to answer. Reading is enough to
answer these basic questions. Options are tricky here which makes this question
difficult to answer.

Q.34)
Who among the following propounded the idea of a managerial grid?

a. George & Graen


b. Hersey & Blanchard
c. Huneryager & Heckman
d. Tannenbaum & Schmidt
e. Blake & Mouton

Answer – (e)
Explanation –
haha!! ‘LEADERSHIP’ this time. Another chapter that you never going to miss if you
are preparing for the RBI examination.
This question clearly is factual. Here the examiner wants to know how much familiar
you are with your preparation by asking just a simple question. Through this
question, we understood that it is important to know the names of people who gave
different theories. If we get lucky, examiners can ask these kinds of questions for 1
mark.

Q.35)
Alderfer’s ERG theory is an extension of Maslow’s need hierarchy theory. What R
stands for in the respective theory?

a. Relative
b. Relatedness
c. Receiving
d. Recurring
e. Recovering

Answer – (b)
Explanation –
The ‘MOTIVATION’ chapter can save you during the Phase 2 crisis J. You must have
observed till now how much this chapter is important by seeing the number of
questions.
Just like the previous year this question is also completely factual. You ought to know
what ERG stands for,
E stands for Existence
R stands for Relatedness and
G stands for Growth.
You need to make sure that you know the full form well. We don’t know what the
examiner can ask J Hence, an in-depth study is important.

1 Markers
Q.36)
¾¾¾¾ records each and every component of the job that an individual has to
perform in a given set-up

a. Job description
b. Job role or job analysis
c. Job specification
d. Job evaluation
e. Job appraisal

Answer – (a)
Explanation –
This question was asked from the topic “JOB ANALYSIS & DESIGN” subpart of
Human Resource Management. We have different terminology here, for instance,
Job description, the Job role or Job analysis, Job specification, Job Evaluation and
Job Appraisal. In these kinds of questions if the concepts are not clear then it will be
difficult to answer. Here the answer is a job description which is a subpart of job
analysis. You’re lucky this is not part of the RBI syllabus anymore.

Q.37)
In an organisation, generally employees are resistant to change. Resistance to
change could be overcome on an enduring basis by systematically planning and
implementing the process of change. To be an effective change in the organisation
Kurt Lewin has introduced a model of planned change.
Read the following options below and mark the first stage of the same model.
a. Change
b. Resistance
c. Compliance
d. Unfreeze
e. Refreeze

Answer – (d)
Explanation –
This was asked from the topic “ORGANISATIONAL CHANGE”. This a very important
topic from a both objective and descriptive point of view. DO NOT MISS IT EVER.
Hey, I am saying it again DO NOT SKIP THIS TOPIC. This is a factual-based
question, easy to answer. You must be wondering HOW? Because the examiner is
asking only for the steps to bring planned change from us. Kurt Lewin’s planned
change model is very important.

Q.38)
Which of the following is a type of communication which does not have any definite
pattern or direction and flows at every level to satisfy needs?

a. Formal Communication
b. Verbal Communication
c. Oral Communication
d. Grapevine Communication
e. Visual Communication

Answer – (d)
Explanation –
Another important chapter “COMMUNICATION”. Communication itself is a very easy
topic to understand and students generally get full marks, you must have had a
sense by now by reading the question. Grapevine communication is a form of
informal communication which arises from the social interaction of people. It does
not have any definite path which is why it can flow in every direction. From an exam
point of, types of communication are important – both formal and informal.

Q.39) Fill in the blank with an appropriate option below.


¾¾¾¾¾ occurs when an individual reports wrongdoing in an organisation such as
financial misconduct or discrimination.

a. Insider Trading
b. Selling
c. Whistleblowing
d. Trading
e. Vigilante
Answer – (c)
Explanation –
Whistleblowing is a part of “CORPORATE GOVERNANCE”. Corporate governance
in itself is a beautiful topic. This question was asked from the Whistle Blowing
Protection Act. It is a pretty easy question to answer as the question is based on
“terminology”. This puts light on the fact how much it is important to be well versed in
terminologies and have clear basics. Students generally do, they try to be specific to
the syllabus only I am not saying it is not the correct way to prepare for your
examination all I am saying is if study the basics well then it will be a piece of cake
for you to read subsequent topics.

Q.40)
Which of the following leadership theories suggests that an individual with traits
relevant to leadership emerges as an effective leader?

a. Trait Theory
b. Fiedler’s Contingency Model
c. Hersey and Blanchard’s Situational Theory
d.
d. House’s Path Goal Theory
e. Charismatic Leadership

Answer – (a)
Explanation –
Again same chapter “LEADERSHIP”. Here in the form of a question, a
characteristic/feature of a theory is given and we have to answer the name of a
theory. This question is specific to theories of leadership and in order to answer
these types of questions understanding different theories is essential. However, we
can also eliminate some of the options just by reading for instance contingency
means to depend on the situation, same for option c), under charismatic leadership
there is charisma in a leader’s personality. We can also apply these tricks to answer
a tricky question.

Q.41)
“ It is the leader who gives instructions only after consulting the group and also
believes in the decentralising decision-making process”?

a. Democratic Leadership Style


b. Paternalistic Leadership Style
c. Charismatic Leadership
d.
d. Servant Leadership
e. Transactional Leadership
Answer – (a)
Explanation –
Again same chapter “LEADERSHIP”. In leadership, we have four different kinds of
leaders and the question is asking one of them. Generally, students get confused
between different styles. The quick way to answer is to know the keywords related to
every style. For instance, if I talk about transactional leadership, here keyword is the
“carrot and stick approach” So generally wherever the “carrot and stick approach” is
mentioned it is transactional leadership only.

Q.42)
In which of the following organisation structure, each specialist is supposed to give
his functional advice to all other foremen and workers?

a. Line Organisation
b. Functional Organisation
c. Line and Staff Organisation
d.
d. Either A & C
e. All the above

Answer – (a)
Explanation –
This question was from a topic called “ORGANISATIONAL STRUCTURE”. Some
students find this topic tricky to answer however you do not need to worry this topic
is not in the syllabus anymore. Let us talk about the question now. It is a
conceptual-based question and here also keywords will help you to answer the
question that happened in this question.

Q.43)
Which type of organisation structure is suitable for a large-scale organisation having
diversified activities requiring a high degree of specialisation in operations?

a. Project Structure
b. Division Structure
c. Functional Structure
d. Matrix Structure
e. Network Structure

Answer – (c)
Explanation –
This question was from a topic called “ORGANISATIONAL STRUCTURE”. A student
can easily answer the question by the keyword mentioned in the question, here we
have two ‘diversified activities’ and ‘high degree of specialisation’. Try to remember
keywords with respect to every topic.

Q.44)
What MBO stands for?

a. Management by Obsession
b. Management by Objective
c. Management by Opportunity
d. Management by Occasion
e. None of the Above

Answer – (b)
Explanation –
This question is asked from the topic “MANAGEMENT BY OBJECTIVE”. This is an
important topic. Now let us quickly discuss this question. Here examiner is testing
your basic knowledge regarding the subject. For instance, when we try to learn any
language we start from the basis same applies here. For the sake of passing the
examination, some students generally study selectively which is not a fine approach
to preparation.

Q.45)
Which of the leadership theories of management focuses on how the leader provides
subordinates with a path to achieve valued goals?

a. Transactional Theory
b. Path-Goal Theory
c. Contingency Theory
d.
d. Transformational Theory
e. Behavioural Theory

Answer – (b)
Explanation –
I was watching a movie lately and in that movie, it was mentioned in Hindi I am
translating that into English for you “the answer lies in the question only”, the same
goes for this question. In question, it is clearly motioned that “a leader provides
subordinates with a path to achieve valued goals” this statement is sufficient to
answer the questions, i.e., Path-Goal Theory.

Q.46)
Which of the following involves the power to reward, promote or grant key
resources?
a. Coercive Power
b. Reward Power
c. Legitimate Power
d. Expert Power
e. Referent Power

Answer – (b)
Explanation –
This question was asked from the topic “POWER” as we have already discussed the
same sort of question in previous questions. Here the answer is a reward power
which you can simply figure out by the keywords which are in this question promote
or grant key resources.

Q.47)
Control system which automates the process of control is known as:

a. Cybernetics
b. Auto-control
c. Splunk control
d. Both a) and b)
e. None of the above

Answer – (a)
Explanation –
This question is a part of the topic called “COMMUNICATION”. Cybernetics was in
news before the examination and the examiner took the opportunity to test how well
you are aware of your surroundings. The learning from this question is to have a
good grasp of financial and current awareness.

Q.48)
An organization structure being implemented without considering vertical or
horizontal bounds across departments and even with an external environment is
called ----------------.

a. Division organisation
b. Boundaryless organisation
c. Line and staff organization
d. Network organization
e. Learning organization

Answer – (b)
Explanation –
Another question from the same topic is called “ORGANISATIONAL STRUCTURE”.
An organisational structure is a system that outlines how specific activities are
handled to fulfil a strategic mission. Rules, roles, and obligations are all parts of
these activities. Why I am explaining this here OS because only then you will be able
to understand the different categories of organisational structure. As I said earlier in
previous questions conceptual clarity is important. If you have conceptual clarity you
will be able to solve maximum questions.

Q.49)
What that process is called through which the conversion of scanned documents and
image-based information like handwritten, typed, or printed text into a digital,
machine-readable text format?

a. Optical Code Recognition


b. Optical Character Reader
c. Optical Character Recognition
d. Optical Code Reader
e. Optical Character Recorder

Answer – (c)
Explanation –
This question is asked from the topic “FINTECH”. It is a static question. Topic Fintech
is itself an important topic not only for examination purposes but also for interviews.
As the world is moving towards the digital era so the relevance of these topics is
increasing. One thing you can do is keep yourself updated with current affairs so that
you get familiar with these terminologies.

Q.50)
Lately Small Industries Development Bank of India (SIDBI) has launched the
‘Udyami Mitra Portal’. What is the objective behind this scheme?

a. To improve the accessibility of credit & handholding services to the MSMEs


b. To provide digital literacy to MSME
c. To build awareness about intellectual property
d. To promote innovation, rural property, and entrepreneurship
e. To enable the establishment of a strong manufacturing base in the MSME
sector

Answer – (a)
Explanation –
This question is a part of the topic called “REGULATORY BODIES OF INDIA”. This
is a current affairs-based question. SIDBI launched this scheme in December 2017.
That is why I always encourage students to be well-versed with finance current
affairs so that you do not miss any marks.

Q.51)
Public-Private partnerships involve collaboration between a government agency and
a private sector company. In what kind of project do these parties generally get
involved?

a. Small business firms


b. Large corporates
c. Small corporates
d. Start-ups
e. Large infrastructure funds

Answer – (e)
Explanation –
This question is asked from the topic called “PUBLIC-PRIVATE PARTNERSHIP”. It is
a pure static question. In order to solve these kinds of questions merely basic
understanding is enough because it comes in the introduction only when you start
reading about PPP. You cannot apply “TRIAL & ERROR” here.

Q.52)
Which of the following monitor the United Payment Interface (UPI) in India?

a. RBI
b. NPCI
c. IBA
d. SEBI
e. Ministry of Finance

Answer – (b)
Explanation –
This is a pure static-based question. Here examiner wants to know how well you can
connect the dots and remember the important details. The UPI system has exploded
in popularity since its introduction in 2016. Keeping in mind this popularity RBI
convert the current affairs-based question into a factual one.
The National Payments Corporation of India (NPCI) an organization in charge of
managing retail payments and settlement systems in India.

Q.53)
As per Budget 2018, What is the revised fiscal deficit target for 2018-19?

a. 3.1% of GDP
b. 3.2% of GDP
c. 3.3% of GDP
d. 2.1% of GDP
e. 3.5% of GDP

Answer – (c)
Explanation –
This question is a part of the topic called “FRBM ACT”. It was established in 2003 to
reduce the fiscal deficit. It aims to introduce transparency in India’s fiscal
management system. The Act’s long-term objective is for India to achieve fiscal
stability and to give the Reserve Bank of India (RBI) flexibility to deal with inflation in
India.
You must remember these specific targets and frequent revision can help you here.

Q.54)
On July 1st, 2017, the Goods and Services Tax was implemented in India. With GST
Government of India introduced certain slab rates. Which one of the following is the
highest slab rate?

a. 18%
b. 24%
c. 28%
d. 19%
e. 25%

Answer- (c)
Explanation –
This question is from the topic “Goods & Services Tax”. It is a static question in the
form of current affairs. GST was a hot topic at that time and RBI took this as an
opportunity to ask in the examination. The learning from this question is that not only
static but also financial current affairs help in the examination. It is imperative to read
the news every day so that you do not miss any marks.

Q.55)
To capture structural changes in the Indian economy and improve the quality,
coverage, and representativeness of the indices the base year for calculation of WPI
is revised to –

a. 2010 - 2011
b. 2009 – 2010
c. 2011 – 2012
d. 2017 - 2018
e. 2018 - 2019

Answer – (c)
Explanation –
This question is from the chapter “INFLATION”. This is again a current affairs-based
question. WPI stands for Wholesale Price Index, which reflects changes in the
average prices of goods at the wholesale level — that is, commodities sold in bulk
and traded between businesses or entities rather than goods bought by consumers.
If you follow news on a regular basis or any channel which covers financial news in
detail, then you will never lose marks.

Q.56)
Under IBC, 2016 what is the maximum time limit for the final resolution of an entity?

a. 100
b. 180
c. 200
d. 365
e. 270

Answer – (e)
Explanation –
This question is from the topic called “INSOLVENCY AND BANKRUPTCY CODE”.
This is a factual question to answer.

Q.57)
Fill in the blank with an appropriate word.
¾¾¾¾¾¾ is the use of small amounts of capital from many individuals to finance a
new business venture.

a. Hedge Funding
b. Sponsorship
c. Crowd Funding
d. Auctioning
e. Exhibition

Answer – (c)
Explanation –
It is a question from the chapter “FinTech”. As I mentioned earlier topic like “FinTech”
is gaining prominence in our society because these new terminologies are emerging
like “crowdfunding”. Again, I would like to reiterate the fact newspaper reading is
essential if you want to clear any examination. PLEASE DO THAT.

Q.58)
What is the amount of standard deduction announced in the recent budget for FY
2018 – 19 in lieu of the present exemption in respect of transport allowance and
reimbursement of miscellaneous medical expenses to salaries taxpayers?
a. 70,000
b. 50,000
c. 40,000
d. 20,000
e. 15,000

Answer – (c)
Explanation –
This is a question from your topic “BUDGETING”. It is a pure current affairs-based
question. Always keep yourself well versed with the important announcement of
“BUDGET” or to save your time you can also watch a video regarding the same

Q.59)
As per the given data, which bank is suitable for investment if the criteria of the P/E
ratio is followed?

Banks Share price per share EPS


Axis Bank 1000 60
ICICI Bank 1298 52
HDFC Bank 2011 67
CITI Bank 827 42
Yes Bank 1550 70

a. Axis Bank
b. ICICI Bank
c. HDFC Bank
d. CITI Bank
e. Yes Bank

Answer – (c)
Explanation –
As you can see this is a numerical based question. Only conceptual understanding
can help you in interpreting these questions. Here the question is about the P/E ratio,
it is the ratio of share price of a stock to its earnings per share (EPS). PE ratio is one
of the most popular valuation metrics of stocks. It provides indication whether a stock
at its current market price is high or cheap.

Q.60)
What is the target rate of retail Inflation in India?
a. 4% +/- 2%
b. 3% +/- 1%
c. 4% +/- 1%
d. 6% +/- 3%
e. 7% +/- 2%

Answer – (a)
Explanation –
This question is a part of the topic called “INFLATION”. This is a completely
static-based question. If we see the question, this question is basic to answer. This is
the ABC of inflation which you must read before you move on to the theoretical part
and then its applications.

Q.61)
Which of the following is/are not a liquidity tool?

a. Repo Rate
b. Reverse Repo Rate
c. Cash Reserve Ratio
d. Interest Rate Swap
e. Marginal Standing Facility

Answer – (d)
Explanation –
This question is a part of the topic “Indian Banking System”. This is a
conceptual-based question. WHY? Because unless you are familiar with these
different terminologies for instance, what is repo rate? What is a marginal standing
facility? etc., you will not be able to answer such questions.

Q.62)
Which of the following is responsible for approving the resolution plan submitted by
insolvency professionals?

a. NCLT
b. COC
c. NCLAT
d. Corporate Debtor
e. None of the Above

Answer – (b)
Explanation –
This question was asked from the topic called “RISK MANAGEMENT”, sub-topic
Insolvency and Bankruptcy Code. This is a factual-based question. You need to
know in detail about IBC in order to attempt this. Generally, students get confused
with the answer, they feel NLCT is the answer but it is not the case. I assume you
will not mark NCLT.

Q.63)
Fill in the blank with an appropriate option below.
The fiscal deficit target for the year 2018-19 is ¾¾¾.

a. 2.2%
b. 3.3%
c. 4.4%
d. 5.5%
e. 6.6%

Answer – (b)
Explanation –
This question is a part of the topic called “FRBM Act”. This is a current affairs-based
question. Keep yourself updated with finance current affairs is the key to attempt
these questions.

Q.64)
As per Union Budget 2018-19, exemptions of interest income for senior citizens on
deposits with banks and post offices are proposed to be increased from Rs.10,000 to
¾¾¾.

a. Rs.20,000
b. Rs.30,000
c. Rs.40,000
d. Rs.50,000
e. Rs.55,000

Answer – (d)
Explanation –
This question is a part of the topic called “BUDGETING”. This is again a current
affairs-based question. And I know what you need to do, Revise, Revise and Revise.

Q.65)
Who among the following is the chairperson of the GST Council?

a. Prime Minister
b. Finance Minister
c. Minister of State for Finance
d. Secretary, Department of Revenue
e. Chairman, Central Board of Indirect Taxes and Customs
Answer – (b)
Explanation –
This question is a part of the topic called “GOODS AND SERVICES TAX”. GST is an
important topic as it came as a revolutionary taxation system. This is a factual
question. Through this, the examiner wants to know how well-versed you are in the
important stuff going around you. As this is a static question, no conceptual clarity is
required to solve this kind of question. J
Economic and Social Issues
15 Markers

Q.1) Despite spread of urbanization in villages, challenges such as race, caste, gender discrimination still
persist.

Q.2) What is meant by economic reforms? Discuss the Economic reforms in India from 1990s till
date.

Q.3) “Financial stability is essential to any financial institution/business.” Discuss in the light of
financial stability report of RBI and economic reforms in view of COVID outbreak in India.

10 Markers

Q.1)
What are the challenges associated with rural entrepreneurship? How young enterprises can solve
these problems?

Q.2)
Write a short note on current monetary policy framework in India

Q.3)
What is NBFC? How is it different from banks? What are the five types of NBFCs registered with
RBI?

Finance and Management


15 Markers

Q.1)
What is communication? What are the common barriers to interpersonal communication in an
organization? How can these barriers be reduced by a manager?

Q.2)
Why is change resisted in an organization? What can the management do to overcome it?

Q.3)
What do you understand by blockchain technology. In the current era, where blockchain is at its
peak, will it disrupt banking?

10 Markers

Q.1) What is targeted long-term repo operations (TLTRO)?


[Type here]

Q.2) Describe India’s response to the Global Financial Crisis 2008.

Q.3) What is the importance of ethical behavior in society and throw some light on the manager’s
role in encouraging ethical behavior in an organization.

English Descriptive
Q.1)

Write an essay on any one of the following topics in about 600 Words. (40 Marks)

• “Peace cannot be kept by force; it can only be achieved by understanding” - Elaborate.

• Role of female entrepreneurs in India .

• Advantages and disadvantages of renewable energy. How the OSOWOG initiative will help
in eliminating these disadvantages.

• India has become a medical destination for the world.

Q.2)

Write a precis from the given paragraph in about 180 words. (30 Marks)

As controversies swirl about funding, resources, motives and methods for spaceflight, it is well to
consider the consequences of exploring space – and of choosing not to do so. Earlier essays have
already analyzed in broad terms some of the potential consequences of not exploring, especially
the lack of creativity that an inward-looking society may bring, by analogy with Ming China. Others
have pointed out that the Age of Discovery was only a mixed blessing, especially when it came to
culture contact.

The consequences of space exploration as already undertaken stand before us for examination.
They occur on many levels: commercial applications, education and inspiration to youth,
applications satellites, scientific benefits, and philosophical implications. All are open to analysis,
and as we approach the fiftieth anniversary of the Age of Space, we should examine, with
historical objectivity, precisely what the impact of the Age of Space has been.
One feature unlikely to be paralleled with the Age of Discovery, at least in the near future, is
contact with other cultures. Shipboard observers often carefully noted exotic plants and animals
seen during the course of their voyages, and the ultimate experience was contact with exotic
humans.

In the Age of Space, the search for microbial life has been a main driver of space exploration, in
particular with regard to Mars, but also now extended to more exotic environments like the Jovian
moon Europa. With that search for life on new worlds, planetary protection protocols have been
put in place, both for our own planet and others.

Contact with intelligent extraterrestrials beyond the solar system will remain a more remote
possibility, and when and if it happens, we should learn from the history of culture contacts on
pg. 1
[Type here]

Earth.

But the immediate impact of the Space Age is far more diverse than the ultimate discovery of life
in space. In her new book Rocket Dreams: How the Space Age Shaped Our Vision of a World
Beyond, Marina Benjamin argues that space exploration has shaped our worldviews in more ways
than one. "The impact of seeing the Earth from space focused our energies on the home planet in
unprecedented ways, dramatically affecting our relationship to the natural world and
our appreciation of the greater community of mankind and prompting a revolution in our
understanding of the Earth as a living system," she wrote.

Benjamin thinks it is no coincidence that the first Earth Day on April 20, 1970, occurred in the
midst of the Apollo program; or that one of the astronauts developed a new school of spiritualism;
or that people "should be drawn to an innovative model for the domestic economy sprung free
from the American space program by NASA administrator James Webb." Exploration
shapes world views and changes cultures in unexpected ways, and so does lack of exploration.

Space has had more tangible impacts on society. To take only one, imagine where we would be
without applications satellites. We now take for granted photographs of weather and Earth
resources data from space, as well as navigation and worldwide communications made possible
by satellite.

Along with human and robotic missions, the late twentieth century will be remembered collectively
as the time when humans not only saw the Earth as a fragile planet against the backdrop of
space, but also utilized near-Earth space to study the planet's resources, to provide essential
information about weather, and to provide means for navigation that was both life-saving and
had enormous economic implications. Worldwide satellite communications brought the world
closer together, a factor difficult to estimate from a cost-benefit analysis. Names like Landsat,
GOES (Geostationary Operational Environmental Satellites), Intelsat and Global Positioning
System may not be household words, but they affect humanity in significant ways not
always appreciated.

Important as they are, applications satellites pale in significance to what space may represent for
the future of humanity. While some argue that robotic spacecraft are cheaper and less risky than
human spaceflight, it is my belief that humans will nevertheless follow robotic reconnaissance as
night follows day.

Humans will not be content with a Space Odyssey carried out by robotic surrogates, any more
than the other great voyages of human history. Robots extend the human senses but will not
replace the human mind in the foreseeable future, even with advances in artificial intelligence.
HAL in Arthur C. Clarke's famous novel and movie, was not as smart as he thought and will not be
for a long time. As President Bush said in announcing his new initiative in January 2004, humans
will spread through the solar system, fulfilling the vision of what British philosopher
Olaf Stapleton 55 years ago called "interplanetary man."

Eventually humans will spread into the cosmos at large. One cannot set a timeline, but by 3001
"interstellar humanity" will likely follow. We do not know what surprises and challenges we will find.
But they will be there, and humans will revel in them. That is the nature of humans with their inbuilt
curiosity and penchant for exploration, one might say the very definition of what it is to be human.

Historians and social scientists have analyzed this kind of argument, and not all agree that the
utopian ideal of spreading humanity to outer space is a valid reason for going, or that utopia is
what we will build when we get there. In a democratic society such arguments must be fully
pg. 2
[Type here]

voiced. Others have demonstrated the complex relation of such space goals to social, racial and
political themes. One such study is De Witt Kilgore's recent book Astro futurism: Science,
Race and Visions of Utopia in Space. In this book Kilgore examines the work of Wernher von
Braun, Willy Ley, Robert Heinlein, Arthur C. Clarke, Gentry Lee, Gerard O'Neill and Ben Bova,
among others in what he calls the tradition of American Astro futurism.

Such studies remind us that, like it or not, the idea of space exploration has been woven into the
fabric of society over the last 50 years, even as exploration has raised our cosmic consciousness.
The historical analysis of that transformation, in ways large and small, should help us make
informed choices about our future in space.

Q.3)

Read the following comprehension and answer the questions that follow (5X6 = 30 Marks)

Financial derivatives are financial instruments that are linked to a specific financial instrument or
indicator or commodity, and through which specific financial risks can be traded in financial
markets in their own right. Transactions in financial derivatives should be treated as separate
transactions rather than as integral parts of the value of underlying transactions to which they may
be linked. Two prominent market players in derivatives are hedgers and speculators/traders.
Hedgers are the underlying asset owners, who wish to transfer the future price fluctuation risk,
while speculators are the risk consumers, who take positions in derivative contracts based on the
predictions of future movement of prices of the underlying asset. Speculators hold derivative
positions with or without owning the underlying assets. Arbitrageurs are the third category market
participants, whose approach is to risk-proof themselves. They take advantage of the price
difference in a product in two different market locations. This trade takes place where the buyer
purchases an asset for a cheaper pr ice in one market/location and arranges to sell the same
simultaneously in a different market/location at a higher price.

The value of a financial derivative derives from the price of an underlying item, such as an asset or
index. Unlike debt instruments, no principal amount is advanced to be repaid and no investment
income accrues. Financial derivatives are used for a number of purposes including risk
management, hedging, arbitrage between markets, and speculation.

Forwards are contracts signed between two parties to buy or sell the underlying asset on any
future date for a price agreed upon on the date of signing of the contract. Forwards are
decentralized and customizable contracts based on the requirements of the parties involved and
are traded over the counter (OTC) and not on any exchanges.

Futures are standardized forward contracts which are traded through any regulated exchanges.
Forwards and futures are a commitment to buy or sell the asset ..

pg. 3
[Type here]

Financial derivatives enable parties to trade specific


financial risks (such as interest rate risk, currency, equity
and commodity price risk, and credit risk, etc.) to other
entities who are more willing, or better suited, to take or
manage these risks—typically, but not always, without
trading in a primary asset or commodity. The risk embodied
in a derivatives contract can be traded either by trading the
contract itself, such as with options, or by creating a new
contract which embodies risk characteristics that match, in a
countervailing manner, those of the existing contract owned.
This latter is termed offset ability and occurs in forward
markets. Offset ability means that it will often be possible to
eliminate the risk associated with the derivative by creating a
new, but "reverse", contract that has characteristics that
countervail the risk of the first derivative. Buying the new
derivative is the functional equivalent of selling the first
derivative, as the result is the elimination of risk. The ability
to replace the risk on the market is therefore considered the
equivalent of tradability in demonstrating value. The outlay
that would be required to replace the existing derivative
contract represents its value—actual offsetting is not
required to demonstrate value.

Who participates in the derivative market?


Discuss the three uses of the derivative.
What is the difference between forward and the future market?
What is offset ability?
From where do financial derivatives derive their price?

pg. 4
ESI FM Objective Question Paper
2023
[Type here]

Economic & Social Issues


2 Markers

I.1-4) X is an international organization that deals with the global rules of trade between nations. It
is successor to the General Agreement on Tariffs and Trade (GATT), which was in place since 1948.
It has its headquarters in Geneva, Switzerland, and serves as the principal forum for trade
negotiations, dispute settlement, and the development of global trade rules.

X facilitates trade negotiations among its member countries to develop and update trade
agreements. These negotiations, known as trade rounds, aim to lower trade barriers and address
various trade-related issues to promote free and fair trade. It also administers a set of agreements
that lay down the rules for international trade in goods, services, and intellectual property. The
most significant of these agreements are the General Agreement on Tariffs and Trade (GATT) for
goods, the General Agreement on Trade in Services (GATS) for services, and the Agreement on
Trade-Related Aspects of Intellectual Property Rights (TRIPS).

Q.1)
Which round of X talks faced various challenges throughout its negotiations and has remained
uncompleted within the originally intended timeframe?

(a) Uruguay Round


(b) Doha Round
(c) Nairobi Round
(d) Buenos Aires Round
(e) Beijing Round

Answer – (b)

Q.2)
When was X mentioned in above paragraph established?

(a) 1 July 1997


(b) 1 October 1996
(c) 1 January 1995
(d) 1 March 1994
(e) 1 September 1993

Answer - (c)

pg. 1
[Type here]

Q.3)
Who is the incumbent Director-General of the X mentioned in the above passage?

(a) Peter Sutherland


(b) Ngozi Okonjo-Iweala
(c) Roberto Azevêdo
(d) Mike Moore
(e) Renato Ruggiero

Answer - (b)

Q.4)
Which of the following is the highest decision-making body of X mentioned in the above passage?

(a) Goods and Services Council


(b) TRIPS Council
(c) Dispute Settlement Body
(d) General Council
(e) Ministerial Conference

Answer – (e)

Explanation –

Questions 1 to 4 are based on World Trade Organization. The questions were asked from the topic
which is directly mentioned in the syllabus – “International Economic Institutions – IMF and World
Bank – WTO – Regional Economic Cooperation”. Hence, the questions are justified because the RBI
did not go out of syllabus.

Now, talking about the questions –


• Question number 1 was difficult and in-depth. To solve this, you need an in-depth
knowledge of the topic.
• Question number 2 was a factual question and again was a difficult one.
• Question number 3 was very easy because when Iweala became the DG of WTO, she was in
news for a long time because she is the first woman and first African to lead the
organization.
• Question number 4 was moderate because when you are studying any organization, it is
expected that you are covering certain basic points about that organizations viz; year of
establishment, headquarters, chief, highest decision-making body etc.

All the 4 questions were covered in my ESI Live classes. The chapter “International economic
institutions” was discussed in detail by Manish Sir in his live class.
So, for the upcoming exam, you need to keep a check on the syllabus and should not leave a single
topic.
pg. 2
[Type here]

I.5-9) X mission was launched by Hon’ble Prime Minister Narendra Modi on 20 October 2022 at
Kevadia, Gujarat. It is India-led global mass movement to nudge individual and community action
to protect and preserve the environment.

X focuses on bringing about behaviour changes in individuals for combating climate change. Its
objective is a global movement for transitioning from a model of destructive and mindless
consumption to deliberate and mindful utilization of resources. Ministry of Environment, Forest
and Climate Change is the nodal ministry for national-level coordination and implementation of
this Mission.

X also recognises that Indian culture and living traditions are inherently sustainable. It highlights
importance of conserving our precious natural resources and living in harmony with nature are
emphasised in our ancient scriptures.

Energy consumption is a significant contributor to greenhouse gas emissions, which is the primary
cause of climate change. By reducing energy use, we can reduce our carbon footprint and help
mitigate the effects of climate change

Q.5)
Identify the Mission X mentioned in the above passage

(a) Mission Saubhagya


(b) Mission AWARe
(c) Mission for Global Green
(d) Mission on Strategic Knowledge for Climate Change
(e) Mission LiFE

Answer - (e)

Q.6)
At which of the following international event in 2021 was the Mission X mentioned in above
passage announced by Prime Minister?

(a) UNFCCC COP24, Katowice


(b) UNFCCC COP25, Madrid
(c) UNFCCC COP26, Glasgow
(d) UNFCCC COP27, Sharm El Sheikh
(e) UNFCCC COP28, Dubai

Answer - (c)

pg. 3
[Type here]

Q.7)
Which of the following is not amongst seven themes of the Mission X?

(a) Reducing use of single-use plastic


(b) Adopting healthy lifestyle
(c) Sustainable food systems
(d) Planting Trees
(e) Energy saving

Answer - (d)

Q.8) X Mission envisions three core shifts towards sustainability in three phases. Match the correct
pairs

A. Phase 1 – i. Change in Policy


B. Phase 2 – ii. Change in Supply
C. Phase 3 – iii. Change in Demand

(a) A-iii, B-ii, C-i


(b) A-i, B-ii, C-iii
(c) A-ii, B-iii, C-i
(d) A-i, B-iii, C-ii
(e) A-iii, B-i, C-ii

Answer - (a)

Q.9)
Which of the following is the first country to include mission X in its Nationally Determined
Contribution?

(a) South Africa


(b) USA
(c) Australia
(d) India
(e) China

Answer – (d)

Explanation –

Questions from 5 to 9 were purely Current Affairs based Questions. They are based on Mission
LiFE. This topic was in news so many times and is very significant initiative of Government of India
to assist the world in combating climate change.

pg. 4
[Type here]

So, it definitely forms part of ESI. These types of important initiatives are thoroughly covered in PIB
247 in detail in the form of sessions and PDFs, covering every aspect of topic.

Now, let’s talk about the questions –

• Question number 5 was easy because the candidate had to only identify the mission and for
that various hints were given in the passage.
• Question 6 was easy for those who were following PIB sessions being conducted by Manish
Sir on YouTube. This was discussed a lot of time in PIB classes.
• Question number 7 was also discussed in PIB sessions. There are 7 seven themes of Mission
LiFE viz; save water, save energy, reduce waste, reduce e-waste, reduce single-use plastics,
adopt sustainable food systems, and adopt healthy lifestyles. Hence, the answer to this
question is “Planting trees” which is not one of the seven themes of Mission LiFE.
• Question number 8 was about the three phases of Mission LiFE viz; Change in Demand
(Phase 1), Change in Supply (Phase 2) and Change in Policy (Phase 3).
• Question number 9 was on difficult lines. Although the information asked in the question
was from a PIB release yet I would say it was a difficult question. The question is justified
because the answer to the question is our pyara Bharat i.e. “India”

I.10-13) X report is released twice a year by Y organisation, providing an assessment of the global
economic situation. It provides an assessment of the global economic situation, including
projections for economic growth, inflation, fiscal balances, and other macroeconomic indicators
for both individual countries and regions

X report also presents economic forecasts for the near and medium-term for a wide range of
countries and regions. These forecasts are based on various factors, including historical data,
policy assumptions, and current economic conditions. It also contains an in-depth analysis of
individual countries and regions, including discussions on their economic performance, policies,
and potential risks and vulnerabilities.

In addition to providing economic analysis and projections, X report also offers policy
recommendations to address economic challenges and promote sustainable economic growth.
These recommendations are aimed at policymakers in member countries and international
institutions.

Q.10)
Identify the report X mentioned in above paragraph.

(a) Global Economic Gap


(b) Global Economic Prospects
(c) World Economic Outlook
(d) World Economic Situation and Prospects
(e) World Economic and Social Outlook

pg. 5
[Type here]

Answer- (c)

Q.11)
As per the report X, mentioned in above paragraph, advanced economies are expected to see an
especially pronounced growth slowdown, from 2.7 percent in 2022 to ____________ percent in
2023. Fill the gap

(a) 1.8 percent


(b) 1.3 percent
(c) 1.5 percent
(d) 1.7 percent
(e) 1.2 percent

Answer - (b)

Q.12)
The database for above Y report mentioned in above paragraph is created during biannual
exercise, which begins in _______ and _______ of each year and results released in the April and
September/October publication. Fill the gaps

(a) February and August


(b) May and October
(c) June and December
(d) January and June
(e) April and November

Answer - (d)

Q.13)
Which of the following are three critical missions of Y organisation that releases X report
mentioned in the above paragraph?

1) Furthering international monetary cooperation


2) Encouraging the expansion of trade and economic growth
3) Discouraging policies that would harm prosperity
4) Reducing poverty in developing countries
5) Promoting and facilitating the liberalization of trade

(a) 3, 4 and 5
(b) 2, 3 and 5
(c) 1, 4 and 5
(d) 2, 3 and 4
(e) 1, 2 and 3

pg. 6
[Type here]

Solution – (e)

Explanation –

Question number 10 to question number 13 are based on World Economic Outlook released by
IMF. In RBI Grade B Exam, at least two or three questions are asked from the major reports and
indices in the ESI paper.

Generally, the examiner asks questions from recent indices and reports which appeared in the
news before the exam. In this exam also, the examiner asked questions from a significant report
i.e. World Economic Outlook.

Talking about the questions –

• Question number 10 was easy to attempt because the candidate had to only identify the
report and for that various hints were given. Here, I would like to draw your attention
towards a fact – “Only those candidates could identify the report who studied the report
comprehensively”. So, I would suggest you to go through the reports in detail as provided
by me in Spotlight magazine and in PIB.
• Question number 11 was very difficult and was an in-depth question. Most of the
candidates faced difficulty in attempting this question.
• Question number 12 was doable for such candidates who studied the basic facts about the
report. The fact asked in the question is a well-known fact and hence, the question is
justified.
• In Question number 13, the examiner went out of the track. Instead of asking question from
the report, he framed this question from the organization. So, for the upcoming aspirants,
before the examiner plays with you, you should be ready to play with him. You need to be
prepared for any question that an examiner might throw at you.

I.14-16) These specific tribal groups in India have been facing extreme social, economic, and
geographic isolation and are highly vulnerable to exploitation and marginalization compared to
regular Scheduled Tribe. They usually reside in remote and geographically difficult areas, which
often contribute to their isolation from mainstream society.

These tribal groups typically have a small population, distinct cultural practices, languages, and
traditions that need protection and preservation. This makes them more susceptible to external
pressures and cultural dilution. They also face significant poverty, illiteracy, and limited access to
basic services such as healthcare and education.

This tribal group has been created by Government of India with the purpose of better improving
the living standards of endangered tribal groups based on priority. Moreover, Government of India
is giving special focus on providing targeted welfare measures and developmental initiatives to
uplift these tribal groups.

pg. 7
[Type here]

Q.14)
The developmental work for these tribal groups mentioned in above paragraph are being taken
under which of the following priorities of Saptarishi (7 Priorities) of Union Budget 2023-24?

(a) Unleashing the Potential


(b) Youth Power
(c) Green Growth
(d) Infrastructure and Investment
(e) Reaching the Last Mile

Answer - (e)

Q.15)
Which of the following state does not have the presence of these tribal groups mentioned in the
above paragraph?

(a) Gujarat
(b) Assam
(c) Maharashtra
(d) Tamil Nadu
(e) Kerala

Answer – (b)

Q.16)
Which of the following criteria are being used by Government of India to identify these tribal
groups mentioned in above paragraph?

1) Pre-agricultural level of technology


2) Low level of literacy
3) Economic Backwardness
4) Declining or Stagnant Population
5) Distinct cultural practices

(a) 1, 3, 4 and 5
(b) 2, 3 and 5
(c) 2, 3, 4 and 5
(d) 1, 2, 3 and 4
(e) 1, 2, 3, 4 and 5

Answer – (d)

pg. 8
[Type here]

Explanation –

Question numbers 14 to 16 were the most difficult questions in 2023 ESI paper. Let me explain
why. The passage was given in such a way that it became very difficult for the candidates to
identify the special category of tribes mentioned in the passage. The Tribes mentioned in the
passage are Particularly Vulnerable Tribal Groups. Now, questions from PVTGs were asked
because, in Union Budget 2023-24, the government of India announced PM PVTG Development
Mission with an outlay of Rs.15000 Crores. Hence, the questions from this topic were justified.

Now talking about the questions –

• Question number 14 was an analytical question. Students generally learn the facts and do
not try to apply those facts. Here, in this question, the examiner expected to apply the facts
which the candidates have learnt from the Budget 2023-24
• Question number 15 is not at all justified. There are various states in the country where
PVTGs are found. The examiner should not have gone this deep. However, if you want to
clear this examination, the examiner has directly challenged you to cover the topics
comprehensively. And I know that you can do it and you will do it.
• Question number 16 was easy and doable. The information asked in the question was
covered by Manish sir in his budget 2023-24 session.

I.17-20) This portal for registration of Unorganised Workers across the country was launched on
26th August 2021. It aims to build comprehensive National Database of Unorganised Workers
(NDUW) and migrant workers in the country seeded with Aadhaar.

It serves as a centralized database of all unorganized workers including Construction Workers,


Migrant Workers, Gig and Platform workers, Street Vendors, Domestic Workers, Agriculture
Workers, etc.

The portal is proving to be huge boost towards last mile delivery of the welfare schemes of
unorganised workers. It is also improving the implementation efficiency of the social security
services for the unorganized workers and integration of Social Security Schemes meant for UWs.

Q.17) How many unorganised sector workers have enrolled on this NDUW portal mentioned in the
above paragraph?

(a) 31.45 crore


(b) 28.87 crore
(c) 24.58 crore
(d) 19.45 crore
(e) 16.67 crore

Answer – (b)

pg. 9
[Type here]

Q.18) This NDUW portal mentioned in the above paragraph has been recently integrated with
application developed by Ministry of Electronics and Information Technology and ___________.
Fill the gap

(a) National Payments Corporation of India


(b) National e-Governance Division
(c) Common Services Centres
(d) Unified Mobile Application for New-age Governance
(e) National Knowledge Network

Answer – (b)

Q.19) Which of the following services can be accessed through this NDUW portal mentioned in the
above paragraph?

1) Employment Opportunities
2) Skilling
3) Apprenticeship
4) Pension Scheme
5) Digital Skilling and State Schemes

(a) 3, 4 and 5
(b) 1, 2, 3 and 4
(c) 1, 3, 4 and 5
(d) 2, 3, 4 and 5
(e) 1, 2, 3, 4 and 5

Answer - (e)

Q.20)
Which of the following ministries has recently added new feature in the portal to provide access to
child education and women centric schemes to the migrant workers?

(a) Ministry of women and child development


(b) Ministry of labour and employment
(c) Ministry of social justice and empowerment
(d) Ministry of health and family welfare
(e) Ministry of education

Answer – (b)

pg. 10
[Type here]

Explanation –

Question numbers 17 to 20 were doable. The questions were regarding the e-Shram portal which
was continuously in the news for 1 year before the exam. Also, the portal was discussed in very
much detail in PIB sessions by Manish Sir.
The portal benefits the migrants and hence the questions were justified because this topic is a part
of the social issues portion of ESI paper.

Talking about the questions –


• All the questions were directly asked from a PIB release of April month. This tells us the
importance of PIB for this exam. “8 Marks from a single PIB release”, isn’t it huge? Hence, I
would again suggest you comprehensively cover the PIB sessions being conducted by
Manish Sir.

1 Markers

Q.21)
Which of the following statement are correct with respect to Sovereign Gold Bond Scheme?

1) Investors are paid fixed rate of 2.5% per annum, payable semi-annually on nominal value
2) Issue price is less by Rs.50 per gram for investors subscribing online and pay through digital
mode
3) A certificate of holding will be issued at the redemption after 8 years
4) These bonds are not sold through offices of Scheduled Foreign Banks

(a) 2 and 3 Only


(b) 3 and 4 Only
(c) 1 and 2 Only
(d) 1 and 4 Only
(e) 2 and 4 Only

Answer – (c)

Explanation –

I hope you know that government schemes form an integral part of RBI Grade B exam, specially for
ESI paper. The ESI paper always consists of a significant number of questions from government
schemes.
Now, talking about this scheme. The scheme is directly being implemented by RBI and hence, I
should not tell its importance. On one hand, it is a scheme and on the other, it is being
implemented by RBI.
Questions from this scheme have been asked a lot of time in previous examinations as well and
will be asked in future also.

pg. 11
[Type here]

The scheme is being covered in my government schemes PDF and PIB sessions by Manish sir. So,
you don’t have to worry about the coverage of this scheme.

Q.22)
The Prime Minister’s Scholarship Scheme is being implemented to encourage technical and post-
graduate education Who among the eligible is/are eligible under the scheme?

1) Students who have taken admission in 1st year (except Lateral Entry & Integrated Course) are
only eligible to apply for PMSS
2) Wards of Civilians including Para Military Personnel
3) Widows of Ex Servicemen & Ex Coast Guard personnel.

(a) Only 1 and 3


(b) Only 2 and 3
(c) Only 1 and 2
(d) Only 3
(e) Only 2

Answer – (a)

Explanation –

This question was a surprise for the candidates. This scheme was neither in news nor it is a
landmark scheme of government of India. But, such surprises are expected in all the competitive
exams.
The categories of beneficiaries are –
• Students who have taken admission in 1st year (except Lateral Entry & Integrated Course)
are only eligible to apply for PMSS. Students should have scored 60% and above in
Minimum Educational Qualification i.e. 10+2 / Diploma / Graduation.
• Students studying in 2nd or subsequent years are not eligible (except for integrated courses
- where 1st portion is academic and 2nd portion is integrated as professional course).
• Students who are Dependent Wards / Widows of Ex Servicemen & Ex Coast Guard
personnel.
• Wards of Civilians including Para Military Personnel are NOT ELIGIBLE.

Q.23)
An NGO is trying to open Pradhan Mantri Jan Aushadhi Kendra. Which of the following will be the
benefits available to this NGO under the scheme?

1) It will be given incentive @ 15% of monthly purchase made from PMBI by these PMBJKS subject
to ceiling of Rs 12,000/- per month
2) It will be provided 20% margin on MRP (excluding taxes) of each drug
3) In case it is special beneficiary, it will receive incentive of Rs.2 Lakhs

pg. 12
[Type here]

(a) 1 Only
(b) 1, 2 and 3 Only
(c) 1 and 2 Only
(d) 1 and 3 Only
(e) 2 and 3 Only

Answer – (e)

Explanation –

This question is also based on a government scheme. This was asked because this scheme was in
news few months before the exam.
Statements 2 and 3 are correct. However, statement 1 is incorrect because incentives are given
subject to a ceiling of Rs.15,000 per month.

Q.24)
The Scheme X was launched by the Government of India on 5 April 2016 to promote
entrepreneurship at grassroot level focusing on economic empowerment and job creation. Who
amongst of the following can avail benefits under the scheme?

(1) Mohan who works in construction work wants to open his start-up to give benefits of housing
to rural sector
(2) Sunita and Sarita are sisters they belong to scheduled tribe and they want to avail a loan of
50,00,000
(3) Savitri belongs to marginal community and wants to avail loan for her start-up under the
scheme

(a) 1 and 2 Only


(b) 1 Only
(c) 2 and 3 Only
(d) 1 and 3 Only
(e) 1, 2 and 3 Only

Answer – (c)

Explanation –

This question was completely justified in the exam. It is based on Stand-Up India scheme, which is
one of the landmark schemes of the government of India. All such schemes were discussed in very
much detail in government schemes revision sessions conducted by Manish sir just before the
exam.

pg. 13
[Type here]

Under the scheme, government of India provides bank loans ranging from Rs.10 lakhs to Rs.1 crore
to at least one SC or ST borrower and at least one-woman borrower per branch for setting up of a
greenfield enterprise.

From the above description, it is very clear that Mohan is not eligible under the scheme.

Q.25)
Rajni who is 35 years old has two daughters Rina (12 Years) and Tina (8 years). She lost her
husband recently and is availing benefits under Swadhar Greh Scheme. Which of the following is
true?

1) Rajni can only take the benefits of the scheme till 45 years
2) Rina cannot stay with Rajni as she is above 10 years
3) Rajni can only take the benefits till 3 years

(a) 1 and 2 Only


(b) 2 and 3 Only
(c) 3 Only
(d) 1 and 3 Only
(e) 2 Only

Answer – (c)

Explanation –

This was a deep question, however, a doable one. I am calling it doable because Manish Sir
covered this scheme in the required detail in his revision session conducted just before the exam.

The question was based on Swadhar Greh which is now called as ‘Shakti Sadan’.

Following are the features of Swadhar Greh –


• Unmarried girls of any age and boys up to the age of 12 years would be allowed to stay in
Shakti Sadan with their mothers
• A woman can stay up to a maximum period of 3 years. Staying beyond 3 years can be
granted on need basis by the concerned District Magistrate
• The older women above 55 years of age can be accommodated for maximum period of 5
years after which they will have to be shifted to old age homes or other similar institutions.
• Shakti Sadan also provides Anti Human Trafficking Units for repatriation of victims of
trafficking and of commercial sexual exploitation.
• There is a provision for Home for widows also to provide the widows a safe and secure
place of stay, health services, nutritious food, legal and counselling services.

Considering above mentioned features, only statement 3 is true.

pg. 14
[Type here]

I.26) Harshit has a daughter Neha, who is 8 years old. Harshit is worried about her higher studies
and marriage. He wants to save money for her future but has no idea where he should invest or
save the money. Harshit met his friend Narender at a market where they started discussion about
their own daughter’s future. Narender told Harshit about an account which is meant for girl child.
In this account minimum Rs.250 and Maximum Rs.1.5 lakh can be deposited. The scheme under
which this account is being opened is operational since 2015 by ministry of women and child
development. Harshit was satisfied with Narender’s suggestion and he has opened the account
mentioned by Narender in the name of her daughter Neha.

Q.26)
With respect to the scheme mentioned in the given information, identify the correct statement/s

(1) The scheme is meant for the girl child below 10 years of age.
(2) Premature withdrawal is limited to 50% of the balance that was at the end of the preceding
financial year.
(3) Maximum validity of account is 14 years

(a) Only 1 and 3


(b) Only 1 and 2
(c) Only 2 and 3
(d) Only 2
(e) Only 3

Answer – (b)
Explanation –

The scheme mentioned in the passage is Sukanya Samriddhi Yojana. The scheme was launched in
2015 to ensure a bright future for the girl child by facilitating their education and marriage
expenses. It is being implemented by ministry of finance.
Statement 3 is incorrect because validity of the account is 21 years.

I.27) This is Central Sector scheme with 100% funding from Government of India.
It became operational from 1.12.2018. Under it, an income support is provided to all land holding
farmer families. In this case, farmer family is husband, wife and minor children. Identification of
eligible beneficiary families under this scheme is done by State Government and UT
administration. On basis of this, fund is directly transferred to the bank accounts of the
beneficiaries.

Q.27)
Identify scheme mentioned in the above passage.

(a) PM Vibrant Villages Programme


(b) PM Kisan Maan Dhan Yojana
(c) PM Kisan Samman Nidhi

pg. 15
[Type here]

(d) PM Krishi UDAN scheme


(e) National Scheme of Welfare of Farmers

Answer – (c)
Explanation –

The scheme mentioned in the passage is Pradhan Mantri Kisan Samman Nidhi. Under this scheme,
eligible farmers receive direct income support of Rs. 6,000 per year in three equal installments of
Rs. 2,000 each. It is being implemented by the Ministry of Agriculture & Farmers Welfare.

This was simple doable question. I am calling it doable because Manish Sir had covered this
scheme more than 100 times in every dimension during regular session and even in revision
session conducted just before the exam.

I.28) X initiative has been launched by Ministry of Civil Aviation in December 2022 for a biometric
boarding system using Facial biometric verification Technology. It aims at providing a seamless and
hassle-free experience for passengers at airports. Its main objective is to enhance the passenger
experience by eliminating the need for manual verification of tickets and ID at multiple touch
points and to achieve better throughput through existing infrastructure using a Digital Framework.

Q.28)
Identify X initiative mentioned in the above paragraph?

(a) Digi Suvidha


(b) Digi Travel
(c) Digi Checkin
(d) Digi Flight
(e) Digi Yatra

Answer – (e)
Explanation –

The X initiative mentioned in the above passage is Digi Yatra. It was initially launched at three
airports, New Delhi, Bengaluru, and Varanasi, followed by Vijayawada, Kolkata, Hyderabad, and
Pune in April 2023. This was a simple and doable question.

Q.29)
Eklavya Model Residential School (EMRS) is a government initiative in India aimed at providing
quality education to tribal children. These schools are fully funded by the Ministry of Tribal Affairs.
EMRS focuses on the holistic development of students by providing them with a conducive
learning environment, modern infrastructure, and trained teachers. Government has planned to
build EMRS in every block with 50% or more ST population and at least 20,000 tribal persons. How
much capital cost for construction of EMRS is provided by Government in case of Northeast, Hilly
and LWE affected areas?

pg. 16
[Type here]

(a) Rs. 50 crores


(b) Rs. 48 crores
(c) Rs. 37 crores
(d) Rs. 25 crores
(e) Rs. 17 crores

Answer – (b)
Explanation –

Tribal Affairs Ministry in coordination with National Education Society for Tribal Students (NESTS),
an autonomous organization to manage and implement the scheme of EMRS. The approved
capital cost for construction of EMRS is Rs.37.80 crores for plain areas which can go upto Rs. 48.00
crores in case of Northeast, Hilly and LWE affected areas.

This was little bit tricky question, because this scheme was in news in multiple times, including
Union Budget 2023-24. But the examiner in this question didn’t focus on basic aspect of the
scheme, but went in-depth, asking cost of construction of EMRS.

This is the level of depth you must study while covering any important welfare scheme.
But you don’t have to worry because, Manish Sir had covered this scheme in depth during regular
PIB 247 session and again during revision session conducted just before the exam.

Q.30)
Manoj is 38 years old. He has six members in his family. He is involved in construction work. He got
to know about Pradhan Mantri Jan Arogya Yojana which is the world’s largest health insurance
scheme. He also found that this scheme aims at providing a health cover of Rs. 5 lakhs per family
per year for secondary and tertiary care hospitalization to poor and vulnerable families. Finding
out importance of this scheme, he enrolled himself and his family under this scheme. Which of the
following statements are correct about benefits of Scheme to Manoj?

1) Only five members of his family can avail the benefits


2) Only his dependent parents can avail the benefits
3) He cannot avail the benefits as he has crossed the age limit

(a) Only 1 and 2


(b) Only 2 and 3
(c) Only 1 and 3
(d) None is correct
(e) All are correct

Answer – (d)
Explanation –

pg. 17
[Type here]

PM Jan Arogya Yojana is the world’s largest health insurance scheme fully financed by the
Government of India. It provides insurance cover of Rs.5 lakh for secondary and tertiary health
care in empanelled public and private hospitals. It covers 3 days pre hospitalisation and 15 days
post hospitalisation expenses are covered.

It should be noted that there is no restriction on the family size, age or gender to avail benefits of
this scheme. So, all above three above statements are incorrect. Moreover, all pre – existing
diseases are covered under this scheme.

This was easy, doable question because Manish Sir covered this scheme in the required detail in
his revision session conducted just before the exam.

Finance and Management


2 Markers

I.1-5) Read the passage below and answer the following questions.

Motivation is a force that pushes people to work with a high level of commitment and focus, even if
things are going against them. Motivation translates into a certain kind of human behaviour. In
short, motivation is the driving force behind human actions. There are many different forces that
guide and direct our motivations. Abraham Maslow postulated that a person will be motivated when
all his needs are fulfilled. McClelland affirms that we all have ____ motivating drivers.

Leadership style is the reflection of the leader’s behavioural pattern that enables them to manage
their employees by directing and motivating them towards a common vision. As per Kurt Lewin,
there are three major leadership styles, however, these theories were further broadened. Power.
French and Raven (1959) gave five power dynamics (or bases of power). The contingency theory of
leadership states that effective leadership is contingent upon the situation at hand. It depends on
whether an individual's leadership style befits the situation.

Q.1)
Which of the following is the highest level need of Maslow's Hierarchy Theory of Motivation?

(a) Self Actualisation Needs


(b) Esteem Needs
(c) Social Needs
(d) Safety Needs
(e) Physiological Needs

Answer – (a)
Explanation –
pg. 18
[Type here]

This question is from one of the most important chapters of Management called “MOTIVATION”.
The RBI Grade B exam is incomplete without the chapter Motivation. NEVER EVER skip this topic ☺
Let us talk about the question now, the question is from one of the theories called ‘Maslow Hierarchy
of Needs- Theory of Motivation', in order to attempt such questions first, one should know the
correct sequence or hierarchy of needs. Seconds, what do these needs actually mean? If someone
knows these two things then it will be a piece of cake to answer this question.
The level of the question was easy.

Q.2)
McClelland’s Model of Motivation proposed how many needs?

(a) 1
(b) 2
(c) 3
(d) 4
(e) 5

Answer – (c)
Explanation –
This question is from one of the most important chapters of Management called “MOTIVATION”.
The RBI Grade B exam is incomplete without the chapter Motivation. NEVER EVER skip this topic ☺
Let us talk about the question now, the question is from one of the theories called ‘McClelland
Theory of Motivation’. The question is pretty basic in nature, the question wants you to answer how
many needs he proposed and here are the three needs:
Need for achievement, Need for power and Need for Affiliation.
The level of the question was easy.

Q.3)
Douglas McGregor developed a theory of motivation on the basis of a hypothesis relating to human
behaviour. On one hand, managers assume employees are lazy and have an inherent dislike of work.
On the other hand, work is as natural as play if the conditions are favourable and the average person
does not inherently dislike work.
Which of the theory is being talked about above?

(a) Vroom’s Expectancy Theory


(b) Theory Z
(c) ERG Theory
(d) Theory X and Theory Y
(e) Porter and Lawler Expectancy Model

Answer – (d)
Explanation –
This question is from one of the most important chapters of Management called “MOTIVATION”.
The RBI Grade B exam is incomplete without the chapter Motivation. NEVER EVER skip this topic ☺

pg. 19
[Type here]

Let us talk about the question now, the question is from one of the theories called ‘Theory X and
Theory Y’. The question is pretty basic in nature.

Q.4)
Which of the following are three leadership styles given by Kurt Lewin?

(a) Authoritarian, Influencing, Power


(b) Autocratic, Democratic, Laissez Faire Leader
(c) Authorization, directive, participatory
(d) Supportive, Aggressive, Authorization
(e) None of the Above

Answer – (b)
Explanation –
This question is from one of the most important chapters of Management called “LEADERSHIP”. The
RBI Grade B exam is incomplete without the chapter Leadership. NEVER EVER skip this topic ☺
Let us talk about the question now, pretty basic in nature as the question is asking you to figure out
3 basic styles of leadership. This topic comes under the most basic topics. So one thing which we can
learn here is that DO NOT LEAVE ANYTHING FOR THE EXAMS as even the most basic topics of which
we do not have any expectation might come up in the exam.

Q.5)
Which of the following is/are part of contingency theory?

(a) Fiedler Contingency Theory


(b) House’s Path Goal Theory
(c) Managerial Grid of Blake and Mouton
(d) a) and b)
(e) a) and c)

Answer – (d)
Explanation –
This question is from one of the most important chapters of Management called “LEADERSHIP”. The
RBI Grade B exam is incomplete without the chapter Leadership. NEVER EVER skip this topic ☺
Let us talk about the question now, pretty basic in nature as the question wants you to identify
which theories fall under contingency theories of leadership.

I. (6 – 10) Read the following passage and answer the following questions.

Rohan is an employee who has changed many jobs due to his poor vocabulary and he is unable to
connect with clients on calls. Barriers to effective communication can result in confusion which can
lead to incorrect information being conveyed or miscommunication which can lead to loss of
business. Various issues faced by the organisation are (i) structure, rules and regulations present in
the organisation. (ii) lack of trust of superior in subordinate (iii) attention issue (iv) lack of skills in

pg. 20
[Type here]

the translator where he does not know both languages. structure, rules and regulations present in
the organisation. Various types of communication barriers are 1. Semantic barriers 2. Psychological
barriers 3. Organisational barriers 4. Personal barriers

Q.6)
Which of the following is/are personal barriers?

(a) only I)
(b) only ii)
(c) only i) and iii)
(d) only ii) and iii)
(e) only iii)

Answer – (b)

Q.7)
Which of the following barrier is faced by Rohan?

(a) Semantic
(b) Physiological
(c) Organisational
(d) Personal
(e) Physical

Answer – (a)

Q.8)
Which of the following barriers are concerned with problems and obstructions in the process of
encoding and decoding the message into words or impressions?

(a) Semantic Barriers


(b) Status Barriers
(c) Organisational Barriers
(d) Perceptual Barriers
(e) Personal Barriers

Answer – (a)

Q.9)
Which of the following is an example of Non-verbal communication?

(a) Memos and documents


(b) Emails
(c) Body Language and Gestures

pg. 21
[Type here]

(d) Posters and Advertisements


(e) Graphics, Infographics, and Arts

Answer – (c)

Q.10)
Which of the following best represents an example of an organizational barrier in a workplace?

(a) A lack of proper communication channels between different departments.


(b) An individual's personal fear of public speaking.
(c) A disagreement between coworkers over a project's direction.
(d) A delay in a project due to unforeseen external factors
(e) None of the Above

Answer – (a)
Explanation –

This paragraph was from one of the most important topics called ‘COMMUNICATION’, in this
paragraph the examiner covered all the important topics under this particular chapter be it Verbal
and Non-Verbal Communication, Barriers of Communication etc.
Just like the chapters Motivation and Leadership, this one is also one of the most important chapters
of exams and please NEVER EVER leave this topic if you want to score well or make it to the cut-off
list.

I.11-15) Read the following paragraph below and answer the questions.
The government initiated scheme X in August 2014 to provide universal banking services for every
unbanked household, based on the guiding principles of banking the unbanked, securing the
unsecured, funding the unfunded and serving un-served and under-served areas. PMJJBY is a
government-backed Life insurance scheme in India. In order to avail of the benefits offered by the
Pradhan Mantri Jeevan Jyoti Bima Yojana policy, it is mandatory to link your Aadhaar Card to the
participatory bank account.

Q.11)
Based on the above passage, identify the Scheme that has been launched on 15th August, 2014:

(a) Pradhan Mantri Jan Arogya Yojana (PMJAY)


(b) Pradhan Mantri Jan Dhan Yojana (PMJDY)
(c) Pradhan Mantri Jan Bhagidari Yojana
(d) Pradhan Mantri Jan Man Yojana
(e) Pradhan Mantri Jan Manthan Yojana

Answer – (b)

pg. 22
[Type here]

Q.12)
What is the life insurance limit under PMJJBY?
(a) Rs. 1,00,000
(b) Rs. 4,00,000
(c) Rs. 3,00,000
(d) Rs. 2,00,000
(e) Rs. 2,50,000

Answer – (d)

Q.13)
What is the age limit in the PMJJBY?
(a) 40 years
(b) 60 years
(c) 50 years
(d) 55 years
(e) 65 years

Answer – (c)

Q.14)
What is the maximum amount of pension that is received under APY?
(a) Rs. 1,000
(b) Rs. 2,000
(c) Rs. 3,000
(d) Rs. 5,000
(e) Rs. 4,000

Answer – (d)

Q.15)
What is the amount of premium that is paid in the PMSBY?
(a) 40
(b) 50
(c) 20
(d) 35
(e) 45
Answer- (c)
Explanation –

This question is from the topic of “Financial Inclusion”. It is directly related to RBI’s syllabus and thus
becomes important for the exam. This is a scheme-based question. Schemes form part of Current
Affairs. Regularly updating yourself will help you solve such questions. You should watch PIB sessions

pg. 23
[Type here]

and schemes sessions regularly. These are available on youtube. PMJJBY offers a renewable one
year term life cover of ₹ 2.00 Lakh to all the subscribing bank account holders in the age group of
18-50 years, covering death due to any reason, for a premium of ₹ 330/- per annum per subscriber,
to be auto debited from the subscriber's bank account. The Scheme is available to people in the age
group 18 to 70 years with a bank account who give their consent to join / enable auto-debit on or
before 31st May for the coverage period 1st June to 31st May on an annual renewal basis. Aadhar
would be the primary KYC for the bank account. Pradhan Mantri Jan Dhan Yojana is a financial
inclusion program of the Government of India open to Indian citizens, that aims to expand affordable
access to financial services such as bank accounts, remittances, credit, insurance and pensions.

16-20) Read the following paragraph below and answer the questions.

These guidelines are applicable on Private sector banks including Small Finance Banks (SFBs),
Payments Banks (PBs), wholly-owned subsidiaries of foreign banks and foreign banks operating in
India under branch mode. The Chair of the board shall be a/an ________ . (a) Audit Committee of
the Board (ACB)-The ACB shall be constituted with only _____ . The Chair of the board shall not be
a member of the ACB. The ACB shall meet with a quorum of three members. Nomination and
Remuneration Committee (NRC)-The board shall constitute an NRC made up of only NEDs. The NRC
shall meet with a quorum of three members. Risk Management Committee of the Board (RMCB).The
board shall constitute an RMCB with a majority of NEDs. At least half of the members attending the
meeting of the RMCB shall be independent directors of which at least one member shall have
professional expertise/ qualification in ______.

Q.16)
Who shall be the chair of the board?
(a) Non-Executive Director
(b) Executive Director
(c) Independent Director
(d) Promotor Director
(e) Small Shareholder Director
Answer – (c)

Q.17)
What is the quorum of the meeting of the board?
(a) one half members
(b) one third members
(c) two third members
(d) two fifth members
(e) one tenth members

Answer – (b)

pg. 24
[Type here]

Q.18)
Which of the following members are comprised in the Nomination committee?
(a) Independent Director
(b) Non Executive Director
(c) Executive Director
(d) Shareholder Director
(e) Promoter Director
Answer – (b)

Q.19)
At least half of the members attending the meeting of the RMCB shall be independent directors of
which at least one member shall have professional expertise/ qualification in ____.
(a) Risk Management
(b) Corporate Management
(c) Credit Management
(d) Financial Management
(e) Audit

Answer – (a)

Q.20)
Which of the following is the guidelines of RBI is the passage talking about?

(a) Risk Management


(b) Corporate Management
(c) Corporate Governance
(d) Outsourcing of IT services
(e) Audit

Answer- (c)

Explanation –

These 5 questions are from the chapter of ‘Corporate Governance in Banking’. The questions are
asked directly from RBI’s guidelines on corporate governance in Banking. Corporate Governance in
banking is an important topic given in the syllabus. It could be done through our course or by
personal research aswell.
Chair and meetings of the Board. The Chair of the board shall be an independent director. In the
absence of the Chair of the board, the meetings of the board shall be chaired by an independent
director. The quorum for the board meetings shall be one-third of the total strength of the board or
three directors, whichever is higher. At least half of the directors attending the meetings of the
board shall be independent directors.

pg. 25
[Type here]

1 Markers

Q.21)
Organized, self-disciplined and achievement-oriented are the characteristics of which of the traits
of the Big Five Traits Model?

(a) Conscientiousness
(b) Emotional Stability
(c) Agreeableness
(d) Extroversion
(e) Openness to experience

Answer – (a)
Explanation –

This question is a part of one of the important chapters of management and i.e., “PERSONALITY’.
This question is from an important topic called ‘Big Five Personality Traits’. This topic is specifically
mentioned in the syllabus of RBI Grade B so it would be my recommendation to not leave this topic.
It is doable.
Let us talk about the question now. In order to attempt this, question a student should know first,
what different traits of personality we have (all of which I have mentioned in the options) and in
addition to those key words related to every trade. For example, here key words given are
“organized, self-disciplined and achievement-oriented”, If a student is not familiar with the
keywords one will not be able to attend to such questions.
The level of the question was easy.

Q.22)
Fill in the blank with an appropriate option below.
A conflict between emotional requirements and real emotions is known as ____________.

(a) Emotional Conflict


(b) Emotional Dissonance
(c) Emotional Management
(d) Perception Defence
(e) Cognitive Dissonance

Answer – (b)
Explanation –
This question is from the chapter “EMOTIONAL INTELLIGENCE”. With the growing importance of soft
skills, the importance of Emotional Quotient is also increasing. This is the objective of RBI for adding
this topic to your syllabus.
This is a direct question as the examiner wants to find the correct keyword which is Emotional
Dissonance in this case. Students might get confused here with the options in the exam but the
correct answer is option b)
pg. 26
[Type here]

Emotional dissonance refers to the internal conflict or incongruity between the emotions a person
feels and the emotions they express outwardly. It occurs when individuals are required to display
emotions that are inconsistent with their true feelings or emotions.
The level of the question was easy.

Q.23)
Which of the following theories holds that the ends or consequences of an act determine whether
the act is good or bad?

(a) Ethics of Rights Theory


(b) Ethics of Justice Theory
(c) Virtue Ethics Theory
(d) Deontological Theory
(e) Teleological Theory

Answer – (e)
Explanation –
This question is from one of the most important chapters called ‘ETHICS’. Questions from this topic
do come both in objective and descriptive format every year so it would be advice to all my students
to not leave this topic NEVER EVER.
Let us talk about the question now, the question is from “Theories of Ethics” which is a quite an
important topic in this chapter. The question is pretty basic in nature and it’s a no-brainer for those
students who studied for exams dedicatedly.
The level of the question was easy.

Q.24)
What kind of principle focuses on the kind of value displaces by others?

(a) Caring
(b) Right
(c) Justice
(d) Distribution
(e) Love

Answer – (a)
Explanation –
This question is from one of the most important chapters called ‘ETHICS’. Questions from this topic
do come both in objective and descriptive format every year so it would be advice to all my students
to not leave this topic NEVER EVER.
Let us talk about the question now. The question is from the topic ‘Values’. The question is pretty
basic in nature. A simple understanding of values will be sufficient to answer the question.

pg. 27
[Type here]

Q.25)
Calculate the Return on Assets (ROA), if Income – is Rs 100 Lakhs and Asset size = is Rs 500 lakhs.

(a) 10
(b) 50
(c) 20
(d) 25
(e) 15
Answer - (c)
Explanation-
Accountancy/ Accounting Ratios was a new topic introduced in the syllabus. This question is from
the chapter of ‘Accounting ratios’. A very easy question…. To solve such questions, you should learn
the formulas, practice questions and ask doubts if you have any.

Q.26)
The NaBFID Act, 2021 has been set up as a Development Financial Institution (DFI) to support the
development of which part of the economy?

(a) Finance
(b) Infrastructure
(c) Banking
(d) Roadways
(e) Export
Answer - (b)
Explanation-
A very easy question about a Development Finance Institute. It was taught to students through
RBI247 session plenty of times. This question can seem to be of current affairs, however it is a new
development Finance Institute, and these are something an RBI aspirant should know. A pretty basic
question.

Q.27)
What is the maximum limit that a resident individual can freely remit per financial year under the
Liberalised Remittance Scheme?

(a) USD 250000


(b) USD 50000
(c) Rs 250000
(d) Rs 50000
(e) Rs 25000
Answer - (a)
Explanation-

pg. 28
[Type here]

Liberalized Remittance scheme is very important. It was in the news for the last 1 year for different
reasons. We were expecting a difficult question from this scheme, however, this is the easiest
possible question that could have been made from this scheme.

Q.28)
Which of the following is an example of Alternative Sources of Finance (ASF)?
(a) Forfaiting
(b) Bank Loan
(c) Credit
(d) Government Security
(e) Corporate Bonds
Answer - (a)
Explanation -
This is again a straightforward question. The concept is taught in a static section. One of the easiest
questions that could have been made from this chapter is ‘Alternate source of finance’. The options
were also not close enough. Easily identifiable answer.

Q.29)
Which of the following is a type of Non-Tax Revenue?
(a) Income Tax
(b) Customs duties
(c) GST
(d) Interest
(e) Excise duties
Answer - (d)
Explanation-
Again, an easy question. Probably one of the easiest question that could have been made from this
topic. These kinds of questions give you free marks. It is asked from ESI- Chapter of BUDGETING.

Q.30)
Mr A has taken a loan of Rs 25,000 from Mr B, which is payable after 3 months. Mr. B has to give Rs.
25,000 to Mr. X. So, instead of repaying to him, Mr. B has asked Mr. A to give those money directly
to Mr. X. Which of the following negotiable instrument will they form?
(a) Certificate of Deposits
(b) Commercial Papers
(c) Bills of Exchange
(d) Promissory Notes
(e) Cheques
Answer - (c)
Explanation-

pg. 29
[Type here]

This question is covered both in the Accountancy section as well as finance- instruments. Negotiable
and exchangeable instruments is an easy topic and is an important topic. It was a very easy question.
It could have been solved with a basic understanding of the topic.
A bill of exchange is an instrument in writing containing an unconditional order, signed by the maker,
directing a certain person to pay on demand or at fixed or determinable future time a certain sum
of money only to, or to the order of, a certain person or to the bearer of the instrument

pg. 30
[Type here]

pg. 31
[Type here]

pg. 32
[Type here]

pg. 33
[Type here]

pg. 34
[Type here]

pg. 35
TITLE OF THE DOCUMENT/TOPIC
(IN CAPS)
[Type here]

Q.1)
Write an essay on any one of the following topics in 300 Words (40 Marks)

• Advantages of UPI over money transfer


• Financial inclusion: A requirement for all ages in India
• Crowd funding: The new phenomenon in financing
• Positive and Negative effects of demonetization on Indian economy
• Measures for promoting microfinance in India

Q.2)
Write a precis from the given paragraph in about 180 words. (30 Marks)

India’s agricultural economy is facing a crisis due to the global economy. This is part of the
economic cycle. There is a surplus of sugar, rice, wheat, fruits and vegetables, onions and pulses
in the global market because of which prices have come down. In such a situation, we have to
provide relief to agriculture. We will divert part of the agricultural workforce to the energy and
power sector, which has imports of over Rs 8 lakh crore. We will make ethanol and bio-CNG, and
develop ethanol economy. At the same time, during this crisis we stand with our farmers in
support. Marginal farmers who have up to five acres of land will get Rs 6,000 per year. This
addition to farmland income will be a great relief to the poor.

A key feature of the budget is that it has prioritised infrastructure. How to accelerate development
in these areas and strengthen the fundamental structure of our economy are key questions. The
work that our government has done in the transport sector, road sector, railways, airports, river
ports and inland waterways is changing the picture of the country.

The decision to give Rs 3,000 as pension to workers in the unorganised sector and provide
income-tax relief to the middle-class including service professionals of this country is a
revolutionary decision by the government. The total beneficiaries of this budget will be in excess of
40-50 crore. This is the first time that the government is giving relief to such a large proportion of
the population.

Our revenues have also increased. In January, our GST receipts crossed Rs 1,03,000 crore and
the number of income-tax returns filed has doubled to over 6.85 crore. Since revenue receipts are
also growing, I don’t think there will be a problem with rising fiscal deficit.

Some problems do arise with the business cycle, but these issues can be overcome. In the road
sector, there are issues in obtaining financial closures, but we are meeting with stakeholders and
resolving them. This is why projects are being awarded and work is beginning. To overcome
obstacles and keep moving forward is the key to leadership. Under Bharatmala, we have
completed almost 6,000km of roads. By March, we will be in a position to begin work on projects
worth Rs 3.5-4 lakh crore across roads, shipping and waterways.

When I became minister, 403 road projects worth Rs 3.85 lakh crore were stuck. We have
managed to solve 95% of this problem. By March we are trying to achieve a rate of road
construction of 40km per day. I had set the target for five years of our government – we hope to
achieve it.

I had the good fortune of running the Pradhan Mantri Gram Sadak Yojana when Atalji was the
prime minister. I believe that when every village is connected by roads, then there will be true
development in our villages. Our government has been successful in achieving this, and this has
pg. 1
[Type here]

yielded great results.

To provide irrigation to farmers, we had planned to complete 96 projects, out of which 75 will be
completed in the coming two months. I believe we will be able to bring 1.88 crore hectares of land
under irrigation next year. The country is changing. In Pune, the municipal corporation recently
purchased 150 electric buses. The running cost of electric buses in Pune is Rs 40 per kilometre
while BEST (in Mumbai) costs Rs 115 per km. I believe that in future everyone will understand that
electric transport can be cheaper as well. We have completed an agreement with Transport for
London. In London, there are 17 operators, with both double- and single-decker buses. The
system is operated privately.

Q.3) Read the following comprehension and answer the questions that follow (5X6 = 30 Marks)

Two weeks after Japan's trade minister gave the all-clear to restart nuclear power plants that had
been shut for maintenance, Naoto Kan, the prime minister, ordered on July 6th that they should
first undergo rigorous stress tests. The inverted sequence showed that only a cursory examination
had taken place. Hideo Kishimoto, a mayor in Southwestern Japan who had earlier given his local
power company permission to restart the Genkai nuclear power plant, retracted his approval, “I
can't trust the government," he said.

It is a refrain heard throughout Japan, aimed not only at national politicians but also at the power
companies, bureaucrats, academics and the media who had given assurances that the country’s
nuclear plants were disaster- proof. A country that has long been governed by Informal bonds of
trust is seeing them start to fray. The meltdown at the Fukushima Dai-ichi power plant is forcing a
re-examination of Japan’s most influential institutions.

The credibility gap bedevils the utilities. Tokyo Electric (TEPC0), the utility that serves the capital
and runs the Fukushima plant, has been accused of withholding data from the start, Including from
the prime minister. And the energy firms have a record of spotty safety standards and cover-ups
stretching back years. Yet their Image worsened in recent days when it transpired that Kyushu
Electric, which operates Gankal, asked thousands of employees to pose as ordinary citizens and
send E-mails and faxes in support of reopening reactors at a public meeting in June that was
televised live. The attempt to manipulate public sentiment, exposed by a rare whistle-blower,
angered the public and energised the media.

Japan's food supply is safe. But pockets of doubt have crept in, owing to a mishandling of safety
inspections. On July 13th the Tokyo Metropolitan Government said that beef contaminated with
radioactive caesium more than six times above the safety limit was sold and possibly consumed. It
followed Initial reports that the meat never made it to the market. Though the quantity was small
(only a few cows, it appears so far) and the health risk said to be non-existent, it raises suspicions.
When radiation above European safety limits was found in tea from Shizuoka in June, a
prefectural official asked the retailer, Radishbo-ya, to keep quiet so as not to harm local growers.

The crisis of confidence in Japanese authority is still at the seedling stage . Yet lately, there have
been nightly televised exposes of the gulf between official reassurances and the worrying reality.
Even the belated stress tests raise eyebrows: they will be undertaken by the utilities themselves
and checked by two regulatory agencies that previously failed to supervise the utilities properly. On
July 13th Mr. Kan said he wants Japan to reduce nuclear power on safety grounds if not to get rid
of it altogether. A few months ago, such a policy, was unthinkable, because of the risk of power
shortages. The public increasingly supports it— but not Mr. Kan.

pg. 2
[Type here]

(a) What can be inferred from the last sentence of the passage?
(b) What is the overall inference that can be drawn from the passage?
(c) What does the inverted sequence in the passage refer to?
(d) In the recent crisis in Japan, what were the evident actions?
(e) “It is a refrain heard throughout Japan”. What does this mean?

pg. 3
[Type here]

pg. 4
[Type here]

pg. 5
[Type here]

pg. 6
[Type here]

pg. 7
TITLE OF THE DOCUMENT/TOPIC
(IN CAPS)
[Type here]

Q.1)
Write an essay on any one of the following topics in 300 Words (40 Marks)

• Advantages of UPI over money transfer


• Financial inclusion: A requirement for all ages in India
• Crowd funding: The new phenomenon in financing
• Positive and Negative effects of demonetization on Indian economy
• Measures for promoting microfinance in India

Q.2)
Write a precis from the given paragraph in about 180 words. (30 Marks)

India’s agricultural economy is facing a crisis due to the global economy. This is part of the
economic cycle. There is a surplus of sugar, rice, wheat, fruits and vegetables, onions and pulses
in the global market because of which prices have come down. In such a situation, we have to
provide relief to agriculture. We will divert part of the agricultural workforce to the energy and
power sector, which has imports of over Rs 8 lakh crore. We will make ethanol and bio-CNG, and
develop ethanol economy. At the same time, during this crisis we stand with our farmers in
support. Marginal farmers who have up to five acres of land will get Rs 6,000 per year. This
addition to farmland income will be a great relief to the poor.

A key feature of the budget is that it has prioritised infrastructure. How to accelerate development
in these areas and strengthen the fundamental structure of our economy are key questions. The
work that our government has done in the transport sector, road sector, railways, airports, river
ports and inland waterways is changing the picture of the country.

The decision to give Rs 3,000 as pension to workers in the unorganised sector and provide
income-tax relief to the middle-class including service professionals of this country is a
revolutionary decision by the government. The total beneficiaries of this budget will be in excess of
40-50 crore. This is the first time that the government is giving relief to such a large proportion of
the population.

Our revenues have also increased. In January, our GST receipts crossed Rs 1,03,000 crore and
the number of income-tax returns filed has doubled to over 6.85 crore. Since revenue receipts are
also growing, I don’t think there will be a problem with rising fiscal deficit.

Some problems do arise with the business cycle, but these issues can be overcome. In the road
sector, there are issues in obtaining financial closures, but we are meeting with stakeholders and
resolving them. This is why projects are being awarded and work is beginning. To overcome
obstacles and keep moving forward is the key to leadership. Under Bharatmala, we have
completed almost 6,000km of roads. By March, we will be in a position to begin work on projects
worth Rs 3.5-4 lakh crore across roads, shipping and waterways.

When I became minister, 403 road projects worth Rs 3.85 lakh crore were stuck. We have
managed to solve 95% of this problem. By March we are trying to achieve a rate of road
construction of 40km per day. I had set the target for five years of our government – we hope to
achieve it.

I had the good fortune of running the Pradhan Mantri Gram Sadak Yojana when Atalji was the
prime minister. I believe that when every village is connected by roads, then there will be true
development in our villages. Our government has been successful in achieving this, and this has
pg. 1
[Type here]

yielded great results.

To provide irrigation to farmers, we had planned to complete 96 projects, out of which 75 will be
completed in the coming two months. I believe we will be able to bring 1.88 crore hectares of land
under irrigation next year. The country is changing. In Pune, the municipal corporation recently
purchased 150 electric buses. The running cost of electric buses in Pune is Rs 40 per kilometre
while BEST (in Mumbai) costs Rs 115 per km. I believe that in future everyone will understand that
electric transport can be cheaper as well. We have completed an agreement with Transport for
London. In London, there are 17 operators, with both double- and single-decker buses. The
system is operated privately.

Q.3) Read the following comprehension and answer the questions that follow (5X6 = 30 Marks)

Two weeks after Japan's trade minister gave the all-clear to restart nuclear power plants that had
been shut for maintenance, Naoto Kan, the prime minister, ordered on July 6th that they should
first undergo rigorous stress tests. The inverted sequence showed that only a cursory examination
had taken place. Hideo Kishimoto, a mayor in Southwestern Japan who had earlier given his local
power company permission to restart the Genkai nuclear power plant, retracted his approval, “I
can't trust the government," he said.

It is a refrain heard throughout Japan, aimed not only at national politicians but also at the power
companies, bureaucrats, academics and the media who had given assurances that the country’s
nuclear plants were disaster- proof. A country that has long been governed by Informal bonds of
trust is seeing them start to fray. The meltdown at the Fukushima Dai-ichi power plant is forcing a
re-examination of Japan’s most influential institutions.

The credibility gap bedevils the utilities. Tokyo Electric (TEPC0), the utility that serves the capital
and runs the Fukushima plant, has been accused of withholding data from the start, Including from
the prime minister. And the energy firms have a record of spotty safety standards and cover-ups
stretching back years. Yet their Image worsened in recent days when it transpired that Kyushu
Electric, which operates Gankal, asked thousands of employees to pose as ordinary citizens and
send E-mails and faxes in support of reopening reactors at a public meeting in June that was
televised live. The attempt to manipulate public sentiment, exposed by a rare whistle-blower,
angered the public and energised the media.

Japan's food supply is safe. But pockets of doubt have crept in, owing to a mishandling of safety
inspections. On July 13th the Tokyo Metropolitan Government said that beef contaminated with
radioactive caesium more than six times above the safety limit was sold and possibly consumed. It
followed Initial reports that the meat never made it to the market. Though the quantity was small
(only a few cows, it appears so far) and the health risk said to be non-existent, it raises suspicions.
When radiation above European safety limits was found in tea from Shizuoka in June, a
prefectural official asked the retailer, Radishbo-ya, to keep quiet so as not to harm local growers.

The crisis of confidence in Japanese authority is still at the seedling stage . Yet lately, there have
been nightly televised exposes of the gulf between official reassurances and the worrying reality.
Even the belated stress tests raise eyebrows: they will be undertaken by the utilities themselves
and checked by two regulatory agencies that previously failed to supervise the utilities properly. On
July 13th Mr. Kan said he wants Japan to reduce nuclear power on safety grounds if not to get rid
of it altogether. A few months ago, such a policy, was unthinkable, because of the risk of power
shortages. The public increasingly supports it— but not Mr. Kan.

pg. 2
[Type here]

(a) What can be inferred from the last sentence of the passage?
(b) What is the overall inference that can be drawn from the passage?
(c) What does the inverted sequence in the passage refer to?
(d) In the recent crisis in Japan, what were the evident actions?
(e) “It is a refrain heard throughout Japan”. What does this mean?

pg. 3
[Type here]

pg. 4
[Type here]

pg. 5
[Type here]

pg. 6
[Type here]

pg. 7
Q.1)
Write an essay on any one of the following topics in 300 Words (40 Marks)

● Is water crisis due to ground water depletion a serious issue?


● Role of department of Animal Husbandry, Dairying, and Fisheries
● SBI Banks Merger
● Importance of teamwork in an organization

Q.2)
Write a precis from the given paragraph in about 180 words. (30 Marks)

A global squeeze on emerging market assets has forced the Reserve Bank of India to take
actions that sometime appear at conflict with each other. To help cool rising bond yields, the
central bank said late last week that it will buy bonds for the first time in 18 months, infusing
cash into financial markets. But that counters moves to prop up a sliding currency -- by selling
dollars from reserves and buying up rupees, effectively draining liquidity.

The weaker currency, by driving up import costs, is adding to inflation risks at a time when oil
prices are already climbing. That’s leading policy makers to warn of higher interest rates
sooner rather than later -- a hawkish bias that is acting as an upward pressure on bond yields,
which of course, the RBI is seeking to contain.The RBI isn’t alone in this conundrum. Emerging
markets from Argentina to Turkey have all faced similar tough choices as a stronger dollar and
rising U.S. Treasury yields push investors to dump higher yielding assets. Developments in the
bond and foreign exchange markets have complicated the monetary policy setting, raising
questions about the RBI’s primary objective,” said Priyanka Kishore, lead Asia economist at
Oxford Economics Ltd. in Singapore.

The RBI’s main goal is to keep inflation around the 4 percent midpoint of its target band. While
inflation has eased in recent months, policy makers are wary of rising risks and have started to
flag a possible interest rate hike this year. Kishore said there’s still a fair chance of an increase
in June. The RBI’s juggling act has come into sharp focus with the rupee’s 5.2 percent slump
against the dollar this year, the most in Asia. Foreigners have been selling Indian bonds,
putting downward pressure on the currency.
To counter that, the central bank has tapped foreign reserves to support the rupee. But that
takes liquidity out of the market, and along with rising demand for currency notes among
locals, it’s led to cash in the banking system dwindling. To ease that crunch, the central bank is
now buying bonds to inject liquidity back into the market.

Back in 2016, the infusion of cash through debt purchases was hailed as the biggest shield to
the market. This time around, it has investors questioning the timing of the move when liquidity
is still surplus. It is very difficult to figure out what the monetary policy is targeting,” Jahangir
Aziz, head of emerging market economics at JPMorgan Chase and Co., said in an interview to
BloombergQuint last week. “Is it targeting the inflation rate, is it targeting the 10-year rate or is
it targeting financial stability?”
Part of the dilemma stems from the fact that the RBI acts as the government’s debt manager
and, as a consequence, has to balance competing needs: keep borrowing costs low to ensure
that a Rs 5.6 trillion ($83 billion) debt program proceeds smoothly, while also curbing prices as
an inflation-targeting central bank. It also has to ensure the good health of India’s banking
industry, especially state-run lenders who are the biggest investors in government debt.

Q.3)
Read the following comprehension and answer the questions that follow (5X6 = 30 Marks)

For two thousand years, moralities have rested upon a traditional metaphysical dichotomy, that
between virtue and vice. But what generally is vice? It is a linguistic sign, comprising three basic
semantic structures for each of its three possible embodiments.

The first. Vice is a passion, a natural love for something that a humane-being corporally and
psychologically cannot live without: fame, money, wine, women etc. It is a separate passion that
makes people blind, that hypertrophies itself and that usurps the place of the whole, disrupting
the normal harmony of different inner necessities and interests.

These passions and objects of longings are socially quite normal in themselves. In this case
vice expresses the growing gap between strongly expressed natural inclinations and public
norms. Passions turn into vices, a human-being symbiosis of corporal orders, psychological
habits and states of consciousness.

Consider the cases of a weak resistance to the appeal of alcohol, or peculiarities of the
metabolism, sexual constitution or temperament. Self-interest, hungering for drugs, ambition,
voluptuousness - in Russian, all these words include the notion 'love'. This is the passion of
nature, it is irrational, and we understand that, feeling sorry for its victims, muttering only 'never
make promises you cannot keep'.

The second kind of sign represented is Vice as a defect, a psychological lack, a consequence of
the absence of culture, or of self-consciousness, or lack of work of the self-consciousness on
oneself (laziness, apathy of soul, its dissoluteness). Cowardice, recklessness, anger,
ruthlessness, shamelessness, irresponsibility, prodigality, callousness, lack of will et
cetera. Other people's attitude to these 'defects' is less tolerant than in the first case. They feel
the failure to counteract them on the part of their carriers is also 'irrational' yet here reason
should have had more say.

The third sign. This interprets vice as an unequivocal and chronic departure not only from social
norms, but also from anthropological ones, which are always wider and more tolerant. Thus, we
have vice as perversion, hatred, ill-intention, gloating, malice, criminality, debauchery,
dissipation. Both the enslavement of the man by passion and the enlarging of the crack of
defect into a deep sincere break, becoming an obscenity. The first two senses of the word 'vice'
are within the framework of social norms that is, society tolerates them. Not so the third.
(a) What does the term “Hypertrophies” signify as given in the passage?
(b) What conclusion can be drawn from the information given in the passage?
(c) Why are the first two manifestations of the vice acceptable by society and not the third?
(d) How, according to the passage, ‘Vice’ is a passion?
(e) What generally is vice, according to the passage?
2019
Q.1)
Write an essay on any one of the following topics in 300 Words (40 Marks)

● Is water crisis due to ground water depletion a serious issue?


● Role of department of Animal Husbandry, Dairying, and Fisheries
SBI Banks Merger
Importance of teamwork in an organization

Q.2)
Write a precis from the given paragraph in about 180 words. (30 Marks)

A global squeeze on emerging market assets has forced the Reserve Bank of India to take
actions that sometime appear at conflict with each other. To help cool rising bond yields, the
central bank said late last week that it will buy bonds for the first time in 18 months, infusing
cash into financial markets. But that counters moves to prop up a sliding currency -- by selling
dollars from reserves and buying up rupees, effectively draining liquidity.

The weaker currency, by driving up import costs, is adding to inflation risks at a time when oil
prices are already climbing. That’s leading policy makers to warn of higher interest rates
sooner rather than later -- a hawkish bias that is acting as an upward pressure on bond yields,
which of course, the RBI is seeking to contain.The RBI isn’t alone in this conundrum. Emerging
markets from Argentina to Turkey have all faced similar tough choices as a stronger dollar and
rising U.S. Treasury yields push investors to dump higher yielding assets. Developments in the
bond and foreign exchange markets have complicated the monetary policy setting, raising
questions about the RBI’s primary objective,” said Priyanka Kishore, lead Asia economist at
Oxford Economics Ltd. in Singapore.

The RBI’s main goal is to keep inflation around the 4 percent midpoint of its target band. While
inflation has eased in recent months, policy makers are wary of rising risks and have started to
flag a possible interest rate hike this year. Kishore said there’s still a fair chance of an increase
in June. The RBI’s juggling act has come into sharp focus with the rupee’s 5.2 percent slump
against the dollar this year, the most in Asia. Foreigners have been selling Indian bonds,
putting downward pressure on the currency.
To counter that, the central bank has tapped foreign reserves to support the rupee. But that
takes liquidity out of the market, and along with rising demand for currency notes among
locals, it’s led to cash in the banking system dwindling. To ease that crunch, the central bank is
now buying bonds to inject liquidity back into the market.

Back in 2016, the infusion of cash through debt purchases was hailed as the biggest shield to
the market. This time around, it has investors questioning the timing of the move when liquidity
is still surplus. It is very difficult to figure out what the monetary policy is targeting,” Jahangir
Aziz, head of emerging market economics at JPMorgan Chase and Co., said in an interview to
BloombergQuint last week. “Is it targeting the inflation rate, is it targeting the 10-year rate or is
it targeting financial stability?”
Part of the dilemma stems from the fact that the RBI acts as the government’s debt manager
and, as a consequence, has to balance competing needs: keep borrowing costs low to ensure
that a Rs 5.6 trillion ($83 billion) debt program proceeds smoothly, while also curbing prices as
an inflation-targeting central bank. It also has to ensure the good health of India’s banking
industry, especially state-run lenders who are the biggest investors in government debt.

Q.3)
Read the following comprehension and answer the questions that follow (5X6 = 30 Marks)

For two thousand years, moralities have rested upon a traditional metaphysical dichotomy, that
between virtue and vice. But what generally is vice? It is a linguistic sign, comprising three basic
semantic structures for each of its three possible embodiments.

The first. Vice is a passion, a natural love for something that a humane-being corporally and
psychologically cannot live without: fame, money, wine, women etc. It is a separate passion that
makes people blind, that hypertrophies itself and that usurps the place of the whole, disrupting
the normal harmony of different inner necessities and interests.

These passions and objects of longings are socially quite normal in themselves. In this case
vice expresses the growing gap between strongly expressed natural inclinations and public
norms. Passions turn into vices, a human-being symbiosis of corporal orders, psychological
habits and states of consciousness.

Consider the cases of a weak resistance to the appeal of alcohol, or peculiarities of the
metabolism, sexual constitution or temperament. Self-interest, hungering for drugs, ambition,
voluptuousness - in Russian, all these words include the notion 'love'. This is the passion of
nature, it is irrational, and we understand that, feeling sorry for its victims, muttering only 'never
make promises you cannot keep'.

The second kind of sign represented is Vice as a defect, a psychological lack, a consequence of
the absence of culture, or of self-consciousness, or lack of work of the self-consciousness on
oneself (laziness, apathy of soul, its dissoluteness). Cowardice, recklessness, anger,
ruthlessness, shamelessness, irresponsibility, prodigality, callousness, lack of will et
cetera. Other people's attitude to these 'defects' is less tolerant than in the first case. They feel
the failure to counteract them on the part of their carriers is also 'irrational' yet here reason
should have had more say.

The third sign. This interprets vice as an unequivocal and chronic departure not only from social
norms, but also from anthropological ones, which are always wider and more tolerant. Thus, we
have vice as perversion, hatred, ill-intention, gloating, malice, criminality, debauchery,
dissipation. Both the enslavement of the man by passion and the enlarging of the crack of
defect into a deep sincere break, becoming an obscenity. The first two senses of the word 'vice'
are within the framework of social norms that is, society tolerates them. Not so the third.

(a) What does the term “Hypertrophies” signify as given in the passage?
(b) What conclusion can be drawn from the information given in the passage?
(c) Why are the first two manifestations of the vice acceptable by society and not the third?
(d) How, according to the passage, ‘Vice’ is a passion?
(e) What generally is vice, according to the passage?
2019
Q.1)
Write an essay on any one of the following topics in 300 Words (40 Marks)

● Is water crisis due to ground water depletion a serious issue?


● Role of department of Animal Husbandry, Dairying, and Fisheries
SBI Banks Merger
Importance of teamwork in an organization

Q.2)
Write a precis from the given paragraph in about 180 words. (30 Marks)

A global squeeze on emerging market assets has forced the Reserve Bank of India to take
actions that sometime appear at conflict with each other. To help cool rising bond yields, the
central bank said late last week that it will buy bonds for the first time in 18 months, infusing
cash into financial markets. But that counters moves to prop up a sliding currency -- by selling
dollars from reserves and buying up rupees, effectively draining liquidity.

The weaker currency, by driving up import costs, is adding to inflation risks at a time when oil
prices are already climbing. That’s leading policy makers to warn of higher interest rates
sooner rather than later -- a hawkish bias that is acting as an upward pressure on bond yields,
which of course, the RBI is seeking to contain.The RBI isn’t alone in this conundrum. Emerging
markets from Argentina to Turkey have all faced similar tough choices as a stronger dollar and
rising U.S. Treasury yields push investors to dump higher yielding assets. Developments in the
bond and foreign exchange markets have complicated the monetary policy setting, raising
questions about the RBI’s primary objective,” said Priyanka Kishore, lead Asia economist at
Oxford Economics Ltd. in Singapore.

The RBI’s main goal is to keep inflation around the 4 percent midpoint of its target band. While
inflation has eased in recent months, policy makers are wary of rising risks and have started to
flag a possible interest rate hike this year. Kishore said there’s still a fair chance of an increase
in June. The RBI’s juggling act has come into sharp focus with the rupee’s 5.2 percent slump
against the dollar this year, the most in Asia. Foreigners have been selling Indian bonds,
putting downward pressure on the currency.
To counter that, the central bank has tapped foreign reserves to support the rupee. But that
takes liquidity out of the market, and along with rising demand for currency notes among
locals, it’s led to cash in the banking system dwindling. To ease that crunch, the central bank is
now buying bonds to inject liquidity back into the market.

Back in 2016, the infusion of cash through debt purchases was hailed as the biggest shield to
the market. This time around, it has investors questioning the timing of the move when liquidity
is still surplus. It is very difficult to figure out what the monetary policy is targeting,” Jahangir
Aziz, head of emerging market economics at JPMorgan Chase and Co., said in an interview to
BloombergQuint last week. “Is it targeting the inflation rate, is it targeting the 10-year rate or is
it targeting financial stability?”
Part of the dilemma stems from the fact that the RBI acts as the government’s debt manager
and, as a consequence, has to balance competing needs: keep borrowing costs low to ensure
that a Rs 5.6 trillion ($83 billion) debt program proceeds smoothly, while also curbing prices as
an inflation-targeting central bank. It also has to ensure the good health of India’s banking
industry, especially state-run lenders who are the biggest investors in government debt.

Q.3)
Read the following comprehension and answer the questions that follow (5X6 = 30 Marks)

For two thousand years, moralities have rested upon a traditional metaphysical dichotomy, that
between virtue and vice. But what generally is vice? It is a linguistic sign, comprising three basic
semantic structures for each of its three possible embodiments.

The first. Vice is a passion, a natural love for something that a humane-being corporally and
psychologically cannot live without: fame, money, wine, women etc. It is a separate passion that
makes people blind, that hypertrophies itself and that usurps the place of the whole, disrupting
the normal harmony of different inner necessities and interests.

These passions and objects of longings are socially quite normal in themselves. In this case
vice expresses the growing gap between strongly expressed natural inclinations and public
norms. Passions turn into vices, a human-being symbiosis of corporal orders, psychological
habits and states of consciousness.

Consider the cases of a weak resistance to the appeal of alcohol, or peculiarities of the
metabolism, sexual constitution or temperament. Self-interest, hungering for drugs, ambition,
voluptuousness - in Russian, all these words include the notion 'love'. This is the passion of
nature, it is irrational, and we understand that, feeling sorry for its victims, muttering only 'never
make promises you cannot keep'.

The second kind of sign represented is Vice as a defect, a psychological lack, a consequence of
the absence of culture, or of self-consciousness, or lack of work of the self-consciousness on
oneself (laziness, apathy of soul, its dissoluteness). Cowardice, recklessness, anger,
ruthlessness, shamelessness, irresponsibility, prodigality, callousness, lack of will et
cetera. Other people's attitude to these 'defects' is less tolerant than in the first case. They feel
the failure to counteract them on the part of their carriers is also 'irrational' yet here reason
should have had more say.

The third sign. This interprets vice as an unequivocal and chronic departure not only from social
norms, but also from anthropological ones, which are always wider and more tolerant. Thus, we
have vice as perversion, hatred, ill-intention, gloating, malice, criminality, debauchery,
dissipation. Both the enslavement of the man by passion and the enlarging of the crack of
defect into a deep sincere break, becoming an obscenity. The first two senses of the word 'vice'
are within the framework of social norms that is, society tolerates them. Not so the third.

(a) What does the term “Hypertrophies” signify as given in the passage?
(b) What conclusion can be drawn from the information given in the passage?
(c) Why are the first two manifestations of the vice acceptable by society and not the third?
(d) How, according to the passage, ‘Vice’ is a passion?
(e) What generally is vice, according to the passage?
Q.1)

Where was the G7 summit 2019 held?

(a) Biarritz, France

(b) Amsterdam, Netherlands

(c) Rome, Italy

(d) Madrid, Spain

(e) Dublin, Ireland

Answer – (a)

Explanation –

The summit took place in the August of 2019 and the exam was held in November 2019. You should
not skip the summits because if you do that you have a very big loss. In these types of questions, you
cannot do the guess work because if you are totally unaware about the summit you may not be able
to guess. But if you have heard or read even a little bit about the summit, I guess, you will be able to
guess the answer.

Q.2)

Where is the headquarter of UNESCO?

(a) New York, US

(b) Washington DC, US

(c) Paris, France

(d) Vienna, Austria

(e) Geneva, Switzerland

Answer – (c)

Explanation –

Nothing can be easier than the headquarters’ question, that too, from the UN and its important
organizations. So, don’t take a chance of neglecting the static general awareness portion, which
includes the headquarters, capitals, currencies, name of parliaments, tiger reserves etc.

Q.3)

Manila is the capital of which country?

(a) Philippines
(b) Brunei Darussalem

(c) Laos PDR

(d) Timor Leste

(e) Samoa

Answer – (a)

Explanation –

Again, a question on static general awareness. So, understand the point that there are many
questions on the capitals, currencies, and sanctuaries etc. Static GK also a significant share in the
exam.

Q.4)

Who is the brand ambassador of MasterCard?

(a) MS Dhoni

(b) Rohit Sharma

(c) Sachin Tendulkar

(d) Virat Kohli

(e) Jasprit Bumrah

Answer – (a)

Explanation –

This news of appointment of MS Dhoni as the brand ambassador of MasterCard. So, this shows that
the 3-4 months prior to the exam are of utmost importance. To be on a safer side, prepare six
months’ current affairs.

Q.5)

Where is the Nakki Lake located?

(a) Madhya Pradesh

(b) Rajasthan

(c) Karnataka

(d) Gujarat

(e) Assam
Answer – (b)

Explanation –

This question might appear as a static GK question but it is not so. It is rather a current affairs’
question. So, always remember to look for the background facts related to the news to get good
marks in the exam.

Q.6)

Which of the following pair of banks is being merged with Union Bank of India?

(a) Andhra Pradesh

(b) Syndicate Bank

(c) Corporation Bank

(d) Both A and B

(e) Both A and C

Answer – (e)

Explanation –

Banks’ merger was the utmost important news of that time. If such major announcements are made
by the government you need to keep a track of the news, even if the announcement was made
months ago. For example, the amalgamation of Regional Rural Banks is announced and going on. So,
keep a track of such news.

Q.7)

Which state does the consumer affair and food distribution minister Ram Vilas Paswan belong to?

(a) Madhya Pradesh

(b) Jharkhand

(c) Chhattisgarh

(d) Bihar

(e) Odisha

Answer – (d)

Explanation –
This is not a current affairs question. Rather, it is a question to check your knowledge on a broad
level. So, reading newspapers and developing awareness about the things going around you is
general awareness and this is only tested through such questions. So, if you are not able to make a
guess, don’t attempt this question as you have negative marking as well.

Q.8)

What is the percentage of gross NPA in banking sector in India, according to RBI’s Annual Report
2018-19?

(a) 6.3%

(b) 7.7%

(c) 5.6%

(d) 9.1%

(e) 8.7%

Answer – (d)

Explanation –

This is also a question from news. Firstly, the months prior to exam are important. All such types of
news and questions are covered in Spotlight.

Q.9)

Bali Jatra festival is observed by which state?

(a) Assam

(b) Odisha

(c) West Bengal

(d) Tripura

(e) Kerala

Answer – (b)

Explanation –

This festival was in news around the exam’s time. So, all the state festivals, emblems, and symbols
which are in news are important. Sometimes, such questions are also asked when the state is in the
news.
Q.10)

With Which sport is Manju Rani related to?

(a) Boxing

(b) Athletics

(c) Archery

(d) Wrestling

(e) Badminton

Answer – (a)

Explanation –

Manju Rani was in the news around this time. So, cover the important personalities in news. The
guess work would not be possible in this question if you haven't heard about Manju Rani before. So,
Not attempting such questions is the only way to tackle these questions because remember you
have your negative marking as well.

Q.11)

14th Nov is internationally celebrated as?

(a) World Diabetes Day

(b) Worl Day of War Orphans

(c) International Day of Education

(d) World Radio Day

(e) International Mother Language Day

Answer – (a)

Explanation –

Important days hold a very significant place in the RBI grade b examination, and we have seen this in
the morning shift as well. So, what is the teaching here that you need to cover the important days
thoroughly. For remembering important days, you can create a CHEAT SHEET for yourself.

Q.12)

To which country does cricketer Ben stokes belong?

(a) Netherlands

(b) England
(c) Croatia

(d) New Zealand

(e) Australia

Answer – (b)

Explanation –

Important personalities which are in the news have been asked repeatedly in the examination of RBI
grade b. Therefore, it is very important for all the candidates to prepare the important personalities
from every sphere for your examination. Spotlight magazine covers all such news which can be
asked in the exam. To remember these kinds of news you can create a CHEAT SHEET or short notes
for yourself.

Q.13)

For which sector does the RBI launched Central Information System for Banking Infrastructure
(CISBI)?

(a) Public Banks

(b) Cooperative banks

(c) Small Finance Banks

(d) Payment Banks

(e) Regional Rural Banks

Answer – (b)

Explanation –

Since, it is the RBI examination therefore the news related to RBI are very obvious to be asked. For
all the news related to RBI you can rely either on the spotlight magazine or you can create a CHEAT
SHEET for yourself. creating a CHEAT SHEET is must for such examinations because you have a lot of
facts to be covered for the exam.

Q.14)

Who has authored The Daughter from a Wishing Tree book?

(a) Sudha Murthy

(b) Kunal Basu

(c) Wajida Tabassum

(d) Aanchal Malhotra


(e) Amitav Ghosh

Answer – (a)

Explanation –

Books and their authors have been asked in the examination for a long period of time. You can
witness such questions in every type of examination. Therefore, you should keep a track of all the
books which are in the news. For keeping a track of the new launches, you can either rely on any
specific exam related magazine or read newspapers daily and create your own short notes from
them.

Q.15)

Where is the Kaiga national park located?

(a) Madhya Pradesh

(b) Karnataka

(c) Telangana

(d) Andhra Pradesh

(e) Maharashtra

Answer – (b)

Explanation –

National parks, wildlife sanctuaries, biosphere reserves, solar power plants, ecological sensitive
zones are all a part of the static general awareness. From such questions you can yourself judge the
proportion of questions asked from the static general awareness. Therefore, the lesson here is to
cover the static general awareness as well. You can choose any good book for that for example you
can also choose Lucent’s book.

Q.16)

Which state does the kuchipudi dance form belong to?

(a) Uttar Pradesh

(b) Assam

(c) Arunachal Pradesh

(d) Andhra Pradesh

(e) Uttar Pradesh


Answer – (d)

Explanation –

State related static facts are asked in the examination. This question is the direct evidence of it. Now
you don't have to cover the static facts such as the folk dance, state bird, state animal, state flower
etc. for every state. You just need to focus on the states which are in news and collect the static facts
related to those states. For static facts related to states Wikipedia is a good source which can help
you in finding the state related static facts easily.

Q.17)

Where was the BRICS cultural meeting held?

(a) Berlin, Germany

(b) Curitiba, Brazil

(c) Rio de Janeiro, Brazil

(d) Milan, Italy

(e) Belgrade, Serbia

Answer – (b)

Explanation –

Meetings and summits of important international organizations are often asked in the examination.
So not only cover their but cover their important meetings as well. Brics cultural meeting is an
integral part of the Brics forum, that is why, this question has been asked here. Not only the
important meetings of the organisations are asked but the meetings hosted by India with other
nations alongside the important summit have been asked in RBI’s examination only. So, your duty is
to cover the bilateral meetings which Indian leaders hold with other nations alongside these
important meetings and summits.

Q.18)

With which sport is the Subroto Cup associated?

(a) Badminton

(b) Football

(c) Athletics

(d) Wrestling

(e) Basketball
Answer – (b)

Explanation –

Often students make this mistake of ignoring the important sports related news. This is a very big
blunder that students often make, according to me. So, awards, sports, books, events etc. are
important for you to cover. You can refer a good magazine tailor-made specifically for RBI, Sebi
Nabard’s examination or you can follow a good newspaper like the livemint, the Hindu, The Indian
Express, economic times, Business Standard etc.

Q.19)

Which country has launched the Gaofen Satellite?

(a) Philippines

(b) Croatia

(c) South Africa

(d) China

(e) Saudi Arabia

Answer – (d)

Explanation –

International affairs hold a significant position in this examination. You can cover the international
affairs from spotlight magazine or any other magazine which specifically caters to the banking
students. This question can be attempted by making a guesswork as well. If you are a little bit aware
of the Mandarin language or if you use the word association technique then also you can associate
Gaofen word with China. Such guess works you can attempt in the examination but be careful
because you have negative marking in the exam as well.

Q.20)

What is the capital of Sierra Leone?

(a) Freetown

(b) Lusaka

(c) Mbabane

(d) Nouakchott

(e) Porto Novo

Answer – (a)
Explanation –

This is a static general awareness question. the very basic static awareness question is related to
capitals, currencies, national parks, EC. So, covering them is a must. the sources which you can use
to cover the static general awareness are Lucent’s book, Manorama’s yearbook etc. Always
remember that in order to retain a lot of facts you should be revising the facts again and again.

Q.21)

With which country does India conduct Vajra Prahar military exercise?

(a) Germany

(b) USA

(c) France

(d) Italy

(e) UK

Answer – (b)

Explanation –

This is a defence news and a very basic question. Always cover the defence exercises conducted
before the examination. Defence related exercises do not only constitute your current affairs but
they are also a part of your static general awareness. Therefore, it is not necessary that an exercise
which has been recently conducted will only be asked in the examination. Rather, the examiner may
ask you about the defence exercises which India conducts. So, cover all the exercises of Indian
armed forces. The list of Indian exercises is also given on PIB. You can use the website for
remembering defence exercises.

Q.22)

Which country has launched the Minuteman III intercontinental satellite?

(a) USA

(b) Japan

(c) South Korea

(d) Qatar

(e) Germany

Answer – (a)

Explanation –
This question is from the international current affairs. In this paper only, we have witnessed that two
questions have been asked from the international current affairs and both the questions are from
the space sector. This highlights that whatever is happening in the space sphere is important for this
examination. We don’t know whether the examiner is more interested in the country or in the space
sector but the lesson to learn from here is that space related news and the news of international
current affairs are asked and therefore you should cover them. You can cover such type of news
either from a good magazine or from different newspapers like livemint, economic times, Indian
Express etc.

Q.23)

Which country does the Nobel prize winner Peter Handke belongs to? (Recently he won Nobel prize
in literature.)

(a) Austria

(b) Slovenia

(c) Serbia

(d) Germany

(e) Canada

Answer – (a)

Explanation –

Questions from the Nobel Prize are asked in the morning shift as well. So, Nobels are a must. Nobel
winners, their nationalities, and their researches are important.

Q.24)

What is the reason for awarding William G Kaelin, Gregg L Semenza and Peter J Ratcliff with the
Nobel Prize in medicine 2019?

(a) For their discoveries of how cells sense and adapt to oxygen availability.

(b) For his discoveries concerning the genomes of extinct hominins and human evolution

(c) For their discoveries of receptors for temperature and touch

(d) For the discovery of Hepatitis C virus

(e) For their discoveries of how cells sense and adapt to oxygen availability

Answer – (a)

Explanation –
One more question has been asked from the Nobel prizes. This time the question is asking about the
reason for which William G Kaelin, Gregg L Semenza and Peter J Ratcliff have won the prize.

Q.25)

Which state has hosted the India International Cooperatives Trade Fair?

(a) Delhi

(b) Madhya Pradesh

(c) Gujarat

(d) Assam

(e) Karnataka

Answer – (a)

Explanation –

This question is again from the important events. All the events, summits, and national level
conferences organised in or buy India are important. In order to cover all the events of the past 6
months, preceding the date of examination, you can Select a good source. Spotlight magazine covers
all such news.

Q.26)

Bernandine Evaristo has been awarded with the Booker Prize 2019 for which book?

(a) The Seven Moons of Maali Almeida

(b) Shuggie Bain

(c) Girl, Woman and Other

(d) The Promise

(e) The Sellout

Answer – (c)

Explanation –

This is a question from the awards section. All the awards, the country giving the awards, the
recipients, and the recipients’ nationalities are all important. This question is a direct and factual
question. Therefore, it cannot be attempted by doing the guesswork. So, in such type of questions if
you know the answer only then attempt otherwise you should leave these questions. You should be
aware of this fact that you don't have to score 100% marks in general awareness you just need to
clear the cut off. Therefore, if you will target at least 70 percent marks in the GA section of RBI grade
b then you are on the safer side.

Q.27)

Which country has abolished the Kafala System?

(a) UAE

(b) Bahrain

(c) Qatar

(d) Saudi Arabia

(e) Oman

Answer – (c)

Explanation –

This question is again from the international current affairs. This system was in the news that is why
this question has been asked in this examination. Now you can clearly see in the options all the
countries belong to the Middle East region therefore the guess work is not easy here. Therefore, the
suggestion here is to cover the international current affairs thoroughly.

Q.28)

Which ministry has launched the Dhruv- Pradhan Mantri Innovative Learning Program?

(a) Ministry of HRD

(b) Ministry of Electronics and IT

(c) Ministry of Skill Development and Entrepreneur

(d) Ministry of Information and Broadcasting

(e) Ministry of Communications

Answer – (a)

Explanation –

This question is from the government schemes. All the government schemes are very important for
the phase one and the Phase two of RBI grade b. You can create your own short notes for
remembering the key facts related to the government schemes.

Q.29)
Which country will host the G7 Summit 2020?

(a) USA

(b) Germany

(c) Japan

(d) France

(e) Italy

Answer – (a)

Explanation –

This question is from the international current affairs. This is from the important events and summits
topic. Now, the advice here for all of you is not only to cover these summits but also cover the
organization’s background facts Like the headquarters, the year of formation, the number of current
members, the recent initiatives of the organization EC.

Q.30)

Where is Kiaga Atomic Power Station located?

(a) Madhya Pradesh

(b) Karnataka

(c) Arunachal Pradesh

(d) Gujarat

(e) Tamil Nadu

Answer – (b)

Explanation –

this question is from the starting general awareness. Atomic power stations, thermal power stations,
solar power stations etc. are all a part of the static general awareness.

Q.31)

Which bank has launched the ‘FD Health’- fixed deposit scheme?

(a) Axis Bank

(b) ICICI bank

(c) Bank of India


(d) Union Bank of India

(e) State Bank of India

Answer – (b)

Explanation –

This question is a part of your banking awareness. All the news related to the banks, including new
initiatives, new partnerships, mergers and acquisitions are all relevant for this examination. You can
find the banking awareness related current affairs on a number of competitive websites. Here, you
should be selective in choosing your source of information. So, spend time on choosing a good
source of current affairs and once you have chosen the website or the magazine then stick to it. It is
highly advisable in your interest only not to shift from source to source.

Q.32)

In which state does the Shinyuu Maitri exercise between India and Japan held?

(a) Arunachal Pradesh

(b) Meghalaya

(c) Himachal Pradesh

(d) Uttarakhand

(e) West Bengal

Answer – (e)

Explanation –

Now this is the second question from the defence exercises. defence related exercise conducted by
India with different countries are provided on the PIB website. You just have to search for it and you
will get the list of defense related exercises.

Q.33)

What does Article 22 elaborate?

(a) Protection against arrest and detention in certain cases

(b) State shall secure a social order for the promotion of welfare of the people

(c) Provision for free and compulsory education for children

(d) Oath or affirmation by the Vice President

(e) Abolition or creation of Legislative Councils in States


Answer – (a)

Explanation –

RBI grade b examination does not only test your banking awareness or static awareness. Rather, it
tests your broader understanding of things going around you.

Q.34)

Where was the women’s world boxing championship held?

(a) Saint Petersburg, Russia

(b) Ulan Ude, Russia

(c) Samara, Russia

(d) Vladivostok, Russia

(e) Moscow, Russia

Answer – (b)

Explanation –

It is a sports-related news. It is a direct question. Major sports events should be thoroughly covered
by you.

Q.35)

How many languages does the new version of BHIM 2.0 support?

(a) 3

(b) 7

(c) 5

(d) 4

(e) 9

Answer – (a)

Explanation –

The app was launched in three languages which are Haryanvi, Konkani and Bhojpuri. This question
has been asked from the national current affairs. This question is from the new initiatives' topic. All
these kinds of news can be covered from some good newspapers like Livemint, Indian Express,
Business Standard, Economic Times, The Hindu etc. You can also choose a tailor-made current
affairs' magazine as well. You just need to know that whichever source you choose you should stick
to that and create your own short notes. You can make a guess work here, if you are aware about it.

Q.36)

To which country does cricketer Jennie Gunn belong?

(a) England

(b) South Africa

(c) Gambia

(d) New Zealand

(e) Australia

Answer – (a)

Explanation –

It is a sports-related news. It is a direct question. Major sports events should be thoroughly covered
by you.

Q.37)

What is the new minimum net worth threshold for Bharat Bill Payment Operating Unit under the
new on-tap authorization guidelines of RBI?

(a) Rs 100 crore

(b) Rs 500 crore

(c) Rs 750 crore

(d) Rs 800 crore

(e) Rs 400 crore

Answer – (a)

Explanation –

This question has been asked from the banking current affairs. This question is from the guidelines
and framework's topic. All these kinds of news can be covered from some good newspapers like
Livemint, Indian Express, Business Standard, Economic Times, The Hindu etc. You can also choose a
tailor-made current affairs' magazine as well. You just need to know that whichever source you
choose you should stick to that and create your own short notes. You cannot make a guess work
here, if you are not aware about it.
Q.38)

Which organization has published the ‘South Asia Economic Focus, Making (De) Centralization
Works’?

(a) International Monetary Fund

(b) European Central Bank

(c) World Bank

(d) USAID

(e) Japan International Cooperation Agency

Answer – (c)

Explanation –

This question has been asked from the reports and indices. All these kinds of news can be covered
from some good newspapers like Livemint, Indian Express, Business Standard, Economic Times, The
Hindu etc. You can also choose a tailor-made current affairs' magazine as well. You just need to
know that whichever source you choose you should stick to that and create your own short notes.
You can make a guess work here, if you are a little bit aware about it.

Q.39)

Merchant Discount Rates are exempted for organization with an annual turnover of more than ___?

(a) Rs 60 crore

(b) Rs 50 crore

(c) Rs 70 crore

(d) Rs 65 crore

(e) Rs 40 crore

Answer – (b)

Explanation –

This question has been asked from the banking current affairs. This question is from the guidelines
and framework's topic. All these kinds of news can be covered from some good newspapers like
Livemint, Indian Express, Business Standard, Economic Times, The Hindu etc. You can also choose a
tailor-made current affairs' magazine as well. You just need to know that whichever source you
choose you should stick to that and create your own short notes. You cannot make a guess work
here, if you are not aware about it.
Q.40)

Which payments bank has recently shut its operation?

(a) Aditya Birla Idea Payments Bank

(b) Fino Payments Bank

(c) India Post Payment Bank

(d) Jio Payment Bank

(e) NSDL Payment Bank

Answer – (a)

Explanation –

This question has been asked from the banking current affairs. This question is from the banks-
related information. All these kinds of news can be covered from some good newspapers like
Livemint, Indian Express, Business Standard, Economic Times, The Hindu etc. You can also choose a
tailor-made current affairs' magazine as well. You just need to know that whichever source you
choose you should stick to that and create your own short notes. You cannot make a guess work
here, if you are not aware about it.

Q.41)

With which bank has Home Credit India partnered for co-lending?

(a) Dhanlaxmi Bank

(b) Karur Vyasa Bank

(c) Bandhan Bank

(d) Canara Bank

(e) HSBC Bank

Answer – (b)

Explanation –

This question has been asked from the banking current affairs. This question is from the banks'
collaborations and new partnerships. All these kinds of news can be covered from some good
newspapers like Livemint, Indian Express, Business Standard, Economic Times, The Hindu etc. You
can also choose a tailor-made current affairs' magazine as well. You just need to know that
whichever source you choose you should stick to that and create your own short notes. You can
make a guess work here, if you are a little bit aware about it.
Q.42)

With which company has Airtel Payment Bank partnered to launch the “Mosquito Disease Protection
Policy”?

(a) HDFC Ergo General Insurance

(b) Aditya Birla Sun Life Insurance Co. Ltd.

(c) TATA AIG Life Insurance Co. Ltd.

(d) HDFC Standard Life Insurance Co. Ltd.

(e) SBI Life Insurance Co. Ltd.

Answer – (a)

Explanation –

This question has been asked from the banking current affairs. This question is from the banks' new
initiatives. All these kinds of news can be covered from some good newspapers like Livemint, Indian
Express, Business Standard, Economic Times, The Hindu etc. You can also choose a tailor-made
current affairs' magazine as well. You just need to know that whichever source you choose you
should stick to that and create your own short notes. You can make a guess work here, if you are a
little bit aware about it.

Q.43)

With which bank has Bandhan Bank partnered to launch the co-branded credit card?

(a) CitiBank

(b) HSBC Bank

(c) DSB

(d) Standard Chartered Bank

(e) Bank of India

Answer – (d)

Explanation –

This question has been asked from the banking current affairs. This question is from the banks' new
initiatives. All these kinds of news can be covered from some good newspapers like Livemint, Indian
Express, Business Standard, Economic Times, The Hindu etc. You can also choose a tailor-made
current affairs' magazine as well. You just need to know that whichever source you choose you
should stick to that and create your own short notes. You can make a guess work here, if you are a
little bit aware about it.
Q.44)

This is the ___ time that Tokyo is hosting summer Olympics 2020?

(a) 10th

(b) 9th

(c) 2nd

(d) 7th

(e) 5th

Answer – (c)

Explanation –

It is a sports-related news. It is a direct question. Major sports events should be thoroughly covered
by you.

Q.45)

Where is Green Park Stadium located?

(a) Indore, Madhya Pradesh

(b) Kanpur, Uttar Pradesh

(c) Rourkela, Odisha

(d) Guwahati, Assam

(e) Imphal, Manipur

Answer – (b)

Explanation –

In this question, we have observed that the direct questions are most often asked in the exam.
These types of direct and short sentenced questions do not leave any scope of any hint or guess
work. So, the best strategy to tackle such questions is to leave them because you have negative
marking in the exam. Taking such a risk can cost you a lot.

Q.46)

Which state celebrates the Shirui Lily festival?

(a) Nagaland

(b) Manipur
(c) Tripura

(d) Assam

(e) Kerala

Answer – (b)

Explanation –

In this question, we have observed that the direct questions are most often asked in the exam.
Here, through word association, you can try to make a guess if you are not very sure about the
answer. Although, preparing the current affairs of past six months before the exam would be the
best strategy to tackle such questions.

Q.47)

Cosmonaut Alexei Leonov has passed away. Which country does he belong to?

(a) Germany

(b) Netherlands

(c) Denmark

(d) Kazakhstan

(e) Russia

Answer – (e)

Explanation –

In this question, we have observed that the direct questions are most often asked in the exam. Here,
through word association, you can try to make a guess if you are not very sure about the answer.
The name appears to be very Russian/Central Asian in nature.

Q.48)

Which is the second highest speaking language after Hindi in India, according to Census 2011?

(a) Bengali

(b) Marathi

(c) Santhali

(d) Tamil

(e) Telugu
Answer – (a)

Explanation –

This is a direct question which has been asked from the census. It is an integral part of the exam’s
general awareness syllabus.

Q.49)

Where is the Padmaja Naidu Himalayan Zoological Park located?

(a) Chhattisgarh

(b) Jharkhand

(c) West Bengal

(d) Maharashtra

(e) Odisha

Answer – (c)

Explanation –

This is a direct question which has been asked from the static GA. It is an integral part of the exam’s
general awareness syllabus.

Q.50)

Central Vigilance Commission has formed a committee, headed by T M Bhasin for examining bank
frauds of over ___?

(a) Rs 100 crore

(b) Rs 600 crore

(c) Rs 75 crore

(d) Rs 50 crore

(e) Rs 20 crore

Answer – (d)

Explanation –

This question has been asked from the banking current affairs. This question is from the framework
and guidelines topic. All these kinds of news can be covered from some good newspapers like
Livemint, Indian Express, Business Standard, Economic Times, The Hindu etc. You can also choose a
tailor-made current affairs' magazine as well. You just need to know that whichever source you
choose you should stick to that and create your own short notes. You cannot make a guess work
here, if you are not aware about it.

Q.51)

Which organization releases the State of World’s Children Report 2019?

(a) UNICEF

(b) UNESCO

(c) Kids Rights

(d) Save The Children Fund

(e) Make-A-Wish Foundation

Answer – (a)

Explanation –

This question has been asked from the indices and reports current affairs. All these kinds of news
can be covered from some good newspapers like Livemint, Indian Express, Business Standard,
Economic Times, The Hindu etc. You can also choose a tailor-made current affairs' magazine as well.
You just need to know that whichever source you choose you should stick to that and create your
own short notes.

Q.52)

Which country has the lowest area among all UTs, according to the Census 2011?

(a) Daman and Diu

(b) Chandigarh

(c) Delhi

(d) Dadra and Nagar Haveli

(e) Lakshadweep

Answer – (e)

Explanation –

This is a direct question which has been asked from the Census. It is an integral part of the exam’s
general awareness syllabus.

Q.53)
Abiy Ahmed Ali has been awarded Nobel Peace Prize 2019 for promoting peace with which nation?

(a) Eritrea

(b) Namibia

(c) Canada

(d) Zambia

(e) Mauritius

Answer – (a)

Explanation –

It is an award-related news. It is a direct question. Major awards should be thoroughly covered by


you.

Q.54)

Who has been awarded with the Lifetime Achievement Award at India Sports Honours?

(a) Deepika Kumari

(b) Sania Mirza

(c) Milkha Singh

(d) PV Sindhu

(e) Jhulan Goswami

Answer – (c)

Explanation –

This is a question from the awards section. All the awards, the country giving the awards, the
recipients, and the recipients’ nationalities are all important. This question is a direct and factual
question. Therefore, it cannot be attempted by doing the guesswork. So, in such type of questions if
you know the answer only then attempt otherwise you should leave these questions. You should be
aware of this fact that you don't have to score 100% marks in general awareness you just need to
clear the cut off. Therefore, if you will target at least 70 percent marks in the GA section of RBI grade
b then you are on the safer side.

Q.55)

Who has been awarded the best actor award, along with Ayushman Khurana, at the 66th national
Awards 2019?

(a) Vicky Kaushal


(b) Hritik Roshan

(c) Varun Dhawan

(d) Tiger Shroff

(e) Ishan Khattar

Answer – (a)

Explanation –

This is again a question from the awards section. All the awards, the country giving the awards, the
recipients, and the recipients’ nationalities are all important. This question is a direct and factual
question.

Q.56)

Mahathir Mohamed is the Prime Minister of which country?

(a) Malaysia

(b) Philippines

(c) Thailand

(d) Indonesia

(e) Cambodia

Answer – (a)

Explanation –

This is a static general awareness question. the very basic static awareness question is related to
capitals, currencies, national parks, EC. So, covering them is a must. the sources which you can use
to cover the static general awareness are Lucent’s book, Manorama’s yearbook etc. Always
remember that in order to retain a lot of facts you should be revising the facts again and again.

Q.57)

Who has become the first Indian women cricketer to complete 20 years in International Cricket?

(a) Jhulan Goswami

(b) Ekta Bisht

(c) Shikha Pandey

(d) Taniya Bhatia

(e) Mithali Raj


Answer – (e)

Explanation –

Often students make this mistake of ignoring the important sports related news. This is a very big
blunder that students often make, according to me. So, awards, sports, books, events etc. are
important for you to cover. You can refer a good magazine tailor-made specifically for RBI, Sebi
Nabard’s examination or you can follow a good newspaper like the livemint, the Hindu, The Indian
Express, economic times, Business Standard etc.

Q.58)

Jagdeep Dhankar is the governor of which state?

(a) Kerala

(b) West Bengal

(c) Assam

(d) Arunachal Pradesh

(e) Odisha

Answer – (b)

Explanation –

State related static facts are asked in the examination. This question is the direct evidence of it. You
just need to focus on the states which are in news and collect the static facts related to those states.
For static facts related to states Wikipedia is a good source which can help you in finding the state
related static facts easily.

Q.59)

Which country does King William and Queen Maxima belong to?

(a) Finland

(b) Germany

(c) Netherlands

(d) Denmark

(e) Sweden

Answer – (c)
Explanation –

Important personalities which are in the news have been asked repeatedly in the examination of RBI
grade b. Therefore, it is very important for all the candidates to prepare the important personalities
from every sphere for your examination. Spotlight magazine covers all such news which can be
asked in the exam. To remember these kinds of news you can create a CHEAT SHEET or short notes
for yourself.

Q.60)

With which country is the Frank Worrell Trophy related to apart from Australia?

(a) West Indies

(b) Australia

(c) New Zealand

(d) Fiji

(e) Solomon Islands

Answer – (a)

Explanation –

Important personalities which are in the news have been asked repeatedly in the examination of RBI
grade b. Therefore, it is very important for all the candidates to prepare the important personalities
from every sphere for your examination. Spotlight magazine covers all such news which can be
asked in the exam. To remember these kinds of news you can create a CHEAT SHEET or short notes
for yourself.

Q.61)

Who has written the “How to Avoid a Climate Disaster: The Solutions We Have and the
Breakthroughs We Need” book?

(a) Bill Gates

(b) Elon Musk

(c) Jeff Bezos

(d) Warren Buffett

(e) Tim Cook

Answer – (a)

Explanation –
Books and their authors have been asked in the examination for a long period of time. You can
witness such questions in every type of examination. Therefore, you should keep a track of all the
books which are in the news. For keeping a track of the new launches, you can either rely on any
specific exam related magazine or read newspapers on a daily basis and create your own short notes
from them.

Q.62)

Mark Vincent Hurd has passed away. He was the ex-ceo of which company?

(a) General Electric

(b) Oracle

(c) Bellatrix Aerospace

(d) IBM

(e) Microsoft

Answer – (b)

Explanation –

Important personalities which are in the news have been asked repeatedly in the examination of RBI
grade b. Therefore, it is very important for all the candidates to prepare the important personalities
from every sphere for your examination. Spotlight magazine covers all such news which can be
asked in the exam. To remember these kinds of news you can create a CHEAT SHEET or short notes
for yourself.

Q.63)

Which state has topped the India Innovation Index 2019?

(a) Madhya Pradesh

(b) Arunachal Pradesh

(c) Karnataka

(d) Gujarat

(e) Uttar Pradesh

Answer – (a)

Explanation –

This question has been asked from the reports and indices. This question is from an Indian index. All
the reports and indices released during the six months prior to the exam date are important. All
these kinds of news can be covered from some good newspapers like Livemint, Indian Express,
Business Standard, Economic Times, The Hindu etc.

Q.64)

Where is the headquarters of Interpol located?

(a) Lyon, France

(b) Paris, France

(c) Marseille, France

(d) Toulouse, France

(e) Strasbourg, France

Answer – (a)

Explanation –

This is a static general awareness question. the very basic static awareness question is related to
capitals, currencies, national parks, EC. So, covering them is a must. the sources which you can use
to cover the static general awareness are Lucent’s book, Manorama’s yearbook etc. Always
remember that in order to retain a lot of facts you should be revising the facts again and again.

Q.65)

Who has topped the Edelgive Hurun India Philanthropy List 2019?

(a) Ratan Tata

(b) Gautam Adani

(c) Shiv Nadar

(d) Narayan Murthy

(e) Azim Premji

Answer – (c)

Explanation –

This question has been asked from the rankings. All the reports, indices, rankings released during the
six months prior to the exam date are important. All these kinds of news can be covered from some
good newspapers like Livemint, Indian Express, Business Standard, Economic Times, The Hindu etc.

Q.66)
Where has world’s first AI university been established?

(a) Qatar

(b) UAE

(c) Bahrain

(d) Kuwait

(e) Madagascar

Answer – (b)

Explanation –

It belongs to the international current affairs. Such questions are covered in the Spotlight magazine.
You can cover such types of news from different newspaper. Whichever source you choose, always
remember that you have to do multiple revisions to retain the facts.

Q.67)

To which country has India gifted a pair of Mi-24 helicopters?

(a) Afghanistan

(b) Gambia

(c) Mauritius

(d) Mozambique

(e) Argentine

Answer – (a)

Explanation –

This is a defence news and a very basic question. In this question, we have observed that the direct
questions are most often asked in the exam. These types of direct and short sentenced questions do
not leave any scope of any hint or guess work. So, the best strategy to tackle such questions is to
leave them because you have negative marking in the exam. Taking such a risk can cost you a lot.

Q.68)

Where is Our Lady of glory Church located?

(a) Mumbai

(b) Kolkata

(c) Panaji
(d) Bangalore

(e) Bhuvneshwar

Answer – (a)

Explanation –

This question might appear as a static GK question but it is not so. It is rather a current affairs’
question. So, always remember to look for the background facts related to the news to get good
marks in the exam.

Q.69)

Where is the headquarters of International Boxing Association located?

(a) Berlin, Germany

(b) Lausanne, Switzerland

(c) Brussels, Belgium

(d) Ankara, Turkey

(e) Baku, Azerbaijan

Answer – (b)

Explanation –

Again, this question might appear as a static GK question but it is not so. It is rather a current affairs’
question. So, always remember to look for the background facts related to the news to get good
marks in the exam.

Q.70)

What is the theme of International Girl Child Day 2019?

(a) My voice, our equal future

(b) Digital generation

(c) All girls deserve better—the world deserves better

(d) EmPOWER Girls: Before, during and after crises

(e) Girl force: Unscripted and unstoppable

Answer – (e)
Explanation –

The same question was asked in the Morning shift of RBI Grade B 2019. This highlights the
importance of major days.

Q.71)

Which company has for the first time crossed the Rs 9 lakh crore market capitalization limit?

(a) Reliance

(b) Tata Group

(c) Mahindra Group

(d) Infosys

(e) ITC Ltd

Answer – (a)

Explanation –

This question has been asked from the banking and finance current affairs. This is a company-specific
question. All these kinds of news can be covered from some good newspapers like Livemint, Indian
Express, Business Standard, Economic Times, The Hindu etc. You can also choose a tailor-made
current affairs' magazine as well. You just need to know that whichever source you choose you
should stick to that and create your own short notes. You cannot make a guess work here, if you are
not aware about it.

Q.72)

Which bank has launched the DigiSmart card?

(a) Standard Chartered Bank

(b) HSBC Bank

(c) Dhanlaxmi Bank

(d) Bank of India

(e) DBS

Answer – (a)

Explanation –

This question has been asked from the banking and finance current affairs. This is a question from
the new launches made by banks. All these kinds of news can be covered from some good
newspapers like Livemint, Indian Express, Business Standard, Economic Times, The Hindu etc. You
can also choose a tailor-made current affairs' magazine as well. You just need to know that
whichever source you choose you should stick to that and create your own short notes. You cannot
make a guess work here, if you are not aware about it.

Q.73)

Which country has announced to leave OPEC by 2020?

(a) Ecuador

(b) Brazil

(c) Bolivia

(d) Suriname

(e) Venezuela

Answer – (a)

Explanation –

It belongs to the international current affairs. Such questions are covered in the Spotlight magazine.
You can cover such types of news from different newspaper. Whichever source you choose, always
remember that you have to do multiple revisions to retain the facts.

Q.74)

Paektu Mountain is the highest mountain range of which country?

(a) South Africa

(b) North Korea

(c) Zambia

(d) Mozambique

(e) Mauritius

Answer – (b)

Explanation –

This question might appear as a static GK question but it is not so. It is rather a current affairs’
question. So, always remember to look for the background facts related to the news to get good
marks in the exam.

Q.75)
With which sport is Li Xuerui associated?

(a) Badminton

(b) Wrestling

(c) Athletics

(d) Boxing

(e) Tennis

Answer – a

Explanation –

It is a sports-related news. It is a direct question. Major sports events should be thoroughly covered
by you.

Q.76)

Which company has planned to invest $1 billion in India to set up a local payment processing center?

(a) Visa

(b) Mastercard

(c) Rupay

(d) American Express

(e) Apple

Answer – (b)

Explanation –

This question has been asked from the banking and finance current affairs. This is a company-specific
question. All these kinds of news can be covered from some good newspapers like Livemint, Indian
Express, Business Standard, Economic Times, The Hindu etc. You can also choose a tailor-made
current affairs' magazine as well. You just need to know that whichever source you choose you
should stick to that and create your own short notes. You cannot make a guess work here, if you are
not aware about it.

Q.77)

Where is the headquarters of International Solar Alliance located?

(a) Gurugram, Haryana

(b) Noida, Uttar Pradesh


(c) Kutch, Gujarat

(d) Indore, Madhya Pradesh

(e) Puri, Odisha

Answer – (a)

Explanation –

This is a static general awareness question. the very basic static awareness question is related to
capitals, currencies, national parks, EC. So, covering them is a must. the sources which you can use
to cover the static general awareness are Lucent’s book, Manorama’s yearbook etc. Always
remember that in order to retain a lot of facts you should be revising the facts again and again.

Q.78)

Who has been awarded with the Golden Shoe award 2019?

(a) Lionel Messi

(b) Cristiano Ronaldo

(c) Sadio Mane

(d) Mohamed Salah

(e) Erling Haaland

Answer – (a)

Explanation –

Often students make this mistake of ignoring the important sports related news. This is a very big
blunder that students often make, according to me. So, awards, sports, books, events etc. are
important for you to cover. You can refer a good magazine tailor-made specifically for RBI, Sebi
Nabard’s examination or you can follow a good newspaper like the livemint, the Hindu, The Indian
Express, economic times, Business Standard etc.

Q.79)

Which airline has become the world’s first airline to use Taxibot on A320 aircraft with passengers on-
board?

(a) Air India

(b) GoAir

(c) SpiceJet

(d) VIstara
(e) AirAsia

Answer – (a)

Explanation –

This is a company-specific question. All the company-specific news released during the six months
prior to the exam date are important.

Q.80)

State trading corporation of India operates under which ministry?

(a) Ministry of Commerce

(b) Ministry of Textiles

(c) Ministry of Information and Broadcasting

(d) Ministry of Finance

(e) Ministry of Corporate Affairs

Answer – (a)

Explanation –

In this question, we have observed that the direct questions are most often asked in the exam. Here,
you can guess the answer as well because trade comes under Ministry of Commerce.
Question Paper 1

RBI Grade B 2019 Phase 1 – General Awareness (Morning Shift)


1. 5th World Parliament of Science Religion and [Covered in the Course? – Yes in October
Philosophy was held in – Spotlight]
Ans. Pune
[Covered in the Course? – Yes in October 8. Helsinki is the capital of –
Spotlight] Ans. Finland
[Covered in the Course? – No]
2. To commemorate 550th Birth anniversary of
__________, UNESCO decided to translate and 9. SCO meeting in November 2019 is taking place at
publish anthology of his writings into world which place?
languages. Ans. Tashkent, Uzbekistan
Ans. Guru Nanak Dev [Covered in the Course? – Yes in October
[Covered in the Course? – Yes in September Spotlight]
Spotlight]
10. Pakyong airport is at which place?
3. The 2019 Nobel Chemistry Prize was based on Ans. Sikkim
Ans. Lithium ion Batteries [Covered in the Course? – No]
[Covered in the Course? – Yes in October
Spotlight] 11. Express FD is launched by which bank?
Ans. Axis Bank
4. What is the theme for UN International Day of Girl [Covered in the Course? – Yes in October
Child? – Spotlight]
Ans. “Girl Force – Unscripted and Unstoppable”
[Covered in the Course? – Yes in October 12. Theme of World’s Teachers Day – Young Teachers
Spotlight] Ans. The future of Profession
[Covered in the Course? – Yes in October
5. Where are the Top 3 cleanest railway stations Spotlight]
located?
Ans. State – Rajasthan 13. UN Habitat headquarters are located in Ans.
[Covered in the Course? – Yes in October Nairobi, Kenya
Spotlight] [Covered in the Course? – No]

6. Conference of Parties of UNCCD has been recently 14. Vikram Sarabhai Library located in?
taken over by which country from China – Ans. IIM Ahmedabad
India [Covered in the Course? – Yes in October
[Covered in the Course? – Yes in June Spotlight] Spotlight]

7. Niti Aayog and _____ collaborated for 15. Central Pollution Control Board collaborated with
disinvestment in PSU – _______ to make first E-waste Clinic.
Ans. DIPAM (Dept of Investment and Public Asset
Mgt)

Prepare for RBI, SEBI & NABARD, UPSC & UGC NET Exam info@anujjindal.in; +91 9999466225
RBI Grade B 2019 Phase 1 – General Awareness (Morning Shift) 2

Ans. Bhopal, Madhya Pradesh [Covered in the Course? – Yes in October


[Covered in the Course? – Yes in October Spotlight]
Spotlight]
24. What was the tagline of the mega life insurance
16. Mascot for 2020 Winter Olympics to be held in campaign?
China Ans. ‘Sabse Pehle Life Insurance’ [Covered in the
Ans. Bing Dwen Dwen Course? – Yes in October Spotlight]
[Covered in the Course? – Yes in September
Spotlight] 25. Lalji Tondon is the governor of
Ans. Madhya Pradesh
17. Floating basketball court is in which city? [Covered in the Course? – No]
Ans. Mumbai
[Covered in the Course? – Yes in October 26. RastrIya Ekta Diwas is celebrated for whom?
Spotlight] Ans. Vallabhbhai Patel
[Covered in the Course? – Yes in October
18. 50th World Economic Forum Meet Spotlight]
Ans. Davos, Switzerland
[Covered in the Course? – Yes in September 27. Where is Rudrasagar lake?
Spotlight] Ans. Tripura
[Covered in the Course? – Yes in September
19. Credit Saison India based out of which place? Spotlight]
Ans. Bengaluru
[Covered in the Course? – Yes in October 28. Headquarters of Laxmi Vilas Bank are located in
Spotlight] Ans. Chennai
[Covered in the Course? – Yes in Static GK]
20. 2019 Japan Open Title is won by whom?
Ans. Novak Djokovic 29. Warfare exercise has been organised with PLA.
[Covered in the Course? – Yes in July Spotlight] PLA is the army of?
Ans. China (People’s Liberation Army) THERE WAS
21. Sarah Taylor is the cricketer of which country? NO WARFARE EXERCISE CONDUCTED BY CHINA
Ans. England
[Covered in the Course? – No] 30. IAEA secretariat – headquarters
Ans. Vienna, Austria
22. Article 15 relates to [Covered in the Course? – Yes in Static GK]
Ans. Prohibition of Discrimination based on the
grounds of caste, religion, creed, etc. 31. According to World Population Prospect,
[Covered in the Course? – No] population will grow by 2 billion in how many
years?
23. The 150th Gandhian challenge is made by NITI Ans. 30 Years (By 2050)
Aayog and which other organization? [Covered in the Course? – Yes in June Spotlight]
Ans. UNICEF

Prepare for RBI, SEBI & NABARD, UPSC & UGC NET Exam info@anujjindal.in; +91 9999466225
RBI Grade B 2019 Phase 1 – General Awareness (Morning Shift) 3

32. World Cotton Day event took place in which city? Ans. By 2025
Ans. Geneva [Covered in the Course? – Yes in October
[Covered in the Course? – Yes in October Spotlight]
Spotlight]
41. NETRA was launched by whom?
33. 7th World Hindu Economic Forum held in which Ans. ISRO
city? [Covered in the Course? – Yes in September
Ans. Mumbai Spotlight]
[Covered in the Course? – Yes in September
Spotlight] 42. Greta Thunberg belongs to which country
Ans. Sweden
34. Kaleshwaram Lift Irrigation Project is built in [Covered in the Course? - No]
which state?
Ans. Telangana 43. RBI rejected the merger of India Bulls and ____
[Covered in the Course? – Yes in September Ans. Lakshmi Vilas Bank
Spotlight] [Covered in the Course? – Yes in October
Spotlight]
35. Edgar Chagwa is the president of which country?
Ans. Zambia 44. PV Sindhu has been appointed as the Brand
[Covered in the Course? – No] Ambassador of
Ans. Visa Card
36. Professional Cricket Association Player of the year [Covered in the Course? – Yes in September
Ans. Ben Stokes Spotlight]
[Covered in the Course? – No]
45. First Lt Governor of Ladakh
37. Himvijay exercise took place in which state? Ans. RK Mathur
Ans. Arunachal Pradesh [Covered in the Course? – Yes in October
[Covered in the Course? – Yes in September Spotlight]
Spotlight]
46. UN Settlement Agency/ UN HABITAT
38. NTPC solar park is in which state? headquarters –
Ans. Kutch, Gujarat
[Covered in the Course? – Yes in October 47. First Bank of the country to use Robotic Hand to
Spotlight] count currency
Ans. ICICI Bank
39. Social Stock Exchange is proposed by whom? [Covered in the Course? – Yes in August Spotlight]
Ans. SEBI
[Covered in the Course? – Yes in July & September 48. South East Asia’s largest solar power plant is
Spotlight] located in which country?
Ans. Vietnam
40. 5 GDP health is achieved by which year?

Prepare for RBI, SEBI & NABARD, UPSC & UGC NET Exam info@anujjindal.in; +91 9999466225
RBI Grade B 2019 Phase 1 – General Awareness (Morning Shift) 4

[Covered in the Course? – Yes in September 57. What is the name of exercise conducted by Indian
Spotlight] Army in Ladkah? Ans. Chang Thang
[Covered in the Course? – Yes in September
49. NTPC proposed to build a mega power project Spotlight]
with a capacity of 5000 GW in which state? [NO]
58. Where will be G20 summit 2020 be held?
50. PNB is proposed to be merged with Ans. Saudi Arabia
Ans. United Bank of India [Covered in the Course? – Yes in October
[Covered in the Course? – Yes in September Spotlight]
Spotlight]
59. Autobiography “Lesson life taught me
51. Question related to Kounotori8 Unknowingly”
[Covered in the Course? – Yes in September Ans. Anupam Kher
Spotlight] [Covered in the Course? – Yes in june Spotlight]

52. 7th Tiger Reserve Ratapani Tiger Reserve 60. Theme for “International Women’s day” Ans.
Ans. Madhya Pradesh “Think equal, build smart, innovate for change”
[Covered in the Course? –No] [Covered in the Course? – Yes in March Spotlight]

53. Tanegashima Space Center is located in which 61. Capital requirement for Small Finance Bank
country Ans. Rs 200 crore
Ans. Japan [Covered in the Course? – Yes in September
[Covered in the Course? – Yes in September Spotlight]
Spotlight]
62. Name of Rohit Sharma’s Rhino Initiative –
54. James Peebles, Michel Mayor and Didier Queloz Rohit4Rhino launched with
have been awarded Ans. WWF (World Wide Fund) India and Animal
Ans. Nobel Prize 2019 in Physics [Covered in the Planet
Course? – Yes in October Spotlight] [Covered in the Course? – Yes in September
Spotlight]
55. Apache Helicopters were delivered by which US
company 63. ______ has the largest diaspora of World
Ans. Boeing Migration Stock
[Covered in the Course? – Yes in May Spotlight] Ans. India
[Covered in the Course? – Yes in September
56. INS Khanderi has been commissioned recently Spotlight]
from
Ans. Mazegaon Dockyard 64. Revised name of Reliance Mutual Fund Ans.
[Covered in the Course? – Yes in September Nippon India Mutual Fund
Spotlight] [Covered in the Course? – Yes in October
Spotlight]

Prepare for RBI, SEBI & NABARD, UPSC & UGC NET Exam info@anujjindal.in; +91 9999466225
RBI Grade B 2019 Phase 1 – General Awareness (Morning Shift) 5

65. Piyush Goyal – Member of Rajya Sabha from [Covered in the Course? – Yes in August & October
which state? Spotlight]
Ans. Maharashtra
[Covered in the Course? – No] 74. Question related to
Ans. Courageous Journalism Award
66. Ekuverin Exercise held in [Covered in the Course? – No]
Ans. Pune
[Covered in the Course? – Yes in October 75. HL Trivedi died recently
Spotlight] Ans. He was renowned – Nephrologist
[Covered in the Course? – No]
67. Recently GOI and LIC has infused Rs 9300 cr in
which bank? 76. National Nutrition Survey, led by the Ministry of
Ans. IDBI Bank Health and Family Welfare has been launched in
[Covered in the Course? – Yes in September collaboration with Ans. UNICEF
Spotlight] [Covered in the Course? – Yes in October
Spotlight]
68. Rajnath Singh visited Defence Expo in which
country? 77. India has offered an additional $ 236 million credit
to _______ for setting up a 1.5 million tonne per
69. Recently which state cooperative bank merger has annum oil refinery in that country
been approved in Ans. Mongolia
[Covered in the Course? – Yes in October
Ans. Kerala State cooperative Bank [Covered in Spotlight]
the Course? – Yes in October Spotlight]
78. A new book titled “150 Years of Celebrating the
70. Director of Gully Boy Mahatma the South African Legacy”, by Fakir
Ans. Zoya Akhtar Hassen launched
[Covered in the Course? – Yes in July & September Ans. Johannesburg, South Africa
Spotlight] [Covered in the Course? – Yes in October
Spotlight]
71. Industry 4.0 Pilot Project between
Ans. Ministry of Railways and IIT Kanpur
[Covered in the Course? – Yes in September
Spotlight]

72. Nirmala Sheroron was banned. She belongs to


Ans. Athletics
[Covered in the Course? – No]

73. Muni Bond launched


Ans. SEBI

Prepare for RBI, SEBI & NABARD, UPSC & UGC NET Exam info@anujjindal.in; +91 9999466225
RBI GRADE B 2020-21 GK ANALYSIS
BOTH SHIFTS (MEMORY BASED)

BY- ANUJ JINDAL


KEY BENEFITS OF OUR COURSES

Our course structure includes a lot of perks that are otherwise


unavailable elsewhere.
It is a comprehensive guide to help you crack the paper & secure
your dream position.

We provide personal solutions all queries using a Telegram group wherein Anuj Jindal
himself will clarify your doubts.

We curate the learning strategies of past year toppers to help you learn from the
success of the best

2
MORNING SHIFTS
1. Myoko festival is celebrated in which state?
Ans – Arunachal Pradesh

2. India has improved its ranking in global innovation index 2020 to 48 from ___ in 2019?
Ans – 52nd

Covered in Indices and Reports 2021

3. What is India's global position in terms of energy consumption as per the India energy
outlook 2021 report?
Ans – 4th

Covered in Spotlight February 2021

4. Which of the following is not a Labour code?


Ans – Option related to Migration

Covered in Spotlight

5. Hiranandani Group companies will set up an integrated logistics and hyperscale data centre
park called Greenbase and Yotta. Where will this center be located?
Ans – West Bengal

6. Jeff Bezos has stepped down from his position of Chief Executive of the Amazon and Andy
Jassy has been appointed as the new CEO. How did Amazon started in its initial days?
Ans – Bookstore

3
7. Ayushman sahakar scheme was launched to improve the infrastructure of ___ sector with
the help of cooperatives in the country?
Ans – Healthcare

Covered in Spotlight October 2020

8. Which app was launched by the finance ministry to help people locate financial services at
any location?
Ans – Jan Dhan Darshak

Covered in Current Affairs 2018

9. In 2015, which of the following company became the first microfinance company in India to
receive a universal banking license?
Ans – Bandhan Bank

10. At which place is the Ranthambhore National Park located in Rajasthan?


Ans – Sawai Madhopur

Covered in Static GK 2021 (National Parks, Wildlife Sanctuaries and Biosphere Reserve
doc)

11. What is the limit of capital expenditure for Indian Railways as per the union budget of 2021
to 2022?
Ans – Rs 1.07 Tn

12. Denmark's first energy island will be based on which energy?


Ans – Wind Energy

Covered in Spotlight February 2021

4
13. Which Indian PSU has been ranked as the best employer in the “world’s best employer
2020” list by Forbes?
Ans – NTPC

Covered in Spotlight October 2020

14. When is the National Cadet Corps raising day observed?


Ans – November 22

Covered in Spotlight November 2020

15. Which bank has launched “Gram Sampark Abhiyan”?


Ans – Punjab National Bank

Covered in Spotlight October 2020

16. What is the __ C approach in the National Strategy for Financial Education 2020-25 of RBI?
Ans – 5C

Covered in Spotlight August 2020

5C Approach – Content, Capacity, Community, Communication, Collaboration.

17. What does ‘R’ stands for in the full form of CRA? (CRA is the agency registered under
PFRDA.)
Ans – Record Keeping

18. What is the minimum limit for RTGS?


Ans – Rs 2 lakh

Covered in the course

5
19. Which one of the following is not a feature of PM-Kusum scheme?
Ans – Not Available

Covered in Govt schemes

20. Which of the following place was hit by Cyclone Nivar?


Ans – Tamil Nadu and Puducherry

Covered in Spotlight November 2020

21. Where will the advanced manufacturing hub (AMHUB) be established in India?
Ans – Tamil Nadu

Covered in Spotlight November 2020

22. Who among the following is not eligible to avail the benefit of pm kisan scheme?
Ans – Not available

Covered in Govt schemes

23. Which one of the following is a feature of senior citizen savings scheme?
Ans – Not Available

24. What is the corpus of post metric scholarship scheme for SC students?
Ans - Rs 59048 crores

Covered in Spotlight December 2020

25. Recently, CM of Puducherry, V. Narayanasamy, has submitted his resignation. How many
seats does the Puducherry assembly have?
Ans – 30 member

6
26. What is the nationality of the director general of World Trade Organization?
Ans – Nigeria

Covered in Spotlight February 2021

27. Which of the following is not a parameter of the digital payment index released by RBI?
Ans – Not Available

Covered in Spotlight January 2021

• Parameters of DPI – total 5 which are,


® (Payment Enablers – 25% weightage)
® (Payment Infrastructure: Supply side factors – 15% weightage)
® (Payment Infrastructure: Demand side factors – 10% weightage)
® (Payment Performance – 45% weightage)
® (Consumer Centricity – 5% weightage)

28. What is the art of making charts and Maps called?


Ans – Cartography

29. Recently, HCL Technologies has joined the ___ revenue club?
Ans - $10 Billion

30. What is the minimum and maximum limit of Cash Reserve Ratio?
Ans – No maximum and minimum limit

31. NAVRITIH certification course is related to which of the following sectors?


Ans- Construction Technology

7
Covered in Spotlight January 2021

32. Where will multi modal logistics park and multi modal transport hub be set up?
Ans – Noida, UP

33. Which edition of INS Karanj was given to Indian Navy by Mazagon Dock Limited?
Ans – 3rd

Covered in Spotlight February 2021

34. Which of the following small finance bank has got permission from SEBI to launch its IPO in
the month of December 2020?
Ans – Suryodaya SFB

35. What is the term used for ATMs that are owned and operated by non-bank entities?
And – White Label ATM

Covered in the course

36. Whose birth anniversary is celebrated as good governance day on December 25 by India?
Ans – Atal Bihari Vajpayee

Covered in Spotlight December 2020

37. Which one of the following is not a policy rate?


Ans – SLR

38. Which one of the following is not a correct feature of i-mobile app of ICICI Bank?
Ans - it can be used by the customers of ICICI Bank only

8
Covered in Spotlight December 2020

39. What is the target of national education policy in terms of gross enrolment ratio at the level
of pre-school to secondary by 2030 and higher education by 2035 respectively?
Ans – 100% ; 50%

Covered in National Education Policy doc

40. Which state was ranked at the second position in terms of supporting MSMEs after
Maharashtra?
Ans – Tamil Nadu

Covered in Spotlight November 2020

41. Where will the National Authority of Ship Recycling be located?


Ans – Gandhinagar, Gujarat

Covered in Spotlight October 2020

42. According to the recently released guidelines of RBI for HFCs, non-deposit taking HFCs
with an asset size of Rs 5000 crores- Rs 10,000 will have to reach a minimum LCR of ___
by December 1, 2021 and to ____ by December 1, 2025?
Ans – 30% and 100%

Covered in Spotlight February 2021

43. In which state will India’s first Sand Dune park be located?
Ans – Goa

Covered in October 2020

9
44. What is the current Marginal Standing Facility rate?
Ans – 4.25%

Covered in Spotlight February 2021

45. Women self-help group is the primary block of which of the following scheme?
Ans – DAY-National Rural Livelihood Mission

Covered in Govt schemes

46. The minimum net owned fund limit of housing finance companies has been recently
proposed by RBI from ___ to Rs 20 crores?
Ans – RS 10 crores

Covered in Spotlight October 2020

47. Where is India’s first fire park being built?


Ans – Odisha

Covered in Spotlight January 2021

48. 2021 commemorates the __ anniversary of the signing of Ramsar Convention on


Wetlands?
Ans – 50th

Covered in Spotlight October 2020

49. Which of the following two places have been connected through Atal Tunnel?
Ans – Manali – Leh

Covered in Spotlight September, October 2020, January 2021

10
50. Which company has announced to launch the Engineering course recently?
Ans – L&T

51. With which bank has Bayer’s Better Life Farming initiative partnered for providing holistic
financial solutions to small holders farmers and rural farmer communities?
Ans – Axis Bank

Covered in Spotlight September 2020

52. What is Bank Credit to GDP ratio as on March 2020?


Ans – Not Available

53. What is the designation of Anil Soni in WHO Foundation?


Ans – Executive Chairperson

Covered in Spotlight December 2020

54. Tata Steel has announced to expand its production capacity to ___ to cater to the
increasing demand of steel?
Ans – 40 million tonnes (Feb 2021)

55. SBI has announced to increase its home loan portfolio to Rs 7 lakh crores by which year?
Ans – 2024 (Feb 2021)

56. In January 2021, West Indies beat Bangladesh with a score line of 395 for 3 wickets, which
is the ___________ highest chase in test series.
[a] Third
[b] Second
[c] Fourth

11
[d] Fifth
[e] Sixth

Ans - Fifth

57. As per the recent recommendation of RBI’s internal working group on corporate
governance of private sector banks, the share of ______ should be increased from 15% to
26% in the private banks in 15 years?
Ans – Promoters

Covered in Spotlight November 2020

58. Carin Energy Co. has recently won a case at the Hague, Netherlands. Where is the
company located?
Ans – UK

59. Recently, Adani Group has bought stake of ACSA Global Limited (ACSA) and Bidvest in
Mumbai Airport. Which country do these companies belong to?
Ans – South Africa

Covered in Spotlight September 2020

60. In which year was the banking ombudsman scheme launched?


Ans –1995

Covered in Spotlight December 2019 (in Feb 2021, annual report on Ombudsman has been
released.)

61. Which of the following banks is the leading player in credit card industry?
Ans – HDFC Bank

12
62. Which country is not a member of 5 eyes alliance?
Ans – India

Covered in Spotlight December 2020

63. Hope mission has been launched for which planet?


Ans – Mars (UAE)

64. What is the venue of Asian games 2034?


Ans – Saudi Arabia

Covered in Spotlight December 2020

65. Interest on which investment scheme is not taxable?


Ans – Public Provident Fund (PPF)

Covered in the course

66. What is the maximum limit of exemption under section 80 C of the Income Tax Act?
Ans - Rs 1.5 lakh

Covered in SEBI Grade A series

67. Which of the following is the highest valued bank in India?


OR

68. Which of the following is the top ranker in BFSI rankings?


Ans – HDFC Bank

69. One Question was on PM Vaya Vandana Yojana.


Covered in Govt schemes

13
AFTERNOON SHIFT
1. Where is Girnar ropeway situated?
Ans - Junagadh district, Gujarat

Covered in Spotlight Oct 2020

2. What is the share of BRICS countries in world trade?


Ans- 18% approx.

Covered in Spotlight November 2020

3. Which one of the following is not a pillar in the budget 2021-2022?


[a] Minimum Govt & Maximum Governance

[b] Domestic Savings

[c] Innovation & R&D

[d] Physical & financial capital & infrastructure

[e] Health & well-being

Ans – B (Domestic Savings)

Five Pillars – as per Budget FY22

o Health & well-being,


o Physical & financial capital & infrastructure,
o Inclusive development for aspirational India,
o Reinvigorating human capital,
o Innovation & R&D,
o Minimum Govt & Maximum Governance.

14
Covered in Budget 2021-22

4. Which of the following is incorrect about Prime Minister Employment Generation Program?
[a] Maximum cost for manufacturing is Rs 20 lakhs

[b] Only new and viable projects considered

[c] It is implemented by KVIC

[d] Indians above 18 years of age and VIII pass are eligible to apply for loans above Rs 5 lakhs for
services business

[e] 15% subsidy is provided on cost to general category in urban areas

Ans – A (Maximum cost for manufacturing is Rs 20 lakhs)

o The maximum cost of the project/unit admissible in manufacturing sector is ₹ 25 lakhs and
in the business/service sector, it is ₹ 10 lakhs.
Covered in Govt Schemes – Schemes Made Easy

5. Who among the following won the Padma Vibhushan 2021?


Ans – BB Lal

Covered in Spotlight Jan 2021

6. Which of the following statement is incorrect about the electoral bonds?


[a] These bonds can be purchased by citizens of India or entities incorporated in India

[b] It can be purchased in both modes – single and jointly

[c] The validity of these bonds is 10 days

[d] It can be encashed by political parties through their bank A/Cs

[e] None of the Above

15
Ans – C (The validity of these bonds is 10 days)

o The validity is 15 days from the date of issue.

7. 2021 census is the 16th census and ____ after independence?


Ans – 8th

8. Which one of the following is not among the top 5 wheat producing states?
[a] Haryana

[b] Bihar

[c] Madhya Pradesh

[d] Punjab

[e] Uttar Pradesh

Ans – B (Bihar)

9. Which of the following statements is true about Pradhan Mantri Fasal Bima Yojana?
[a] Subsidy will be given to Kharif crops farmers with a premium payable up to 2%

[b] It replaces NAIS and MNAIS

[c] Farmers whose KCC is sanctioned should be compulsorily covered under PMFBY scheme

[d] Difference between the premium paid by farmer and actual insurance premium is shared
between the central and state govt in 70:30 ratio

[e] All of the Above

Ans – B (It replaces NAIS and MNAIS)

Covered in Govt schemes

10. How many southern states are in the top 5 rankings of the innovation index 2020?
Ans – 4

16
Covered in PIB 247 and Spotlight

11. Which global development institution is focused on the private sector in developing
country?
Ans – International Finance Corporation

Covered in Static General Awareness

12. What does ‘A’ stand for in the full form of guard?
Ans – Appropriate Regulation and Supervision

o GUARD, viz., - Governance Oversight, Utile Technology Investment, Appropriate


Regulation and Supervision, Robust Collaboration and Developing necessary IT, cyber
security skills set.
Covered in Spotlight September 2020

13. What is the upper limit per UPI transaction in India?


[a] Rs 2 lakh

[b] Rs 5 lakhs

[c] Rs 0.50 lakh

[d] Rs 3 lakhs

[e] Rs 1 lakh

Ans – E (At present, the upper limit per UPI transaction is Rs.1 Lakh.)

14. What is the objective of PM-WANI scheme?


[a] Healthcare

[b] Essential Services

[c] Food Distribution

17
[d] Internet Connectivity

[e] Payment system

Ans – D (internet connectivity)

Covered in Spotlight December 2020

15. Which of the following sustainable development goal is for gender equality?
Ans – SDG 5

Covered in Spotlight

16. Which of the following is not a strategic sector as identified in the union budget 2021-22?
[a] Banking, Insurance, Financial Services

[b] Transportation and Telecommunication

[c] Power, Petrol, Coal and Mineral

[d] Media and Entertainment

[e] Atomic energy, space and defence

Ans – D (Media and Entertainment)

Covered in Budget

17. Which one of the following is not among the CLMV countries?
[a] Laos, , and

[b] Cambodia

[c] Myanmar

[d] Malaysia

[e] Vietnam

Ans – D (Malaysia)

18
Covered in Spotlight December 2020

18. SLBC is chaired by whom?


[a] Regional Director of RBI

[b] Governor

[c] Chief Minister

[d] Chair / MD of Convenor bank

[e] Deputy commissioner

Ans – D (Chair / MD of Convenor bank)

Master Circular of RBI of July 2020

State Level Bankers’ Committee (SLBC) has been constituted in April 1977.

19. What is the theme of UN international anti-corruption day?


Ans - United Against Corruption / Recover with Integrity

Covered in Spotlight December 2020

20. Bimal Jalan was the governor of RBI from ___ to ___?
[a] 1992 -97

[b] 1997- 2003

[c] 2003-08

[d] 2008-13

[e] 1985- 1990

Ans – B (1997 to 2003)

21. What does CIMS stand for?

19
Ans - Coal Import Monitoring System

Covered in Spotlight December 2020

22. NRO balance can be paid to the account holder if the account has been maintained for
____ months?
Ans – 6 months

RBI Notification 2016

23. Stand-Up India Scheme Facilitates bank loans between _____ to at least one Scheduled
Caste (SC) or Scheduled Tribe (ST) borrower and at least one woman borrower per bank
branch for setting up a greenfield enterprise?
[a] Rs 25-100 Lakhs

[b] Rs 10-100 lakhs

[c] Rs 10 -50 lakhs

[d] Rs 5 to 50 lakhs

[e] Rs 1 to 10 lakh

Ans – B (10 lakh and 100 Lakhs)

Covered in Govt schemes – schemes made easy

24. What is the priority sector lending limit that the bank are mandated to give to start up in the
agricultural and allied activities sector?
Ans - Rs 50 crore

25. How much is the investment of Tesla in Bitcoin in February 2021?


Ans - $1.5 billion

20
26. How many aspirational districts are covered POSHAN 2.0?
Ans – 112 districts (all aspirational districts)

27. Which is the first novel of Amitav Ghosh?


Ans - The Circle of Reasons

28. Which company has got contract from NASA to establish telecom network on moon?
[a] Vodaphone

[b] Nokia

[c] Orange SA

[d] AT&T

[e] Verizon

Ans – B (Nokia)

Covered in Spotlight October 2020

29. Hubble telescope (which is run by NASA and ESA) has found a galaxy. In the month of
January 2021, the telescope captured some images of the galaxy. What is the nickname of
that galaxy?
[a] Dark Galaxy

[b] Spiral Galaxy

[c] Lost Galaxy

[d] Swirling Galaxy

[e] Brown Galaxy

Ans – C (Lost Galaxy)

30. What is the lending target of DFI as per the union budget of 2021-2022?

21
[a] 5 lakh crore in 3 years

[b] 2 lakh crore in 2 years

[c] 4 lakh crore in 2 years

[d] 10 lakh crore in 5 years

[e] 6 lakh crore in 4 years

Ans – A (5 lakh crore in 3 years)

31. What is the minimum age prescribed for players by ICC?


[a] 18 years

[b] 21 years

[c] 15 years

[d] 11 years

[e] 13 years

Ans – C (15 years)

32. Which one of the following is not an indigenous game included in the khelo India youth
games 2021?
Ans – Silambattam

o Gatka, Kalaripayattu, Thang-Ta and Mallakhamba along with Yogasana were added in the
Khelo India Youth Games.
Covered in Spotlight December 2020

33. Wavel Ramkalawan of Indian-orgin was appointed as the President of Seychelles. Where is
Seychelles located?
[a] Arabian Sea

[b] Atlantic Ocean

22
[c] Pacific Ocean

[d] Bay of Bengal

[e] Indian Ocean

Ans – E (Indian Ocean)

34. Which country does Kyle Jamieson belong to?


Ans - New Zealand

35. In which state is Kalanamak rice found?


Ans - Uttar Pradesh

36. Under which scheme are the property cards distributed?


Ans – PM – Svamitva Scheme

Covered in Spotlight, Govt schemes, and PIB 247

37. Where is sur sarovar located?


Ans - Uttar Pradesh

Covered in Spotlight November 2020

38. Which of the following statements is true about Pradhan Mantri Atma Nirbhar Swasth
Bharat scheme?
[a] Investment in Primary, Secondary, Tertiary healthcare

[b] Outlay – Rs 64180 crore

[c] Period of the scheme – 4 years

[d] Addition to NHM

[e] Strengthen existing health institutions and create new

23
Ans – B (Outlay – Rs 64180 crore)

Covered in Budget

39. Kasigo Rabada – the winner of Purple Cap during IPL 2020- has played from which team?
Ans – Delhi Capitals

40. Kasturba Gandhi Balika Vidyalaya is a scheme launched for school girls for SC, ST, OBC
and minority communities for which of the following classes?
Ans – Class 6 to 12

41. What is the duration of the Production-Linked Incentive Scheme?


Ans – FY 2020-21 to FY 2028-29

Covered in Spotlight and PIB 247

42. As of December 2020, which PSU has the largest share of market capitalization?
Ans – Not Available

43. Comprehensive Economic Cooperation and Partnership Agreement has been signed
between India and Mauritius recently. It is the __ edition of such an agreement signed by
India with any African country?
Ans - First

Covered in Spotlight and PIB 247

44. Recently, in a military coup over the failure of Burmese govt to handle Covid-19 pandemic,
the heads of the government including Aung San Suu Kyi were detained. Which of the
following country does not share border with Myanmar?

24
Ans – Vietnam

45. In which strategic sectors will the government maintain a bare number of public sector
undertaking and not divest its stake?
Ans – Not Available

o List of Strategic Sectors - Atomic energy, space and defence; transport and
telecommunications; power, petroleum, coal and other minerals; and banking, insurance
and financial services in non-strategic sectors
Covered in Budget

46. Janet Yellen – the Treasury Secretary of USA – was the earlier chairperson of which
organization?
Ans – US Federal Reserve

47. ISO 9000 is related to which of the following sectors?


Ans – Quality Management and Quality Assurance

48. What is the impact of Atmanirbhar package in monetary terms?


Ans – Rs 20 lakh crores

Covered in Spotlight and PIB 247

49. What is the export value target set by Spices Board of India by 2025 and 2030
respectively?
Ans - $5 Bn; $10 Bn

50. Which state has signed a letter of understanding with UN World Food Program to achieve
the food and nutritional goals?

25
Ans – Rajasthan

Covered in Spotlight November 2020

51. Which bank went live as the financial service provider under the account aggregator
framework of RBI?
Ans – IndusInd Bank

Covered in October 2020

52. Jigmet Dolma from India secured 4th position in the Dhaka Marathon 2021. From which
state does he belong to?
Ans – Ladakh

53. What is India’s rank in the Global Climate Risk Index in 2019 and 2018?
Ans – 7th and 5th respectively

Covered in Spotlight December 2020 & Indices & Reports 2021

54. When is the International Tea Day observed?


Ans – May 21

Covered in Spotlight December 2020

55. Central govt has announced to celebrate Netaji Subhash Chandra Bose’s birth anniversary
as ___ every year on January 23?
Ans – Parakram Divas

Covered in Spotlight January 2021

56. Which one is the only public sector enterprise in the reinsurance sector?

26
Ans – General Insurance Corporation of India (GIC)

57. What is the name of the 2nd offshore petrol vessel given to Indian Coast Guard by the Goa
Shipyard Ltd?
Ans – ICGS Sujeet

OR

What is the name of the 5th and last offshore petrol vessel given to Indian Coast Guard by the Goa
Shipyard Ltd?

Ans – ICGS Saksham

Covered in Spotlight December 2020

58. Recently, Kaja Kallas has become the first female PM of Estonia. In which year did Estonia
get its independence from Soviet Union?
Ans – 1991

Covered in Spotlight January 2021 (but only appointment, capital and currency)

59. What does ‘A’ stand for in the full form of NDEAR, which was envisaged in the Union
Budget 2021-22?
Ans - National Digital Education Architecture (NDEAR)

Covered in Budget

60. Govt has recently sold its 26.12% stake in Tata Communications Ltd, erstwhile VSNL,
through offer for sale. How much percentage did the govt sell in the company in 2002,
along with the management control?
Ans – 25%

Covered in Spotlight February 2021 (divestment is covered; history is not covered)

27
61. After the third five year plan, there was a period of plan holiday (1966-69), during which the
government could not implement the five year plan due to war, inflation, and unrest. The
Fourth Five year plan was introduced in which year?
Ans – 1969

Covered in the course

62. Which one of the following forces is not a part of the Central Armed Police Forces?
Ans – Territorial Army

o There are seven forces under CAPF:


(i) Central Reserve Police Force,
(ii) Central Industrial Security Force
(iii) National Security Guards
(iv) Border Security Force,
(v) Indo-Tibetan Border Police,
(vi) Sashastra Seema Bal,
(vii) Assam Rifles.
Covered in Spotlight march 2020

63. On which bank did RBI impose a penalty of Rs 1 crore for unauthorized ATM operations?
Ans – PNB

Covered in Spotlight November 2020

64. Recently, Vidyut Mohan has won the UNEP’s Young Champion of the Earth Award. How
many Asians won the award in 2020?
Ans – Three

o Seven winners were in total


Covered in Spotlight December 2020

28
65. Which one is the longest bridge of India?
Ans – Dhubri-Phulbari (connecting Assam and Meghalaya)

66. Who is the 2nd most powerful woman after Angela Merkel in the Forbes Most Powerful
Women’s list?
Ans – Christine Lagarde

67. NS for finance education relies on ___? (doubtful question)

INTERPRETATION

A) Level – difficult
B) Type of questions – Statement based long questions
C) UPSC like General Studies Questions were asked.
D) Majority questions were from October, November, December 2020.
E) Questions were asked in depth.
F) Not only the current rankings but previous year rankings, historical facts about companies,
organizations and general studies questions have been asked in both shifts.
G) Questions covered in Morning shift – 45 / 69 (Doable question – 39)
H) Questions covered in afternoon shift – 41 / 67 (Doable questions – 35)

29
Check our website www.anujjindal in for enrolment,
Course details and other updates!

Or

Log on to our mobile application.

30
“HALL OF FAME”

31
RBI

550+ Students cleared RBI Phase 1


300+ Students clear RBI Phase 2
48 Students got selected in RBI

32
SEBI

33
NABARD

34
UGC NET JRF

35
RBI GRADE B 2023 PHASE 1
EXAM ANALYSIS
(AFTERNOON SHIFT)
[Type here]

General Awareness
Q.1)
India’s BSR system needs to be robust, timely, comprehensive and open to change, stated
Michael Patra, deputy governor of Reserve Bank of India (RBI). The BSR Code is a seven-digit
code that is provided to the registered banks by the RBI. The first three digits in the code identify
the bank, while the following four digits identify the bank branch. This code helps banks to keep a
clear record of every online payment made towards tax. BSR system has metamorphosed into a
sound and comprehensive reporting system, generating a wide array of useful statistics.
Combined with bank branch statistics, it has supported the post-nationalisation expansion of the
Indian banking system. In October 2022, the BSR completed __ years of its existence. RBI had
organized an event to celebrate the anniversary.
(a) 50 years
(b) 10 years
(c) 20 years
(d) 30 years
(e) 15 years
Answer – (a)
Explanation –
In 2022, the Basic Statistical Return (BSR) system completed 50 years since its inception. To
commemorate this milestone, the RBI deputy governor delivered a speech at the conference on
‘BSR@50 organised in Mumbai. The exam of RBI Grade B 2023 Phase 1 took place on July 9,
2023. This question is stating that not only the six months prior to the exam but all the events and
initiatives of RBI which are in the news in the past one year are important.

Q.2)
As per the Annual Report of RBI (released in May 2023), the Reserve Bank successfully
completed the combined gross market borrowings of the central and the state governments to the
tune of Rs 21,79,392 crore during 2022-23, which was 19.2 per cent higher than the previous
year. During 2022-23, securities ranging from 2 to ____ years tenor (original maturity) were issued
with the objective of catering to the demand from various investors in different maturity buckets.
Floating Rate Bonds (FRBs) of 7-year and 13- year tenors (original maturity) were also reissued
during the year. The share of FRBs in total issuances during 2022-23 was 2.5 per cent as
compared to 7.8 per cent in the previous year.
(a) 20
(b) 15
(c) 30
(d) 40
(e) 10
Answer – (d)
Explanation –
Annual Report of RBI needs to be covered for RBI’s exam - this is something that goes without
saying. RBI is the regulator of banks in India. Therefore, you can expect questions from this report
in any kind of banking exam at any level - be it assistant, clerk, PO etc. Annual reports of the

pg. 1
[Type here]

regulators (RBI, SEBI, NABARD, IRDAI, PFRDA, and IFSCA) are very crucial. They can be asked
in depth in any kind of exam.

Q.3)
The Global Gender Gap Index annually benchmarks the current state and evolution of gender
parity across four key dimensions (Economic Participation and Opportunity, Educational
Attainment, Health and Survival, and Political Empowerment). It is the longest-standing index
tracking the progress of numerous countries’ efforts towards closing these gaps over time since its
inception in 2006. This year, the 17th edition of the Global Gender Gap Index benchmarks gender
parity across 146 countries, providing a basis for robust cross-country analysis. Further, examining
a subset of 102 countries that have been included in every edition of the index since 2006
provides a large constant sample for time-series analysis. Which country achieved the full gender
parity in this year’s index?
(a) Norway
(b) Iceland
(c) Finland
(d) Denmark
(e) Sweden
Answer – (b)
Explanation –
This question is from the indices and reports topic, which is an integral part of your Phase 1 and
Phase 2 syllabus of RBI. Be prepared for such lengthy questions in future. Try to develop a fast-
reading habit.

Q.4)
Recently, a South American country conferred its highest civilian honour to President Droupadi
Murmu- The Grand Order of the Chain of Yellow Star, the first Indian to receive this award. The
country in question is a small country on the northeastern coast of South America. It's defined by
vast swaths of tropical rainforest, Dutch colonial architecture and a melting-pot culture. It is a
tropical country covered in rainforests. Its extensive tree cover is vital to the country's efforts to
mitigate climate change and maintain carbon negativity. Identify it.
(a) Guyana
(b) Brazil
(c) Suriname
(d) Chile
(e) Peru
Answer – (c)
Explanation –
This question is a very basic current affairs question. The President of our nation has received the
highest civilian honour of another country – a very expected question, isn’t it? The only challenge
here is that the question is lengthy and a little twisted with a lot of unnecessary information.
Therefore, I recommend all of you to read as much as you can to at least complete 65-70 such
long questions in 25 minutes.

Q.5)
pg. 2
[Type here]

Towns of Export Excellence is a status provided to those towns which produce and export goods
worth a minimum value in a specific sector. Sectors include handicraft, handloom, seafood,
pharmaceutical, fisheries, apparels, coir, leather products etc. As per the Foreign Trade Policy,
towns that produce goods worth at least ____ can be recognized as towns of export excellence,
based on their growth potential for exports. However, for Town of Export Excellence (TEE) in
Handloom, Handicraft, Agriculture and Fisheries sector, the threshold limit is____.
(a) Rs 500 crore; Rs 100 crore
(b) Rs 250 crore; Rs 50 crore
(c) Rs 450 crore; Rs 150 crore
(d) Rs 900 crore; Rs 450 crore
(e) Rs 750 crores; Rs 150 crores
Answer – (e)
Explanation –
This question has been asked from the Foreign Trade Policy released in March 2023, w.e.f., 1st
April 2023. This question is an expected question. New towns of excellence were also announced
in this year’s policy and asking the criteria for this category is not a difficult or an unexpected
question. The lesson that all of you should learn is that prepare all the important reports, policy
frameworks, master directions released in the past one year from the date of examination.

Q.6)
According to reports, a trilateral highway connecting Kolkata to ___ will be built and available for
use in four years. Commerce ministers from numerous countries explored this possibility at a
recent meeting held by the Indian Chamber of Commerce (ICC) and the Ministry of External
Affairs (MEA). The highway has immense potential to promote tourism and cultural exchanges
among the countries it connects. This road will facilitate the exploration of historical landmarks and
cultural assets in the countries it connects. The trilateral highway project was first proposed and
approved at a ministerial meeting in April 2002. The 2,800-kilometre-long highway aimed at
boosting multilateral trade.
(a) Hanoi
(b) Bangkok
(c) Vientiane
(d) Naypyidaw
(e) Phnom Penh
Answer – (b)
Explanation –
The trilateral highway project was first proposed and approved at a ministerial meeting of India,
Myanmar and Thailand in April 2002. The highway will be connecting Kolkata to Bangkok via
Myanmar. This is also not an unexpected question. The RBI has developed a tendency to go
beyond the surface. Therefore, it asks the questions from the static initiatives (like this), macro-
economic developments, and overarching scenarios/topics. Try to cover every topic holistically.

Q.7)
During the visit of COP 26 President Hon’ble Alok Sharma to India on 21-22 July 2022, NITI
Aayog launched two significant initiatives, which are ____ mobile application to raise awareness
on electric mobility and the Report on Advanced Chemistry Cell Battery Reuse and Recycling
pg. 3
[Type here]

Market in India. To bolster this low carbon infrastructure, NITI Aayog has been collaborating with
the UK Government in the areas of e-vehicles, charging infrastructure and battery storage. India is
the fifth largest and fastest growing vehicle market in the world, providing massive potential for
electric vehicle uptake. The shift to Zero Emission Vehicles (ZEVs) is well underway and
accelerating, creating 100,000s of new e-mobility jobs, driving down technology costs, ensuring
the air we breathe is cleaner and reducing our reliance on imported fuels.
(a) e-Amrit
(b) e-Change
(c) e-Speed
(d) e-Mile
(e) e-Run
Answer – (a)
Explanation –
Again, the question has been asked from an initiative which has macro-economic implications.
India is working towards net zero-emissions, SDG attainment, energy transition, climate change
mitigation, and e-vehicles. Considering the focus of India, this question is relevant and not
unexpected. So, not only facts but do keep an eye on the policies of the govt which can become a
question in your exam (like this), even if the intiative or campaign was launched 2-3-5 years back,
doesn’t matter.

Q.8)
According to the Economic Survey of 2022-23, the Indian economy appears to have moved on
after its encounter with the pandemic, staging a full recovery in FY22 ahead of many nations and
positioning itself to ascend to the pre-pandemic growth path in FY23. Yet in the current year, India
has also faced the challenge of reining in inflation that the European strife accentuated. Measures
taken by the government and RBI, along with the easing of global commodity prices, have finally
managed to bring retail inflation below the RBI upper tolerance target in November 2022. Despite
these, agencies worldwide continue to project India as the fastest-growing major economy with a
growth rate falling in an average range of _____ and _____ for the year ending ___.
(a) 6.5-7.0 per cent; March 31, 2023
(b) 7.0-7.5 per cent; March 31, 2022
(c) 6.4-6.7 per cent; March 31, 2025
(d) 8.1-8.5 per cent; March 31, 2023
(e) 8.2-9.0 per cent; March 31, 2023
Answer – (a)
Explanation –
This question is from the latest Economic Survey. It is a very important topic of your current affairs
syllabus for almost every type of exam. So, in-depth coverage of such reports is important to clear
this exam.

Q.9)
Cruise tourism has seen an immense surge in recent years in India and worldwide. Seeing the
potential of this sector, India, on June 5, inaugurated its first international cruise vessel, ____.
Flagged off from _____ to Sri Lanka, the cruise will cover three ports during the 5-day cruise. This
also means that Indians no longer need to go abroad to board a ship to get international cruise
pg. 4
[Type here]

experience. Inaugurating the cruise, Union Minister of Ports, Shipping & Waterways Sarbananda
Sonowal called it a historic step for the maritime sector, adding that the initiative will further boost
tourism and generate employment.
(a) MV Kirpan; Bengaluru
(b) MV Empress; Chennai
(c) MV Imperial Blue; Mysore
(d) MV Neon; Mumbai
(e) MV Prestige; Surat
Answer – (b)
Explanation –
On the World Environment Day on June 5, 2023, India has launched its first international cruise.
The exam was held on July 9, 2023. One month prior to the exam date, this news came out. All
the news coming in the 4 months prior to the exam become very important.

Q.10)
TRAI in its endeavour to curb menace of spams through Unsolicited Commercial Communication
(UCC) has taken various measures in recent past such as cleaning data in respect of PE Headers
and message content templates, tagging variables in message content templates etc. In
continuation of above actions, TRAI has now issued a Direction to all the Access Providers to
develop and deploy the DCA facility for creating a unified platform and process to register
customers consent digitally across all service providers and Principal Entities. What is the full form
of DCA?
(a) Digital Content Amalgamation
(b) Demand-based Content Acquisition
(c) Derivative Consent Acquiring
(d) Digital Consent Acquisition
(e) Digital Content Admission
Answer – (d)
Explanation –
This news also came out in June 2023. That’s why it has been asked in RBI’s exam. In such type
of guidance or policies, in-depth coverage is not required because telecom sector is not
mainstream to the RBI’s working. Therefore, you can just cover the main points of such guidelines.
These guidelines were entirely focused on DCA facilities.

Q.11)
Mr. Girish Chandra Murmu, the Comptroller and Auditor General of India (CAG), has been re-
elected as External Auditor of the ______ for a ____ year term. CAG’s appointment is a
recognition of its standing among the international community as well as its professionalism, high
standards, global audit experience and strong national credentials. This is the second major
international audit assignment for the CAG this year following his selection for the post of External
Auditor (2024-2027) of International Labour Organization (ILO) in Geneva earlier this year in
March 2023.
(a) International organization for migrants; 2 years
(b) World Bank; 2 years
(c) World Health Organization; 4 years
pg. 5
[Type here]

(d) International Organization of Supreme Audit Institutions; 5 years


(e) FATF; 2 years
Answer – (c)
Explanation –
This is a May 2023 news. Often students ignore the appointments, awards, sports, and
miscellaneous sections of the Spotlight magazine, which, in my opinion, is a very big mistake an
aspirant commit. The consequence is in front of you, in the form of this question. So, for every type
of exam, the recommendation is to cover these sections thoroughly. They are equally important as
the international, national, banking news sections.

Q.12)
According to Morgan Stanley, India is already the fastest-growing economy in the world with three
megatrends in the country, global offshoring, digitalization, and energy transition putting the
country on the path to unprecedented economic growth and that India will be one of only three
economies in the world that can generate more than $400 billion annual economic output growth
from 2023 onward which will rise to more than $500 billion after 2028. In light of this, India’s
participation in various international forums renders an opportunity to the South Asian nation to put
forth its interest and participate in the decision-making processes of these forums. India is a
member of various international organizations; it is still endeavouring to become member of other
forums. Name the forum which does not have India as its member.
(a) SAARC
(b) IPEF
(c) SCO
(d) OECD
(e) WEF
Answer – (d)
Explanation –
This is a static GK question. Just knowing the total number of countries of international and
regional organizations would not serve your purpose now. You also have to look for India’s
membership and role in the international organizations.

Q.13)
According to the data collated by Department for Promotion of Industry and Internal Trade (DPIIT),
incentives have been given to PLI beneficiaries of the 14 sectors. Of the 14 sectors, only 8 have
performed well and six have not. Which is not amongst six non-performing sectors?
(a) Telecommunications
(b) Steel
(c) Textile
(d) White Goods
(e) Battery
Answer – (a)
Explanation –
According to data collated by the Department for Promotion of Industry and Internal Trade (DPIIT),
incentives have been given to PLI beneficiaries in eight of the 14 sectors — mobile manufacturing,

pg. 6
[Type here]

information technology (IT) hardware, pharmaceutical drug, bulk drug, medical device,
telecommunications, food product, and drone.
Progress has been sluggish for the remaining six sectors — steel, textile, battery, white goods,
solar photovoltaic, and automotive — and are yet to receive incentives.
This question is from the govt schemes. The news article was published in June 2023. Hence, it
has all the reasons to be a part of your question paper. Not only the basics about the scheme but
the current news about it should also be covered by you.

Q.14)
What makes a country worth living is not just economic hegemony and achievements of the state,
it is the wealth of the culture of the people, accumulated over centuries of wisdom about how to
conduct life. The ancient Indian civilisation is the source of all the extensions of what we call as the
Indian subcontinent. The geographic spread covered under this banner stretches far towards the
Gulf on one side and towards the expanse of Indo-Pacific on the other. In the present scenario,
countries like India and China are jostling for influence over the ports and island nations of the
Indian Ocean. The pivot of a major chunk of economics in the extended neighbourhood is the
Indian Ocean because many resources like oil, gas, raw materials and goods pass through it.
Over a period of time, for political, economic and security reasons, various blocks and groupings
were formed for cooperation on several fronts. The two main groupings we have in the Indo-
Pacific region are ASEAN and BIMSTEC. Which of the following countries are common members
of these two regional blocs?
(1) Thailand
(2) Cambodia
(3) Myanmar
(4) Laos
(5) Vietnam
(a) 1 and 2 Only
(b) 2 and 3 Only
(c) 1 and 3 Only
(d) 3 and 4 Only
(e) 4 and 5 Only
Answer – (c)
Explanation –
Again, a question from the static GK and, that too, from international/regional organizations. This
topic is also important for the Phase 2 ESI paper. Although, RBI has created an ESI-level question
in the Phase 1 only. Nevertheless, cover the international and regional organizations thoroughly,
especially those which are in news.

Q.15)
Since the implementation of monetary policy framework in 2016, the inflation rate has crossed
upper threshold limit various times. The RBI informs central government if the inflation rate
crosses upper threshold limit of ……………….. for ………………… quarters?
(a) 4%, 2 Quarters
(b) 6%, 3 Quarters
(c) 3%, 2 Quarters
pg. 7
[Type here]

(d) 5%, 3 Quarters


(e) 6%, 2 Quarters
Answer – (b)
Explanation –
This question highlights the need for you to dig deep into the functioning of RBI.

Q.16)
Major private sector banks continued to expand branch network in the financial year ended March
2023, while public sector banks (PSB) saw a decline in number of branches. Which of the
following scheduled commercial banks has the highest increase in the number of branches in FY
2022-23?
(a) Kotak Mahindra
(b) ICICI
(c) HDFC
(d) Bandhan Bank
(e) SBI
Answer – (c)
Explanation –
1st - Largest private lender HDFC Bank added nearly 1,500 branches, closing the year with over
7,800, making it the largest number of new branch additions by a scheduled commercial bank in
the country.
2nd – Axis Bank had 4,903 domestic branches as on March 2023, compared to 4,758 in the year-
ago period. (Addition of 200 branches approx.)
3rd – State Bank of India added only 139 new branches during the last financial year. As of March
2023, SBI had 22,405 branches, compared to 22,266 in the year-ago period.
As per RBI data, the number of public sector bank branches has been decreasing steadily in
recent years while it is increasing for private banks. From 87,892 in March 2020, the number of
PSB branches fell to 86,311 in FY21 and 84, 258 in FY22. On the other hand, the number of
private bank branches increased from 34,794 in FY20 to 37,872 in FY23.

Q.17)
The Digital Payment Index (DPI) is a metric used to measure the adoption and growth of digital
payments within a country. It provides an assessment of the digital payment ecosystem, taking
into account various parameters such as the number of digital transactions, the value of digital
payments, the availability of digital payment infrastructure, and the level of consumer awareness
and confidence in using digital payment methods. What is the base period of the digital payment
index?
(a) March 2018
(b) March 2017
(c) September 2018
(d) September 2017
(e) March 2019
Answer – (a)
Explanation –
The base period is March 2018.
pg. 8
[Type here]

Q.18)
RBI has been organizing various public awareness campaigns for promoting financial literacy and
safe banking practices. Which of the following mascots are used by RBI for public awareness
campaigns?
1) Money Kumar
2) Ms. Money
3) Mr. Money
4) Mrs. Paisa
5) Ms. Paisa
(a) Only 1 and 4
(b) Only 2, 3 and 4
(c) Only 1, 2 and 4
(d) Only 1 and 2
(e) Only 3 and 4
Answer – (d)
Explanation –
Again, a question from RBI’s functioning. This is taken from the Annual Report of RBI.

Q.19)
e-way bill is a document required to be carried by a person in charge of the conveyance carrying
any consignment of goods of value exceeding ______ as mandated by the Government in terms
of section 68 of the Goods and Services Tax Act read with rule 138 of the rules framed
thereunder. It is generated from the GST Common Portal by the registered persons or transporters
who causes movement of goods of consignment before commencement of such movement.
(a) 25000
(b) 50000
(c) 12500
(d) 75000
(e) 35000
Answer – (b)
Explanation –
This is an important banking and economy section question. You should cover all such news
thoroughly.

Q.20)
According to union budget 2023-24, 50-years interest free loans is given to states for
……………… In budget 2023-24, the total amount sanctioned has been increased to Rs.1.3 Lakh
Crores from ………………… in the previous year.
(a) CAPEX, 1 Lakh Crore
(b) Investment, 50K Crore
(c) Subsidy, 1 Lakh Crore
(d) Credit Guarantee, 35000 Crore
(e) CAPEX, 50000 Crore
Answer – (a)
pg. 9
[Type here]

Explanation –
Union Budget is an integral part of the syllabus. Cover it thoroughly for every phase of the exam.

Q.21)
Finastra has partnered with a bank to support the latter’s new integrated corporate banking portal,
‘FYN’, using Finastra’s Unified Corporate Portal solution. The Unified Corporate Portal harnesses
the power of Finastra’s Corporate Channels framework, enabling banks to build a seamless
experience for their corporate clients spanning account services, payments, trade, supply chain
finance and lending. Banks can unify data across multiple, product-specific portals and back-office
systems to create a single view of transactions, positions and balances, enabling self-service and
driving operational efficiency. The data is presented via persona-based dashboards, with insightful
widgets and the ability for users to tailor their experience. Which bank has launched this ‘FYN’
portal?
(a) HDCF
(b) Kotak Mahindra Bank
(c) Bandhan Bank
(d) ICICI
(e) SBI
Answer – (b)
Explanation –
Bank-related detail questions are usually asked in this exam. So, remember major portals of the
banks. This news was released in October 2022.

Q.22)
Prudential guidelines aim to address credit risk concentration in non-banking financial companies
(NBFCs). These guidelines focus on identifying and managing large exposures, which refer to the
sum of all exposure values of an NBFC-UL (an NBFC placed in the upper layer) to a counterparty
or group of connected counterparties. The guidelines outline the limits on exposure of Non-
Banking Financial Companies – Upper Layer (NBFC-UL) to single counterparties and groups of
connected counterparties. For single counterparties, the sum of all exposure values of an NBFC-
UL must not be higher than ___ percent of the NBFC-UL’s eligible capital base at all times.
(a) 15%
(b) 8%
(c) 20%
(d) 15%
(e) 12%
Answer – (c)
Explanation –
This is from the policy guidelines of RBI for NBFCs. Do cover all the policy guidelines, circulars,
and master directions of RBI released in the last one year for both the phases.

Q.23)
The Indian Council of Agricultural Research (ICAR) and an e-commerce platform have signed a
Memorandum of Understanding (MoU) to combine strengths and create synergy between the two
organizations for guiding farmers on scientific cultivation of different crops for optimum yield and
pg. 10
[Type here]

income. Under the MoU, ICAR will provide technological backstopping to the farmers through the
network of the e-commerce platform. This will help improve farmers' livelihood and boost crop
yield. Identify the platform.
(a) Amazon
(b) Flipkart
(c) Reliance
(d) Snap Deal
(e) E-Biz
Answer – (a)
Explanation –
Such type of news is covered in the Spotlight magazine. MOUs are also important and this
question is evidence of it. So, cover the MOUs as well.

Q.24)
The European Central Bank (ECB) is the central bank for the eurozone, the group of nineteen
countries who use the euro common currency. Its mandate is to maintain price stability by setting
key interest rates and controlling the union’s money supply. The 1992 Maastricht Treaty created
the European System of Central Banks (ESCB), which comprises the ECB and the twenty-eight
national central banks of the European Union (EU), including those from countries that do not use
the euro. Under the ESCB sits the Euro system, which comprises the ECB and the national central
banks of eurozone countries. The ECB took over responsibility for monetary policy in the euro
area in 1999, two years before the euro was introduced into circulation. Recently, the ECB, which
is headquartered in ___, celebrated its ___ anniversary.
(a) Geneva, 50th
(b) Lyon, 25th
(c) Frankfurt, 25th
(d) Madrid, 75th
(e) Barcelona, 60th
Answer – (c)
Explanation –
Established: 1 June 1998
Location: Frankfurt (Germany)
Again, a question from the international organization. Since, it was in news due to its anniversary,
that’s why, this question has been asked.

Q.25)
A bull market is a period of time in financial markets when the price of an asset or security rises
continuously. The commonly accepted definition of a bull market is when stock prices rise by 20%.
Traders employ a variety of strategies, such as increased buy and hold and retracement, to profit
off bull markets. The opposite of a bull market is a bear market, when prices trend downward.
Recently, the Indian stock market also witnessed a bull run, whereby an Indian company’s stock
price per share has crossed the Rs 1 Lakh valuation, for the first time in the history of Indian stock
market. Identify the company.
(a) Titan
(b) Reliance
pg. 11
[Type here]

(c) Adani
(d) HDFC
(e) MRF
Answer – (e)
Explanation –
Economy related news are always asked in this exam. This news was published in June 2023.

Q.26)
The goals and vision of the RBI are categorized in the Payments Vision 2025 documents into five
anchor goalposts – Integrity, Inclusion, Innovation, Institutionalization and Internationalization. The
theme of Payment Vision 2025 was “E-Payments for …………………., Everywhere and
………………”
(a) Everyone, Every time
(b) Every time, Everyone
(c) Everyday, Everyone
(d) Every day, Every year
(e) Every Year, Every Day
Answer – (a)
Explanation –
The Vision document was released in Jan 2023. It is one of the key documents released by RBI.
Therefore, you should cover all such documents of RBI.

Q.27)
In June 2023, a financial inclusion dashboard named ‘Antardrishti’ was launched. The dashboard,
as its name suggests, provides the necessary knowledge to evaluate and track the development
of financial inclusion by recording relevant data. This tool also makes it possible to assess the
degree of financial exclusion at a local level across the nation so that such places may be
addressed. Which of the following organizations has launched the dashboard?
(a) NABARD
(b) RBI
(c) SIDBI
(d) SEBI
(e) NHB
Answer – (b)
Explanation –
This is also a very easy (if we ignore the length of the question) and expected question. In June
month only, RBI had launched this initiative.

Q.28)
El Niño and La Niña are opposite phases of what is known as the El Niño-Southern Oscillation
(ENSO) cycle. Indian Ocean ________, sometimes referred to as the Indian Nino, is similar to the
El Nino phenomenon, occurring in the relatively smaller area of the Indian Ocean between the
Indonesian and Malaysian coastline in the east and the African coastline near Somalia in the west.
(a) Current
(b) Dipole
pg. 12
[Type here]

(c) Springs
(d) Nino
(e) Nina
Answer – (b)
Explanation –
The Indian Ocean Dipole (IOD) is defined by the difference in sea surface temperature between
two areas (or poles, hence a dipole) – a western pole in the Arabian Sea (western Indian Ocean)
and an eastern pole in the eastern Indian Ocean south of Indonesia. The IOD affects the climate
of Australia and other countries that surround the Indian Ocean Basin, and is a significant
contributor to rainfall variability in this region. Lately, questions from geography have become a
favourite of RBI. So, keep a track over news related to countries’ geographies and basic
geography facts.

Q.29)
The Union Minister Education, Skill Development and Entrepreneurship, Shri Dharmendra
Pradhan released the India Rankings 2023 in June 2023. Which of the following statements is
incorrect with respect to the NIRF rankings?
(a) Indian Institute of Technology Madras retains its 1st position in Overall Category for fifth
consecutive year
(b) Indian Institute of Science, Bengaluru tops the Universities Category
(c) All India Institute of Medical Sciences (AIIMS), New Delhi occupies the top slot in Medical
(d) National Law School of India University, Bengaluru retains its first position in Law
(e) St. Stephens is the best college among all colleges
Answer – (e)
Explanation –
This is from the indices and reports topic. It is very important and needs to be covered.

Q.30)
The Reserve Bank of India publishes half-yearly reports on management of foreign exchange
reserves as part of its efforts towards enhanced transparency and levels of disclosure. These
reports are prepared half yearly with reference to the position as at end-March and end-
September each year. The present report (40th in the series) is with reference to the position as at
end-March 2023. Among other topics, the report identifies the risks associated with the
deployment of foreign exchange reserves. Identify the risk which affect the forex deployment.
1) Operational
2) Internal
3) Market
4) Currency
5) Liquidity
(a) 1, 2, and 4
(b) 2, 3, and 5
(c) 1, 3, and 4
(d) 3 and 4
(e) 1, 3, 4, and 5
Answer – (e)
pg. 13
[Type here]

Explanation –
The report “Half Yearly Report on Management of Foreign Exchange Reserves (October 2022 -
March 2023)” was released in the month of May 2023. The risks on deployment of forex reserves
include credit risk, market risk, liquidity risk and operational risk and the systems.

Q.31)
Which of the following companies’ majority of assets under management (AUM) are invested in
central government securities, followed by equities and state government securities? Out of the
total, 37.45 percent, or Rs 14.81 trillion are invested in central government securities as of
September. Around 24.77 percent of the AUM, or Rs 9.80 trillion, have been invested in equities
as of September while in FY21 total equity investment was worth Rs 7.97 trillion, or 21.67 percent
of its total AUM.
(a) LIC
(b) NHB
(c) SIDBI
(d) SBI
(e) HDFC
Answer – (a)
Explanation –
This information was revealed in the draft red herring prospectus of LIC (submitted in February
2022).

Q.32)
Foreign Trade Policy (2023) is a policy document which is based on continuity of time-tested
schemes facilitating exports as well as a document which is nimble and responsive to the
requirements of trade. It is based on principles of 'trust' and 'partnership' with exporters. Which of
the following are the Pillars of India’s Foreign Trade Policy 2023?
1) Incentives to Remission
2) Export promotion through collaboration
3) Ease of Doing Business
4) Emerging areas
(a) Only 1, 3 and 4
(b) Only 3 and 4
(c) All 1, 2, 3 and 4
(d) Only 1, 2 and 4
(e) Only 1, 2 and 3
Answer – (c)
Explanation –
FTP is an important policy document. The question is easy because it is not an in-depth question
as such.

Q.33)
Under SEBI (Mutual Funds) Regulations, 1996, Mutual Funds are permitted to charge certain
operating expenses for managing a mutual fund scheme – such as sales & marketing / advertising
expenses, administrative expenses, transaction costs, investment management fees, registrar
pg. 14
[Type here]

fees, custodian fees, audit fees – as a percentage of the fund’s daily net assets. All such costs for
running and managing a mutual fund scheme are collectively referred to as ‘Total Expense Ratio’
(TER). It is calculated as a percentage of scheme’s average ………………….?
(a) Net Asset Value
(b) Net owned fund
(c) Liabilities
(d) Operational Expenses
(e) Investment
Answer – (a)
Explanation –
The question is again forcing you to go into the depth of the news and understand the static
concept/mechanism behind the news or topic.

Q.34)
Household savings play a vital role in India's financial sector for multiple reasons. Firstly, they
provide a stable and reliable source of funds for the banking sector. Banks utilize these savings as
deposits, which they can lend to businesses and individuals, promoting economic activities and
investments. Secondly, household savings contribute to the overall liquidity and stability of the
financial system, reducing the dependence on external sources of funding. The accumulation of
household savings can lead to increased domestic savings rates, which is vital for reducing
reliance on foreign capital and achieving sustainable economic development. The Annual Report
of the RBI describes the landscape of the household sector’s savings in detail. Which of the
following financial instruments constituted the highest share in the household sector’s savings in
2022-23?
(a) Currency
(b) Deposits
(c) Shares and Debentures
(d) Insurance Funds
(e) Provident and Pension funds
Answer – (b)
Explanation –
As per the Annual Report, the distribution of the household savings is given below.

Q.35)
CBDC has the potential to bring significant changes to the economy and the financial sector. It can
enhance financial inclusion by providing access to digital payment services for unbanked
populations. Additionally, CBDCs can streamline payment systems, reduce transaction costs,
increase transparency, and enable central banks to implement monetary policies more effectively.
Which of the following statements is correct regarding pilot launch of CBDC in India?
(a) Pilot of whole sale segment was launched on 1st December 2022
(b) Pilot of whole sale segment was launched on 1st November 2022
(c) Pilot of both whole sale and retail segment was launched on 1st December 2022
(d) Pilot of retail segment was launched on 1st January 2023
(e) Pilot of retail segment was launched on 1st November 2022
pg. 15
[Type here]

Answer – (b)
Explanation –
Again, all the decisions which can have macro-economic or pan-India implications are important
for this exam.

Q.36)
Small savings schemes are of significant importance in India due to several reasons. They
promote a savings culture among individuals, fostering financial stability. These schemes offer
safe and accessible investment options, especially for small investors in rural and semi-urban
areas. Additionally, they provide attractive interest rates and secure returns, making them a
popular choice for risk-averse investors. Which of the following small savings scheme is providing
highest interest rate (8.2%) for quarter starting from 1st July and ending on 30th September?
(a) Senior Citizen Savings Scheme
(b) Kisan Vikas Patra
(c) National Savings Certificate
(d) PPF
(e) Post office time deposit
Answer – (a)
Explanation –
The rates were updated on June 30, 2023. The rates of the highest paying schemes are –

Q.37)
MAVEN is a NASA spacecraft orbiting Mars to study the loss of that planet's atmospheric gases to
space, providing insight into the history of the planet's climate and water. What does “V” stand for
in MAVEN?
(a) Visible
(b) Viable
(c) Volatile
(d) Visibility
(e) Volatility
Answer – (c)
Explanation –
MAVEN (Mars Atmosphere and Volatile EvolutioN) mission was launched in November 2013 and
entered Mars’ orbit in September 2014. The mission’s goal is to explore the planet’s upper
atmosphere, ionosphere, and interactions with the Sun and solar wind to explore the loss of the
Martian atmosphere to space. It was in news in the month of June 2023.

Q.38)
The Atlantic Declaration and accompanying Action Plan form the basis of a new type of innovative
partnership across the full spectrum of ___ and ___ counties’ economic, technological,
commercial and trade relations; a first of its kind, and which demands the joint leadership and
imagination to realize in full. It will constitute a new economic security framework covering ever-
closer cooperation on critical and emerging technologies and stronger protective toolkits. And it
will also strengthen our alliance across defense, science, health security, and space.
(a) USA and UK
pg. 16
[Type here]

(b) Netherlands and USA


(c) USA and India
(d) Australia and USA
(e) Australia and Japan
Answer – (a)
Explanation –
This is also a June 2023 news.

Q.39)
Net Owned Funds (NOF) is a term commonly used in the context of financial institutions,
particularly non-banking financial companies (NBFCs) in India. It refers to the difference between
the total assets and the outside liabilities of an institution. In simple terms, NOF represents the net
worth or capital of the financial institution, including the shareholders' equity and accumulated
reserves. It indicates the financial strength and stability of the institution and is an important
parameter for regulatory compliance and risk assessment. The Reserve Bank of India (RBI) had
fixed the minimum Net Owned Fund (NOF) size for housing finance companies to ___?
(a) 25 Crores
(b) 50 Crores
(c) 100 Crores
(d) 100 Lakhs
(e) 25 Lakhs
Answer – (a)
Explanation –
This guideline was released in October 2020. As per the guidelines, the housing finance
companies (HFCs) holding a Certificate of Registration (CoR) and having an NOF of less than Rs
25 crore will be required to achieve NOF of Rs 15 crore by March 31, 2022 and Rs 25 crore by
March 31, 2023. Since, the date of effect was March 2023, that’s why, this question has been
asked from you.

Q.40)
B20 refers to the Business 20, which is an engagement group that brings together business
leaders from the G20 member countries. It serves as a platform for dialogue between the business
community and G20 policymakers to discuss global economic challenges and provide
recommendations for policy formulation. The B20 plays a crucial role in representing the interests
of the private sector and providing valuable insights on key issues such as trade, investment,
infrastructure, digitalization, and sustainable development. Who chaired the B20 grouping under
India’s G20 Presidency?
(a) Mukesh Ambani
(b) Suresh Kotak
(c) N. Chandrasekharan
(d) Anil Pradhan
(e) Amitabh Bhattacharya
Answer – (c)
Explanation –

pg. 17
[Type here]

Although the news of appointment was in December, the meetings of B20 took place in the later
months. Hence, this question is neither difficult nor unexpected.

Q.41)
Union Cabinet has approved constitution and empowerment of Inter-Ministerial Committee under
the Chairpersonship of Minister of Cooperation for facilitation of the “World’s Largest Grain
Storage Plan in Cooperative Sector”. This facilitation will be achieved by convergence of various
schemes of which of the following ministries?
1) Ministry of Agriculture and Farmers Welfare
2) Ministry of Commerce and Industry
3) Ministry of Consumer Affairs, Food and Public Distribution
4) Ministry of Chemicals and Fertilizers
5) Ministry of Food Processing Industries
(a) 1, 2 and 3 Only
(b) 2, 3 and 4 Only
(c) 3, 4 and 5 Only
(d) 2, 3 and 5 Only
(e) 1, 3 and 5 Only
Answer – (e)
Explanation –
This question is from the May 2023 news. Again, an expected and easy question, keeping aside
the length.

Q.42)
The significant expansion of digital lending in India drove RBI to issue the Digital Lending
guidelines (DL guidelines) in September 2022. The DL guidelines struck a balance between the
need for an innovative and inclusive system of lending and protecting the customer’s interest. The
DL guidelines focused on three main areas: Customer protection and conduct requirements;
Technology and data requirement; and Regulatory framework. Now, in 2023, the RBI has released
the guidelines on DLG in digital lending. Identify the correct statement with regards to the DLG
guidelines.
1) RBI has allowed default loss guarantee (DLG) arrangement among banks, NBFCs, and lending
service providers in the digital lending space
2) The FLDG is an arrangement whereby a third party such as a financial technology (fintech)
player (LSP) compensates lenders if the borrower defaults.
3) The RBI has allowed banks to accept DLG in digital lending only if the guarantee is in the form
of a cash deposit, or fixed deposits in a bank with a lien in favour of the RE, or a bank guarantee in
favour of the RE.
4) Banks and NBFCs should ensure that the total amount of DLG cover on any outstanding
portfolio does not exceed 5% of the amount of that loan portfolio.
5) The guidelines segregate the entities working in the digital lending sector into five categories
based on their FLDG exposures.
(a) Only 1, 2 and 5
(b) Only 2, 4, and 5
(c) Only 1, 2, 3 and 4
pg. 18
[Type here]

(d) Only 2, 4 and 5


(e) Only 1, 3, 4, and 5
Answer – (c)
Explanation –
The fifth statement is wrong. No such information was unveiled in the guidelines.

Q.43)
As per the RBI’s Annual Report, _____ witnessed an improvement, reflecting the higher degree of
transmission of monetary policy to lending rates than to deposit rates in the rising interest rate
cycle.
(a) CASA
(b) LCR
(c) PCR
(d) CRAR
(e) NIM
Answer – (e)
Explanation –
The Net Interest Margin is the correct answer.

Q.44)
Macroeconomic indicators are of utmost importance as they serve as essential barometers for
understanding the overall economic health and performance of a country. These indicators, such
as GDP growth rate, inflation rate, unemployment rate, and trade balance, provide valuable
insights into the state of the economy, helping policymakers and analysts make informed
decisions. In this regard, the Reserve Bank has been conducting the survey of professional
forecasters (SPF) on macroeconomic indicators since September 2007. The latest survey has
kept the real gross domestic product (GDP) growth forecast for 2023-24 has been retained at
____ per cent; it is expected to grow by ____ per cent in 2024-25.
(a) 6% and 6.4%
(b) 5.7% and 6.2%
(c) 4.9% and 7.2%
(d) 7% and 6.4%
(e) 6.9% and 7.3%
Answer – (a)
Explanation –
The responses, for 82nd round of the survey, were received during May 13-June 2, 2023, wherein
thirty-nine panellists participated. This is again from RBI’s recent reports.

Q.45)
The legislative assembly holds immense importance in a federal structure as it serves as a
representative body that reflects the will and aspirations of the people within a particular state or
region. Through elected representatives, it provides a platform for citizens to voice their concerns
and interests, ensuring their representation in the decision-making process. The legislative
assembly plays a crucial role in enacting laws and regulations, which govern various aspects of
the state or region. It acts as a forum for debates and discussions, facilitating the formulation of
pg. 19
[Type here]

policies that address the diverse needs and challenges faced by the population. Which state’s
term of legislative assembly will not end till January 2024?
(a) Rajasthan
(b) Chhattisgarh
(c) Madhya Pradesh
(d) Haryana
(e) Telangana
Answer – (d)
Explanation –
The scope of RBI’s exam is increasing. Questions picked up from elections, constitution, and
sometimes, acts are asked. Keep a tight watch over the news. Many states are going to witness
elections in end of this year.

Q.46)
NHB RESIDEX, India’s first official housing price index, was an initiative of the National Housing
Bank (NHB) undertaken at the behest of the Ministry of Finance, Government of India. The index
was formulated under the guidance of a Technical Advisory Committee (TAC) comprising of
stakeholders from the housing market. The scope has been widened under NHB RESIDEX brand,
to include housing price indices (HPI), land price indices (LPI) and building materials price indices
(BMPI), and also housing rental index (HRI). What is the latest base year for calculating the
housing price index?
(a) 2015-16
(b) 2008-09
(c) 2012-13
(d) 2011-12
(e) 2017-18
Answer – (e)
Explanation –
With effect from April 2018, a new series with FY2017-18 as new base year has been published
and is updated up to the current quarter. This question is a little bit unexpected. Although, this
index is released periodically.

Q.47)
Planned urbanization is necessary to accommodate population growth, promote economic
development, and ensure sustainable and inclusive cities. It allows for efficient allocation of
resources, proper infrastructure, and improved quality of life for urban residents. Without planned
urbanization, cities can face challenges such as overcrowding, inadequate services, and
environmental degradation. In this regard, the Government of India has launched a host of
schemes to work towards planned urbanization. Identify the schemes which were launched, in this
regard, in the year 2015.
1) AMRUT
2) Smart Cities
3) PMAY-Urban
4) Swachh Bharat Mission
5) DAY-NULM
pg. 20
[Type here]

(a) Only 1, 2, and 3


(b) Only 2, 4, and 5
(c) Only 1 and 3
(d) Only 2 and 5
(e) Only 2, 3, and 5
Answer – (a)
Explanation –
A basic question from schemes has been asked.

Q.48)
Recently, the capital markets regulator Sebi proposed to tweak the current definition of
unpublished price sensitive information (UPSI) to bring regulatory certainty and uniformity in
compliance for the listed companies in respect of the identification of certain events as UPSI. The
regulator suggested that the current definition of UPSI be amended, and the disclosures as
required under Regulation 30 of LODR (Listing Obligations and Disclosure Requirements) be
brought under it. Which of the following information will be deemed as ‘material information’ once
the regulation 30 of LODR is applied on UPSI?
1) merger and acquisition
2) fraud or default by promoters
3) change in director and key managerial personnel
4) board meeting pertaining to dividends and financial results
5) initiation of forensic auditing
(a) Only 1 and 2
(b) Only 2 and 3
(c) Only 1, 2, 3, 4, and 5
(d) Only 1, 2, 3, and 4
(e) Only 2, 3, and 4
Answer – (c)
Explanation –
This consultation paper was released in May 2023. The question is in-depth. Policies of the
regulators need to be covered thoroughly.

Q.49)
Indian-origin professor Joyeeta Gupta is among the two scientists who have been named for the
prestigious Spinoza Prize, also known as the Dutch Nobel Prize, given by the Dutch Research
Council. Gupta, the faculty professor of Sustainability and professor of Environment and
Development in the Global South at the University of Amsterdam, received the prize for her
outstanding, pioneering, and inspiring scientific work in which she focuses on a just and
sustainable world. Name the award.
(a) Spinoza Prize
(b) Wolf Prize
(c) Breakthrough Prize
(d) Fields Medal
(e) Turing Award
Answer – (a)
pg. 21
[Type here]

Explanation –
The news was released in June 2023.

Q.50)
The 2023 Malaysia Masters was a badminton tournament that took place at the Axiata Arena,
Kuala Lumpur, Malaysia, from 23 to 28 May 2023 and had a total prize of US $420,000. What is
the category of this tournament?
(a) Super 500
(b) Super 1000
(c) Super 750
(d) Super 300
(e) Super 100
Answer – (a)
Explanation –
Again, a question from the May 2023 news.

Q.51)
FSSAI has developed State Food Safety Index to measure the performance of states on various
parameters of Food Safety. This index is based on performance of State/ UT on five significant
parameters, namely, Human Resources and Institutional Data, Compliance, Food Testing –
Infrastructure and Surveillance, Training & Capacity Building and Consumer Empowerment. The
Index is a dynamic quantitative and qualitative benchmarking model that provides an objective
framework for evaluating food safety across all States/UTs. Name the top 3 states in the larger
category in this index.
(a) Tamil Nadu, Odisha, and Haryana
(b) Haryana, Punjab and Uttar Pradesh
(c) Gujarat, Assam, and Maharashtra
(d) Kerala, Punjab, and Tamil Nadu
(e) Maharashtra, Telangana, and Karnataka
Answer – (d)
Explanation –
This index was released prior to the exam date. This is a part of the indices and reports topic.

Q.52)
The National Statistical Office (NSO), Ministry of Statistics and Programme Implementation
(MoSPI) published the Provisional Estimates (PE) of National Income for 2022- 23 and Quarterly
Estimates of Gross Domestic Product (GDP) for the quarter January-March of 2022-23 (Q4 2022-
23). What is the share of agriculture and allied sector in the Gross Value Added of the entire
country in 2022-23?
(a) 2.3%
(b) 4%
(c) 5.4%
(d) 3.8%
(e) 8.1%
Answer – (b)
pg. 22
[Type here]

Explanation –
This was released in May 2023.

Q.53)
India’s Services exports set a new record of USD 254.4 Billion in the financial year 2021-2022.
The new record achieved in 2021-22 beats the previous high of USD 213.2 Billion in 2019-20.
Also, Services exports hit an all-time monthly high of USD 26.9 Billion in March 2022. Which of the
following is the top contributor in Services exports during April-December 2021?
(a) Telecommunications
(b) Business Process Outsourcing
(c) Freight
(d) Air transport
(e) Travel
Answer – (a)
Explanation –
Telecommunications, computer, and information services, other business services and Transport
are the top contributors in Services exports during April-December 2021. This is an unexpected
question because the data is quite old. There are one or two questions which are out of context.

Q.54)
According to the data released by the Department for Promotion of Industry and Internal Trade
(DPIIT), the foreign direct equity investments into India in fiscal 2023 shrank 21.67% from the
previous year to $46.03 billion. The FDI equity inflows were $58.77 in 2021-22. Which are top
three source countries of FDI for India?
(a) Singapore, Mauritius, and US
(b) Ireland, UK, and Denmark
(c) UK, Saudi Arabia, and Oman
(d) UAE, Mauritius, and Cayman Islands
(e) US, Ireland, and Cayman Islands
Answer – (a)
Explanation –
Singapore emerged as the top investor with $17.2 billion in FDI in the fiscal year, followed by
Mauritius at $6.1 billion and the US at $6.04 billion.

Q.55)
The Twenty Points Program was a significant policy initiative introduced by the Government of
India in 1975. It aimed to address various socio-economic challenges and promote social justice
and equality. The program included twenty specific points that covered a wide range of areas,
including agriculture, industry, employment, education, housing, and healthcare. Some key
objectives of the program were to alleviate poverty, reduce income inequalities, provide
employment opportunities, improve access to basic amenities, and empower marginalized
sections of society. Recently, the Ministry of Statistics and Programme Implementation (MoSPI)
has released the latest progress report of the Twenty Points Program. Of the 14 parameters
tracked quarterly under the Twenty Point Programme, the government achieved a more than ____
percent target completion ("very good") in six parameters.
pg. 23
[Type here]

(a) 90%
(b) 70%
(c) 65%
(d) 40%
(e) 55%
Answer – (a)
Explanation –
The news was published in May 2023.

Q.56)
Sovereign Gold Bonds are the government securities denominated in grams of gold. They are
substitutes for holding physical gold. Investors have to pay the issue price in cash and the bonds
will be redeemed in cash on maturity. The Bond is issued by Reserve Bank on behalf of
Government of India. The quantity of gold for which the investor pays is protected, since he
receives the ongoing market price at the time of redemption/ premature redemption. Identify the
incorrect statement regarding the SGB scheme.
(a) Person resident in India as defined under Foreign Exchange Management Act, 1999 are
eligible to invest in SGB
(b) Eligible investors include individuals, HUFs, trusts, universities and charitable institutions
(c) An investor/trust can buy 4 Kg/20 Kg worth of gold every year as the ceiling has been fixed on
a fiscal year (April-March) basis.
(d) The Bonds bear interest at the rate of 5.50 per cent (fixed rate) per annum on the amount of
initial investment.
(e) Bonds are sold through offices or branches of Nationalised Banks, Scheduled Private Banks,
Scheduled Foreign Banks, designated Post Offices, Stock Holding Corporation of India Ltd.
(SHCIL) and the authorised stock exchanges either directly or through their agents
Answer – (d)
Explanation –
The interest rate is 2.5%.

Q.57)
Which of the following organization celebrates its foundation day on July 1st?
(a) ICAI
(b) ICAR
(c) ICSI
(d) IARI
(e) IRRI
Answer – (a)
Explanation –
ICAI celebrates its foundation day on July 1st.

Q.58)
The US is set to rejoin ______ after a four-year absence from the global body that the country
abandoned during the Donald Trump presidency over what his administration called “anti-Israeli
bias”. The US has left the organization in 2018.
pg. 24
[Type here]

(a) IMF
(b) World Bank
(c) UNESCO
(d) WEF
(e) WIPO
Answer – (c)
Explanation –
US will join UNESCO.

Q.59)
A petaflop is a unit of measurement used to quantify the processing speed of a computer or
supercomputer. It represents one quadrillion floating-point operations per second (FLOPS). In
simple terms, a petaflop means a computer or supercomputer can perform one quadrillion
calculations or mathematical operations in a single second. It is a measure of computational power
and represents a significant milestone in the advancement of technology and computing
capabilities. Supercomputers that can achieve petaflop speeds are capable of tackling complex
scientific simulations, data analysis, and high-performance computing tasks that require massive
computational power. What is the numerical representation of a petaflop?
(a) 10^10
(b) 10^5
(c) 10^15
(d) 10^12
(e) 10^9
Answer – (c)
Explanation –
One petaFLOPS is equal to 1,000,000,000,000,000 (one quadrillion) FLOPS, or one thousand
teraFLOPS.

Q.60)
Prime Minister Narendra Modi in May 2023 released a special commemorative Rs 75 coin to mark
the inauguration of the new parliament building at the inauguration event held at the Lok Sabha
chamber. Commemorative coins are often released to commemorate significant events and
showcase unique designs that reflect the occasion they represent. These coins hold great value
for coin collectors, serving as treasured collectables. This coin is a made up of an alloy of four
different metals. Which of the following is not among its constituent metals?
(a) Silver
(b) Gold
(c) Zinc
(d) Nickel
(e) Copper
Answer – (b)
Explanation –
This is again an expected question.

Q.61)
pg. 25
[Type here]

The National Pension Scheme (NPS) is a voluntary and contributory pension scheme introduced
by the Government of India. It is designed to provide individuals with a sustainable income after
retirement. The scheme is open to both the employed and self-employed segments of the
population and offers two types of accounts: Tier-I and Tier-II. The NPS has gained popularity due
to its transparent and regulated framework, low-cost structure, and potential for wealth
accumulation over the long term. It provides individuals with a systematic and structured approach
to retirement planning, promoting financial security and independence during their post-
employment years. As per the Economic Survey of 2022-23, the average monthly net
subscriptions increased from 61.9 thousand in 2021 to ____ thousand in 2022 for the April-
October period.
(a) 71.8
(b) 63.2
(c) 45.9
(d) 55.7
(e) 89.6
Answer – (b)
Explanation –
This question is again from the Economic Survey.

Q.62)
Foreign reserves, also known as foreign exchange reserves or forex reserves, are assets held by
a country's central bank, typically in the form of foreign currencies and other internationally
accepted instruments. These reserves play a crucial role in maintaining economic stability and
safeguarding a nation's financial system. India’s forex reserves comprise of gold, foreign currency,
reserve tranche, and special drawing rights. Where is the 437-metric tonne gold held overseas by
RBI?
1) Bank of England
2) Bank of international settlement
3) Federal reserve
4) European Central Bank
(a) 1 and 2
(b) 2 and 3
(c) 3 and 4
(d) 2 and 4
(e) 1 and 3
Answer – (a)
Explanation –
This news article was released in May 2023.

Q.63)
To facilitate transparency and informed decision-making among the investors, markets regulator
Sebi mandated additional requirements for the issuance and listing of ____ bonds. This is one of
the sub-categories of 'green debt security'. These bonds are generally used for raising funds for
transitioning to a more sustainable form of operations in line with India's intended nationally
determined contributions.
pg. 26
[Type here]

(a) masala
(b) transition
(c) muni
(d) agency
(e) convertible
Answer – (b)
Explanation –
In May 2023, SEBI mandated additional requirements for the issuance and listing of transition
bonds.

Q.64)
NBFC-MFI stands for Non-Banking Financial Company-Microfinance Institution. It refers to a type
of financial institution that operates as a non-bank entity and provides microfinance services to
low-income individuals and underserved communities. The Reserve Bank of India (RBI) regulates
and supervises NBFC-MFIs to ensure their adherence to responsible lending practices, client
protection principles, and overall financial stability in the microfinance sector. RBI has mandated
Minimum Net Owned Funds of ____ for the NBFC-MFIs operating in the North-East region.
(a) Rs 2 crore
(b) Rs 1 crore
(c) Rs 5 crore
(d) Rs 3 crore
(e) Rs 4 crore
Answer – (a)
Explanation –
As per the RBI’s website, Minimum Net Owned Funds of Rs.5 crore. (For NBFC-MFIs registered in
the North Eastern Region of the country, the minimum NOF requirement shall stand at Rs. 2
crore).

Q.65)
The DRS is a technology-based system used in cricket to assist the match officials in their
decision-making. DRS was preceded by a system to allow on-field umpires to refer some
decisions to the third umpire to be decided using TV replays, in place since November 1992. What
is the full form of DRS?
(a) Decision Review System
(b) Data Recovery System
(c) Direct Registration System
(d) Digital Radio System
(e) Designated Representative Service
Answer – (a)
Explanation –
This is also an in-depth question from sports sections.

Q.66)
SWAMIH was announced on 6th November 2019 by the Finance Minister Smt. Nirmala
Sitharaman. The Union Cabinet cleared a proposal to set up a ‘Special Window’ in the form of AIF
pg. 27
[Type here]

to provide priority debt financing for completion of stalled housing projects. SBICAP Ventures was
assigned the role of Investment Manager for this special window. Who is the chief investment
officer of SWAMIH fund?
(a) Ashish Chandak
(b) Adwait Kasbekar
(c) Irfan Kazi
(d) Girish Sarwate
(e) Yashni Parikh
Answer – (c)
Explanation –
This is a question from the government schemes.

Q.67)
PM-KUSUM scheme stands for Pradhan Mantri Kisan Urja Suraksha evam Utthaan Mahabhiyan.
It is a government initiative launched in 2019 in India. The scheme aims to support farmers in
installing solar-powered agriculture pumps and promote the use of renewable energy in the
agricultural sector. The PM-KUSUM scheme plays a vital role in rural development, improving
farmer income, and contributing to the overall goal of achieving a clean and green energy future in
India. Identify the correct statement with regards to the scheme.
1) The Scheme consists of three components. Component A of the scheme aims at developing
10,000 MW of solar capacity through installation of small Solar Power Plants of individual plants of
capacity up to 2 MW.
2) The scheme is implemented till 31.03.2026
3) Solar energy-based power plants (SEPP) of capacity 500 kW to 2 MW will be setup by
individual farmers/ group of farmers/ cooperatives/ panchayats/ Farmer Producer Organisations
(FPO)/Water User associations (WUA)
(a) Only 1
(b) Only 2 and 3
(c) Only 1, 2, and 3
(d) Only 3
(e) Only 1 and 3
Answer – (c)
Explanation –
All the statements are correct.

Q.68)
The Smart Cities Mission was launched by the Hon’ Prime Minister on 25 June, 2015. The main
objective of the Mission is to promote cities that provide core infrastructure, clean and sustainable
environment and give a decent quality of life to their citizens through the application of ‘smart
solutions’. 100 cities have been selected to be developed as Smart Cities through a two-stage
competition. Identify the correct statement with regards to the scheme.
1) The Mission is operated as a Centrally Sponsored Scheme. Central Government will give
financial support to the extent of Rs. 48,000 crores over 5 years i.e., on an average Rs.100 crore
per city per year.

pg. 28
[Type here]

2) Additional finances for this scheme are raised through convergence, from ULBs’ own funds,
grants under Finance Commission, innovative finance mechanisms such as Municipal Bonds,
other government programs and borrowings.
3) The Smart Cities Mission has conceptualized 'DataSmart Cities'-Strategy to leverage the
potential of data to address complex urban challenges in 100 Mission Cities.
(a) Only 1 and 3
(b) Only 3
(c) Only 1 and 2
(d) Only 2 and 3
(e) Only 1, 2, and 3
Answer – (e)
Explanation –
All the statements are correct.

Q.69)
The SAI20 Summit of the SAI20 Engagement Group under India's G20 Presidency was organized
in Goa. Shri Girish Chandra Murmu, the Comptroller and Auditor General (CAG) of India and the
Chair of the Supreme Audit Institutions-20 (SAI20) Engagement Group, delivered the concluding
remarks. Which of the following were the themes of discussion of the meeting?
(a) Trade & Investment and Energy
(b) Blue Economy and Artificial Intelligence
(c) Transport & Communication and Tourism
(d) Technology and Public Health
(e) Fisheries and Agriculture
Answer – (b)
Explanation –
This is also an expected question.

Q.70)
A climate clock refers to a visual representation or digital display that provides real-time
information about the impacts of climate change. It typically shows various climate-related
statistics, such as greenhouse gas emissions, temperature rise, sea-level rise, deforestation rates,
and other indicators. Where is the climate clock located?
(a) New York
(b) Stockholm
(c) Dublin
(d) Glasgow
(e) Cardiff
Answer – (a)
Explanation –
In April 2023, CSIR- National Botanical Research Institute (NBRI), Lucknow will participate in an
attempt to create a world record in an event titled ‘World’s Largest Global Climate Clock Assembly’
on its premises. On the other hand, the climate clock in New York showed the time left for
emission control was 6 years and 102 days. Similarly, the time shown to reach 2 degrees Celsius
global warming was 24 years and 19 days.
pg. 29
[Type here]

Q.71)
UPI stands for Unified Payments Interface. It is a real-time payment system in India that enables
instant and seamless fund transfers between different banks and payment service providers. With
its simple and interoperable nature, UPI has revolutionized digital payments in India, facilitating
person-to-person transfers, merchant payments, bill payments, and other financial services
through a single platform. It has played a crucial role in promoting financial inclusion and digital
adoption, contributing to the growth of a cashless economy in the country. Which of the following
statement(s) is/are correct with regards to the UPI?
1) UPI was launched in 2016 in India
2) Immediate money transfer through mobile device round the clock 247 and 365 days is available
3) Virtual address of the customer for Pull & Push provides for incremental security with the
customer is not required to enter the details such as Card no, Account number; IFSC etc.
4) In the UPI system, a Pay Request is a transaction where the initiating customer is pushing
funds to the intended beneficiary. Payment Addresses include Mobile Number & MMID, Account
Number & IFSC and Virtual ID.
(a) Only 1 and 3
(b) Only 1, 2, 3, and 4
(c) Only 2, 3, and 4
(d) Only 2 and 3
(e) Only 1, 3, and 4
Answer – (b)
Explanation –
All the statements are correct about UPI.

Q.72)
The Payments Infrastructure Development Fund (PIDF) Scheme is an initiative introduced by the
Reserve Bank of India (RBI) to promote the development of digital payments infrastructure in the
country. Under this scheme, the RBI provides financial support and grants to eligible entities, such
as banks, non-bank payment system operators, and other eligible stakeholders, to enhance the
acceptance and accessibility of digital payments. Identify the correct statement with regards to the
PIDF.
1) PIDF is governed by an ex-officio Advisory Council
2) The tenor of the PIDF scheme is three years from January 01, 2021, extendable by two further
years, if necessary
3) Increasing payments acceptance infrastructure by adding 30 lakh touch points – 10 lakh
physical and 20 lakh digital payment acceptance devices every year
(a) Only 1, 2, and 3
(b) Only 2 and 3
(c) Only 1
(d) Only 1 and 2
(e) Only 3
Answer – (a)
Explanation –
All the statements are correct about PIDF.
pg. 30
[Type here]

Q.73)
Gross Domestic Saving (GDS) refers to the total amount of savings generated within a country
over a specific period, typically a year. It is a key macroeconomic indicator that reflects the portion
of national income not consumed but instead saved by households, businesses, and the
government. GDS includes savings in various forms, such as personal savings, corporate profits
set aside for investment, and government savings from budget surpluses. It is expressed as a
percentage of ____.
(a) Total Savings
(b) Private Final Consumption Expenditure
(c) Private Income
(d) National Income
(e) Gross Domestic Product
Answer – (e)
Explanation –
Gross Domestic Saving is GDP minus final consumption expenditure. It is expressed as a
percentage of GDP.

Q.74)
A clearing corporation, also known as a clearinghouse, is an entity that acts as an intermediary in
financial markets to facilitate the clearing and settlement of trades. Its primary function is to ensure
the smooth and efficient completion of transactions between buyers and sellers. Which of the
following entities is not recognized by SEBI?
(a) Indian Clearing Corporation Ltd.
(b) Metropolitan Clearing Corporation of India Ltd
(c) Multi Commodity Exchange Clearing Corporation Ltd
(d) National Commodity Clearing Ltd
(e) Kotak Clearing Corporation Ltd
Answer – (e)
Explanation –
Only E is not a clearing corporation recognized by SEBI.

Q.75)
The Bharat Bill payment system is a Reserve Bank of India (RBI) conceptualised system driven by
National Payments Corporation of India (NPCI). It is a one-stop ecosystem for payment of all bills
providing an interoperable and accessible “Anytime Anywhere” Bill payment service to all
customers across India with certainty, reliability and safety of transactions. Which of the following
statement(s) is/are a feature of the BBPS?
1) Bharat BillPay transaction can be initiated through multiple payment channels like Internet,
Internet Banking, Mobile, Mobile-Banking, Mobile Wallets, Kiosk, ATM, Bank Branch, Agents and
Business Correspondents, by just looking at the Bharat BillPay logo
2) Bharat BillPay is an integrated ecosystem connecting banks and non-banks in bills aggregation
business, Billers, payment service providers and retail Bill outlets.
3) Bharat BillPay facilitates seamless payment of bills through any channel: digital and physical.
(a) Only 1
pg. 31
[Type here]

(b) None of those given as options


(c) Only 2
(d) Only 2 and 3
(e) Only 1 and 3
Answer – (b)
Explanation –
All the statements are correct.

Q.76)
Which of the following Article of the Indian Constitution has described the division of powers
between Union and State governments with respect to the Union, State, and Concurrent lists?
(a) Article 246
(b) Article 134
(c) Article 108
(d) Article 55
(e) Article 113
Answer – (a)
Explanation –
Only option A is the correct answer.

Q.77)
This Country Partnership Strategy (CPS), 2023-2027 for India is well aligned with India’s national
development priorities to be achieved by 2047, when the country marks 100 years of
independence. Economic and sector reforms and national flagship programs embody the national
priorities. The CPS will advance Strategy 2030’s seven operational priorities. As it aims to help
India return to its development trajectory, it will support better health, education, and social
protection and improved urban livability and rural development while ensuring gender
empowerment. Which multilateral development agency has released the strategy?
(a) WEF
(b) IMF
(c) WB
(d) OECD
(e) ADB
Answer – (e)
Explanation –
ADB released the strategy.

Q.78)
Which of the following is the top gold producing countries in the world?
(a) China
(b) India
(c) UAE
(d) Oman
(e) Bahrain
Answer – (a)
pg. 32
[Type here]

Explanation –
The top gold producing country is China.

Q.79)
Trade Receivables electronic Discounting System (TReDS) is an online electronic platform and an
institutional mechanism for factoring of trade receivables of MSME sellers. It enables discounting
of invoices through an auction mechanism to ensure prompt realisation of trade receivables. How
many parties are involved in the TReDS system?
(a) Four
(b) Seven
(c) Five
(d) Three
(e) Two
Answer – (a)
Explanation –
Buyers, sellers, financiers, and insurers are the participating entities in the TReDS platform. This
news is from the June 2023 month.

Q.80)
Domestic Systemically Important Insurers (D-SIIs) refer to insurers of such size, market
importance, and domestic and global interconnectedness, whose distress or failure would cause a
significant dislocation in the domestic financial system. Therefore, the continued functioning of D-
SIIs is critical for the uninterrupted availability of insurance services to the national economy. In
March 2023, IRDAI released the list of domestic-systematically important insurers. Identify the
insurers from the list.
(a) Life Insurance Corporation of India
(b) General Insurance Corporation of India
(c) New India Assurance Co. Ltd
(d) Only A and B
(e) Only A, B, and C
Answer – (e)
Explanation –
All the three insurers are correct.

English
Q.81)
Four statements have been mentioned below. One or more statements may contain an error. It
may contain a grammatical and/or a contextual error. Identify the INCORRECT statement.
1) The Neoclassical style refers to a movement in art, architecture, and design that emerging in
the 18th and early 19th centuries to revive and emulate the ideals of classical antiquity.

pg. 33
[Type here]

2) Restoration comedies were known for their wit, satire, and social commentary. They often
portrayed the manners and social mores of the upper class and were characterized by their clever
wordplay, risqué humor, and intricate plots.
3) Miracle plays, also known as mystery plays, were a form of medieval drama that emerged in
Europe during the Middle Ages. These plays were religious in nature and were performed by
members of the clergy and local communities.
4) The term "Dark Ages" is generally used to refer to a period of cultural, intellectual, and
economic decline or stagnation. It is often characterized by a decline in centralized authority,
political instability, economic hardships, and a relative lack of cultural and intellectual progress
compared to earlier and later periods.
(a) Only 1
(b) Only 2 and 3
(c) Only 1, 2, and 4
(d) Only 2 and 4
(e) Only 3
Answer – (a)
Explanation –
‘Emerging’ is wrong usage. ‘Emerged’ should be used.

Q.82)
In the question below, a sentence has been divided into different parts. Rearrange the parts to
form a meaningful sentence.
(P) In the intricate tapestry of human history, traversing vast epochs marked by triumphs and
tribulations, the interplay of diverse cultures, ideologies, and
(Q) knowledge, technological advancements, and collective aspirations continually shape the
tapestry of our existence,
(R) forging intricate connections between past, present, and future, as we navigate the perennial
quest for progress, harmony, and understanding amidst the ebb and flow of the human experience
(S) socio-political systems has engendered multifaceted narratives replete with paradigm shifts,
oscillating power dynamics, and profound transformations, wherein the ceaseless pursuit of
(a) RPQS
(b) SQPR
(c) RPSQ
(d) PSQR
(e) QSRP
Answer – (d)
Explanation –
The correct sequence is PSQR.

Q.83)
In the question below, a sentence has been divided into different parts. Rearrange the parts to
form a meaningful sentence.
(R) Amidst the vast expanse of the universe, with its swirling galaxies, cosmic phenomena, and
enigmatic celestial bodies dancing to the symphony of gravity, electromagnetic forces, and the
mysterious dark energy,
pg. 34
[Type here]

(S) humanity, driven by an insatiable curiosity and thirst for knowledge, has embarked on a
relentless quest to unravel the secrets of existence, delving into the intricate realms of quantum
mechanics, exploring the cosmos
(P) through telescopes that pierce the cosmic veil, conducting experiments at particle colliders that
reveal the fundamental building blocks of matter, and venturing into the depths of the ocean where
ancient
(Q) lifeforms hold the keys to understanding our planet's past and its potential future, as we strive
to grasp the essence of our place in the cosmos and the meaning that lies hidden within the fabric
of reality.
(a) PRQS
(b) RSPQ
(c) QSPR
(d) SPQR
(e) SQPR
Answer – (b)
Explanation –
The correct sequence is RSPQ.

Q.84)
In the question mentioned below, a paragraph is divided into different sentences. Rearrange the
sentences to form a coherent paragraph.
(1) With each passing moment, we teeter on the precipice of irreversible consequences, where the
ravenous appetite for energy conspires with the existential threat of climate change. The
dexterous choreography of this transition demands not only the orchestration of technological
advancements and innovative solutions but also the harmonization of political will, economic
viability, and societal engagement.
(2) As we navigate the treacherous terrain strewn with vested interests, formidable barriers, and
divergent perspectives, we must rise above complacency and rally behind the transformative
potential that lies dormant within this audacious endeavor.
(3) The labyrinthine challenge of shifting from fossil fuels to renewable energy sources requires
meticulous planning, colossal investments, and the fortitude to disrupt long-established systems
entrenched in the annals of industrialization.
(4) The road to an energy transition is arduous, fraught with complexities, but it is through our
unwavering resilience and unwavering commitment that we shall pave the path towards a
sustainable future, forging a legacy that transcends time, and kindles the embers of hope for
generations yet unborn.
(5) In the relentless pursuit of a sustainable future, the imperative for an energy transition emerges
as an unwavering mandate, demanding audacious action and resolute determination.
(a) 14235
(b) 25431
(c) 53124
(d) 43215
(e) 54123
Answer – (c)
Explanation –
pg. 35
[Type here]

The correct sequence of paragraph is 53124.


(5) In the relentless pursuit of a sustainable future, the imperative for an energy transition emerges
as an unwavering mandate, demanding audacious action and resolute determination.
(3) The labyrinthine challenge of shifting from fossil fuels to renewable energy sources requires
meticulous planning, colossal investments, and the fortitude to disrupt long-established systems
entrenched in the annals of industrialization.
(1) With each passing moment, we teeter on the precipice of irreversible consequences, where the
ravenous appetite for energy conspires with the existential threat of climate change. The
dexterous choreography of this transition demands not only the orchestration of technological
advancements and innovative solutions but also the harmonization of political will, economic
viability, and societal engagement.
(2) As we navigate the treacherous terrain strewn with vested interests, formidable barriers, and
divergent perspectives, we must rise above complacency and rally behind the transformative
potential that lies dormant within this audacious endeavor.
(4) The road to an energy transition is arduous, fraught with complexities, but it is through our
unwavering resilience and unwavering commitment that we shall pave the path towards a
sustainable future, forging a legacy that transcends time, and kindles the embers of hope for
generations yet unborn.

Q.85)
In the question mentioned below, a paragraph is divided into different sentences. Rearrange the
sentences to form a coherent paragraph.
(1) Challenging this entrenched paradigm necessitates a paradigm shift in urban planning and
design, recognizing the multifaceted dimensions of gender and embracing an inclusive approach
that fosters safety, accessibility, and empowerment for all individuals.
(2) To forge equitable societies, we must dismantle the gendered infrastructure, rebalancing the
scales of spatial justice, and championing an era where every person, irrespective of gender, can
flourish and thrive within the architectural fabric of our shared world.
(3) The manifestation of gendered infrastructure materializes in the subtle design choices that
shape our urban landscapes, reinforcing power dynamics and further marginalizing certain
communities.
(4) The built environment, often an embodiment of patriarchal norms, has historically perpetuated
gender inequities, perpetuating spatial biases and engendering disparities.
(5) Gendered infrastructure, a subject of profound socio-cultural significance, demands rigorous
examination within the intricate tapestry of societal systems.
(6) As the framework of cities and public spaces fails to accommodate diverse needs, it
perpetuates a systemic exclusion of women, exacerbating their vulnerability and hindering their
access to essential services and opportunities.
(a) 123654
(b) 543612
(c) 654321
(d) 412356
(e) 543126
Answer – (b)
Explanation –
pg. 36
[Type here]

The correct sequence is 543612.


(5) Gendered infrastructure, a subject of profound socio-cultural significance, demands rigorous
examination within the intricate tapestry of societal systems.
(4) The built environment, often an embodiment of patriarchal norms, has historically perpetuated
gender inequities, perpetuating spatial biases and engendering disparities.
(3) The manifestation of gendered infrastructure materializes in the subtle design choices that
shape our urban landscapes, reinforcing power dynamics and further marginalizing certain
communities.
(6) As the framework of cities and public spaces fails to accommodate diverse needs, it
perpetuates a systemic exclusion of women, exacerbating their vulnerability and hindering their
access to essential services and opportunities.
(1) Challenging this entrenched paradigm necessitates a paradigm shift in urban planning and
design, recognizing the multifaceted dimensions of gender and embracing an inclusive approach
that fosters safety, accessibility, and empowerment for all individuals.
(2) To forge equitable societies, we must dismantle the gendered infrastructure, rebalancing the
scales of spatial justice, and championing an era where every person, irrespective of gender, can
flourish and thrive within the architectural fabric of our shared world.

Q.86)
In the question below, two sentences have been mentioned with a blank in each sentence. Use
one of the phrases given in the options to fill in the blanks.
1) Despite his notable contributions to the project, John's colleagues often felt overshadowed as
his penchant for self-promotion and desire to _____ during presentations and discussions
undermined the collaborative spirit and overshadowed the collective efforts of the team.
2) Amidst the intricate dance of egos and aspirations within the creative industry, certain
individuals unabashedly ______, their insatiable hunger for attention overshadowing the collective
brilliance of their peers, stifling collaboration, and perpetuating a culture of self-centeredness
(a) Hue and Cry
(b) Dragging a crate
(c) Hog the limelight
(d) Burn the bridge
(e) Sleeping like a log
Answer – (c)
Explanation –
Hog the limelight means to steal the limelight. It fits best in the sentences.

Q.87)
In the question below, two sentences have been mentioned with a blank in each sentence. Use
one of the phrases given in the options to fill in the blanks.
1) In the realm of economic policy, policymakers strive to strike a delicate balance between
allocating resources efficiently and maximizing the ________, wherein the optimal utilization of
available funds yields the greatest impact and value for the desired outcomes, navigating the fine
line between cost-effectiveness, equitable distribution, and long-term sustainability.
2) As consumers increasingly seek value and efficiency in their purchasing decisions, businesses
must constantly innovate and refine their offerings to provide a compelling __________,
pg. 37
[Type here]

leveraging competitive pricing strategies, product differentiation, and exceptional customer


experiences to maximize perceived value and establish long-term customer loyalty.
(a) Bang for the buck
(b) Fish rots from the head
(c) To bite the dust
(d) Elephant in the room
(e) Slay your dragon
Answer – (a)
Explanation –
Bang for the buck means to value your money. It is apt for both the sentences.

Q.88)
In the question below, two sentences have been mentioned with a blank in each sentence. Use
one of the phrases given in the options to fill in the blanks.
1) Despite the severity of the crime committed, the lenient sentencing of a mere community
service order for a repeat offender can be seen as nothing more than a ______, failing to serve as
an adequate deterrent or impart a sense of justice to the victims and society at large.
2) The regulatory agency's decision to impose a nominal fine on the corporation found guilty of
gross negligence and environmental violations is seen by many as nothing more than a symbolic
_______, lacking the teeth to truly hold the company accountable for its actions and ensure
meaningful change in their practices.
(a) A Taste of Your Own Medicine
(b) Slap on the wrist
(c) Back to the Drawing Board
(d) All Greek to me
(e) Baker's Dozen
Answer – (b)
Explanation –
A Slap on the Wrist means A very mild punishment. It fits best in both sentences.

Instructions for Q.89)


Read the passage and answer the questions that follow.
Third world countries, a term that originated during the Cold War era, refer to the developing
nations that faced economic, social, and political challenges in the aftermath of colonialism and
globalization. These countries have diverse histories and trajectories, shaped by colonization,
post-colonial struggles, and attempts at development. The history of third world countries is
marked by the exploitation of resources, cultural suppression, and political subjugation by colonial
powers. The aftermath of independence brought hopes of progress and self-determination, but
many nations grappled with the daunting task of nation-building and overcoming deep-rooted
socio-economic disparities.
Despite the challenges, third world countries possess considerable growth prospects. Several
nations have made significant strides in areas such as education, healthcare, and infrastructure.
Economic reforms, investments in human capital, and advancements in technology have opened
doors for innovation, entrepreneurship, and foreign investments. The emergence of regional
cooperation and trade blocs has also fostered economic integration and increased market
pg. 38
[Type here]

opportunities. However, third world countries continue to face a myriad of challenges. Poverty,
income inequality, and lack of access to basic services remain persistent issues. Political
instability, corruption, and inadequate governance hinder progress and create hurdles for
sustainable development. Environmental degradation, climate change, and natural disasters pose
additional threats to these nations, affecting their agricultural productivity, water resources, and
overall resilience.
To overcome these challenges and pave the way forward, third world countries must prioritize
inclusive and sustainable development. Strengthening governance structures, promoting
transparency, and combating corruption are essential for creating an enabling environment for
growth. Investment in education and skills development can empower the workforce and foster
innovation. Enhanced access to healthcare, clean water, and sanitation is critical for improving the
quality of life. Furthermore, third world countries must leverage technological advancements to
bridge the digital divide and foster digital literacy. Sustainable agricultural practices, renewable
energy initiatives, and climate resilience strategies should be prioritized to mitigate environmental
risks. Collaboration and partnerships with international organizations, developed countries, and
the private sector can provide valuable support in terms of finance, technology transfer, and
capacity-building. In summary, the history, growth prospects, challenges, and the way forward for
third world countries are complex and multifaceted. Achieving sustainable development requires
addressing systemic issues, embracing innovation, and fostering inclusive growth. It necessitates
a global commitment to promoting equity, justice, and shared prosperity.
In addition to economic and social challenges, third world countries also face issues related to
population growth and urbanization. Rapid population growth puts immense pressure on limited
resources and infrastructure, leading to overcrowded cities, inadequate housing, and strained
public services. Urbanization presents both opportunities and challenges, as it can drive economic
growth and innovation but also exacerbate income inequality and social disparities if not properly
managed. Effective urban planning, investment in sustainable urban infrastructure, and social
welfare programs are crucial for creating inclusive and livable cities.
Another critical aspect in the development of third world countries is the empowerment and
inclusion of women. Gender inequality remains a significant obstacle, limiting women's access to
education, healthcare, and economic opportunities. By ensuring equal rights, addressing cultural
norms, and providing support for women's empowerment, third world countries can unlock the full
potential of their human capital and foster inclusive growth. Women's participation in decision-
making processes, entrepreneurship, and leadership roles is key to driving sustainable
development and social progress.
The way forward for third world countries also necessitates a focus on cultural preservation and
promotion. Indigenous knowledge, traditions, and cultural heritage play a vital role in identity
formation and community cohesion. Integrating cultural preservation efforts into development
strategies can enhance social cohesion, promote cultural diversity, and foster a sense of pride and
belonging. Embracing cultural tourism, supporting local artisans and craftsmen, and preserving
historical sites contribute to sustainable development while safeguarding cultural heritage for
future generations.
Moreover, international cooperation and fair global trade are essential for the progress of third
world countries. Addressing trade imbalances, promoting fair trade practices, and reducing
barriers to market access can create opportunities for economic growth and poverty reduction.
Access to affordable finance and technology transfer are crucial for developing industries and
pg. 39
[Type here]

fostering innovation. Multilateral institutions and global partnerships should strive to create a level
playing field, providing support and resources to ensure the equitable participation of third world
countries in the global economy.
Q.89)
What are some of the challenges related to population growth and urbanization in third world
countries?
(a) Limited access to education and healthcare
(b) Overcrowded cities and inadequate infrastructure
(c) Political instability and inadequate governance
(d) Exploitation of natural resources by colonial powers
(e) Strong economic growth and technological advancements
Answer – (b)
Explanation –
From the options, only option B fits in best.

Instructions for Q.90)


Read the passage and answer the questions that follow.
Third world countries, a term that originated during the Cold War era, refer to the developing
nations that faced economic, social, and political challenges in the aftermath of colonialism and
globalization. These countries have diverse histories and trajectories, shaped by colonization,
post-colonial struggles, and attempts at development. The history of third world countries is
marked by the exploitation of resources, cultural suppression, and political subjugation by colonial
powers. The aftermath of independence brought hopes of progress and self-determination, but
many nations grappled with the daunting task of nation-building and overcoming deep-rooted
socio-economic disparities.
Despite the challenges, third world countries possess considerable growth prospects. Several
nations have made significant strides in areas such as education, healthcare, and infrastructure.
Economic reforms, investments in human capital, and advancements in technology have opened
doors for innovation, entrepreneurship, and foreign investments. The emergence of regional
cooperation and trade blocs has also fostered economic integration and increased market
opportunities. However, third world countries continue to face a myriad of challenges. Poverty,
income inequality, and lack of access to basic services remain persistent issues. Political
instability, corruption, and inadequate governance hinder progress and create hurdles for
sustainable development. Environmental degradation, climate change, and natural disasters pose
additional threats to these nations, affecting their agricultural productivity, water resources, and
overall resilience.
To overcome these challenges and pave the way forward, third world countries must prioritize
inclusive and sustainable development. Strengthening governance structures, promoting
transparency, and combating corruption are essential for creating an enabling environment for
growth. Investment in education and skills development can empower the workforce and foster
innovation. Enhanced access to healthcare, clean water, and sanitation is critical for improving the
quality of life. Furthermore, third world countries must leverage technological advancements to
bridge the digital divide and foster digital literacy. Sustainable agricultural practices, renewable
energy initiatives, and climate resilience strategies should be prioritized to mitigate environmental
risks. Collaboration and partnerships with international organizations, developed countries, and
pg. 40
[Type here]

the private sector can provide valuable support in terms of finance, technology transfer, and
capacity-building. In summary, the history, growth prospects, challenges, and the way forward for
third world countries are complex and multifaceted. Achieving sustainable development requires
addressing systemic issues, embracing innovation, and fostering inclusive growth. It necessitates
a global commitment to promoting equity, justice, and shared prosperity.
In addition to economic and social challenges, third world countries also face issues related to
population growth and urbanization. Rapid population growth puts immense pressure on limited
resources and infrastructure, leading to overcrowded cities, inadequate housing, and strained
public services. Urbanization presents both opportunities and challenges, as it can drive economic
growth and innovation but also exacerbate income inequality and social disparities if not properly
managed. Effective urban planning, investment in sustainable urban infrastructure, and social
welfare programs are crucial for creating inclusive and livable cities.
Another critical aspect in the development of third world countries is the empowerment and
inclusion of women. Gender inequality remains a significant obstacle, limiting women's access to
education, healthcare, and economic opportunities. By ensuring equal rights, addressing cultural
norms, and providing support for women's empowerment, third world countries can unlock the full
potential of their human capital and foster inclusive growth. Women's participation in decision-
making processes, entrepreneurship, and leadership roles is key to driving sustainable
development and social progress.
The way forward for third world countries also necessitates a focus on cultural preservation and
promotion. Indigenous knowledge, traditions, and cultural heritage play a vital role in identity
formation and community cohesion. Integrating cultural preservation efforts into development
strategies can enhance social cohesion, promote cultural diversity, and foster a sense of pride and
belonging. Embracing cultural tourism, supporting local artisans and craftsmen, and preserving
historical sites contribute to sustainable development while safeguarding cultural heritage for
future generations.
Moreover, international cooperation and fair global trade are essential for the progress of third
world countries. Addressing trade imbalances, promoting fair trade practices, and reducing
barriers to market access can create opportunities for economic growth and poverty reduction.
Access to affordable finance and technology transfer are crucial for developing industries and
fostering innovation. Multilateral institutions and global partnerships should strive to create a level
playing field, providing support and resources to ensure the equitable participation of third world
countries in the global economy.
Q.90)
Why is the empowerment and inclusion of women crucial for the development of third world
countries?
(a) It perpetuates gender inequality and social disparities
(b) It leads to overcrowded cities and inadequate housing
(c) It fosters inclusive growth and unlocks human capital
(d) It exacerbates income inequality and poverty
(e) It promotes corruption and political instability
Answer – (c)
Explanation –
From the options given, option C is the best fit.

pg. 41
[Type here]

Instructions for Q.91)


Read the passage and answer the questions that follow.
Third world countries, a term that originated during the Cold War era, refer to the developing
nations that faced economic, social, and political challenges in the aftermath of colonialism and
globalization. These countries have diverse histories and trajectories, shaped by colonization,
post-colonial struggles, and attempts at development. The history of third world countries is
marked by the exploitation of resources, cultural suppression, and political subjugation by colonial
powers. The aftermath of independence brought hopes of progress and self-determination, but
many nations grappled with the daunting task of nation-building and overcoming deep-rooted
socio-economic disparities.
Despite the challenges, third world countries possess considerable growth prospects. Several
nations have made significant strides in areas such as education, healthcare, and infrastructure.
Economic reforms, investments in human capital, and advancements in technology have opened
doors for innovation, entrepreneurship, and foreign investments. The emergence of regional
cooperation and trade blocs has also fostered economic integration and increased market
opportunities. However, third world countries continue to face a myriad of challenges. Poverty,
income inequality, and lack of access to basic services remain persistent issues. Political
instability, corruption, and inadequate governance hinder progress and create hurdles for
sustainable development. Environmental degradation, climate change, and natural disasters pose
additional threats to these nations, affecting their agricultural productivity, water resources, and
overall resilience.
To overcome these challenges and pave the way forward, third world countries must prioritize
inclusive and sustainable development. Strengthening governance structures, promoting
transparency, and combating corruption are essential for creating an enabling environment for
growth. Investment in education and skills development can empower the workforce and foster
innovation. Enhanced access to healthcare, clean water, and sanitation is critical for improving the
quality of life. Furthermore, third world countries must leverage technological advancements to
bridge the digital divide and foster digital literacy. Sustainable agricultural practices, renewable
energy initiatives, and climate resilience strategies should be prioritized to mitigate environmental
risks. Collaboration and partnerships with international organizations, developed countries, and
the private sector can provide valuable support in terms of finance, technology transfer, and
capacity-building. In summary, the history, growth prospects, challenges, and the way forward for
third world countries are complex and multifaceted. Achieving sustainable development requires
addressing systemic issues, embracing innovation, and fostering inclusive growth. It necessitates
a global commitment to promoting equity, justice, and shared prosperity.
In addition to economic and social challenges, third world countries also face issues related to
population growth and urbanization. Rapid population growth puts immense pressure on limited
resources and infrastructure, leading to overcrowded cities, inadequate housing, and strained
public services. Urbanization presents both opportunities and challenges, as it can drive economic
growth and innovation but also exacerbate income inequality and social disparities if not properly
managed. Effective urban planning, investment in sustainable urban infrastructure, and social
welfare programs are crucial for creating inclusive and livable cities.
Another critical aspect in the development of third world countries is the empowerment and
inclusion of women. Gender inequality remains a significant obstacle, limiting women's access to
education, healthcare, and economic opportunities. By ensuring equal rights, addressing cultural
pg. 42
[Type here]

norms, and providing support for women's empowerment, third world countries can unlock the full
potential of their human capital and foster inclusive growth. Women's participation in decision-
making processes, entrepreneurship, and leadership roles is key to driving sustainable
development and social progress.
The way forward for third world countries also necessitates a focus on cultural preservation and
promotion. Indigenous knowledge, traditions, and cultural heritage play a vital role in identity
formation and community cohesion. Integrating cultural preservation efforts into development
strategies can enhance social cohesion, promote cultural diversity, and foster a sense of pride and
belonging. Embracing cultural tourism, supporting local artisans and craftsmen, and preserving
historical sites contribute to sustainable development while safeguarding cultural heritage for
future generations.
Moreover, international cooperation and fair global trade are essential for the progress of third
world countries. Addressing trade imbalances, promoting fair trade practices, and reducing
barriers to market access can create opportunities for economic growth and poverty reduction.
Access to affordable finance and technology transfer are crucial for developing industries and
fostering innovation. Multilateral institutions and global partnerships should strive to create a level
playing field, providing support and resources to ensure the equitable participation of third world
countries in the global economy.
Q.91)
What role do partnerships with international organizations and developed countries play in the
development of third world countries?
(a) They contribute to corruption in the government
(b) They promote economic dependency and hinder growth
(c) They provide valuable financial and technological support
(d) They exacerbate progress in the socio-economic sectors
(e) They discourage innovation and entrepreneurship
Answer – (c)
Explanation –
Option C is the correct answer.

Instructions for Q.92)


Read the passage and answer the questions that follow.
Third world countries, a term that originated during the Cold War era, refer to the developing
nations that faced economic, social, and political challenges in the aftermath of colonialism and
globalization. These countries have diverse histories and trajectories, shaped by colonization,
post-colonial struggles, and attempts at development. The history of third world countries is
marked by the exploitation of resources, cultural suppression, and political subjugation by colonial
powers. The aftermath of independence brought hopes of progress and self-determination, but
many nations grappled with the daunting task of nation-building and overcoming deep-rooted
socio-economic disparities.
Despite the challenges, third world countries possess considerable growth prospects. Several
nations have made significant strides in areas such as education, healthcare, and infrastructure.
Economic reforms, investments in human capital, and advancements in technology have opened
doors for innovation, entrepreneurship, and foreign investments. The emergence of regional
cooperation and trade blocs has also fostered economic integration and increased market
pg. 43
[Type here]

opportunities. However, third world countries continue to face a myriad of challenges. Poverty,
income inequality, and lack of access to basic services remain persistent issues. Political
instability, corruption, and inadequate governance hinder progress and create hurdles for
sustainable development. Environmental degradation, climate change, and natural disasters pose
additional threats to these nations, affecting their agricultural productivity, water resources, and
overall resilience.
To overcome these challenges and pave the way forward, third world countries must prioritize
inclusive and sustainable development. Strengthening governance structures, promoting
transparency, and combating corruption are essential for creating an enabling environment for
growth. Investment in education and skills development can empower the workforce and foster
innovation. Enhanced access to healthcare, clean water, and sanitation is critical for improving the
quality of life. Furthermore, third world countries must leverage technological advancements to
bridge the digital divide and foster digital literacy. Sustainable agricultural practices, renewable
energy initiatives, and climate resilience strategies should be prioritized to mitigate environmental
risks. Collaboration and partnerships with international organizations, developed countries, and
the private sector can provide valuable support in terms of finance, technology transfer, and
capacity-building. In summary, the history, growth prospects, challenges, and the way forward for
third world countries are complex and multifaceted. Achieving sustainable development requires
addressing systemic issues, embracing innovation, and fostering inclusive growth. It necessitates
a global commitment to promoting equity, justice, and shared prosperity.
In addition to economic and social challenges, third world countries also face issues related to
population growth and urbanization. Rapid population growth puts immense pressure on limited
resources and infrastructure, leading to overcrowded cities, inadequate housing, and strained
public services. Urbanization presents both opportunities and challenges, as it can drive economic
growth and innovation but also exacerbate income inequality and social disparities if not properly
managed. Effective urban planning, investment in sustainable urban infrastructure, and social
welfare programs are crucial for creating inclusive and livable cities.
Another critical aspect in the development of third world countries is the empowerment and
inclusion of women. Gender inequality remains a significant obstacle, limiting women's access to
education, healthcare, and economic opportunities. By ensuring equal rights, addressing cultural
norms, and providing support for women's empowerment, third world countries can unlock the full
potential of their human capital and foster inclusive growth. Women's participation in decision-
making processes, entrepreneurship, and leadership roles is key to driving sustainable
development and social progress.
The way forward for third world countries also necessitates a focus on cultural preservation and
promotion. Indigenous knowledge, traditions, and cultural heritage play a vital role in identity
formation and community cohesion. Integrating cultural preservation efforts into development
strategies can enhance social cohesion, promote cultural diversity, and foster a sense of pride and
belonging. Embracing cultural tourism, supporting local artisans and craftsmen, and preserving
historical sites contribute to sustainable development while safeguarding cultural heritage for
future generations.
Moreover, international cooperation and fair global trade are essential for the progress of third
world countries. Addressing trade imbalances, promoting fair trade practices, and reducing
barriers to market access can create opportunities for economic growth and poverty reduction.
Access to affordable finance and technology transfer are crucial for developing industries and
pg. 44
[Type here]

fostering innovation. Multilateral institutions and global partnerships should strive to create a level
playing field, providing support and resources to ensure the equitable participation of third world
countries in the global economy.
Q.92)
What are the key factors for the way forward in third world countries?
(a) Exploitation of resources and colonial power dynamics
(b) Promoting technological advancements, digital literacy, preservation of cultural and its
promotion
(c) Inciting a downtrend in governance structures
(d) Limited access to education and healthcare
(e) Reduction in foreign investments and trade opportunities
Answer – (b)
Explanation –
Option B is correct as per the passage.

Instructions for Q.93)


Read the passage and answer the questions that follow.
Third world countries, a term that originated during the Cold War era, refer to the developing
nations that faced economic, social, and political challenges in the aftermath of colonialism and
globalization. These countries have diverse histories and trajectories, shaped by colonization,
post-colonial struggles, and attempts at development. The history of third world countries is
marked by the exploitation of resources, cultural suppression, and political subjugation by colonial
powers. The aftermath of independence brought hopes of progress and self-determination, but
many nations grappled with the daunting task of nation-building and overcoming deep-rooted
socio-economic disparities.
Despite the challenges, third world countries possess considerable growth prospects. Several
nations have made significant strides in areas such as education, healthcare, and infrastructure.
Economic reforms, investments in human capital, and advancements in technology have opened
doors for innovation, entrepreneurship, and foreign investments. The emergence of regional
cooperation and trade blocs has also fostered economic integration and increased market
opportunities. However, third world countries continue to face a myriad of challenges. Poverty,
income inequality, and lack of access to basic services remain persistent issues. Political
instability, corruption, and inadequate governance hinder progress and create hurdles for
sustainable development. Environmental degradation, climate change, and natural disasters pose
additional threats to these nations, affecting their agricultural productivity, water resources, and
overall resilience.
To overcome these challenges and pave the way forward, third world countries must prioritize
inclusive and sustainable development. Strengthening governance structures, promoting
transparency, and combating corruption are essential for creating an enabling environment for
growth. Investment in education and skills development can empower the workforce and foster
innovation. Enhanced access to healthcare, clean water, and sanitation is critical for improving the
quality of life. Furthermore, third world countries must leverage technological advancements to
bridge the digital divide and foster digital literacy. Sustainable agricultural practices, renewable
energy initiatives, and climate resilience strategies should be prioritized to mitigate environmental
risks. Collaboration and partnerships with international organizations, developed countries, and
pg. 45
[Type here]

the private sector can provide valuable support in terms of finance, technology transfer, and
capacity-building. In summary, the history, growth prospects, challenges, and the way forward for
third world countries are complex and multifaceted. Achieving sustainable development requires
addressing systemic issues, embracing innovation, and fostering inclusive growth. It necessitates
a global commitment to promoting equity, justice, and shared prosperity.
In addition to economic and social challenges, third world countries also face issues related to
population growth and urbanization. Rapid population growth puts immense pressure on limited
resources and infrastructure, leading to overcrowded cities, inadequate housing, and strained
public services. Urbanization presents both opportunities and challenges, as it can drive economic
growth and innovation but also exacerbate income inequality and social disparities if not properly
managed. Effective urban planning, investment in sustainable urban infrastructure, and social
welfare programs are crucial for creating inclusive and livable cities.
Another critical aspect in the development of third world countries is the empowerment and
inclusion of women. Gender inequality remains a significant obstacle, limiting women's access to
education, healthcare, and economic opportunities. By ensuring equal rights, addressing cultural
norms, and providing support for women's empowerment, third world countries can unlock the full
potential of their human capital and foster inclusive growth. Women's participation in decision-
making processes, entrepreneurship, and leadership roles is key to driving sustainable
development and social progress.
The way forward for third world countries also necessitates a focus on cultural preservation and
promotion. Indigenous knowledge, traditions, and cultural heritage play a vital role in identity
formation and community cohesion. Integrating cultural preservation efforts into development
strategies can enhance social cohesion, promote cultural diversity, and foster a sense of pride and
belonging. Embracing cultural tourism, supporting local artisans and craftsmen, and preserving
historical sites contribute to sustainable development while safeguarding cultural heritage for
future generations.
Moreover, international cooperation and fair global trade are essential for the progress of third
world countries. Addressing trade imbalances, promoting fair trade practices, and reducing
barriers to market access can create opportunities for economic growth and poverty reduction.
Access to affordable finance and technology transfer are crucial for developing industries and
fostering innovation. Multilateral institutions and global partnerships should strive to create a level
playing field, providing support and resources to ensure the equitable participation of third world
countries in the global economy.
Q.93)
What are some of the ways in which the third world countries can address environmental risks and
climate change?
1) By investing in sustainable agriculture and renewable energy
2) By exploiting natural resources for economic growth
3) By depending on international aid and funding
(a) Only 1
(b) Only 1 and 3
(c) Only 2
(d) Only 1 and 2
(e) Only 1, 2, and 3
Answer – (a)
pg. 46
[Type here]

Explanation –
Option A is the right answer.

Instructions for Q.94)


Read the passage and answer the questions that follow.
Third world countries, a term that originated during the Cold War era, refer to the developing
nations that faced economic, social, and political challenges in the aftermath of colonialism and
globalization. These countries have diverse histories and trajectories, shaped by colonization,
post-colonial struggles, and attempts at development. The history of third world countries is
marked by the exploitation of resources, cultural suppression, and political subjugation by colonial
powers. The aftermath of independence brought hopes of progress and self-determination, but
many nations grappled with the daunting task of nation-building and overcoming deep-rooted
socio-economic disparities.
Despite the challenges, third world countries possess considerable growth prospects. Several
nations have made significant strides in areas such as education, healthcare, and infrastructure.
Economic reforms, investments in human capital, and advancements in technology have opened
doors for innovation, entrepreneurship, and foreign investments. The emergence of regional
cooperation and trade blocs has also fostered economic integration and increased market
opportunities. However, third world countries continue to face a myriad of challenges. Poverty,
income inequality, and lack of access to basic services remain persistent issues. Political
instability, corruption, and inadequate governance hinder progress and create hurdles for
sustainable development. Environmental degradation, climate change, and natural disasters pose
additional threats to these nations, affecting their agricultural productivity, water resources, and
overall resilience.
To overcome these challenges and pave the way forward, third world countries must prioritize
inclusive and sustainable development. Strengthening governance structures, promoting
transparency, and combating corruption are essential for creating an enabling environment for
growth. Investment in education and skills development can empower the workforce and foster
innovation. Enhanced access to healthcare, clean water, and sanitation is critical for improving the
quality of life. Furthermore, third world countries must leverage technological advancements to
bridge the digital divide and foster digital literacy. Sustainable agricultural practices, renewable
energy initiatives, and climate resilience strategies should be prioritized to mitigate environmental
risks. Collaboration and partnerships with international organizations, developed countries, and
the private sector can provide valuable support in terms of finance, technology transfer, and
capacity-building. In summary, the history, growth prospects, challenges, and the way forward for
third world countries are complex and multifaceted. Achieving sustainable development requires
addressing systemic issues, embracing innovation, and fostering inclusive growth. It necessitates
a global commitment to promoting equity, justice, and shared prosperity.
In addition to economic and social challenges, third world countries also face issues related to
population growth and urbanization. Rapid population growth puts immense pressure on limited
resources and infrastructure, leading to overcrowded cities, inadequate housing, and strained
public services. Urbanization presents both opportunities and challenges, as it can drive economic
growth and innovation but also exacerbate income inequality and social disparities if not properly
managed. Effective urban planning, investment in sustainable urban infrastructure, and social
welfare programs are crucial for creating inclusive and livable cities.
pg. 47
[Type here]

Another critical aspect in the development of third world countries is the empowerment and
inclusion of women. Gender inequality remains a significant obstacle, limiting women's access to
education, healthcare, and economic opportunities. By ensuring equal rights, addressing cultural
norms, and providing support for women's empowerment, third world countries can unlock the full
potential of their human capital and foster inclusive growth. Women's participation in decision-
making processes, entrepreneurship, and leadership roles is key to driving sustainable
development and social progress.
The way forward for third world countries also necessitates a focus on cultural preservation and
promotion. Indigenous knowledge, traditions, and cultural heritage play a vital role in identity
formation and community cohesion. Integrating cultural preservation efforts into development
strategies can enhance social cohesion, promote cultural diversity, and foster a sense of pride and
belonging. Embracing cultural tourism, supporting local artisans and craftsmen, and preserving
historical sites contribute to sustainable development while safeguarding cultural heritage for
future generations.
Moreover, international cooperation and fair global trade are essential for the progress of third
world countries. Addressing trade imbalances, promoting fair trade practices, and reducing
barriers to market access can create opportunities for economic growth and poverty reduction.
Access to affordable finance and technology transfer are crucial for developing industries and
fostering innovation. Multilateral institutions and global partnerships should strive to create a level
playing field, providing support and resources to ensure the equitable participation of third world
countries in the global economy.
Q.94)
Inclusive and sustainable development is a holistic approach that seeks to address social,
economic, and environmental challenges while ensuring that no one is left behind. It recognizes
that economic growth should be equitable, and progress should be shared by all segments of
society. At its core, inclusive and sustainable development aims to foster social cohesion, reduce
inequalities, protect the environment, and promote long-term prosperity. In light of this, what is the
importance of inclusive and sustainable development for the third world countries?
1) It promotes corruption and political instability
2) It improves governance structures and transparency
3) It hinders economic growth and innovation
4) It exacerbates income inequality and poverty
5) It fosters equitable growth and improves quality of life
(a) Only 1, 3, and 5
(b) Only 2 and 5
(c) Only 2, 4, and 5
(d) Only 1 and 3
(e) Only 2 and 4
Answer – (b)
Explanation –
As per the passage, both the statements are correct.

Instructions for Q.95)


Read the passage and answer the questions that follow.

pg. 48
[Type here]

Third world countries, a term that originated during the Cold War era, refer to the developing
nations that faced economic, social, and political challenges in the aftermath of colonialism and
globalization. These countries have diverse histories and trajectories, shaped by colonization,
post-colonial struggles, and attempts at development. The history of third world countries is
marked by the exploitation of resources, cultural suppression, and political subjugation by colonial
powers. The aftermath of independence brought hopes of progress and self-determination, but
many nations grappled with the daunting task of nation-building and overcoming deep-rooted
socio-economic disparities.
Despite the challenges, third world countries possess considerable growth prospects. Several
nations have made significant strides in areas such as education, healthcare, and infrastructure.
Economic reforms, investments in human capital, and advancements in technology have opened
doors for innovation, entrepreneurship, and foreign investments. The emergence of regional
cooperation and trade blocs has also fostered economic integration and increased market
opportunities. However, third world countries continue to face a myriad of challenges. Poverty,
income inequality, and lack of access to basic services remain persistent issues. Political
instability, corruption, and inadequate governance hinder progress and create hurdles for
sustainable development. Environmental degradation, climate change, and natural disasters pose
additional threats to these nations, affecting their agricultural productivity, water resources, and
overall resilience.
To overcome these challenges and pave the way forward, third world countries must prioritize
inclusive and sustainable development. Strengthening governance structures, promoting
transparency, and combating corruption are essential for creating an enabling environment for
growth. Investment in education and skills development can empower the workforce and foster
innovation. Enhanced access to healthcare, clean water, and sanitation is critical for improving the
quality of life. Furthermore, third world countries must leverage technological advancements to
bridge the digital divide and foster digital literacy. Sustainable agricultural practices, renewable
energy initiatives, and climate resilience strategies should be prioritized to mitigate environmental
risks. Collaboration and partnerships with international organizations, developed countries, and
the private sector can provide valuable support in terms of finance, technology transfer, and
capacity-building. In summary, the history, growth prospects, challenges, and the way forward for
third world countries are complex and multifaceted. Achieving sustainable development requires
addressing systemic issues, embracing innovation, and fostering inclusive growth. It necessitates
a global commitment to promoting equity, justice, and shared prosperity.
In addition to economic and social challenges, third world countries also face issues related to
population growth and urbanization. Rapid population growth puts immense pressure on limited
resources and infrastructure, leading to overcrowded cities, inadequate housing, and strained
public services. Urbanization presents both opportunities and challenges, as it can drive economic
growth and innovation but also exacerbate income inequality and social disparities if not properly
managed. Effective urban planning, investment in sustainable urban infrastructure, and social
welfare programs are crucial for creating inclusive and livable cities.
Another critical aspect in the development of third world countries is the empowerment and
inclusion of women. Gender inequality remains a significant obstacle, limiting women's access to
education, healthcare, and economic opportunities. By ensuring equal rights, addressing cultural
norms, and providing support for women's empowerment, third world countries can unlock the full
potential of their human capital and foster inclusive growth. Women's participation in decision-
pg. 49
[Type here]

making processes, entrepreneurship, and leadership roles is key to driving sustainable


development and social progress.
The way forward for third world countries also necessitates a focus on cultural preservation and
promotion. Indigenous knowledge, traditions, and cultural heritage play a vital role in identity
formation and community cohesion. Integrating cultural preservation efforts into development
strategies can enhance social cohesion, promote cultural diversity, and foster a sense of pride and
belonging. Embracing cultural tourism, supporting local artisans and craftsmen, and preserving
historical sites contribute to sustainable development while safeguarding cultural heritage for
future generations.
Moreover, international cooperation and fair global trade are essential for the progress of third
world countries. Addressing trade imbalances, promoting fair trade practices, and reducing
barriers to market access can create opportunities for economic growth and poverty reduction.
Access to affordable finance and technology transfer are crucial for developing industries and
fostering innovation. Multilateral institutions and global partnerships should strive to create a level
playing field, providing support and resources to ensure the equitable participation of third world
countries in the global economy.
Q.95)
Which of the following is not a challenge faced by third world countries?
(a) Income inequality and poverty
(b) Lack of access to basic services
(c) Political instability and inadequate governance
(d) Climate change and environmental degradation
(e) Strong economic growth and technological advancements
Answer – (e)
Explanation –
Option E is correct.

Instructions for Q.96)


Read the passage and answer the questions that follow.
Third world countries, a term that originated during the Cold War era, refer to the developing
nations that faced economic, social, and political challenges in the aftermath of colonialism and
globalization. These countries have diverse histories and trajectories, shaped by colonization,
post-colonial struggles, and attempts at development. The history of third world countries is
marked by the exploitation of resources, cultural suppression, and political subjugation by colonial
powers. The aftermath of independence brought hopes of progress and self-determination, but
many nations grappled with the daunting task of nation-building and overcoming deep-rooted
socio-economic disparities.
Despite the challenges, third world countries possess considerable growth prospects. Several
nations have made significant strides in areas such as education, healthcare, and infrastructure.
Economic reforms, investments in human capital, and advancements in technology have opened
doors for innovation, entrepreneurship, and foreign investments. The emergence of regional
cooperation and trade blocs has also fostered economic integration and increased market
opportunities. However, third world countries continue to face a myriad of challenges. Poverty,
income inequality, and lack of access to basic services remain persistent issues. Political
instability, corruption, and inadequate governance hinder progress and create hurdles for
pg. 50
[Type here]

sustainable development. Environmental degradation, climate change, and natural disasters pose
additional threats to these nations, affecting their agricultural productivity, water resources, and
overall resilience.
To overcome these challenges and pave the way forward, third world countries must prioritize
inclusive and sustainable development. Strengthening governance structures, promoting
transparency, and combating corruption are essential for creating an enabling environment for
growth. Investment in education and skills development can empower the workforce and foster
innovation. Enhanced access to healthcare, clean water, and sanitation is critical for improving the
quality of life. Furthermore, third world countries must leverage technological advancements to
bridge the digital divide and foster digital literacy. Sustainable agricultural practices, renewable
energy initiatives, and climate resilience strategies should be prioritized to mitigate environmental
risks. Collaboration and partnerships with international organizations, developed countries, and
the private sector can provide valuable support in terms of finance, technology transfer, and
capacity-building. In summary, the history, growth prospects, challenges, and the way forward for
third world countries are complex and multifaceted. Achieving sustainable development requires
addressing systemic issues, embracing innovation, and fostering inclusive growth. It necessitates
a global commitment to promoting equity, justice, and shared prosperity.
In addition to economic and social challenges, third world countries also face issues related to
population growth and urbanization. Rapid population growth puts immense pressure on limited
resources and infrastructure, leading to overcrowded cities, inadequate housing, and strained
public services. Urbanization presents both opportunities and challenges, as it can drive economic
growth and innovation but also exacerbate income inequality and social disparities if not properly
managed. Effective urban planning, investment in sustainable urban infrastructure, and social
welfare programs are crucial for creating inclusive and livable cities.
Another critical aspect in the development of third world countries is the empowerment and
inclusion of women. Gender inequality remains a significant obstacle, limiting women's access to
education, healthcare, and economic opportunities. By ensuring equal rights, addressing cultural
norms, and providing support for women's empowerment, third world countries can unlock the full
potential of their human capital and foster inclusive growth. Women's participation in decision-
making processes, entrepreneurship, and leadership roles is key to driving sustainable
development and social progress.
The way forward for third world countries also necessitates a focus on cultural preservation and
promotion. Indigenous knowledge, traditions, and cultural heritage play a vital role in identity
formation and community cohesion. Integrating cultural preservation efforts into development
strategies can enhance social cohesion, promote cultural diversity, and foster a sense of pride and
belonging. Embracing cultural tourism, supporting local artisans and craftsmen, and preserving
historical sites contribute to sustainable development while safeguarding cultural heritage for
future generations.
Moreover, international cooperation and fair global trade are essential for the progress of third
world countries. Addressing trade imbalances, promoting fair trade practices, and reducing
barriers to market access can create opportunities for economic growth and poverty reduction.
Access to affordable finance and technology transfer are crucial for developing industries and
fostering innovation. Multilateral institutions and global partnerships should strive to create a level
playing field, providing support and resources to ensure the equitable participation of third world
countries in the global economy.
pg. 51
[Type here]

Q.96)
Which of the following best describes the term "third world countries"?
(a) Developed nations that faced economic challenges during the Cold War era
(b) Nations that emerged after the end of colonial rule
(c) Developing countries that struggled with economic, social, and political issues
(d) Countries that were aligned with the Soviet Union during the Cold War
(e) Nations that experienced rapid economic growth in the post-colonial period
Answer – (c)
Explanation –
The first paragraph itself is explaining the meaning of the ‘third world countries’.

Q.97)
In the question below, a phrase has been mentioned. The phrase has been used in following
sentences. One of the sentences is using the phrase inappropriately. Identify the sentence with
the incorrect usage.
“Give a big hand”
1) As the curtains drew to a close on the awe-inspiring performance, the audience, captivated by
the breathtaking display of talent, collectively rose to their feet, unleashing a resounding applause
that reverberated through the auditorium, an exultant gesture to give a big hand to the exceptional
artists who had left an indelible mark on their hearts and minds with their extraordinary skill and
artistry.
2) In the ever-evolving landscape of technological advancements, the field of artificial intelligence
has made tremendous strides, progressing by giving a big hand as cutting-edge algorithms,
sophisticated machine learning models, and massive datasets enable unprecedented
breakthroughs, revolutionizing industries and reshaping the way we live, work, and interact,
propelling humanity into a future where the boundaries of possibility continue to expand at an
astonishing pace.
3) Amidst the tense atmosphere of the boardroom, the formidable CEO, known for her
uncompromising demeanour, unleashed a torrent of scathing criticism, meticulously dissecting
each member's performance with a meticulously articulated barrage of words, effectively giving
them a big hand and leaving no room for doubt about her dissatisfaction, as she sought to instil a
sense of accountability and drive for excellence within the team, albeit through a confrontational
and no-holds-barred approach.
(a) Only 2 and 3
(b) Only 1
(c) Only 1 and 3
(d) Only 2
(e) Only 3
Answer – (a)
Explanation –
The phrase “Give a big hand” means to give one a loud or enthusiastic round of applause, usually
as a show of approval or appreciation.

Q.98)

pg. 52
[Type here]

In the question below, a phrase has been mentioned. The phrase has been used in following
sentences. One of the sentences is using the phrase inappropriately. Identify the sentence with
the incorrect usage.
“Thrash out”
1) In the midst of competing priorities and resource constraints, the ambitious project that had
once held great promise and garnered significant attention found itself thrust into the background,
as shifting circumstances and pressing urgencies necessitated a difficult decision to thrash out,
temporarily setting it aside in favor of more immediate and critical endeavors, with the hope that it
would regain prominence and receive the attention it deserved once the current challenges had
been adequately addressed and resources became available.
2) In the crucible of heated debates and intense negotiations, the stakeholders, representing
diverse perspectives and conflicting interests, engaged in an arduous process of discourse and
deliberation, meticulously analyzing each issue, exploring alternative solutions, and persistently
seeking common ground to thrash out a comprehensive agreement that would not only reconcile
their differences but also chart a path forward, effectively resolving the complex challenges that
had hindered progress and cooperation for far too long
3) As the midnight hour approached and exhaustion settled in, the negotiating parties, driven by a
shared determination to find common ground and overcome seemingly insurmountable
differences, immersed themselves in an intense marathon of deliberation, engaging in rigorous
debates, passionately advocating their positions, and meticulously dissecting each contentious
issue, as they endeavored to thrash out a mutually acceptable agreement, recognizing that only
through persistent dialogue, compromise, and a genuine commitment to finding middle ground
could they transcend the impasse that had stymied progress and forge a path towards a
collaborative and sustainable future.
(a) Only 2
(b) Only 1
(c) Only 3
(d) Only 2 and 3
(e) Only 1 and 2
Answer – (b)
Explanation –
The first sentence is using it incorrectly.

Instructions for Q.99)


A paragraph has been mentioned below with blanks in each sentence of the paragraph. The
blanks have been numbered as (1), (2), (3) etc. Answer the questions that follow the paragraph.
The oil dependency of European countries on Russia has emerged as a complex and multifaceted
issue, ____(1)___ woven into the geopolitical dynamics and energy landscapes of the region. The
vast reserves and strategic geographical positioning of Russia have enabled it to become a major
supplier of oil to numerous European nations, creating intricate interdependencies that carry both
benefits and risks. At the heart of the matter lies the ____(2)____ fact that several European
countries heavily rely on Russian oil imports to meet their energy demands. This heavy
dependence exposes them to potential vulnerabilities, as disruptions in the supply chain or
___(3)____ in oil prices can have significant economic and political ramifications. The
concentration of oil imports from a single supplier raises concerns about energy security,
pg. 53
[Type here]

diversification, and the ability to respond effectively to changing global dynamics. Furthermore, the
oil dependency on Russia also creates intricate webs of political and economic ___(4)___.
Q.99)
Fill in the blank (2).
(a) Contestable
(b) Disputable
(c) Undeniable
(d) Doubtable
(e) Refutable
Answer – (c)
Explanation –
Option C fits in the best in the blank.

Instructions for Q.100)


A paragraph has been mentioned below with blanks in each sentence of the paragraph. The
blanks have been numbered as (1), (2), (3) etc. Answer the questions that follow the paragraph.
The oil dependency of European countries on Russia has emerged as a complex and multifaceted
issue, ____(1)___ woven into the geopolitical dynamics and energy landscapes of the region. The
vast reserves and strategic geographical positioning of Russia have enabled it to become a major
supplier of oil to numerous European nations, creating intricate interdependencies that carry both
benefits and risks. At the heart of the matter lies the ____(2)____ fact that several European
countries heavily rely on Russian oil imports to meet their energy demands. This heavy
dependence exposes them to potential vulnerabilities, as disruptions in the supply chain or
___(3)____ in oil prices can have significant economic and political ramifications. The
concentration of oil imports from a single supplier raises concerns about energy security,
diversification, and the ability to respond effectively to changing global dynamics. Furthermore, the
oil dependency on Russia also creates intricate webs of political and economic ___(4)___.
Q.100)
Fill in the blank (1).
(a) Intricately
(b) Complex
(c) Elaborate
(d) Deliberately
(e) Complicatedly
Answer – (a)
Explanation –
Option A fits in the blank aptly.

Instructions for Q.101)


A paragraph has been mentioned below with blanks in each sentence of the paragraph. The
blanks have been numbered as (1), (2), (3) etc. Answer the questions that follow the paragraph.
The oil dependency of European countries on Russia has emerged as a complex and multifaceted
issue, ____(1)___ woven into the geopolitical dynamics and energy landscapes of the region. The
vast reserves and strategic geographical positioning of Russia have enabled it to become a major
supplier of oil to numerous European nations, creating intricate interdependencies that carry both
pg. 54
[Type here]

benefits and risks. At the heart of the matter lies the ____(2)____ fact that several European
countries heavily rely on Russian oil imports to meet their energy demands. This heavy
dependence exposes them to potential vulnerabilities, as disruptions in the supply chain or
___(3)____ in oil prices can have significant economic and political ramifications. The
concentration of oil imports from a single supplier raises concerns about energy security,
diversification, and the ability to respond effectively to changing global dynamics. Furthermore, the
oil dependency on Russia also creates intricate webs of political and economic ___(4)___.
Q.101)
Fill in the blank (3).
(a) Stability
(b) Fluctuations
(c) Consistency
(d) Steadiness
(e) Steadiness
Answer – (b)
Explanation –
Option B fits in the best.

Instructions for Q.102)


A paragraph has been mentioned below with blanks in each sentence of the paragraph. The
blanks have been numbered as (1), (2), (3) etc. Answer the questions that follow the paragraph.
The oil dependency of European countries on Russia has emerged as a complex and multifaceted
issue, ____(1)___ woven into the geopolitical dynamics and energy landscapes of the region. The
vast reserves and strategic geographical positioning of Russia have enabled it to become a major
supplier of oil to numerous European nations, creating intricate interdependencies that carry both
benefits and risks. At the heart of the matter lies the ____(2)____ fact that several European
countries heavily rely on Russian oil imports to meet their energy demands. This heavy
dependence exposes them to potential vulnerabilities, as disruptions in the supply chain or
___(3)____ in oil prices can have significant economic and political ramifications. The
concentration of oil imports from a single supplier raises concerns about energy security,
diversification, and the ability to respond effectively to changing global dynamics. Furthermore, the
oil dependency on Russia also creates intricate webs of political and economic ___(4)___.
Q.102)
Fill in the blank (4).
(a) Disconnection
(b) Isolation
(c) Separation
(d) interplay
(e) Unrelatedness
Answer – (d)
Explanation –
Option D is the correct answer here.

Q.103)

pg. 55
[Type here]

Four statements have been mentioned below. One or more statements may contain an error. It
may contain a grammatical and/or a contextual error. Identify the CORRECT statement. In case,
all the statements are incorrect, then, mark option E, ‘None of the above’ as the answer.
(a) "The Chimney Sweeper" is a poignant poem written by William Blake during the Romantic era,
which delves into the harsh realities of child labour and societal indifference.
(b) One of the prominent themes in "The Chimney Sweeper" is the exploitation and suffering
endured by young chimney sweepers, who were often impoverished children forced into
hazardous labour.
(c) From a social commentary perspective, Blake critiques the oppressive societal structures and
the dehumanization of individuals within industrialized societies
(d) The poem serves as a call to action, urging readers to question and challenge the unjust
systems that perpetuate such suffering.
(e) None of the above
Answer – (e)
Explanation –
All the statements are grammatically correct. Hence, the option E is the right answer.

Q.104)
Five statements have been mentioned below. One or more statements may contain an error. It
may contain a grammatical and/or a contextual error. Identify the CORRECT statement.
(a) Coleridge did employed rich and sensory imagery throughout the poem, creating a vivid and
immersive experience for the reader. The descriptive language, with its cascading waterfalls,
exotic gardens, and sacred rivers, evokes a sense of awe and wonder.
(b) The fragmented structure of "Kubla Khan" is notable. Coleridge famously will claimed that the
poem was a result of an opium-induced dream, and that he was unable to complete it due to an
interruption.
(c) Coleridge will being portrayed the Khan's pleasure-dome as a symbol of an idealized, utopian
world, where nature and human creativity coexist harmoniously. The poem explores the
boundaries between reality and the imaginative realm, blurring the lines between the conscious
and unconscious mind.
(d) "Kubla Khan" is a complex and evocative poem that delves into the themes of imagination,
nature, creativity, and the boundaries of human experience. Through its mesmerizing imagery,
dreamlike structure, and exploration of the power of the mind, Coleridge invites readers to embark
on a journey through the realms of imagination, challenging conventional boundaries and opening
up new vistas of thought and inspiration.
(e) "Kubla Khan" is an transformative power of imagination and its ability to transport the reader to
fantastical realms.
Answer – (d)
Explanation –
Only option D is grammatically correct.

Q.105)
Which of the following sentences from the options given below is not coherent with the central idea
of the other sentences?

pg. 56
[Type here]

(a) The rewriting of history is a phenomenon that occurs when the narratives, interpretations, and
accounts of past events are revised or altered to align with particular ideologies, political agendas,
or societal perspectives.
(b) This process involves reevaluating and reinterpreting historical events, figures, and narratives
through new lenses, often aiming to highlight previously marginalized voices or challenge
dominant narratives.
(c) The act of rewriting history can be a contentious and complex endeavor. It raises questions
about the subjective nature of historical interpretation and the power dynamics that influence the
shaping of historical narratives.
(d) Those who engage in rewriting history often seek to challenge existing power structures, rectify
historical injustices, or provide a more inclusive representation of diverse experiences. However, it
also raises concerns about the potential manipulation of historical facts and the erosion of
objective truth.
(e) The Mughal Period witnessed its highest peak during the reign of Akbar – The Great. As far as
the most intolerant king is concerned in the history of Mughal Empire, Aurangzeb tops the charts.
Answer – (e)
Explanation –
Only option E is not continuing the idea of the other sentences.

Q.106)
Which of the following sentences from the options given below is not coherent with the central idea
of the other sentences?
(a) Culture repatriation, also known as cultural restitution or cultural heritage repatriation, refers to
the return of artifacts, artworks, and cultural objects to their countries of origin. It is a complex and
often contentious process that involves the reclaiming of cultural treasures that were acquired or
removed during colonialism, imperialism, or illicit trade.
(b) The movement of highly-skilled people is called the brain drain. It refers to the migration of
highly skilled and talented individuals from their home countries to seek better opportunities
abroad. This phenomenon has significant implications for both the source and destination
countries.
(c) Culture repatriation is driven by the recognition of the cultural and symbolic significance of
these objects to their original communities and nations. Many countries and indigenous
communities have advocated for the repatriation of cultural artifacts as a means to restore their
cultural identity, reconnect with their heritage, and rectify historical injustices.
(d) The repatriation process typically involves legal, ethical, and diplomatic considerations. It
requires extensive research, documentation, and negotiations between the parties involved,
including the governments, museums, collectors, and indigenous communities.
(e) Advocates argue that culture repatriation promotes cultural diversity, fosters healing and
reconciliation, and supports the self-determination of communities in determining their cultural
heritage.
Answer – (b)
Explanation –
Only option B is not continuing the idea of the other sentences.

Q.107)
pg. 57
[Type here]

In the question given below, a sentence has been divided into different segments. Some of these
segments contain grammatical errors. Identify the part which contains the errors. Ignore the errors
of punctuation, if any.
The persisting issue of the gender pay gap, characterizing by the disparity in earnings between
men and women in the workforce, (1) / is a complex and multifaceted phenomenon that
encompasses a multitude of interconnected factors, including societal norms (2) /, occupational
segregation, discrimination, limited access to career advancement opportunities, and the
undervaluation (3)/ of traditionally female-dominated industries, all of which contribute to the
perpetuation of an unjust (4)/ and inequitable system that undermine gender equality and hinders
the realization of true economic and social progress (5).
(a) 1 and 5
(b) 2 and 3
(c) 3 and 4
(d) 2 and 5
(e) 1 and 3
Answer – (a)
Explanation –
In the first part, ‘characterizing’ is wrong usage. ‘characterized’ should be used.
In the fifth part, ‘undermines’ should be used.

Q.108)
In the question given below, a sentence has been divided into different segments. Some of these
segments contain grammatical errors. Identify the part which contains the errors. Ignore the errors
of punctuation, if any.
Rural development, a multifaceted and intricate process, entails the comprehensive and
sustainable improvement of social, economic, and environmental conditions in rural areas,
encompassing strategies and interventions (1) / that addressing the unique challenges and
opportunities specific to rural contexts, such as agricultural productivity, access to basic services,
infrastructure development, employment generation, poverty alleviation, (2) /land and natural
resource management, social inclusion, and the empowerment of marginalized communities, all of
which necessitating integrated and participatory approaches that engage local stakeholders,
leverage local knowledge and resources, foster collaboration between government, (3) / civil
society, and the private sector, and prioritize the principles of sustainability, equity, and resilience,
with the ultimate goal of enhancing the quality of life, promoting balanced regional development,
(4)/ and realizing the full potential of rural areas as vibrant and self-sustaining centers of economic
growth, social well-being, and cultural heritage preservation (5).
(a) 1 and 2
(b) 2 and 4
(c) 2 and 3
(d) 4 and 5
(e) 1 and 5
Answer – (c)
Explanation –
In 2nd sentence, ‘addressing’ is used wrongly. ‘address’ should be used.
In the 3rd sentence, ‘necessitating’ is used wrongly. ‘necessitate’ should be used.
pg. 58
[Type here]

Q.109)
In the question given below, a sentence has been divided into different segments. Some of these
segments contain grammatical errors. Identify the part which contains the errors. Ignore the errors
of punctuation, if any.
Artificial intelligence (AI), a rapidly evolving and complex field at the intersection of computer
science, mathematics, and cognitive science, encompasses a diverse range of technologies,
algorithms, and methodologies that aim to mimic or simulate human intelligence, (1) / enabling
machines to perform tasks traditionally requiring human cognitive capabilities, such as learning,
problem-solving, pattern recognition, and decision-making, through the utilizing of advanced
machine learning models, neural networks, natural language processing, and data analytics (2) /,
with profound implications across numerous domains and sectors, including healthcare, finance,
transportation, manufacturing, and entertainment, as it holds the potential to revolutionize (3) /
industries, enhance efficiency, accelerate innovation, and being address complex societal
challenges, while also posing ethical, privacy, and policy considerations regarding data
governance, algorithmic bias, human-machine interactions (4)/, and the future of work,
necessitating robust regulations, responsible development, interdisciplinary collaboration, and
ongoing dialogue to ensure that AI is deployed for the greater good, amplifying human capabilities,
promoting inclusivity, and upholding human values and ethical standards (5).
(a) 2 and 4
(b) 1 and 2
(c) 1 and 3
(d) 2 and 5
(e) 4 and 5
Answer – (a)
Explanation –
In the 2nd sentence, ‘utilizing’ is wrong. ‘utilization’ is right.
In the 4th sentence, ‘being address’ is wrong. ‘address’ is correct.

Q.110)
In the question given below, a sentence has been divided into different segments. Some of these
segments contain grammatical errors. Identify the part which contains the errors. Ignore the errors
of punctuation, if any.
Climate change, drived by the accumulation of greenhouse gases in the atmosphere due to
human activities, is an intricate and multifaceted global issue that has led to a wide array of
consequences, (1)/ including rising global temperatures, altered weather patterns, increased
frequency and intensity of extreme events, disruptions (2) / to ecosystems and biodiversity, shifting
agricultural landscapes, water scarcity, and threats to human health and livelihoods, necessitating
a comprehensive and coordinated response (3) / has encompassing mitigation efforts to reduce
emissions, adaptation strategies to address the impacts, international cooperation, sustainable
development practices, and the engagement (4) / of diverse stakeholders across sectors and
regions to ensure a sustainable and resilient future for generations to come (5).
(a) 2 and 3
(b) 1 and 4
(c) 1 and 2
pg. 59
[Type here]

(d) 3 and 4
(e) 2 and 4
Answer – (b)
Explanation –
In the 1st sentence, ‘drived’ is wrong. ‘driven’ is right.
In the 4th sentence, ‘has encompassing’ is wrong. ‘encompassing’ is right.

Quantitative Aptitude

Instructions for Q.111 to Q.115


Consider the below information about 200 laptops being sold by five different stores amongst A, B,
C, D and E in the month of April 2022.

Q.111)
Average number of laptops sold by E, B, C and D is _____ times of average of number of laptops
sold by A, D, E and C together.
5/6
4/5
7/8
8/9
can’t be determined
Answer - (c)
A + B + C + D + E = 100%
2X% + 20% + 22% + X% + 13% = 100%
3X% + 55% = 100%
3X% = 45%
X = 15%
So, A = 30%
And D = 15%
Now, E + B + C + D = 100 – A = 70%
Also, A + C + D + E = 100 – B = 80%
Required fraction = 7/8

Q.112)
If total laptops sold by K is 150% of that of E and out of the total laptops sold by K, 33.33% are
gaming laptops and rest are non-gaming laptops. Find the number of non-gaming laptops sold by
K.
39
26
21
13

pg. 60
[Type here]

30
Answer - (b)
Laptops sold by E = 13% of 200 = 26
Laptops sold by K = 1.5 x 26 = 39
Non-Gaming laptops sold by K = 2/3 x 39 = 26

Q.113)
Find the central angle (in degrees) for store A.
120
144
72
108
54
Answer - (d)
Laptops sold by A = 30%
Laptops sold by A (in degrees) = 30/100 x 360 = 108

Q.114)
Assume the total number of laptops sold by these 5 stores in May 2022 is 400. The percentage
share of B and E is interchanged while for the rest it is the same as given. Find the difference
between the number of laptops sold by store B and C in May 2022.
32
28
8
36
40
Answer - (d)
Percentage share Laptops sold
A 30% 120
B 13% 52
C 22% 88
D 15% 60
E 20% 80

Q.115)
In the next month, the number of laptops sold by store C increased by 25% and the number of
laptops sold by store D reduced by one-fifth. What is the average of total number of laptops sold
by all the five stores together in the next month?
39
41
42
40
44
Answer - (b)
In the next month,
pg. 61
[Type here]

Increase in laptops sold by C = 0.25 x (22% x 200) = 11


Laptops sold by D = 1/5 x (15% x 200) = 6
Net increase in total laptops sold = 11 – 6 = 5
So, total laptops sold = 200 + 5 = 205
New average = 205/5 = 41

Q.116)
Consider the below series:
12, 21, 46, 95, 176, X, Y
What is the sum of X and Y?
[a] 737
[b] 805
[c] 798
[d] 712
[e] 763
Answer - (e)
The pattern is as follows:
12 + 32 = 21
21 + 52 = 46
46 + 72 = 95
95 + 92 = 176
176 + 112 = 297 = X
297 + 132 = 466 = Y
X + Y = 297 + 466 = 763

Q.117)
Below are given two series which follow certain pattern. There are two missing values X and Y.
Series I: 158, 78, 38, 18, 8, X
Series II: 9, Y, 46, 84, 135, 199
Which of the following statements is/are true?
1. X = 7Y
2. 3 < X < Y < 21
3. 4(Y + 3.5) = 11(X)2 – 1
Both 1 and 2
Both 1 and 3
Only 3
All 1, 2 and 3
None
Answer - (c)
The pattern of series I is as follows:
158 – 80 = 78
78 – 40 = 38
38 – 20 = 18
18 – 10 = 8
8–5=3=X
pg. 62
[Type here]

The pattern of series II is as follows:

So, X = 3
And 7Y = 21 x 7 = 147
So, 3 ≠ 7
Hence, statement 1 is not true.
3 < X < Y < 21 is not true since X = 3 and Y = 21.
So, statement 2 is not true.
4(Y + 3.5) = 11(X)2 – 1
4(21 + 3.5) = 11(9) – 1
4(24.5) = 98
98 = 98
So, statement 3 is true.

Instructions for Q.118 and Q.119


Consider the below two quadratic equations:
I. 4x2 – 16x + C = 0
II. 2y2 – 3y + 1 = 0
One of the roots of equation I is 1.5
Q.118)
Find the sum of the largest roots of equation I and II.
3.5
4.5
3
4
5.5
Answer - (a)
4x2 – 16x + C = 0
Now, substitute 1.5 for x in order to find out the value of ‘C’
4(1.5)2 – 16(1.5) + C = 0
4(2.25) – 16(1.5) + C = 0
9 – 24 + C = 0
C = 15
Sum of roots = -b/a = 16/4 = 4
So, the other root of equation I = 4 – 1.5 = 2.5 (larger root)
Consider 2y2 – 3y + 1 = 0
Roots = {-b ± √(b2 – 4ac)} / 2a
{3 ± √(9 – 8)} / 4
Roots = 4/4, 2/4
Roots = 1, 0.5
So, sum of larger roots = 2.5 + 1 = 3.5

Q.119)
Find the sum of the digits of C.
[a] 5
pg. 63
[Type here]

[b] 4
[c] 8
[d] 6
[e] 7
Answer - (d)
4x2 – 16x + C = 0
Now, substitute 1.5 for x in order to find out the value of ‘C’
4(1.5)2 – 16(1.5) + C = 0
4(2.25) – 16(1.5) + C = 0
9 – 24 + C = 0
C = 15
Sum of digits of C = 1 + 5 = 6

Instructions for Q.120 and Q.121


Certain students comprising of boys and girls are studying in an institute which has three sections
A, B and C. The ratio of boys and girls is 3 : 5. 50% students are in section A. The number of girls
in section B is equal to the number of boys in section C. The number of boys in section A is equal
to the number of students in section B. 30% of the girls are in section C.
Q.120)
Let P = number of girls in section B and let Q = number of boys in section A.
Which of the following statements are correct?
I. P is approximately 67% of Q
II. 33 1/3% of total number of girls = 20% of total boys = P
III. 3P = 2Q
Only II and III
Only I
All I, II and III
Only III
Only I and III
Answer - (e)
Boys Girls Total
A 3x 5x 8x
B 3x – y y 3x
C y 5x – y 5x
Total 6x 10x 16x

30% girls are in section C


So, 0.3(10x) = 5x – y
3x = 5x – y
2x = y
Boys Girls Total
A 3x 5x 8x
B x 2x 3x
C 2x 3x 5x
Total 6x 10x 16x
pg. 64
[Type here]

P = number of girls in section B = 2x


Q = number of boys in section A = 3x
P = 2/3 of Q
2x = 2/3 (3x)
2x = 2x
So, statement I is true.
33 1/3% of total number of girls = 1/3 (10x) = 10x/3
20% of total boys = 0.2(6x) = 1.2x
P = 2x
So, statement II is not true.
3P = 2Q
3(2x) = 2(3x)
6x = 6x
So, statement III is true.

Q.121)
What percent boys are in section B?
33.33%
16.67%
25%
15%
20%
Answer - (b)
Boys Girls Total
A 3x 5x 8x
B X 2x 3x
C 2x 3x 5x
Total 6x 10x 16x

Total boys = 6x
Boys in section B = x
Required percentage = x / 6x = 1/6 = 16.67%

Q.122)
Ansh borrowed 80 books from library A and 120 from library B. Some books are of English
language while some are of regional languages. X% of total books were in English language.
Number of books in regional language are equal to 1.25 times the books in library B. What is the
value of X?
50
33.33
25
20
40
Answer - (c)
pg. 65
[Type here]

Number of books in regional language = 1.25 x 120 = 150


Number of English books = 200 – 150 = 50
Number of English books in percentage = 50/200 x 100 = 25%
So, X = 25

Q.123)
Consider the below given two equations. You are given two quantities below the equations namely
QI and QII. You are required to calculate both the quantities and then compare them.
Y: ax2 – bx + 1 = 0; where 4 < a < b < 8 and ‘a’ and ‘b’ are distinct whole numbers
Z: d/e + c/f – 1 = 3; where 1 < e, c, f < d < 7 and ‘d’, ‘e’, ‘c’ and ‘f’ are all distinct whole numbers
QI: Value of smallest root of Y
QII: Value of reciprocal of smallest number among d, c, e and f
QI > QII
QI < QII
QI ≤ QII
QI ≥ QII
QI = QII or no relation can be established
Answer - (b)
Equation Y will have the smallest roots when a = 5 and b = 7.
Smallest Root = (7 - √(49 – 20)) / 10
QI = Smallest root = 0.161 (approx.)
d/e + c/f – 1 = 3
d/e + c/f = 4
6/4 + 5/2 = 1.5 + 2.5 = 4
Also, 6/3 + 4/2 = 2 + 2 = 4
So, d = 6
e = 4 or 3
c = 5 or 4
f=2
So, ‘f’ is the smallest among the four and its reciprocal is 1/2 = 0.5 = QII
Hence, QII > QI

Q.124)
Below are given two expressions and then two quantities (QI and QII) below it. Compare the two
quantities and mark your answer accordingly.
Expression 1: 1/a + 1/b + 1
Expression 2: a3 – b2 – 4
3 < a < b < 10
And a and b are integers
Q1: Maximum value of expression 1 + Minimum value of expression 2
Q2: (- 9)
QI > QII
QI < QII
QI ≤ QII
QI ≥ QII
pg. 66
[Type here]

QI = QII or no relation can be established


Answer - (b)
1/a + 1/b + 1
The above expression will have the maximum value when ‘a’ and ‘b’ are minimum.
1/a = 1/4
1/b = 1/5
So, 1/4 + 1/5 + 1 = (5 + 4 + 20) / 20 = 29/20
a3 – b2 – 4
The above expression will have minimum value when ‘a’ is minimum and ‘b’ is maximum
So, b = 9 and a = 4
43 – 92 – 4 = 64 – 81 – 4 = -21
So, (29/20 – 21) < (- 9)
Or, QI < QII

Q.125)
Anshu invested Rs. P in scheme X for 3 years at Compound Interest @ 20% per annum and Ronit
invested 36 2/5% of P for T years in scheme Y at Simple Interest @ 10% per annum. Interest
earned is equal in both the schemes.
Which of the following statement(s) is/are correct?
I. P is greater than 3000.
II. T is greater than 19.
III. P is certainly a multiple of 250.
IV. T is less than 25.
I, II and IV only
II, III and IV only
II only
II and IV only
All I, II, III and IV
Answer - (d)
Interest earned by Anshu = P{(1.2)3 – 1}
36 2/5 % of P = 182/500 x P = 91P/250
Interest earned by Ronit = (91P x 10 x T) / (100 x 250)
P{6/5 x 6/5 x 6/5 – 1} = 91PT/2500
216/125 – 1 = 91T/2500
91/125 = 91T/2500
T = 2500/125 = 20 years
We cannot find out the value of ‘P’.

Q.126)
Consider the below statements and then answer the question accordingly:
I: X is a two digit number in which the tenth digit is 'B' and the unit digit is 'A'; where A is greater
than 1 and A < B.
II: Unit digits of A2, A3, A4 are same, where A is unit digit of given number.
III: Multiplication of both 'A' and 'B' is a multiple of 7.
If Y is the number obtained on interchanging the digits of X, then, Y is closest to,
pg. 67
[Type here]

69
58
76
64
74
Answer - (b)
From statement I, X = BA; where A > 1 and A < B
From statement II, A = 5 or 6
So, the number X = B5 or B6
From statement III, B = 7 (7x5 = 35 which is a multiple of 7 and 7x6 = 42 which is also a multiple of
7)
So, X = 75 or 76
Y = 57 or 67
So, 58 is the closest value from the given options.

Q.127)
P litres of mixture contains milk and water, in which water is 25%.
Which of the following statements is/are true?
1. If P/4 Iitres of water is added to the mixture, then water becomes 40% of the total mixture.
2. If 9P/44 Iitres of milk is removed from the mixture and replaced with same quantity of water,
then the ratio of milk and water becomes 6:5.
3. 20% of the mixture is removed and replaced with water. This process is repeated one more
time. The ratio of milk and water now is 24:25.
Only 1
Only 1 and 2
Only 2 and 3
Only 1 and 3
None of these
Answer - (b)
Original Milk = 3P/4
Original water = P/4
In Statement 1,
New quantity of water = P/4 + P/4 = P/2
New ratio of milk and water = 3/4 : 1/2 = 3/4 : 2/4 = 3 : 2
So, water becomes 40% of the total mixture.
Hence, statement 1 is true.
In statement 2,
New quantity of milk = 3P/4 - 9P/44 = 24P/44
New quantity of milk = P/4 + 9P/44 = 20P/44
New ratio = 24 : 20 = 6 : 5
So, statement 2 is also true.
In statement 3,
New quantity of milk = 3P/4 x 4/5 x 4/5 = 12P/25
New quantity of water = P – 12P/25 = 13P/25
Ratio = 12 : 13 = 24 : 26
pg. 68
[Type here]

So, statement III is false.

Q.128)
A circle of radius “r” metre is inscribed in a rectangle of length Y metre such that the circle touches
both the sides of the rectangle. The cost of fencing the circle and rectangle is Rs 4620 and Rs
8820 respectively. The cost of fencing per metre is the same for both circle and rectangle. Find the
perimeter of a square inscribed inside the circle.
2√2Y metres
√2Y/3 metres
3Y/√2 metres
√2Y metres
2Y metres
Answer - (d)
Perimeter of circle = 2 x 22/7 x r
Perimeter of rectangle = 2(l + b)
L=Y
B = 2r (since the diameter of the circle is equal to the breadth of the rectangle)
Let cost of fencing per metre be ‘P’
(2(Y + 2r) x P) / (2 x 22/7 x r x P) = 8820/4620
(Y + 2r)/(22/7 x r) = 21/11
(Y + 2r)/r = 6
Y + 2r = 6r
Y = 4r
Diagonal of the square = diameter of the circle = 2r
So, side of the square = √2r
Perimeter of the square = 4√2r = √2Y metres

Q.129)
In which of the following statements, profit percentage are equal?
I. An article with cost price of Rs 2200 and marked price of Rs 3162.50 which is offered at a
discount of 20%.
II. An article is sold for Rs 8400 whose marked price is Rs 10800 which is 150% of the cost price
of the article.
III. An article is sold at a discount of Rs 1650 which is equal to 20%. The profit earned is Rs 1100.
IV. An article whose cost price is Rs 4200 is sold at a discount of 25% on the list price. The
discount offered in Rs 1610.
II and III only
I and II only
II and IV only
III and IV only
I and IV only
Answer - (e)
From statement I,
SP = 3162.5 x 0.8 = 2530
Profit% = (2530 – 2200) / 2200 = 15%
pg. 69
[Type here]

From statement II,


CP = 10800/1.5 = 7200
Profit% = (8400 – 7200)/7200 = 16.67%
From statement III,
20% of MP = 1650
MP = 1650/0.2 = 8250
SP = 8250 – 1650 = 6600
CP = 6600 – 1100 = 5500
Profit% = 1100/5500 = 20%
From statement IV,
25% of MP = 1610
MP = 1610/0.25 = 6440
SP = 6440 – 1610 = 4830
Profit% = (4830 – 4200)/4200 = 15%

Instructions for Q.130 to Q.134


Consider the below data about 5 watches as follows:
Watches Loss Discount % Marked Price
A ---- 23 1800
B 140 15 ----
C 228 22 ----
D 142 11 2200
E ---- ---- 1200

Average loss of B, C, D and E is 183.25

Q.130)
If the cost price of watch A is 80% of the cost price of watch D, then what is the loss incurred on
selling watch A?
Rs 294
Rs 223
Rs 284
Rs 420
Rs 304
Answer - (a)
SP of watch D = 2200 x 0.89 = 1958
CP of D = 1958 + 142 = 2100
CP of A = 2100 x 0.8 = 1680
SP of A = 1800 x 0.77 = 1386
Loss on A = 1680 – 1386 = Rs 294

Q.131)
Considering the additional information given in the previous question, what is the average loss
incurred on all the five watches?
Rs 204.4
pg. 70
[Type here]

Rs 207.4
Rs 203.4
Rs 205.4
Rs 206.4
Answer - (d)
Average loss on B, C, D and E = 183.25
(140 + 228 + 142 + E) / 4 = 183.25
510 + E = 733
Loss on E = 223
So, average loss = (294 + 140 + 228 + 142 + 223) / 5 = 1027/5 = Rs 205.4

Q.132)
If the loss percentage incurred on watch B is 16%, then the marked price of watch B is
approximately how much percent of the marked price of watch E?
75%
64%
72%
61%
84%
Answer - (c)
16% of CP of B = 140
CP of B = 875
SP of B = 875 – 140 = 735
MP of B = 735/0.85 = 864.7
Required percentage = 864.7/1200 = 72% (approx.)

Q.133)
If the average of the marked price of watches B, C and D is Rs 2400 and the ratio of the marked
price of watches B and C is 16 : 9, then find the sum of the cost price of watches B and C?
Rs 4352
Rs 4264
Rs 5000
Rs 4124
Rs 4492
Answer - (e)
Total MP of B, C and D = 2400 x 3 = 7200
Sum of MP of B and C = 7200 – 220 = 5000
MP of B = 16/25 x 5000 = 3200
MP of C = 9/25 x 5000 = 1800
SP of B = 3200 x 0.85 = 2720
SP of C = 1800 x 0.78 = 1404
CP of B = 2720 + 140 = 2860
CP of C = 1404 + 228 = 1632
CP of B + CP of C = 2860 + 1632 = 4492

pg. 71
[Type here]

Q.134)
If the ratio of cost price and marked price for watch B is 6 : 7, then find the selling price of watch B.
Rs 16380
Rs 16660
Rs 19600
Rs 16800
Rs 17240
Answer - (b)
Let CP be 6y and MP be 7y
So, SP = 6y – 140
Also, SP = 7y x 0.85
Equating both the expressions,
6y – 140 = 7y x 0.85
6y – 140 = 5.95y
0.05y = 140
y = 2800
SP = 6(2800) – 140 = 16660

Instructions for Q.135 to Q.138


A game is being played consisting of three rounds. Only 4 balls are being played in each round.
In round I, on each ball either 2 or 4 points are being scored
In round II, on each ball either 3 or 6 points are being scored
In round III, on each ball either 1 or 5 points are being scored
Difference between points scored on 1st ball in round I and II is 4.
Average points scored on 1st, 2nd and 3rd ball in round I is 3 1/3.
Points scored on 2nd ball in round II is less than points scored on 2nd ball in rounds I and III
individually
Points scored on 3rd ball in rounds II and III together is 11
Points scored on 1st and 4th ball in round III is less than that scored on 2nd and 3rd ball
Runs scored on 4th ball in rounds I, II and III together is 8
Runs scored on all balls on round 3 together is not more than 12.

Q.135)
What is the difference between the runs scored on Ball 1 in round II and runs scored on ball 4 in
round III?
6
5
4
3
2
Answer - (b)
Round I Round II Round III
Ball 1 2 6 1 9
Ball 2 4 3 5 12
Ball 3 4 6 5 15
pg. 72
[Type here]

Ball 4 4 3 1 8
14 18 12

Required difference = 6 – 1 = 5

Q.136)
What is the average of runs scored on all balls in round II?
3
3.75
3.5
4.5
5
Answer - (d)
Round I Round II Round III
Ball 1 2 6 1 9
Ball 2 4 3 5 12
Ball 3 4 6 5 15
Ball 4 4 3 1 8
14 18 12

Average = 18/4 = 4.5

Q.137)
On which ball were the highest runs scored taking all the rounds together?
Ball 1
Ball 2
Ball 3
Ball 4
There are two balls on which same highest number of runs were scored
Answer - (c)
Round I Round II Round III
Ball 1 2 6 1 9
Ball 2 4 3 5 12
Ball 3 4 6 5 15
Ball 4 4 3 1 8
14 18 12

Q.138)
Which of the following statements is correct?
The maximum runs were scored on ball 3 of round III.
The runs scored on ball 4 in round II is more than the runs scored on the same ball in round I.
The total runs scored in round III is 8.
Average runs scored on ball 3 is 5.
The sum of runs scored on ball 3 of round II and ball 2 of round III is 10.
Answer - (d)
pg. 73
[Type here]

Round I Round II Round III


Ball 1 2 6 1 9
Ball 2 4 3 5 12
Ball 3 4 6 5 15
Ball 4 4 3 1 8
14 18 12

Q.139)
Find the ratio of age of Rahul after 6 years and age of Tina before 2 years.
Statement 1: The sum of present ages of Rahul and Tina is 88 years.
Statement 2: Tina is 26 years and 16 years younger than Rahul and Anjali respectively.
Statement 3: Ratio of present ages of Rahul and Anjali is 7:6.
The answer can be calculated by using
I. 1 and 2 together
II. 2 and 3 together
III. 1 and 3 together
Only I
Both I and II
Both I and III
Both II and III
None of these
Answer – (b)
From statement 1,
R + T = 88
From statement 2,
T + 26 = R
T + 16 = A
R – 26 = A – 16
R – A = 10
From statement 3,
R = 7y; A = 6y
Combining 1 and 2,
T = 31; R = 57
So, 1 and 2 together are sufficient.
Combining 2 and 3,
R – A = 10
So, 7y – 6y = 10
y = 10
So, R = 70 and A = 60
Also, T = R – 26
T = 70 – 26 = 44
So, 2 and 3 together are also sufficient.

Q.140)
pg. 74
[Type here]

A car with speed of x km/hr can cover a distance of 2d km in 8 hrs. Speed of a boat in still water is
x km/hr and speed of the current is y km/hr. The boat can travel d km upstream and return to the
initial point in 8(1/3) hrs. Time taken by the boat to cover 72 km downstream is 2 hrs more than to
cover 24 km upstream. Find the time taken by the boat to cover 108km in still water.
i. (2y(x+y))/x
ii. {3y(x-y)} / x
iii. {2x(x+y)} / y
Only i and iii
Only ii
Only i and ii
All i, ii and iii
None of the above
Answer - (c)
Speed of car = x km/hr
So, x = 2d/8 = d/4
Or, d = 4x
d/(x-y) + d/(x+y) = 25/3
or, 4x/(x-y) + 4x/(x+y) = 25/3
(4x2 + 4xy + 4x2 – 4xy)/(x2 – y2) = 25/3
8x2/(x2 – y2) = 25/3
24x2 = 25x2 – 25y2
x2 = 25y2
x = 5y
Also, 72/(x+y) = 2 + 24/(x-y)
72/6y = 2 + 24/4y
12/y = 2 + 6/y
6/y = 2
y=3
So, x = 5y = 15
And d = 4x = 60
Required time = 108/x = 108/15 = 36/5 hours
Now check for the expressions:
(2y(x+y))/x = 6(18)/15 = 108/15; so (i) holds
{3y(x-y)}/x = 9(12)/15 = 108/15; so (ii) holds
{2x(x+y)}/y = 30(18)/3 = 180; so (iii) does not hold

Reasoning

Q.141)
In the word ‘GLASTONBURY’, change every vowel after k to the third consonant after it...and
every consonant to previous letter and then after changing, delete the letters that appear more
than once and vowels if left.
Which of the following letter will come in the middle of the newly formed word?

pg. 75
[Type here]

X
R
K
S
M
Answer - (d)
G–1=F
L–1=K
A=A
S -1 = R
T–1=S
O+3=R
N–1=M
B–1=A
U+3=X
R–1=Q
Y–1=X
The word formed is ‘FKARSRMAXQX’
Now delete the vowels and letters that appear more than once.
FKRSMXQ
So, letter ‘S’ comes in the middle.

Q.142)
Consider the following words which have one blank each. You are required to fill the blanks from
one of the given options so as to form meaningful English word.
K __ F A
R B __ L
__ L G O
E T L __
SATO
LUCW
GOEF
NEWB
EIJM
Answer - (b)
The meaningful words formed are: FALK, BLUR, CLOG and WELT.

Instructions for Q.143 to Q.147


Input: erase 47 92 wound limbo 36 prove 85 28 hasty 63 cards
Step 1: woune erase 47 92 limbo 36 prove 85 hasty 63 cards 29
Step 2: provf woune erase 47 92 limbo 85 hasty 63 cards 29 37
Step 3: limbp provf woune erase 92 85 hasty 63 cards 29 37 48
Step 4: hastz limbp provf woune erase 92 85 cards 29 37 48 64
Step 5: erasf hastz limbp provf woune 92 cards 29 37 48 64 86
Step 6: cardt erasf hastz limbp provf woune 29 37 48 64 86 93
pg. 76
[Type here]

Step 6 is the last step.


Find the final output and various steps for the below given input:
Input: bride 39 master 77 61 nocturnal yatch 25 feel 84 18 train

Q.143)
Which of the following is the third element from the right end in step 3 for the above input?
26
yatci
19
feel
84
Answer - (c)
Input: erase 47 92 wound limbo 36 prove 85 28 hasty 63 cards
Step 1: woune erase 47 92 limbo 36 prove 85 hasty 63 cards 29
In Step 1, the last word as per dictionary (from the given words) has been placed on the extreme
left and the last letter of the word has been replaced with the succeeding letter. Also, the smallest
number has been placed at the extreme right and 1 is added to the number.
Step 2: provf woune erase 47 92 limbo 85 hasty 63 cards 29 37
In Step 2, again the last word as per dictionary (after the word ‘wound’) has been placed at the
extreme left and last letter of the word has been replaced with the succeeding letter. Also, the next
smallest number has been placed at the extreme right and 1 is added to the number.
This process is being repeated till all the words are placed on the left and all the numbers on the
right.
Now,
Input: bride 39 master 77 61 nocturnal yatch 25 feel 84 18 train
Step 1: yatci bride 39 master 77 61 nocturnal 25 feel 84 train 19
Step 2: traio yatci bride 39 master 77 61 nocturnal feel 84 19 26
Step 3: nocturnam traio yatci bride master 77 61 feel 84 19 26 40
Step 4: mastes nocturnam traio yatci bride 77 feel 84 19 26 40 62
Step 5: feem mastes nocturnam traio yatci bride 84 19 26 40 62 78
Step 6: bridf feem mastes nocturnam traio yatci 19 26 40 62 78 85
Step 6 is the final output.
Instep 3, third element from the right end is ‘19’

Q.144)
The elements ‘bride 77 feel’ come together in the same order in which of the below steps?
Step 2
Step 4
Step 5
Step 6
Step 3
Answer - (b)
Input: bride 39 master 77 61 nocturnal yatch 25 feel 84 18 train
Step 1: yatci bride 39 master 77 61 nocturnal 25 feel 84 train 19
Step 2: traio yatci bride 39 master 77 61 nocturnal feel 84 19 26
pg. 77
[Type here]

Step 3: nocturnam traio yatci bride master 77 61 feel 84 19 26 40


Step 4: mastes nocturnam traio yatci bride 77 feel 84 19 26 40 62
Step 5: feem mastes nocturnam traio yatci bride 84 19 26 40 62 78
Step 6: bridf feem mastes nocturnam traio yatci 19 26 40 62 78 85
Step 6 is the final output.

Q.145)
Which of the following elements is fourth to the left of element which is seventh from the left end of
Step 5?
mastes
traio
62
40
nocturnam
Answer - (e)
Input: bride 39 master 77 61 nocturnal yatch 25 feel 84 18 train
Step 1: yatci bride 39 master 77 61 nocturnal 25 feel 84 train 19
Step 2: traio yatci bride 39 master 77 61 nocturnal feel 84 19 26
Step 3: nocturnam traio yatci bride master 77 61 feel 84 19 26 40
Step 4: mastes nocturnam traio yatci bride 77 feel 84 19 26 40 62
Step 5: feem mastes nocturnam traio yatci bride 84 19 26 40 62 78
Step 6: bridf feem mastes nocturnam traio yatci 19 26 40 62 78 85
Step 6 is the final output.
‘84’ is seventh from the left end in Step 5.
The element which is fourth to the left of ‘84’ in step 5 is ‘nocturnam’.

Q.146)
Which of the following elements are placed in the middle in step number 6?
bride 84
77 feel
traio 26
yatci 19
bridf 85
Answer - (d)
Input: bride 39 master 77 61 nocturnal yatch 25 feel 84 18 train
Step 1: yatci bride 39 master 77 61 nocturnal 25 feel 84 train 19
Step 2: traio yatci bride 39 master 77 61 nocturnal feel 84 19 26
Step 3: nocturnam traio yatci bride master 77 61 feel 84 19 26 40
Step 4: mastes nocturnam traio yatci bride 77 feel 84 19 26 40 62
Step 5: feem mastes nocturnam traio yatci bride 84 19 26 40 62 78
Step 6: bridf feem mastes nocturnam traio yatci 19 26 40 62 78 85
Step 6 is the final output.

Q.147)

pg. 78
[Type here]

What is the sum of the fourth element from the right end in step 3 and eighth element from the left
end in step 5?
[a] 103
[b] 96
[c] 168
[d] 38
[e] 161
Answer - (a)
Input: bride 39 master 77 61 nocturnal yatch 25 feel 84 18 train
Step 1: yatci bride 39 master 77 61 nocturnal 25 feel 84 train 19
Step 2: traio yatci bride 39 master 77 61 nocturnal feel 84 19 26
Step 3: nocturnam traio yatci bride master 77 61 feel 84 19 26 40
Step 4: mastes nocturnam traio yatci bride 77 feel 84 19 26 40 62
Step 5: feem mastes nocturnam traio yatci bride 84 19 26 40 62 78
Step 6: bridf feem mastes nocturnam traio yatci 19 26 40 62 78 85
Step 6 is the final output.
Fourth element from the right end in step 3 = 84
Eighth element from the left end in step 5 = 19
Sum = 84 + 19 = 103

Instructions for Q.148 to Q.151


Below question consist of some conclusions followed by some statements in the option. Study the
following information carefully and decide which of the following statement is true for the given
conclusion.

Q.148)
Conclusion:
Some Briefcase are not Umbrella.
Some Umbrella not being Charger is a possibility.
Some Truck can never be Charger.

Some Umbrella are Truck. All Truck are Briefcase. No Briefcase is Umbrella. Some Umbrella are
Charger. All Glass are Charger.
All Briefcase is Umbrella. Some Umbrella are Charger. Some Charger are Truck. All Truck are
Glass. No Glass is Water.
Some Truck are Water. All Water are Umbrella. No Umbrella is Charger. Some Charger are
Briefcase. All Briefcase are Glass.
Only Water are Truck. Some Glass are Briefcase. No Umbrella is Glass. Some Water are
Umbrella. All Briefcase are Charger.
Some Briefcase are Umbrella. All Umbrella are Charger. No Charger is Truck. All Truck are Glass.
Some Glass are not Water.
Answer - (d)
Option B gets eliminated because it says ‘All Briefcase is Umbrella’ whereas in the conclusion it is
given ‘Some Briefcase are not Umbrella’.

pg. 79
[Type here]

Option C is also eliminated because it says ‘No Umbrella is Charger’ which is a definite
conclusion. So the conclusion ‘Some Umbrella not being Charger is a possibility’ does not follow
because ‘no umbrella is charger’ is a definite conclusion and hence not a possibility.
Option E is also eliminated because it contains ‘All umbrella are charger’ whereas in the
conclusion it is given ‘Some Umbrella not being Charger is a possibility.’
Option A is also eliminated as it is not possible to make a diagram of it.
So, option D should be the answer. The basic diagram of it is as follows:

Since only water are truck, so truck cannot be anything else except water. So, all the three
conclusion follows from statements in Option D.

Q.149)
Conclusion:
Some football can never be Chess.
Some Volleyball not being Boxing is a possibility.
Some Boxing can be Cricket.
All Cricket are Football. No Football is Volleyball. Some Boxing are Chess. No Chess is Cricket.
Some Cricket are Football. All Football are Volleyball. Some Chess are Boxing. No Boxing is
Volleyball. No Chess is Cricket.
Only Football are Cricket. Some Football are Chess. No chess is Volleyball. All Volleyball are
Boxing.
Some Cricket are Football. All Football are Chess. Some Chess are Boxing. Some Boxing are
Volleyball.
No Cricket is Football. Some Football are Volleyball. All Boxing are Chess. No Chess is Cricket.
Answer - (a)
Option B is eliminated because it says ‘No Boxing is Volleyball’ which makes the conclusion
‘Some Volleyball not being Boxing is a possibility’ false because ‘no boxing is volleyball’ is a
definite statement and hence we cannot get a possible conclusion between volleyball and boxing.
Option C is eliminated because it contains ‘all volleyball are boxing’ which makes the conclusion
‘some volleyball not being boxing is a possibility’ false.
Option D is eliminated because it says ‘All football are chess’ which makes the conclusion ‘some
football can never be chess’ false.
Option E is eliminated because it says ‘All Boxing are Chess. No Chess is Cricket’ which implies
‘No boxing is Cricket’ which makes the conclusion ‘Some boxing can be cricket’ false.
The basic diagram for Option A is as follows:

We can draw possible diagrams for option A to check for possible conclusions.

Q.150)
Conclusion:
I) Some N are R.
II) All O which are N is also S is a possibility.
III) Some Z are R.
Some M are R. Only a few R are O. No O is N. Some N is S. All S are Z.
Some M are O. Only a few O are R. No R is S. Some S is N. All N are Z.
pg. 80
[Type here]

Some M are O. Only a few O are R. No N are R. Some N are S. All S are Z.
Some O are R. Some M are O. Some R are N. Some N are S. All N are Z.
All Follows.
Answer - (d)
Option C is eliminated because it contains ‘No N are R’ whereas conclusion I contains ‘some N
are R’.
So, Option E is also eliminated.
Option A is also eliminated because it contains ‘No O is N’ which makes conclusion II false.
Option B is also eliminated because ‘some N are R’ and ‘some Z are R’ does not always follow.
This is because option B contains ‘No R is S’.
So, option D is the answer and its basic diagram is as follows:

Q.151) Conclusion:
Some Red are not Carrot.
Some Chili being not Garlic is a possibility.
Some Green are not Tomato.
Some Garlic are not Chili. All Garlic are Red. No Red is Carrot. Some Carrot are Green. All Green
are Tomato.
All Red are Tomato. Some Tomato are Green. No Green is Chili. All Chili are Carrot. Some Garlic
are Carrot.
All Red are Chili. All Tomato are Garlic. Some Garlic are Carrot. No Chili is Carrot. Some Green
are Chili. No Green is Garlic.
All Carrot are Chili. Some Chili are Tomato. No tomato is Green. Some Green are Garlic. No
Garlic is Red.
None of these
Answer - (c)
Option A is eliminated because it contains ‘all green are tomato’ which makes conclusion III false.
From option B, we can draw the below possible diagram which makes conclusion I false.

So, Option B is also eliminated.


From Option C,

So, Option C is the answer as all the conclusions follow from Option C.

Q.152)
Which of the following symbols should be placed in the blank spaces in order to complete the
given expression in such a manner that makes the expressions ‘J ^ F' and 'K ? G' true?
K ? J % H ___ G ^ F ! E
^
+
%
?
Either (a) or (c)
Answer - (e)
pg. 81
[Type here]

‘J ^ F’ means J ≥ F
'K ? G' means K = G
Now, the questions says make both the expressions ‘true’ but not ‘definitely true’.
K ? J % H ___ G ^ F ! E
K > J = H ___ G ≥ F < E
So, either ‘=’ or ‘≥’ will make both the expressions true.
So, Option E is the right answer.

Instructions for Q.153 to Q.157


Each of the questions below consists of a question and two/three statements given below it. You
have to decide whether the data provided in the statements are sufficient to answer the question:
Q.153)
Six boxes namely – P, Q, R, S, T and U are kept one above other. Each box contains different
article viz. Watch, Card, Radio, Book, Mobile and Laptop. All the information given are not
necessary in same order. Box R is kept second from bottom at a gap of two boxes from one
contains Card. Box U, which is neither kept at bottom nor adjacent to one that contains Card, is
kept at a gap of one box from one that contains Watch. Box P contains Book is kept at top. Which
box is kept just below one that contains Radio?
I. Only two boxes are kept between box Q and T, Q which is kept adjacent to box S. Box that
contains Radio is kept below one that contains Laptop. Box Q contains Mobile.
II. Box Q, which is neither kept adjacent to one contains Book nor at bottom, is kept just above one
that contains Laptop. Only one box is kept between one that contains Radio and Mobile.
If the data in Statement I alone is sufficient to answer the question, while the data in Statement II
alone is not sufficient to answer the question.
If the data in Statement II alone is sufficient to answer the question, while the data in Statement I
alone is not sufficient to answer the question.
If the data either in Statement I alone or in Statement II alone is sufficient to answer the question.
If the data in both the Statements I and II together are not sufficient to answer the question.
If the data in both the Statements I and II together are necessary to answer the question.
Answer - (a)
From the initial information in the question, we get:
Box Article
P Book
Card

U
R
Watch

From statement I alone:


Box Article
P Book
S Card
Q Mobile
pg. 82
[Type here]

U Laptop
R Radio
T Watch

Box T is kept just below the one that contains Radio. Hence, I alone is sufficient.
Now using statement II alone:
Box Article
P Book
S/T Card
Q Mobile/radio
U Laptop
R Radio/mobile
T/S Watch
So, II alone is not sufficient.

Q.154)
Seven friends namely – Amar, Rekha, Pinku, Hetal, Romy, Zoya and Monty are sitting in a row
facing north but not necessarily in the same order. How many persons sit left of Monty?
I. Amar sits second to the left of Pinku. Only two persons sit between Romy and Pinku. Rekha sits
immediate left of Amar. Hetal is not an immediate neighbour of Pinku.
II. Zoya sits third to the left of Hetal, who sits immediate left of Romy. Neither Zoya nor Hetal sits
at end of the row. Only two persons sit between Rekha and Pinku, who is an immediate neighbour
of Zoya. Rekha sits at one end of the row.
If the data in Statement I alone is sufficient to answer the question, while the data in Statement II
alone is not sufficient to answer the question.
If the data in Statement II alone is sufficient to answer the question, while the data in Statement I
alone is not sufficient to answer the question.
If the data either in Statement I alone or in Statement II alone is sufficient to answer the question.
If the data in both the Statements I and II together are not sufficient to answer the question.
If the data in both the Statements I and II together are necessary to answer the question.
Answer - (e)
From statement I alone,
Rekha Amar Zoya / Monty Pinku Monty / Zoya Hetal Romy

So, exact position of Monty is not known. Hence, statement I alone is not sufficient.
From statement II alone,
Rekha Monty / Amar Zoya Pinku Amar / Monty Hetal Romy

Again, exact position of Monty is not known. Hence, statement II alone is not sufficient.
Combining I and II,
Rekha Amar Zoya Pinku Monty Hetal Romy

So, four persons sit to the left of Monty.


Hence, I and II together are sufficient.

pg. 83
[Type here]

Q.155)
Six persons namely – P, Q, R, S, T and U lives in six floored building marked 1 to 6 in such a way
that lowermost floor is marked 1 and topmost floor is marked 6. Each person likes different fruits
viz. Orange, Guava, Banana, Mango, Apple and Grapes. All the information are not necessary in
same order. P lives on floor marked 4 at gap of two floor from one who likes Guava. U lives on
even numbered floor at a gap of two floor from one who likes Grapes. Neither P nor U likes
Banana. Q lives on adjacent floor of one who likes Guava. One who lives Banana lives on any
floor above Q. How many person lives below one who likes Apple?
I. Only two person lives between one who likes Mango and R. Neither U nor Q likes Mango. One
who likes Apple and Guava doesn’t lives on adjacent floors. Only two person lives between one
who likes Orange and S.
II. One who likes Orange lives on even numbered floor but not adjacent floor of one who likes
Banana. T lives on adjacent floor of one who likes Orange. S and one who likes Mango lives on
adjacent floor. Only two person lives between one who likes Apple and T.
If the data in Statement I alone is sufficient to answer the question, while the data in Statement II
alone is not sufficient to answer the question.
If the data in Statement II alone is sufficient to answer the question, while the data in Statement I
alone is not sufficient to answer the question.
If the data either in Statement I alone or in Statement II alone is sufficient to answer the question.
If the data in both the Statements I and II together are not sufficient to answer the question.
If the data in both the Statements I and II together are necessary to answer the question.
Answer - (a)
From the initial information in the question, we get
Floor Person Fruit
6 U
5 Banana
4 P
3 Grapes
2 Q
1 Guava

Now using I alone, we get


Floor Person Fruit
6 U Apple
5 S Banana
4 P Mango
3 T Grapes
2 Q Orange
1 R Guava

So, I alone is sufficient.


Now, using II alone, we get two possibilities
Possibility 1:
Floor Person Fruit
6 U Apple
pg. 84
[Type here]

5 S Banana
4 P Mango
3 T Grapes
2 Q Orange
1 R Guava

Possibility 2:
Floor Person Fruit
6 U Mango
5 S Banana
4 P Apple
3 R Grapes
2 Q Orange
1 T Guava

So, statement II alone is not sufficient as we don’t know the exact floor of the person who likes
apple.

Q.156)
What is the direction of point O with respect to point I?
I. Point A is 5 m to the north of point P. Point D is 4 m to the south of point C. Point P is 6 m to the
east of point C.
II. Point B is 3 m to the east of point P. Point I is 6 m to the east of point F. Point B is 4 m to the
north of point F.
III. Point N is 2 m to the north of point M. Point A is 4 m to the west of point M. Point N is 6 m to
the east of point O.
All I, II and III
Both II and III
Only II
II and either I or III
Both I and II
Answer - (a)
Combining all three, we get

So, point O is the north-west direction with respect to point I.

Q.157)
Six persons P, Q, R, S, T, U are sitting around a circular table. Some of them are facing the center
while some are facing away from the center. How many persons are facing inside?
I. P sits third to the right of Q, who is facing outside. Both R and U are immediate neighbour of
each other. S is not an immediate neighbour of Q.
II. U sits second to the left of Q, who is sitting third to the left of P. R sits second to the right of T.
If statements I alone is sufficient to answer the question, but statement II alone is not sufficient to
answer the question.

pg. 85
[Type here]

If statement II alone is sufficient to answer the question, but statement I alone is not sufficient to
answer the question.
If statement either I or II is sufficient to answer the question.
If both the statements I and II taken together are not sufficient to answer the question.
If both the statements I and II taken together are sufficient to answer the question.
Answer - (d)
From statement I:
Case 1:

Case 2:

So, statement I alone is not sufficient.


From statement II:
Case 1:

Case 2:

So, statement II alone is not sufficient.


Combining I and II:

So, even I and II together are not sufficient.

Instruction for Q.158


Below is given a passage followed by certain assumptions. You have to check which of the given
assumptions is implicit in the passage.
Q.158)
Passage: A diversified brick and mortar company recently hired a web designer and created an
interesting online clothing catalogue to take its Clothing business online. As a result, the company
had a huge uptick in the volume of sales in the Clothing segment. The spurt was caused due to
increased orders from digitally literate clients. Now, the company is planning to take another of its
business segment, namely electronic goods, online. Which of the following assumptions hold?
Other Electronic Goods companies doesn't have online presence.
Digitally literate clients will be interested in Electronic Goods.
The same web designer can be used for designing Electronic goods catalogue too.
Other companies do not have quality web designers.
The company can wind up its brick and mortar business.
Answer - (b)
Assumption B holds because it has been mentioned in the passage that “The spurt was caused
due to increased orders from digitally literate clients.” So, it implies that the company assumes
similar response for its electronic goods segment as well.

Q.159)
Statement: Excessive amounts of lead in drinking water, associated with certain types of industrial
pollution, have been shown to cause lung cancer. Ajanta Island has an economy based
pg. 86
[Type here]

completely on subsistence level agriculture. there are no industries in the island. The inhabitants
of Ajanta island have in the last 50 years shown high incidence of lung cancer.
Which of the following can be validly inferred from the above statement?
Inferences:
I. Lead in drinking water is absolutely safe.
II. Lung cancer must have causes other than lead in drinking water.
III. Lead in drinking water must have sources other than industrial pollution.
II only
III only
II or III
II or III or both
none of the above
Answer - (d)
The statement portrays about the impact of lead in drinking water, in causing cancer. Inference I –
states that lead is safe, which contradicts the statement, hence this option can be omitted. Other
inferences II and III talks about the impacts of lead causing cancer, hence option D is the correct
choice.

Q.160)
Statement: Green marketing is concerned with choosing, developing, and promoting commodities
that meet the quality, quantity, and price expectations of consumers without negatively impacting
the environment. The demands, wants, and needs of human beings are limitless; however, the
sources available to satisfy them are limited. A study found that most consumers of green
products are very likely to be educated young people, aware of eco-friendly practices. To put it
simply, it is the marketing of eco-friendly products and services.
Which of the given options will be the best conclusion as per the paragraph given above?
A product's features and benefits should be communicated to buyers.
Bamboo products and products made of corn-starch or those without plastic tend to excite these
individuals.
Green products should be developed, priced and positioned carefully.
Marketers should use effective communication tools to inform buyers about the benefits of
biodegradable products.
A product's eco-friendliness depends on the entire production process that goes into it.
Answer - (d)
A conclusion logically completes an idea that has been mentioned in the paragraph. The last few
lines of the paragraph describe that the people who are aware of the effects of the environment or
the availability of alternative products that do not harm the environment only tend to use them. So,
more awareness must be created. So, Option D is the most suitable answer.

Q.161)
The ministry of sports has been advised by a committee to take the highest award in the field of
sports back from two players who were allegedly involved in match fixing. Which of the following
statements would weaken the argument put forward by the committee to the sports ministry?
A good conduct in the past and a lack of evidence against the players make the case against them
very weak.
pg. 87
[Type here]

The ministry of sports has never declined the recommendations made by the committee earlier.
Taking the award back from the players would set a good example to other players for avoiding
such actions in the future.
There have been past cases where the award had to be taken back from the players owing to
some misconduct later on.
The committee is constituted of some of the most respected from the fields of sports and politics.
Answer - (a)
Option A weakens the argument put forward by the committee as it makes a case for forgiving the
players and not taking back the award from them.

Q.162)
STATEMENT: In the city, over 75% of the people are living in slums and substandard houses,
which is reflection of the housing and Urban Development policy of the government. Substandard
houses are often in severe disrepair. Substandard housing isn't just housing that's unattractive or
outdated, it's housing that poses a risk to the health, safety or physical well-being of occupants,
neighbours, or visitors. Substandard housing increases risk of disease, crime, social isolation and
poor mental health.
Course of Action:
1. There should be a separate Department looking after housing and Urban Development.
2. The policies in regard to the urban housing should be reviewed.
3. The policies regarding housing should also be reviewed so that such problem could be avoided
in rural areas.
Only 1 follows
Only 2 follows
Only 2 & 3 follow
None follows
All follow
Answer - (b)
The statement speaks of the failure of Housing and Urban development policies of the
government. Hence, the policy in regard to the urban housing should be reviewed.

Q.163)
Statement: For-profit colleges serve far fewer students than either public or private non-profit
colleges. At the same time, relative to non-profit colleges, for-profit colleges draw a
disproportionate share of federal and state financial aid, such as tuition grants and guaranteed
loans, for their students. It must be, then, that for-profit colleges enrol a greater proportion of
financially disadvantaged students than do non-profit colleges.
The statement above depends on which of the following assumptions?
Public non-profit colleges and private non-profit colleges enrol a similar proportion of financially
disadvantaged students.
For-profit colleges do not engage in fraudulent practices in helping their students obtain unneeded
federal and state financial aid.
The number of students receiving federal and state financial aid at for-profit colleges is greater
than the number of students receiving federal and state financial aid at non-profit colleges.
For-profit colleges are of similar educational quality as non-profit colleges.
pg. 88
[Type here]

The majority of students at for-profit colleges do not default on repayment of their loans after they
complete college.
Answer - (b)
(A) Differences among non-profit colleges – such as public vs. private – are irrelevant to the
argument.
(B) This is the correct choice. One alternative reason that might explain the disproportionate aid
distribution is that for-profit colleges engaged in fraudulent practices to obtain unneeded financial
assistance for their students. This answer choice asserts that this was NOT in fact the case,
thereby eliminating this alternative explanation and highlighting a key assumption upon which the
statement rests.
(C) The actual number of students receiving aid at for-profit vs. non-profit colleges is irrelevant to
the statement.
(D) The relative educational quality of for-profit vs. non-profit colleges lies outside the scope of the
argument, which is focused solely on differences in financial aid distribution.
(E) Whether students successfully repay their loans after college is immaterial to the claim made
in the statement.

Instruction for Q.164


Below is given a statement followed by three course of actions. You have to check which of these
course of actions follow from the given statement and mark your answer accordingly.
Q.164)
Statement: According to the officials, paucity orphaned with organization has led to the pathetic
condition of this brilliant architectural structure. An organization is considered orphaned when the
organization Owner and all Project Collection Administrators are inactive in Azure Active Directory
(Azure AD) or have left the company. This scenario causes the organization to have no
administrator and no way of transferring administrator rights to another user.
Course of Action:
1. A new architectural structure for the building should be designed.
2. Grant should be given to improve the condition of the structure.
3. The reason for the poor condition of the structure should be found out.
Only 1 follows
Only 2 follows
Only 2 & 3 follow
Only 3 follows
All follow
Answer - (b)
Nothing has been mentioned about architectural structure or any problem in it. So, action 1 does
not follow.
It has been mentioned in the first sentence that paucity is one of the main reason for pathetic
condition and hence course of action 2 follows.
Course of action 3 does not follow because reasons for poor condition have already been
mentioned in the passage.

Q.165)

pg. 89
[Type here]

Statement: The solidity of bridge piers built on pilings depends largely on how deep the pilings are
driven. Prior to 1700, pilings were driven to refusal," that is, to the point at which they refused to go
any deeper. In a 1588 inquiry into the solidity of piers for Venice's Rialto Bridge, it was determined
that the bridge's builder, Antonio Da Ponte, had met the contemporary standard for refusal: he had
caused the pilings to be driven until additional penetration into the ground was no greater than two
inches after twenty-four hammer blows.
Which one of the following can properly be inferred from the passage?
The Rialto Bridge was built on unsafe pilings.
The standard of refusal was not sufficient to ensure the safety of a bridge.
Da Ponte's standard of refusal was less strict than that of other bridge builders of his day.
After 1588, no bridges were built on pilings that were driven to the point of refusal.
It is possible that the pilings of the Rialto Bridge could have been driven deeper even after the
standard of refusal had been met.
Answer - (e)
Option E is the correct answer. As stated in the statement, "he had caused the pilings to be driven
until additional penetration into the ground was no greater than two inches after twenty-four
hammer blows." The statement indicates that additional penetration was possible with a sufficient
number of hammer blows.

Q.166)
Max is clearly an incompetent detective. This is because, he has solved a smaller percentage of
the cases assigned to him in the last 3 years, i.e., only 1 out of 25, than any other detective on the
police force.
Which one of the following, if true, most seriously weakens the argument above?
Max was previously a detective in a police department in another city, and in the 5 years he spent
there, he solved only 1 out of 40 crime cases.
Many of the officers in the police department in which Max serves were hired or promoted within
the last 5 years.
Before he became a detective, Max was a neighbourhood police officer and was highly respected
by the residents of the neighbourhood he patrolled.
Detectives on the police force on which Max serves are provided with extensive resources
including the use of a large computer database, to help them solve crimes.
Because the police chief regards Max as the most capable detective, he assigns him only the
most difficult cases, ones that others have failed to solve.
Answer - (e)
As per the argument, Max is clearly an incompetent detective as he has solved a smaller
percentage of the cases assigned to him in the last 3 years, i.e., only 1 out of 25. This argument
can be weakened by adding an explanation for the low proportion of cases solved by Max and
thus, challenging the premise. Option (e) says that since the police chief regards Max as the most
capable detective, he assigns him only the most difficult cases, ones that others have failed to
solve, and thus, adds the required explanation.

Instructions for Q.167 to Q.171


Study the following information carefully and answer the questions given below:

pg. 90
[Type here]

Twelve people A, B, C, D, E, F, G, H, I, J, K, and L live on twelve different floors of the same


building. The ground floor of the building is numbered 1, the floor immediately above the ground
floor is numbered 2, and so on. The twelve people were born in the years 1965, 1967, 1972, 1976,
1987, 1989, 1990, 1997, 1998, 2000, 2003, and 2005 but not necessarily in the same order.
Calculate their ages with reference to the year 2020. Their marks in a particular test are 80, 25,
75, 64, 84, 83, 55, 70, 90, 76, 59, and 48. There are exactly four people between A and K. The
mark scored by I is equal to his present age and he lives immediately below J. H lives on a prime
numbered floor and the mark obtained by him is 10 more than his age. H’s age is the product of
his floor number and the floor number of F. F is the third youngest in the group and he doesn’t live
below H. The mark obtained by B is a perfect square and one person lives between B and I. If J
had scored one mark more than his present mark, it would have been a perfect square. At least 4
people live between K and D and the age of D is equal to the mark scored by J. The number of
people between B and L is one more than the number of people who live between A and K. L lives
immediately below K. D lives on the ground floor of the apartment.
Exactly two people live between K and I. Two people live between G and L. K and L were born in
consecutive years. The difference between the year of birth and the marks for both K and L is
1914. E who scored 80 marks lives immediately above K.
The age of J is an odd multiple of 11 and G was born immediately after J. The age of B is the
product of the floor numbers of I and H. The person who has scored the maximum number of
marks lives on the floor that is immediately above E and H was born immediately after E. The age
of C is also a multiple of 11. Mark of A is three times the mark scored by H and D has scored one
more than that of A. C has scored more than F.
Q.167)
Who lives on the floor immediately above H?
A
K
C
D
E
Answer - (c)
Floor Person Age Marks
12 B 30 64
11 J 33 48
10 I 55 55
9 G 31 90
8 E 17 80
7 K 23/22 83/84
6 L 23/22 83/84
5 F 20 59
4 C 44 70
3 H 15 25
2 A 53 75
1 D 48 76

Q.168)
pg. 91
[Type here]

How much has F scored in the test?


59
70
80
90
25
Answer - (a)
Floor Person Age Marks
12 B 30 64
11 J 33 48
10 I 55 55
9 G 31 90
8 E 17 80
7 K 23/22 83/84
6 L 23/22 83/84
5 F 20 59
4 C 44 70
3 H 15 25
2 A 53 75
1 D 48 76

Q.169)
Who is the eldest in the group?
A
B
D
I
J
Answer - (d)
Floor Person Age Marks
12 B 30 64
11 J 33 48
10 I 55 55
9 G 31 90
8 E 17 80
7 K 23/22 83/84
6 L 23/22 83/84
5 F 20 59
4 C 44 70
3 H 15 25
2 A 53 75
1 D 48 76

Q.170)
What is the age of the person who has scored 84?
pg. 92
[Type here]

23
22
20
15
Cannot be determined
Answer - (b)
Floor Person Age Marks
12 B 30 64
11 J 33 48
10 I 55 55
9 G 31 90
8 E 17 80
7 K 23/22 83/84
6 L 23/22 83/84
5 F 20 59
4 C 44 70
3 H 15 25
2 A 53 75
1 D 48 76

Q.171)
What is the difference between the marks of G and C?
5
10
15
20
25
Answer - (d)
Floor Person Age Marks
12 B 30 64
11 J 33 48
10 I 55 55
9 G 31 90
8 E 17 80
7 K 23/22 83/84
6 L 23/22 83/84
5 F 20 59
4 C 44 70
3 H 15 25
2 A 53 75
1 D 48 76

Instructions for Q.172 to Q.177


Study the following information carefully and answer the below questions:

pg. 93
[Type here]

Eight persons i.e. A, B, C, D, E, F, G and H are working in three different departments viz.
Production, Management and Finance at different posts i.e. manager and general manager. In the
finance department, only a manager post is available. They all have different ages. The age of the
one who is a general manager is an odd number whereas the age of the one who is a manager is
an even number. The age of each person is a whole number less than 90. All the information is
not necessarily in the same order. Minimum one person and maximum two persons are working at
each post of the given departments.
Note: If it is given that the person who is a general manager or manager works with A or the one
whose age is 65, then it indicates that both the person works in the same department but not
necessarily at the same post.
A is a general manager and works along with E. B is 31 years old and doesn’t work along with A.
The one whose age is 60 years works along with C. G is 53 years old and works along with A. C
neither works in the Management department nor in the Production department. The age of D is
equal to the sum of the ages of B and G. The one who works along with D is 2 years older than A.
The age of F is less than H. The difference between the ages of D and F is 48. F doesn’t work in
the Management department. Both F and D works in different departments. The age of E is twice
the age of C. The difference between the ages of C and H is 36. The age of A is less than 50
years.
Q.172)
Who among the following person works in the Production department?
The one who is 29 years old
The one who works along with H
The one who works along with F at the same post
B
Both (a) and (c)
Answer - (e)
The final arrangement is as follows:
Production Management Finance
General manager A(29), G(53) B(31) ----
Manager E(48), F(36) D(84) C(24), H(60)

Q.173)
Four of the following five are alike in a certain way and thus form a group. Which among the
following one doesn’t belong to that group?
A
G
B
E
F
Answer - (c)
The final arrangement is as follows:
Production Management Finance
General manager A(29), G(53) B(31) ----
Manager E(48), F(36) D(84) C(24), H(60)

pg. 94
[Type here]

All the four, except B, work in the production department.

Q.174)
Which of the following statement is/are true with respect to the final arrangement?
The one who is 24 years old works in the Finance department
A works in the Management department
G is a manager
Both (a) and (e)
The sum of the ages of A and C is less than 52
Answer - (a)
The final arrangement is as follows:
Production Management Finance
General manager A(29), G(53) B(31) ----
Manager E(48), F(36) D(84) C(24), H(60)

Q.175)
What is the sum of the ages of the persons who work in the Management department?
120
221
117
115
None of the above
Answer - (d)
The final arrangement is as follows:
Production Management Finance
General manager A(29), G(53) B(31) ----
Manager E(48), F(36) D(84) C(24), H(60)

Q.176)
How many persons’ age is more than H?
Four
One
Six
Two
Three
Answer - (b)
The final arrangement is as follows:
Production Management Finance
General manager A(29), G(53) B(31) ----
Manager E(48), F(36) D(84) C(24), H(60)

Q.177)
Which of the following statements is incorrect?
Three persons are working as General Managers whereas five persons are working as Managers.
F is elder to C but younger to E.
pg. 95
[Type here]

There is no General Manager in the Finance department.


The age of D is more than twice the age of E.
C is the youngest person whereas D is the eldest person.
Answer - (d)
The final arrangement is as follows:
Production Management Finance
General manager A(29), G(53) B(31) ----
Manager E(48), F(36) D(84) C(24), H(60)

Instructions for Q.178 to Q.183


Study the following information carefully and answer the below questions.
Nine persons – B, C, G, K, M, Q, T, W, and Z visited a tourist place during nine different months
viz.- January, March, April, July, August, September, October, November, and December in the
same year. Each person likes different colours viz. - Pink, Blue, Black, Yellow, Brown, White,
Orange, Red, and Green. All the above information is not necessary in the same order. Three
persons visited between the one who likes Pink and K, who visits in one of the month having an
even number of days. Three persons visited between the one who likes Brown and M, who doesn’t
like Red. The one who likes Brown visits in October. The number of persons visited between T
and the one who likes Brown is one less than the number of persons visited between C and the
one who likes Pink. C neither visited in December nor July. T neither likes Pink nor Red. The one
who likes White visited immediately before C, who neither likes Brown nor Orange. The number of
persons visited between Z and the one who likes White is one more than the number of persons
visited after the one who likes Red. Z visits in any month after the one who likes Blue. Two
persons visited between the one who likes Black and W, who visits in one of the month having an
odd number of days. W, who doesn’t like Pink, visits immediately after the one who likes Green.
Neither G nor Q likes Pink. The number of persons visited between B and the one who likes Blue
is one more than the number of persons visited between G and the one who likes Orange.
Q.178)
How many persons visited between T and the one who likes Brown?
Three
As many persons visited before the one who likes Black.
As many persons visited between the one who likes Green and C.
Either A or C
None of these
Answer - (c)
Month Name Colour
Jan Q Black
Mar B Pink
Apr M Green
Jul W White
Aug C Blue
Sep K Orange
Oct Z Brown
Nov G Red
Dec T Yellow
pg. 96
[Type here]

Q.179) Who among the following person likes Yellow?


Z
The one who visited immediately after the one who likes Orange
K
The one who visited immediately before the one who likes Red.
None of these
Answer - (e)
Month Name Colour
Jan Q Black
Mar B Pink
Apr M Green
Jul W White
Aug C Blue
Sep K Orange
Oct Z Brown
Nov G Red
Dec T Yellow

Q.180)
Who among the following person likes Green?
B
The one who visited three persons after Q
The one who visited two persons before C
Either B or C
G
Answer - (c)
Month Name Colour
Jan Q Black
Mar B Pink
Apr M Green
Jul W White
Aug C Blue
Sep K Orange
Oct Z Brown
Nov G Red
Dec T Yellow

Q.181)
Four of the five are related to each other in a certain way and thus form a group, find the odd one
out?
C
The one who likes Black.
The one who likes Yellow
M
pg. 97
[Type here]

Z
Answer - (d)
Month Name Colour
Jan Q Black
Mar B Pink
Apr M Green
Jul W White
Aug C Blue
Sep K Orange
Oct Z Brown
Nov G Red
Dec T Yellow
All the above except M visited in the month with odd number of days.

Q.182)
Which of the following statement is/are not true?
I. The one who likes Red visits immediately after Z.
II. Three persons visited between M and the one who likes Brown.
III. The one who likes Orange visited immediately before G.
Both II and III
Only III
Only I
Both I and II
None of these
Answer - (b)
Month Name Colour
Jan Q Black
Mar B Pink
Apr M Green
Jul W White
Aug C Blue
Sep K Orange
Oct Z Brown
Nov G Red
Dec T Yellow

Q.183)
Who among the following visited immediately before and after the one who likes Blue?
The one who likes Green and the one who likes Orange
K and the one who likes Brown
The one who likes White and Z
W and the one who likes Orange
M and the one who likes White
Answer - (d)
Month Name Colour
pg. 98
[Type here]

Jan Q Black
Mar B Pink
Apr M Green
Jul W White
Aug C Blue
Sep K Orange
Oct Z Brown
Nov G Red
Dec T Yellow

Instructions for Q.184 to Q.188


Study the information carefully and answer the questions given below.
Twelve people A, B, C, D, E, F, P, Q, R, S, T, and U belong to the same family. They were seated
in a concentric circle such that persons sitting in the outer circle, i.e. P, Q, R, S, T, and U face
inside whereas the persons sitting in the inner circle, i.e. A, B, C, D, E and F face outside. Also the
persons in the two circles were sitting opposite to each other. All the information given are not
necessarily in the same order.
Note: The persons facing each other are not the same gender except for the one who faces B.
Neither A nor B sits adjacent to C who faces her grandchild, who is the only sibling of P. A sits
opposite Q, who has three children, and two of the male children got married. The father of S is
facing S. A sits second to the left of B and faces his wife. The relation between A and F, and T and
P is the same. C is the mother-in-law of T, who sits opposite the one who sits immediate right of
the former. The one who sits immediate right of B faces S. D is the unmarried sister-in-law of T
and faces her only uncle. B and C belong to the same generation. The son of A is R. Immediate
neighbours of C are the opposite genders.

Q.184)
Choose the correct option.
I. P faces the spouse of R.
II. Number of married couples and the number of males are the same.
III. Immediate neighbours of F are female.
Both I and II follow
Only III follows
Both II and III follows
Only I follows
None follows
Answer - (d)
The final seating arrangement is as follows:

The final blood relation chart is as follows:

Q.185)
How is P related to the one who faces F?
Son-in-law
pg. 99
[Type here]

Father
Nephew
Daughter
Niece
Answer - (d)
The final seating arrangement is as follows:

The final blood relation chart is as follows:

Q.186)
How many are seated between R and the sibling of P?
Three
One
As many between B and F
Four
As many between C and B
Answer - (c)
The final seating arrangement is as follows:

The final blood relation chart is as follows:

Q.187)
Four of the following five are alike in a certain way and so form a group. Which one does not
belong to that group?
B
R
F
E
U
Answer - (e)
The final seating arrangement is as follows:

The final blood relation chart is as follows:

Except for U, all belongs to the second generation.

Q.188)
Which of the following statements is incorrect?
The number of females is greater than the number of males.
B is the sister-in-law of F.
T, S and D belong to the same generation.
P is sitting second to the left of U.
D and R are facing each other.
pg. 100
[Type here]

Answer - (d)
The final seating arrangement is as follows:

The final blood relation chart is as follows:

Instructions for Q.189 to Q.193


There are 10 people who are sitting in two parallel rows. In row 1, A, B, C, D and E are sitting
facing north. In row II, P, Q, R, S, T are sitting facing south. They are facing each other. They live
in five different cities amongst Singapore, Berlin, Osaka, Helsinki and Mumbai. One person from
each row is living in one of these cities so that exactly two people are living in one city.
1. B, who is not from Mumbai or Berlin, is sitting on the immediate right of the person who faces
the person from Mumbai.
2. A is not sitting at the extreme end of the row and is sitting third to the left of the person from
Mumbai.
3. Q is sitting at one of the extreme ends of the row and is facing C.
4. E is facing the person who sits second to the right of the person from Mumbai.
5. The person who lives in Singapore sits diagonally opposite to the person from Mumbai.
6. S, who does not live in Singapore, is not an immediate neighbour of T or Q.
7. P, who neither lives in Helsinki nor Berlin, sits opposite to the person who sits to the immediate
left of C.
8. In row 1, the person from Berlin sits second to the right of the person from Helsinki.
9. No two persons from the same city are facing each other.
10. In row 2, the person from Helsinki is sitting to the right of the person from Berlin.

Q.189)
Who among the following are sitting diagonally opposite to each other?
S and D
C and R
T and B
Q and E
P and A
Answer - (d)
T (Singapore) R (Helsinki) S (Mumbai) P (Osaka) Q (Berlin)
E (Helsinki) A (Osaka) D (Berlin) B (Singapore) C (Mumbai)

So, T & C and Q & E are sitting diagonally opposite to each other.

Q.190)
The two persons who are sitting in the middle of the two rows belong to which of the following two
cities?
Osaka and Singapore
Berlin and Mumbai
Helsinki and Berlin
Mumbai and Osaka
pg. 101
[Type here]

Singapore and Mumbai


Answer - (b)
T (Singapore) R (Helsinki) S (Mumbai) P (Osaka) Q (Berlin)
E (Helsinki) A (Osaka) D (Berlin) B (Singapore) C (Mumbai)
S is sitting in the middle of row 2 and belong to Mumbai and D is sitting in the middle of row 1 and
belongs to Berlin.

Q.191)
Four of the following five belong to a group. Identify the one which does not belong to that group.
A - Berlin
S - Osaka
T - Mumbai
B - Berlin
C – Singapore
Answer - (c)
T (Singapore) R (Helsinki) S (Mumbai) P (Osaka) Q (Berlin)
E (Helsinki) A (Osaka) D (Berlin) B (Singapore) C (Mumbai)

All the four, except option C, are sitting adjacent to each other.

Q.192)
Which of the following statements is correct?
A is sitting to the immediate left of the person from Helsinki.
P, who lives is Osaka, is not facing the person from Berlin.
R sits second to the left of the person, who is facing B
In row 1, the person who lives in Singapore is not an immediate neighbour of the person from
Mumbai.
Exactly two people are sitting between Q and the person from Helsinki.
Answer - (b)
T (Singapore) R (Helsinki) S (Mumbai) P (Osaka) Q (Berlin)
E (Helsinki) A (Osaka) D (Berlin) B (Singapore) C (Mumbai)

Q.193)
Who among the following do not belong to the same city?
T and B
Q and D
C and S
P and E
All of the above belong to the same city
Answer - (d)
T (Singapore) R (Helsinki) S (Mumbai) P (Osaka) Q (Berlin)
E (Helsinki) A (Osaka) D (Berlin) B (Singapore) C (Mumbai)

Instructions for Q.194 to Q.199


pg. 102
[Type here]

Study the following information carefully and answer the below questions:
Eight persons- A, B, C, D, E, F, G, and H are sitting on the square table facing the center. Four of
them sit in the corner whereas four of them sit in the middle of the side of the table. They bought
different stones- Emerald, Pearl, Topaz, Jade, Zircon, Garnet, Diamond, and Amethyst. All the
information is not necessarily in the same order.
F sits third to the right of the one who bought Topaz. G sits second to the left of F. The one who
bought Pearl sits immediate right of G. Two persons sit between G and A. E sits second to the left
of the one who bought Amethyst. The one who bought Amethyst neither sits opposite to F nor
adjacent to F. A does not buy Amethyst. E does not buy Pearl. B sits opposite to the one who
bought Emerald. The one who bought Emerald does not sit in the middle of the table. B and F are
not immediate neighbours. The number of persons sit between B and H is the same as the
number of persons sit between A and D when counted from the left of both A and B. H neither
bought Emerald nor Pearl. C sits immediate left of the one who bought Jade. Neither C nor F
bought Garnet. The one who bought Zircon does not sit adjacent to the one who bought Pearl.
Q.194)
What is the position of E with respect to the one who bought Zircon?
Immediate left
Immediate right
Second to the right
Third to the right
Second to the left
Answer - (c)

Q.195)
How many persons are sitting between E and D when counted from the left of D?
Three
Four
Two
One
None
Answer - (c)

Q.196)
Four of the five among the following are similar in such a way to form a group, who among the
following doesn’t belong to the group?
The one who bought Jade
The one who bought Emerald
The one who sits immediate left of G
D
The one who sits third to the right of A
Answer - (e)

All of the following except Option E are sitting at the four corners of the square.
pg. 103
[Type here]

Q.197)
Who among the following person sits second to the right of the one who bought Diamond?
The one who sits immediate left of E
C
B
The one who sits second to the left of G
D
Answer - (b)

Q.198)
Which of the following statement is/are true?
I. H bought Emerald.
II. E sits immediate left of B.
III. D does not buy Diamond.
IV. C sits opposite to the one who bought pearl.
Only I and III
Only III and IV
Only I and II
Only II and III
None of the statements is false
Answer - (d)

Q.199)
Which of the following statements is not true?
The one who bought Garnet sits in the middle of the side of the table.
C sits opposite to the one who bought Amethyst.
D sits between the one who bought Diamond and Amethyst when counted from the left of
Diamond.
E is not an immediate neighbour of the one who bought Topaz.
All of the above are true.
Answer - (d)

Instruction for Q.200


Following are the conditions for selecting a candidate in an organization.
The candidate must –
(i) have passed class 12 exam with at least 60% marks.
(ii) have passed graduation degree in any discipline with at least 55% marks.
(iii) be not less than 21 years and not more than 30 years of age as on 1.7.2016.
(iv) possess a certificate/diploma/degree course in Computer Science.
In the case of candidate who fulfils all conditions except –
(a) at (ii) above but is a post-graduate, case may be referred to the Executive Director (ED).
pg. 104
[Type here]

(b) at (iv) above but has studied Computer Science as one of subjects in graduation, case may be
referred to the Vice President (VP).
Q.200)
Lakshay is a Commerce graduate with 57% marks. He had secured 73% marks class 12
examination. He has studied Computer Science as one of the subjects in class 12. His date of
birth is 22.9.1990.
if the case is to be referred to Executive Head.
if the case is to be referred to Vice President.
if the candidate is to be selected.
if the information is inadequate to take a decision.
if the candidate is not to be selected.
Answer - (e)
(iv) condition is not specified. So move to (b). But this is also not fulfilled as he has studies
computer science in class 12th and not in graduation. So he will not be selected for any job.

pg. 105
[Type here]

pg. 106
[Type here]

pg. 107
[Type here]

pg. 108
[Type here]

pg. 109
[Type here]

pg. 110
General Awareness
Q.1)
Who is the Author of the book “My Country My life”?
(a) L K Advani
(b) Mamata Banerjee
(c) N Chandrababu Naidu
(d) Rahul Gandhi
(e) Prakash Javadekar

Answer – (a)
Explanation –
Books and their authors have been asked in the examination for a long period of time. You can witness
such questions in every type of examination. Therefore, you should keep a track of all the books which
are in the news. For keeping a track of the new launches, you can either rely on any specific exam
related magazine or read newspapers on a daily basis and create your own short notes from them.

Q.2)
Who is the Google policy head of India?
(a) Sukant Vatsa
(b) Archana Gulati
(c) Ritika Singh Verma
(d) Ramesh Kumar
(e) Chandra Sekhar

Answer – (b)
Explanation –
This question is from the appointments section. All the latest appointments prior to the exam are
important. This question is the evidence of how high the level of difficulty is in this examination.
Questions are not restricted to the news itself, rather the questions are being asked about the static
information related to the news. So, prepare the background information of the news.

Q.3)
Singareni coalfields are located in which state?
(a) Telangana
(b) Maharashtra
(c) Andhra Pradesh
(d) Karnataka
(e) Odisha

Answer – (a)
Explanation –
This question might appear as a static GK question but it is not so. It is rather a current affairs’ question.
So, always remember to look for the background facts related to the news to get good marks in the
exam.
Q.4)
Delimitation is defined as the act or process of fixing limits or boundaries of territorial constituencies in
a country or a province having a legislative body. Delimitation is the process of redrawing boundaries of
Lok Sabha and state Assembly seats to represent changes in the population. It’s only after this exercise
has been completed that an election can be conducted. Delimitation has happened in which state
recently?
(a) J&K
(b) Assam
(c) Meghalaya
(d) Telangana
(e) Lakshwadweep

Answer – (a)
Explanation –
This is question from the national news. Policy-related guidelines and frameworks are always important
and need to be covered in detail. Year by Year, the level of the exam is getting tougher. Thus, it requires
more conceptual understanding and fact memorization. You should prepare past six months’ current
affairs thoroughly. In order to cover all the events of the past 6 months, preceding the date of
examination, you can select a good source. Spotlight magazine covers all such news.

Q.5)
Which of the following state is the winner of Santosh Cup?
(a) Uttar Pradesh
(b) Arunachal Pradesh
(c) Assam
(d) Gujarat
(e) Kerala

Answer – (e)
Explanation –
Often students make this mistake of ignoring the important sports related news. This is a very big
blunder that students often make, according to me. So, awards, sports, books, events etc. are important
for you to cover. You can refer a good magazine tailor-made specifically for RBI, Sebi Nabard’s
examination or you can follow a good newspaper like the livemint, the Hindu, The Indian Express,
economic times, Business Standard etc.

Q.6)
Who is the Runner up of Pro kabaddi league held recently?
(a) Patna Pirates
(b) Dabang Delhi
(c) Tamil Thalaivas
(d) Puneri Paltan
(e) Telugu Titans

Answer – (a)
Explanation –
Often students make this mistake of ignoring the important sports related news. This is a very big
blunder that students often make, according to me. So, awards, sports, books, events etc. are important
for you to cover. You can refer a good magazine tailor-made specifically for RBI, Sebi Nabard’s
examination or you can follow a good newspaper like the livemint, the Hindu, The Indian Express,
economic times, Business Standard etc.

Q.7)
In which year was Federal reserve established in USA?
(a) 1913
(b) 1998
(c) 1987
(d) 1967
(e) 1955

Answer – (a)
Explanation –
This question is the evidence of how high the level of difficulty is in this examination. Questions are not
restricted to the news itself, rather the questions are being asked about the static information related to
the news. So, prepare the background information of the news. Now, you have to follow an integrated
approach to prepare for the exam. Merely preparing Phase 1 first won’t help you in clearing even the
Phase 1 itself. You should prepare both the phases together to clear the exam.

Q.8)
What is the ratio of ownership in an RRB by states, centre and sponsor banks?
(a) 50:15:35
(b) 40:30:30
(c) 60:20:20
(d) 75:15:10
(e) 35:15:50

Answer – (a)
Explanation –
The Govt is planning to reduce its holdings in the RRBs. It can be reduced to 51% owned by both centre
and states and 15% by sponsor banks. Rest of the capital can be raised from the market.
This question is a part of your static banking awareness. You need to be thorough with the bank-related
static information to tackle such questions. You can choose a tailor-made current affairs' magazine. You
just need to know that whichever source you choose you should stick to that and create your own short
notes. You cannot make a guess work here, if you are not aware about it.

Q.9)
PM Kisan is a Central Sector scheme with 100% funding from Government of India. Under the scheme an
income support of 6,000/- per year in three equal installments will be provided to all land holding
farmer families. The funds are directly transferred to the bank accounts of the beneficiaries. State
Government and UT administration will identify the farmer families which are eligible for support as per
scheme guidelines. Which of the following is the correct allocation of PM kisan in the recent budget?
(a) Rs 90,000 crore
(b) Rs 18,000 crore
(c) Rs 45,500 crore
(d) Rs 68,000 crore
(e) Rs 57,000 crore
Answer – (d)
Explanation –
The Centre has allocated Rs 68,000 crore for the Pradhan Mantri Kisan Samman Nidhi (PM-KISAN) in the
Union Budget 2022-23.
This is a question which has been asked directly from the government schemes. It is an integral part of
the exam’s general awareness syllabus. Usually, questions from the government schemes are twisted
and elongated for make them tough. To get through such questions, the only strategy you can apply is
thorough preparation of the government schemes. Believe me, there is no other way as far as this
section is concerned.

Q.10)
Where is Louvre Museum Located?
(a) Paris, France
(b) Boston, USA
(c) Berlin, Germany
(d) Stockholm, Sweden
(e) Amsterdam, Netherlands

Answer – (a)
Explanation –
It belongs to the international current affairs. Always remember to look for the background facts related
to the news to get good marks in the exam. You can cover such types of news from different newspaper.
Whichever source you choose, always remember that you have to do multiple revisions to retain the
facts.

Q.11)
National Family Health Survey is a survey carried out on a massive scale across the country to collect
information on many parameters which would ultimately help the Ministry of Health and Family Welfare
(MOHFW) to frame policies and programs to help in the upliftment of the vulnerable groups in India.
National Family Health Survey 5 is the recent round of the survey carried out by the MoH&FW.
According to NFHS-5, what is the male to female ratio in India?
(a) 1020 women for 1000 men
(b) 989 women for 1000 men
(c) 1001 women for 1000 men
(d) 997 women for 1000 men
(e) 1050 women for 1000 men

Answer – (a)
Explanation –
This question has been asked from the reports and indices. This question is from an Indian index. All the
reports and indices released during the six months prior to the exam date are important. All these kinds
of news can be covered from some good newspapers like Livemint, Indian Express, Business Standard,
Economic Times, The Hindu etc.

Q.12)
According to the latest State of World’s Forest report, what is the percentage of world forest cover?
(a) 27^
(b) 19%
(c) 31%
(d) 50%
(e) 42%

Answer – (c)
Explanation –
This question has been asked from the reports and indices. This question is from an Indian index. All the
reports and indices released during the six months prior to the exam date are important. All these kinds
of news can be covered from some good newspapers like Livemint, Indian Express, Business Standard,
Economic Times, The Hindu etc.

Q.13)
What is the Venue of Climate Change Conference to be held in June 2022?
(a) Bonn, Germany
(b) New York, US
(c) Seattle, US
(d) Paris, France
(e) Madrid, Spain

Answer – (a)
Explanation –
International affairs hold a significant position in this examination. You can cover the international
affairs from spotlight magazine or any other magazine which specifically caters to the banking students.

Q.14)
First-generation bio-alcohols (bioethanol) are produced via yeast (Saccharomyces cerevisiae)
fermentation of plant sugars and starches obtained from crops such as sugarcane (Saccharum sp.), sugar
beet, and corn (Zea mays). Obtaining ethanol from food crops is also a practice of producing the first-
generation biofuel. The 1st food-grain based ethanol factory is situated in which state?
(a) Bihar
(b) Madhya Pradesh
(c) Uttar Pradesh
(d) Rajasthan
(e) Gujarat

Answer – (a)
Explanation –
This is a question which has been asked directly from the government schemes. It is an integral part of
the exam’s general awareness syllabus. Usually, questions from the government schemes are twisted
and elongated for make them tough. To get through such questions, the only strategy you can apply is
thorough preparation of the government schemes. Believe me, there is no other way as far as this
section is concerned.

Q.15)
Coal Miners Day is observed on _____ to honour the coal miners. The day is celebrated to highlight the
contributions of coal miners in meeting our energy demands.
(a) April 8
(b) May 4
(c) June 12
(d) July 17
(e) August 19

Answer – (b)
Explanation –
This is a question asked from the important days. So, don't ignore important days and their themes. At
the same time, focus on the major important days should be made. The major days are those which
have been asked repeatedly and which have a huge importance worldwide for eg. World Health Day,
AIDS Day etc.

Q.16)
T+1 was implemented by which country for the first time?
(a) China
(b) Singapore
(c) Japan
(d) USA
(e) Germany

Answer – (a)
Explanation –
In January 2023, India has become the 2nd country in the world to implement the T+1 settlement cycle,
after China.
This question has been asked from the banking current affairs. This question is from the framework and
guidelines topic. Policy-related guidelines and frameworks are always important and need to be covered
in detail.

Q.17)
Street vendors represent a very important constituent of the urban informal economy and play a
significant role in ensuring availability of the goods and services at affordable rates at the door-step of
the city dwellers. The COVID-19 pandemic and consequent lockdowns have adversely impacted the
livelihoods of street vendor. Therefore, the Central Government launched the PM-Svanidhi scheme, a
Central Sector Scheme, which is fully funded by Ministry of Housing and Urban Affairs. Which of the
following statement(s) is/are correct with respect to the objectives of this scheme?
1) To facilitate working capital loan up to Rs 10,000
2) The loan will be provided for a period of 18 months
3) On early repayment, the vendors will be eligible for the next cycle of working capital loan with an
enhanced limit i.e., Rs 15,000

(a) Only 1
(b) Only 1 and 2
(c) Only 2 and 3
(d) Only 3
(e) Only 1 and 3

Answer – (a)
Explanation –
This is a question which has been asked directly from the government schemes. It is an integral part of
the exam’s general awareness syllabus. Usually, questions from the government schemes are twisted
and elongated for make them tough. To get through such questions, the only strategy you can apply is
thorough preparation of the government schemes. Believe me, there is no other way as far as this
section is concerned.

Q.18)
In budget 2022, how much allocation has been made to Jal Jeevan Mission?
(a) Rs 20,000 crore
(b) Rs 60,000 crore
(c) Rs 1,00,000 crore
(d) Rs 49,000 crore
(e) Rs 70,000 crore

Answer – (b)
Explanation –
This is a question which has been asked directly from the government schemes. It is an integral part of
the exam’s general awareness syllabus. Usually, questions from the government schemes are twisted
and elongated for make them tough. To get through such questions, the only strategy you can apply is
thorough preparation of the government schemes. Believe me, there is no other way as far as this
section is concerned.

Q.19)
Repo rate and CRR has been revised recently by RBI. On which date was the revision made and from
which date has It been implemented?
(a) 2th June 2022 and 18st August 2022
(b) 4th May 2022 and 21st May 2022
(c) 18th July 2022 and 1st August 2022
(d) 15th July 2022 and 31st August 2022
(e) 10th June 2022 and 1st September 2022

Answer – (b)
Explanation –
This is a question from the policy rates. Thus, policy rates are important for this exam because they are a
part of the monetary policy. Policy-related guidelines and frameworks are always important and need to
be covered in detail. All these kinds of news can be covered from some good newspapers like Livemint,
Indian Express, Business Standard, Economic Times, The Hindu etc. The Spotlight and RBI 247 magazines
and sessions strive to convey the policies in detail. Whichever source you choose make sure that you
stick to that.

Q.20)
The 2022 edition of The State of the World’s Forests (SOFO) was released on the first day of the World
Forestry Congress, 2022. This year’s report explores the potential of three forest pathways for achieving
green recovery and tackling environmental crises, including climate change and biodiversity loss against
the backdrop of the Glasgow Leaders’ Declaration on Forests and Land Use and the pledge of 140
countries to eliminate forest loss by 2030 and to support restoration and sustainable production and
consumption. The World State of the Forest Report is published by which organization?
(a) FAO
(b) IFAD
(c) UNEP
(d) IUCN
(e) WFA

Answer – (a)
Explanation –
This question has been asked from the reports and indices. This question is from an Indian index. All the
reports and indices released during the six months prior to the exam date are important. All these kinds
of news can be covered from some good newspapers like Livemint, Indian Express, Business Standard,
Economic Times, The Hindu etc.

Q.21)
What is India’s rank in global fish production?
(a) 5th
(b) 2nd
(c) 8th
(d) 9th
(e) 10th

Answer – (b)
Explanation –
With a production of 14.16 million metric tonnes in 2019-20, India maintains its position as the world's
second-largest fish producer. This is question from the national news. National news section includes
important events, summits, conferences, initiatives of national importance. Such types of news are
covered in Spotlight in the national news category.

Q.22)
What is the relationship between Bond yields/rates and prices?
(a) Inverse relationship
(b) Direct relationship
(c) Not related
(d) Proportional relationship
(e) Indirect relationship

Answer – (a)
Explanation –
This question is a part of your static banking awareness. You need to be thorough with the bank-related
static information to tackle such questions. You can choose a tailor-made current affairs' magazine. You
just need to know that whichever source you choose you should stick to that and create your own short
notes. You cannot make a guess work here, if you are not aware about it.

Q.23)
Which of the following statement(s) is/are incorrect about the ASEEM portal?
(i) Ministry of Skill Development and Entrepreneurship (MSDE) has launched ASEEM portal to help
skilled people find sustainable livelihood opportunities
(ii) ASEEM stands for Aatamanirbhar Skilled Employee Employer Mapping
(iii) It will improve the information flow and bridge the demand-supply gap in the skilled workforce
market

(a) Only II and III


(b) Only I and II
(c) Only I, II, and III
(d) None are incorrect
(e) All are incorrect

Answer – (d)
Explanation –
The Ministry of Skill Development and Entrepreneurship (MSDE) has launched ‘Aatamanirbhar Skilled
Employee Employer Mapping (ASEEM)’ portal to help skilled people find sustainable livelihood
opportunities. This is a question which has been asked directly from the government schemes. In this
question, we have observed that the direct questions are most often asked in the exam.

Q.24)
The Indira Gandhi National Open University (IGNOU) has been chosen by the Ministry of Social Justice
and Empowerment, Government of India to set up the DACE centres. The objective of DACE is to
empower the Scheduled Caste (SC) candidates by providing them high quality coaching absolutely free
of cost for civil services Examinations (Prelims and Mains). In DACE portal, E stands for?
(a) Excellence
(b) Enterprise
(c) Expertise
(d) Environment
(e) Expedition

Answer – (a)
Explanation –
DACE stands for Dr Ambedkar Centres of Excellence. DACE SCHEME aims to provide high-quality free
coaching facilities to the Scheduled Caste students for the UPSC Civil Services examinations.
This is a question which has been asked directly from the government schemes. In this question, we
have observed that the direct questions are most often asked in the exam. These types of direct and
short sentenced questions do not leave any scope of any hint or guess work. So, the best strategy to
tackle such questions is to leave them because you have negative marking in the exam. Taking such a
risk can cost you a lot.

Q.25)
The Organisation for Economic Co-operation and Development is an intergovernmental organisation
with 38 member countries to stimulate economic progress and world trade. The majority of OECD
members are high-income economies with a very high Human Development Index (HDI), and are
regarded as developed countries. It is a forum whose member countries describe themselves as
committed to democracy and the market economy, providing a platform to compare policy experiences,
seek answers to common problems, identify good practices, and coordinate domestic and international
policies of its members.
When was the OECD established?
(a) 1961
(b) 1987
(c) 1954
(d) 1988
(e) 1999

Answer – (a)
Explanation –
It belongs to the international organizations. Always remember to look for the background facts related
to the news to get good marks in the exam. So, covering them is a must. the sources which you can use
to cover the static general awareness are Lucent’s book, Manorama’s yearbook etc. Always remember
that in order to retain a lot of facts you should be revising the facts again and again.

Q.26)
The Bay of Bengal Initiative for Multi-Sectoral Technical and Economic Cooperation (BIMSTEC) is a
regional organization. Initially known as BIST-EC (Bangladesh-India-Sri Lanka-Thailand Economic
Cooperation), the organisation is now known as BIMSTEC and comprises seven Member States with the
admission of Myanmar on 22 December 1997, and Bhutan and Nepal in February 2004.
When was BIMSTEC established?
(a) 1989
(b) 1997
(c) 1929
(d) 1990
(e) 1991

Answer – (b)
Explanation –
It belongs to the international organizations. Always remember to look for the background facts related
to the news to get good marks in the exam. So, covering them is a must. the sources which you can use
to cover the static general awareness are Lucent’s book, Manorama’s yearbook etc. Always remember
that in order to retain a lot of facts you should be revising the facts again and again.

Q.27)
Which is the 1st Geological Park in India?
(a) Lamheta
(b) Kurnool
(c) Anantapur
(d) Barmer
(e) Jaisalmer

Answer – (a)
Explanation –
It is a state-specific current affairs’ question. Many questions are asked from the state current affairs.
So, make sure that you prepare the state-specific current affairs thoroughly. Such types of news are
covered in Spotlight.

Q.28)
Under the Reserve Bank of India, Act,1934, RBI is entrusted with the responsibility of conducting
monetary policy in India with the primary objective of maintaining price stability while keeping in mind
the objective of growth. RBI Act, 1934 provides for an empowered monetary policy committee (MPC) to
be constituted by the Central Government for fixing the benchmark interest rate in India. The Monetary
Policy Committee consists of _______ members?
(a) Seven
(b) Ten
(c) Five
(d) Six
(e) Two

Answer – (d)
Explanation –
This question has been asked from the banking current affairs. This question is from the framework and
guidelines topic. Policy-related guidelines and frameworks are always important and need to be covered
in detail. All these kinds of news can be covered from some good newspapers like Livemint, Indian
Express, Business Standard, Economic Times, The Hindu etc. The Spotlight and RBI 247 magazines and
sessions strive to convey the policies in detail. Whichever source you choose make sure that you stick to
that.

Q.29)
National Centre for Financial Education (NCFE) is a Section 8 (Not for Profit) Company promoted by
Reserve Bank of India (RBI), Securities and Exchange Board of India (SEBI), Insurance Regulatory and
Development Authority of India (IRDAI) and Pension Fund Regulatory and Development Authority
(PFRDA), under the aegis of Financial Stability and Development Council (FSDC), Ministry of Finance,
Government of India. When was NCFE established?
(a) 2013
(b) 2018
(c) 2019
(d) 2017
(e) 2012

Answer – (a)
Explanation –
This question is a part of your static banking awareness. You need to be thorough with the bank-related
static information to tackle such questions. You can choose a tailor-made current affairs' magazine. You
just need to know that whichever source you choose you should stick to that and create your own short
notes. You cannot make a guess work here, if you are not aware about it.

Q.30)
ECLGS- Emergency credit line guarantee scheme expanded by _____ to Rs 5 lakh crore?
(a) Rs 50,000 crore
(b) Rs 30,000 crore
(c) Rs 25,000 crore
(d) Rs 20,000 crore
(e) Rs 15,000 crore

Answer – (a)
Explanation –
In 2022, the Union Cabinet, chaired by the Prime Minister, Shri Narendra Modi has approved the
enhancement in the limit of Emergency Credit Line Guarantee Scheme (ECLGS) by Rs 50,000 crore from
Rs. 4.5 Lakh crore to Rs. 5 Lakh crore.
In this question, we have observed that the direct questions are most often asked in the exam. These
types of direct and short sentenced questions do not leave any scope of any hint or guess work. So, the
best strategy to tackle such questions is to leave them because you have negative marking in the exam.
Taking such a risk can cost you a lot.

Q.31)
A Non-profit organization can be setup as per which section of companies act?
(a) Section 8
(b) Section 4
(c) Section 30
(d) Section 28
(e) Section 13

Answer – (a)
Explanation –
This is an out of the blue question. In case, you are a little bit aware about the question, then only try to
make a guess or deploy the elimination technique. Otherwise, the best strategy to deal with such
questions is not to attempt them.

Q.32)
In India, highest employment is generated in which sector after agriculture sector?
(a) Iron and Steel Industry
(b) Textile
(c) Automobile
(d) Software and IT
(e) Pharma

Answer – (b)
Explanation –
Textile or construction sector provides the maximum employment after agriculture sector. This is a
question from Economic Survey. Budget, Economic Survey, and national reports etc. are important from
the exam’s perspective.

Q.33)
How does the Monetary Policy Committee ensure transparency in inflation measurement?
(a) MPC increases/decreases policy rates
(b) MPC prepares the CPI index to measure inflation
(c) MPC monitors the residex to monitor housing prices
(d) Only I and II
(e) Only II and III

Answer – (a)
Explanation –
This question has been asked from the banking static facts. This question is from the Monetary Policy
Committee. Policy-related guidelines and frameworks are always important and need to be covered in
detail. The Spotlight and RBI 247 magazines and sessions strive to convey the policies in detail.

Q.34)
The employee provident fund interest rate was cut to a decade low of ____ in 2021-22 fiscal?
(a) 8.1%
(b) 7.8%
(c) 8.9%
(d) 7.1%
(e) 7.5%

Answer – (a)
Explanation –
This question is a part of your finance awareness. All these kinds of news can be covered from some
good newspapers like Livemint, Indian Express, Business Standard, Economic Times, The Hindu etc. The
Spotlight and RBI 247 magazines and sessions strive to convey the policies in detail. Whichever source
you choose make sure that you stick to that.

Q.35)
If x is 18 years old and wants to have a pension of Rs 5000 when he retires under Atal pension Yojana,
then how much should he contribute regularly under Atal pension Yojana?
(a) 300
(b) 180
(c) 150
(d) 210
(e) 250

Answer – (d)
Explanation –
This is a question which has been asked directly from the government schemes. It is an integral part of
the exam’s general awareness syllabus. Usually, questions from the government schemes are twisted
and elongated for make them tough. To get through such questions, the only strategy you can apply is
thorough preparation of the government schemes. Believe me, there is no other way as far as this
section is concerned.

Q.36)
In order to provide skilling and livelihood opportunities, the Government of India implements various
kinds of schemes from time to time. Recently the finance minister of India Nirmala Sitharaman launched
the DESH Stack e portal. Through this portal, a digital ecosystem will be developed for skilling and
livelihood. What does ‘e’ stand for in the DESH portal?
(a) Enterprise
(b) Ecosystem
(c) Effort
(d) Encore
(e) Establishment

Answer – (b)
Explanation –
This is a question which has been asked directly from the government schemes. It is an integral part of
the exam’s general awareness syllabus. Usually, questions from the government schemes are twisted
and elongated for make them tough. To get through such questions, the only strategy you can apply is
thorough preparation of the government schemes. Believe me, there is no other way as far as this
section is concerned.

Q.37)
Peacekeeping medals were given by the United States to which country?
(a) South Sudan
(b) India
(c) Zambia
(d) Gambia
(e) Kenya

Answer – (a)
Explanation –
This is a question from the awards section. All the awards, the country/organization giving the awards,
the recipients, and the recipients’ nationalities are all important. This question is a direct and factual
question. Therefore, it cannot be attempted by doing the guesswork. So, in such type of questions if you
know the answer only then attempt otherwise you should leave these questions. You should be aware
of this fact that you don't have to score 100% marks in general awareness you just need to clear the cut
off. Therefore, if you will target at least 70 percent marks in the GA section of RBI grade b then you are
on the safer side.

Q.38)
What is the urban and rural sex ratio in India respectively?
(a) Rural- 924; Urban- 918
(b) Rural- 904; Urban- 998
(c) Rural- 933; Urban- 965
(d) Rural- 817; Urban- 980
(e) Rural- 898; Urban- 920

Answer – (a)
Explanation –
This is question from the NFHS-5. Year by Year, the level of the exam is getting tougher. Thus, it requires
more conceptual understanding and fact memorization. You should prepare past six months’ current
affairs thoroughly. In order to cover all the events of the past 6 months, preceding the date of
examination, you can select a good source. Spotlight magazine covers all such news.

Q.39)
When is a bank customer eligible to be insured under DICGC for his deposits made in a bank?
(a) Liquidation or Cancellation of bank's licence
(b) Amalgamation and merger
(c) Reconstruction or arrangement
(d) Only A and B
(e) Only B and C
Answer –
Explanation –
This question has been asked from the banking static facts. This question is from the Monetary Policy
Committee. Policy-related guidelines and frameworks are always important and need to be covered in
detail. The Spotlight and RBI 247 magazines and sessions strive to convey the policies in detail.

Q.40)
Union Minister for Finance and Corporate Affairs, Smt Nirmala Sitharaman, launched the asset
monetisation pipeline of Central ministries and public sector entities: 'National Monetisation Pipeline.
NITI Aayog has developed the pipeline, in consultation with infrastructure line ministries, based on the
mandate for 'Asset Monetisation' under Union Budget 2021-22. NMP estimates aggregate monetisation
potential of _____?
(a) Rs 4 lakh crore
(b) Rs 6 lakh crore
(c) Rs 8 lakh crore
(d) Rs 10 lakh crore
(e) Rs 40 lakh crore

Answer – (b)
Explanation –
This is a question which has been asked directly from the government schemes. It is an integral part of
the exam’s general awareness syllabus. Usually, questions from the government schemes are twisted
and elongated for make them tough. To get through such questions, the only strategy you can apply is
thorough preparation of the government schemes. Believe me, there is no other way as far as this
section is concerned.

Q.41)
What is the minimum NOF requirement for a public company to be incorporated as a Nidhi company?
(a) Rs 20 lakh
(b) Rs 40 lakh
(c) Rs 30 lakh
(d) Rs 15 lakh
(e) Rs 25 lakh

Answer – (a)
Explanation –
Any public company incorporated as Nidhi with a share capital of Rs 10 lakh will have submit NDH-4
form and apply with the central government to be notified as a Nidhi company within 120 days of its
incorporation. Also, the company needs to have at least 200 members and should have a net owned
fund (NOF) of Rs 20 lakh.

Q.42)
How many Beneficiaries are covered in Jal Jeevan Mission?
(a) 4.5 crores
(b) 2.7 crores
(c) 11.1 crores
(d) 3.8 crores
(e) 6.6 crores
Answer – (d)
Explanation –
In this question, we have observed that the direct questions are most often asked in the exam. These
types of direct and short sentenced questions do not leave any scope of any hint or guess work. So, the
best strategy to tackle such questions is to leave them because you have negative marking in the exam.
Taking such a risk can cost you a lot.

Q.43)
In which month in 2022, GST collection has crossed Rs 1.5 Lakh Crore mark?
(a) Rs 1.68 Lakh Crores in April 2022
(b) Rs 2.0 Lakh Crores in March 2022
(c) Rs 1.72 Lakh Crores in July 2022
(d) Rs 1.8 Lakh Crores in August 2022
(e) Rs 2.2 Lakh Crores in June 2022

Answer – (a)
Explanation –
This question is a part of your finance awareness. All these kinds of news can be covered from some
good newspapers like Livemint, Indian Express, Business Standard, Economic Times, The Hindu etc. The
Spotlight and RBI 247 magazines and sessions strive to convey the policies in detail. Whichever source
you choose make sure that you stick to that.

Q.44)
After National Education Policy, what is the allocation for education in Union Budget 2022?
(a) Rs 1,54,000 crore
(b) Rs 1,04,278 crore
(c) Rs 2,90,800 crore
(d) Rs 1,07,900 crore
(e) Rs 3,40,500 crore

Answer – (b)
Explanation –
This is a question which has been asked directly from the government schemes. It is an integral part of
the exam’s general awareness syllabus. Usually, questions from the government schemes are twisted
and elongated for make them tough. To get through such questions, the only strategy you can apply is
thorough preparation of the government schemes. Believe me, there is no other way as far as this
section is concerned.

Q.45)
Azadi se Antyodaya Tak Inter ministerial campaign has been announced for?
(a) 50 days
(b) 60 days
(c) 80 days
(d) 90 days
(e) 75 days

Answer – (d)
Explanation –
This is question from the national news. National news section includes important events, summits,
conferences, initiatives of national importance. Such types of news are covered in Spotlight in the
national news category.

Q.46)
Jharokha scheme has been launched in how many states?
(a) 19
(b) 13
(c) 20
(d) 6
(e) 10

Answer – (b)
Explanation –
Jharokha is a PAN India programme that will be held at 16 locations in 13 states and UTs, as part of the
Azadi ka Amrit Mahotsav. It is a state-specific current affairs’ question. Many questions are asked from
the state current affairs. So, make sure that you prepare the state-specific current affairs thoroughly.
Such types of news are covered in Spotlight.

Q.47)
The SagarMala programme has a target of developing how many kms of inland waterways?
(a) 27,800 km
(b) 18,900 km
(c) 14,500 km
(d) 25,600 km
(e) 10,800 km

Answer – (c)
Explanation –
The Sagarmala programme is the flagship programme of the Ministry of Shipping to promote port-led
development in the country through harnessing India's 7,500 km long coastline, 14,500 km of potentially
navigable waterways and strategic location on key international maritime trade routes. This is question
from the national news. National news section includes important events, summits, conferences,
initiatives of national importance. Such types of news are covered in Spotlight in the national news
category.

Q.48)
How much additional subsidy is allocated to Fertilizer sector by the government?
(a) ₹2.78 lakh crore
(b) ₹2.55 lakh crore
(c) ₹0.80 lakh crore
(d) ₹1.10 lakh crore
(e) ₹2.07 lakh crore

Answer – (d)
Explanation –
In addition to the fertiliser subsidy of ₹1.05 lakh crore in the Budget, an additional amount of ₹1.10 lakh
crore is being provided to further cushion our farmers. This is a question from Union Budget. Budget,
Economic Survey, and national reports etc. are important from the exam’s perspective.

Q.49)
What is the name of the new initiative launched by Indian Air Force and Indian Oil Corporation Ltd.?
(a) Slice Supre Card
(b) Fleet Card - Fuel on Move
(c) OneCard Credit Card
(d) Uni Card 1/3rd card
(e) Gold credit card

Answer – (b)
Explanation –
The Indian Air Force unveiled a new initiative with the Indian Oil Corporation Ltd under which its
convoys will be to refuel at fuel stations of the state-run energy major. The defence ministry said a 'Fleet
Card - Fuel on Move' has been rolled out to facilitate the refuelling of convoys belonging to the IAF.

Q.50)
UPI and PayNow have been integrated to enable users of each of the two fast payment systems to make
instant, low-cost fund transfers on a reciprocal basis without a need to get onboarded onto the other
payment system. Which country does PayNow belong to?
(a) Singapore
(b) Mauritius
(c) USA
(d) Germany
(e) France

Answer – (a)
Explanation –
This question is a part of your banking awareness. All the news related to the banks, including new
initiatives, new partnerships, mergers and acquisitions are all relevant for this examination. You can find
the banking awareness related current affairs on a number of competitive websites. Here, you should be
selective in choosing your source of information. So, spend time on choosing a good source of current
affairs and once you have chosen the website or the magazine then stick to it. It is highly advisable in
your interest only not to shift from source to source.

Q.51)
Bharat Bond ETF is an open-ended Target Maturity Exchange Traded Bond Fund that seeks to track the
returns provided by Nifty BHARAT Bond Index. Which company is tasked with managing the ETF?
(a) Aditya Birla Sun Life Mutual Fund
(b) Franklin Templeton Mutual Fund
(c) Baroda BNP Paribas Mutual Fund
(d) Edelweiss Mutual Fund
(e) Axis Mutual Fund

Answer – (d)
Explanation –
This question is a part of your banking awareness. All the news related to the banks, including new
initiatives, new partnerships, mergers and acquisitions are all relevant for this examination. You can find
the banking awareness related current affairs on a number of competitive websites. Here, you should be
selective in choosing your source of information. So, spend time on choosing a good source of current
affairs and once you have chosen the website or the magazine then stick to it. It is highly advisable in
your interest only not to shift from source to source.

Q.52)
Who is the next chairman of FATF and which country does he belong to?
(a) Trygve Lie from Norway
(b) Dag Hammarskjöld from Sweden
(c) Kurt Waldheim from Austria
(d) Javier Perez de Cuellar from Peru
(e) T Raja Kumar from Singapore

Answer – (e)
Explanation –
This question is from the appointments section. All the latest appointments prior to the exam are
important. This question is the evidence of how high the level of difficulty is in this examination.
Questions are not restricted to the news itself, rather the questions are being asked about the static
information related to the news. So, prepare the background information of the news.

Q.53)
Which country will provide an additional Sustainable development aid worth 10 billion Euros to India to
support the latter’s green growth plan till 2030?
(a) Netherlands
(b) Germany
(c) Denmark
(d) France
(e) United Kingdom

Answer – (b)
Explanation –
It belongs to the international current affairs. Such questions are covered in the Spotlight magazine. You
can cover such types of news from different newspaper. Whichever source you choose, always
remember that you have to do multiple revisions to retain the facts.

Q.54)
How many members are there in OECD?
(a) 38
(b) 43
(c) 67
(d) 80
(e) 55

Answer – (a)
Explanation –
It belongs to the international organizations. Always remember to look for the background facts related
to the news to get good marks in the exam. So, covering them is a must. the sources which you can use
to cover the static general awareness are Lucent’s book, Manorama’s yearbook etc. Always remember
that in order to retain a lot of facts you should be revising the facts again and again.

Q.55)
Which of the following are not MPC committee assumptions in April 2022?
(a) Assumption of a normal monsoon
(b) Moderation on international commodity price pressures
(c) Average crude oil price
(d) Steady economic recovery
(e) Tightened fiscal policy

Answer – (e)
Explanation –
This question has been asked from the banking current affairs. This question is from the framework and
guidelines topic. Policy-related guidelines and frameworks are always important and need to be covered
in detail. All these kinds of news can be covered from some good newspapers like Livemint, Indian
Express, Business Standard, Economic Times, The Hindu etc. The Spotlight and RBI 247 magazines and
sessions strive to convey the policies in detail. Whichever source you choose make sure that you stick to
that.

Q.56)
Which of the following is not the part of Memorandum of a Company?
(a) Board of director Clause
(b) Name Clause
(c) Object Clause
(d) Capital Clause
(e) Liability Clause

Answer – (a)
Explanation –
This is an out of the blue question. In case, you are a little bit aware about the question, then only try to
make a guess or deploy the elimination technique. Otherwise, the best strategy to deal with such
questions is not to attempt them.

Q.57)
How many institutions are members of Financial Stability Board?
(a) 71
(b) 69
(c) 80
(d) 110
(e) 45

Answer – (a)
Explanation –
Financial Stability Board (FSB) is an international body that monitors and makes recommendations about
the global financial system. It was established after the G20 London summit in April 2009 as a successor
to the Financial Stability Forum (FSF). This question might appear as a static GK question but it is not
so. It is rather a current affairs’ question. So, always remember to look for the background facts related
to the news to get good marks in the exam.

Q.58)
Which of the following sector/area is not under the capital expenditure of government?
(a) Interest Payment
(b) Loan Repayment
(c) Grants to other countries
(d) Remittances
(e) Infrastructure development

Answer – (d)
Explanation –
This question has been asked from the static economics. Basic economic concepts are also important for
the exam. So, adopt an integrated approach for tackling this exam.

Q.59)
SBI card signed a deal with which technology company?
(a) Tata Consultancy Services
(b) IBM
(c) Infosys
(d) Wipro
(e) Oracle

Answer – (a)
Explanation –
This is a company-specific question. All the company-specific news released during the six months prior
to the exam date are important.

Q.60)
Which is India’s indigenous train accident system?
(a) Kavach
(b) Netra
(c) Suraksha
(d) Saksham
(e) Nikshay

Answer – (a)
Explanation –
This is question from the national news. National news section includes important events, summits,
conferences, initiatives of national importance. Such types of news are covered in Spotlight in the
national news category.

Q.61)
Which state Khadi Gramodyoga Samyukta Sangha gave the contract of production and supply of the
National Flag?
(a) Uttarakhand Khadi Gramodyoga Samyukta Sangha
(b) Kerala Khadi Gramodyoga Samyukta Sangha
(c) Telangana Khadi Gramodyoga Samyukta Sangha
(d) Karnataka Khadi Gramodyoga Samyukta Sangha
(e) Gujarat Khadi Gramodyoga Samyukta Sangha

Answer – (d)
Explanation –
This is question from the national news. Policy-related guidelines and frameworks are always important
and need to be covered in detail. Year by Year, the level of the exam is getting tougher. Thus, it requires
more conceptual understanding and fact memorization. You should prepare past six months’ current
affairs thoroughly. In order to cover all the events of the past 6 months, preceding the date of
examination, you can select a good source. Spotlight magazine covers all such news.

Q.62)
Project Venus of ISRO is to be Launched in?
(a) December 2024
(b) June 2022
(c) August 2023
(d) September 2025
(e) October 2027

Answer – (a)
Explanation –
This is question from the national news. In this question, we have observed that the direct questions are
most often asked in the exam. These types of direct and short sentenced questions do not leave any
scope of any hint or guess work. So, the best strategy to tackle such questions is to leave them because
you have negative marking in the exam. Taking such a risk can cost you a lot.

Q.63)
Cyclone Asani is named by which country?
(a) Sri Lanka
(b) Thailand
(c) UAE
(d) Saudi Arabia
(e) Indonesia

Answer – (a)
Explanation –
Natural disasters are unfortunate events and not knowing about them is even more unfortunate for an
aspirant. Here, you have seen that yourself. So, prepare the names of the cyclones and other disasters
and their affected areas.

Q.64)
Covid champion award has been bagged by which airport?
(a) Rajiv Gandhi International Airport
(b) Cochin International Airport Limited
(c) Lokpriya Gopinath Bordoloi International Airport
(d) Indira Gandhi International Airport
(e) Veer Savarkar International Airport

Answer – (b)
Explanation –
This is a question from the awards section. All the awards, the country/organization giving the awards,
the recipients, and the recipients’ nationalities are all important. This question is a direct and factual
question. Therefore, it cannot be attempted by doing the guesswork. So, in such type of questions if you
know the answer only then attempt otherwise you should leave these questions. You should be aware
of this fact that you don't have to score 100% marks in general awareness you just need to clear the cut
off. Therefore, if you will target at least 70 percent marks in the GA section of RBI grade b then you are
on the safer side.

Q.65)
Khelo India university championship 2021 was won by which university?
(a) Jain University
(b) Acharya Nagarjuna University
(c) Andhra University
(d) Vikrama Simhapuri University
(e) Bodoland University

Answer – (a)
Explanation –
It is a sports-related news. It is a direct question. Major sports events should be thoroughly covered by
you.

Q.66)
Central Intelligence Agency has appointed ______ as CIA’s first-ever Chief Technology Officer (CTO).
(a) L. Murugan
(b) Chandra Sekhar
(c) Gopal Sadhwani
(d) Nand Mulchandani
(e) Pankaj Salodia

Answer – (d)
Explanation –
This question is from the appointments section. All the latest appointments prior to the exam are
important.

Q.67)
National Forensic science university comes under which ministry?
(a) Ministry of Agriculture and Family Welfare
(b) Ministry of Home Affairs
(c) Ministry of Science and Technology
(d) Ministry of Corporate Affairs
(e) Ministry of Finance

Answer – (b)
Explanation –
This is question from the national news. This question is the evidence of how high the level of difficulty
is in this examination. It requires more conceptual understanding and fact memorization. One more
thing, conceptual understanding will help you in retaining the facts for a longer period of time.

Q.68)
When is the World Environment Day observed?
(a) June 5
(b) August 10
(c) July 18
(d) September 20
(e) April 22

Answer – (a)
Explanation –
This is a question asked from the important days. So, don't ignore important days and their themes. At
the same time, focus on the major important days should be made. The major days are those which
have been asked repeatedly and which have a huge importance worldwide for eg. World Health Day,
AIDS Day etc.

Q.69)
Asia's largest event on civil aviation jointly organised by the Ministry of Civil Aviation and FICCI at the
Begumpet airport in Hyderabad. What is the name of the event?
(a) Asia Air
(b) Fly India
(c) Civil India
(d) Wings India
(e) Aviation India

Answer – (d)
Explanation –
This is question from the national news. National news section includes important events, summits,
conferences, initiatives of national importance. Year by Year, the level of the exam is getting tougher.
Thus, it requires more conceptual understanding and fact memorization. One more thing, conceptual
understanding will help you in retaining the facts for a longer period of time. To understand the
concepts in detail, you can watch the daily current affairs videos on my channel. Such types of news are
covered in Spotlight in the national news category.

Q.70)
How many Debts Recovery Tribunals are presently functioning in India?
(a) 39
(b) 43
(c) 50
(d) 80
(e) 78

Answer – (a)
Explanation –
This is not a current affairs question. Rather, it is a question to check your knowledge on a broad level.
So, reading newspapers and developing awareness about the things going around you is general
awareness and this is only tested through such questions. So, if you are not able to make a guess, don’t
attempt this question as you have negative marking as well.

Q.71)
As per IRDAI, ____ is the second largest general insurance company?
(a) Star Health and Allied Insurance
(b) ICICI Lombard
(c) United India General Insurance
(d) Universal Sompo General Insurance
(e) Oriental General Insurance

Answer – (b)
Explanation –
This is a company-specific question. All the company-specific news released during the six months prior
to the exam date are important.

Q.72)
As per National Education Policy, public investment is to be raised from 4.3 to ____% of GDP?
(a) 8%
(b) 5%
(c) 4.5%
(d) 6%
(e) 5.7%

Answer – (d)
Explanation –
This is question from the national news. Policy-related guidelines and frameworks are always important
and need to be covered in detail. Year by Year, the level of the exam is getting tougher. Thus, it requires
more conceptual understanding and fact memorization. You should prepare past six months’ current
affairs thoroughly. In order to cover all the events of the past 6 months, preceding the date of
examination, you can select a good source. Spotlight magazine covers all such news.

Q.73)
India’s monthly merchandise exports remained above US $ _____ billion for the first time in 2021-22?
(a) $40 billion
(b) $30 billion
(c) $20 billion
(d) $5 billion
(e) $10 billion

Answer – (a)
Explanation –
This question is a part of your banking awareness. All the news related to the banks and economics. You
can find the banking awareness related current affairs on a number of competitive websites. Here, you
should be selective in choosing your source of information. So, spend time on choosing a good source of
current affairs and once you have chosen the website or the magazine then stick to it. It is highly
advisable in your interest only not to shift from source to source.

Q.74)
When is the ‘Food Processing Week 2.0’ celebrated?
(a) April 25 to 30
(b) May 16 to 22
(c) April 1 to 7
(d) May 1 to 7
(e) May 17 to 22

Answer – (a)
Explanation –
This is a question asked from the important days. So, don't ignore important days and their themes.

Q.75)
What is the current market size of the domestic pharmaceutical industry of India?
(a) $100 billion
(b) $50 billion
(c) $30 billion
(d) $65 billion
(e) $71 billion

Answer – (b)
Explanation –
This is question from the national news. National news section includes government’s targets for
specific targets. Such types of news are covered in Spotlight in the national news category.

Q.76)
Who is the new chairperson of Central Board of Direct Taxes (CBDT)?
(a) R. K. Dhawan
(b) Rupinder Brar
(c) Santosh Kumar
(d) Sangeeta Singh
(e) Jaspal Singh

Answer – (d)
Explanation –
This question is from the appointments section. All the latest appointments prior to the exam are
important.

Q.77)
Who is the author of “Leaders Politicians Citizens: Fifty Figures Who Influenced India's Politics” book?
(a) Rasheed Kidwai
(b) Gyan Bhushan
(c) Rajesh Kumar
(d) Santosh Shilpokar
(e) Shoeb Samad
Answer – (a)
Explanation –
Books and their authors have been asked in the examination for a long period of time. You can witness
such questions in every type of examination. Therefore, you should keep a track of all the books which
are in the news. For keeping a track of the new launches, you can either rely on any specific exam
related magazine or read newspapers on a daily basis and create your own short notes from them.

Q.78)
With which country has India agreed to establish a lighthouse cooperation on "Agroecology and
Sustainable Management of Natural Resources” to benefit the rural population and small-scale farmers
in India in terms of income, food security, climate resilience, improved soil, biodiversity, forest
restoration and water availability?
(a) USA
(b) Germany
(c) France
(d) Denmark
(e) UK

Answer – (b)
Explanation –
This is question from the national news. Policy-related guidelines and frameworks are always important
and need to be covered in detail. Year by Year, the level of the exam is getting tougher. Thus, it requires
more conceptual understanding and fact memorization. You should prepare past six months’ current
affairs thoroughly. In order to cover all the events of the past 6 months, preceding the date of
examination, you can select a good source. Spotlight magazine covers all such news.

Q.79)
Where is the ‘Miyan Ka Bada’ railway station located?
(a) Jalore
(b) Barmer
(c) Jhalawar
(d) Sirohi
(e) Karauli

Answer – (b)
Explanation –
It is a state-specific current affairs’ question. Many questions are asked from the state current affairs.
So, make sure that you prepare the state-specific current affairs thoroughly. Such types of news are
covered in Spotlight.

Q.80)
How much additional subsidy for phosphate-based fertilizers?
(a) Rs 50000 crore
(b) Rs 78000 crore
(c) Rs 40000 crore
(d) Rs 25000 crore
(e) Rs 30000 crore
Answer – (c)
Explanation –
This is question from the national news. National news section includes important events, summits,
conferences, initiatives of national importance. Such types of news are covered in Spotlight in the
national news category.

English language
Instruction for Q.1 to Q.5

Read the following passage carefully and answer the given questions. Certain words are given in bold to
help you locate them while answering some of the questions.

On attending a conference, which focused on the role of the services sector in Indian economy I was
amazed. The conference gave a very interesting perspective on the role of the service sector in the
growth of Indian economy in relation to growth rates in agriculture and industry.

The current situation in India is that the growth rate of services has overtaken both agriculture and
industry and is now contributing to more than 50% of GDP. The service sector has the highest growth
rate and is the least volatile sector. Growth is particularly marked in public services, IT and financial
services.

In some areas. the growth rate of the service sector is 40-505 due to increased use of mobile
technologies. India, therefore, has a service-oriented economy. It hasn't traditional growth models as in
China. However, in the process of doing so it has skipped the manufacturing and has jumped straight
from the agriculture stage to service stage, which is also the main reason for the expansion of the
service sector.

In fact, the situation now is such that the growth in the service sector can and will support in the
agriculture and industrial sectors. However, the only setback for Indian economy is the lack of growth in
the manufacturing sector which causes dependence on other countries, which is not so desirable in
terms of job creation and increased prosperity.

Population is also a major concern of the Indian economy as the population of India grows so also does
the number of dependents in the population in both the lower and higher age groups. In such a scenario
of increasing Population, especially in an economy which still recovering from crisis, growth becomes
difficult. For such an economy to grow it has to invest.

Currently, the public sector invests more than it saves. The household sector saves in surplus, but it is
not increasing so it cannot continue to support private and public sectors. There is a massive need to
spend on agriculture and infrastructure development of the country. Apart from that health and
education should also be the priority of the government particularly the education of women to reduce
the birth rate.
Q.1)

Choose the word which is most opposite in meaning to word given in bold as used in the passage.

Volatile

(a) Erratic

(b) Impatient

(c) Stable

(d) Solid

(e) Strained

Answer – (a)

Explanation –

The most opposite meaning of volatile is 'erratic'. The best way to answer such questions is to replace
the highlighted word or the word asked in the question with each option to assess which one fits best in
the context of the line or paragraph.

Q.2)

According to the passage, which of the following is/are true about the impact of increasing population
on Indian economy?

(1) If India attempts to absorb all the labour force, it will impact the growth of service sector negatively
and in turn hamper the economic growth of the county.

(2) As the population of the country increases, the number of dependants in the country also
increase which in turn increases the pressure on the economy.

(3) An increasing population can never lead the economy of the country towards prosperity; in fact, it
can only put strain on the economy of a county.

(a) Only 2

(b) Only 3

(c) 2 and 3

(d) 1 and 3

(e) All of these


Answer – (a)

Explanation –

According to the passage, 'as the population of the country increases, the number of dependants in the
country also increase which in turn increases the pressure on the economy' is true about the impact of
increasing population on Indian economy.

Q.3)

According to the passage, which of the following can be said about the agriculture and industry sectors
in India?

(1) Looking at the growth of the services sector in India it can be safely said that the service sector
will soon be in a position to support both agriculture and industry sector.

(2) The agriculture and the industrial sector of the country have reached their threshold and there
would be no use of further investment in these sectors.

(3) currently, the agriculture and industry sectors contribute lesser as compared to service sector to the
GDP of the country.

(a) only 1

(b) Only 3

(c) 1 and 3

(d)2 and 3

(e) all of these

Answer – (c)

Explanation –

‘Looking at the growth of the service sector in India it can be safely said that the service sector will soon
be in a position to support both agricultural and industry sector’ and ‘currently the agricultural and
industry sector contribute lesser as compared to service sector to the GDP of the country’, are said
about the agriculture and industry sectors in India.

Q.4)
Choose the word which is most similar in meaning to the word given in bold as used in the passage.
Perspective

(a) Viewpoint

(b) Prospect

(c) Attitude

(d) Agreement

(e) Proportion

Answer – (a)

Explanation –

The most similar in meaning to the word 'perspective' is ‘viewpoint’. The best way to answer such
questions is to replace the highlighted word or the word asked in the question with each option to
assess which one fits best in the context of the line or paragraph.

Q.5)

Which of the following is true as per the passage?

(a) India has not followed the conventional model of growth and has moved directly from the agriculture
sector to the service sector.

(b) The service sector of the country is yet to make a mark on the IT and financial sectors of the country

(c) With availability of labour and growth in human skills, the service sector of Indian economy is
booming limitlessly as there is no restriction on movement of labour

(d) India has become self-reliant and does not have to depend on other countries because of the
development in the manufacturing sector.

(e) All of the above

Answer – (d)

Explanation –

India has become self-reliant and does not have to depend on other countries because of the
development in the manufacturing sector' is true statement as per passage.

Instructions for Q.6 to Q.10


In the following passage, some of the words have been left out. Read the passage carefully and fill in the
blanks by selecting the most appropriate alternatives. The question number from which a word is to be
selected out of the given alternatives, is written in each blank space.

Q.6)

The latest surgical tools are certainly fashionable, especially lasers and laparoscopes. But their role in
transforming surgery is ___ (6), as is the excitement they generate. Dr. Sanjay C from Delhi specialises in
remodelling the ____ (7) with what is called an excimer laser. "It's like etching on gold to design
jewellery", he says. "The laser manages to either flatten or raise the cornea, solving the problem of
myopia". Now watch Dr. Harshad P, in his clinic as he gazes not at this ____ (8) but at a television screen,
which his patient too is ____ (9) from the operating table. Controlled from the outside, Dr. Harshad’s
laser is waltzing through his patient’s prostrate; it’s called a laser prostatectomy. A miniature camera
and a beam of light from a fibre-optic wire, both at the end of a laparoscope, allow him to see ____ (10)
than with the naked eyes.

choose the correct option:

(a) questionable

(b) discretionary

(c) unpardonable

(d) unquestionable

(e) indisputable

Answer – (d)

Explanation –

Options a, b and c do not go with the theme, only option d “unquestionable”, matches the
exceptionable role of the laser. The most fundamental step to answer such questions is to build a strong
vocabulary and brush up on your grammar. Learn new phrases, synonyms, antonyms, and idioms every
day. And last but not the least advice is ‘practice more’.

Q.7)

The latest surgical tools are certainly fashionable, especially lasers and laparoscopes. But their role in
transforming surgery is ___ (6), as is the excitement they generate. Dr. Sanjay C from Delhi specialises in
remodelling the ____ (7) with what is called an excimer laser. "It's like etching on gold to design
jewellery", he says. "The laser manages to either flatten or raise the cornea, solving the problem of
myopia". Now watch Dr. Harshad P, in his clinic as he gazes not at this ____ (8) but at a television screen,
which his patient too is ____ (9) from the operating table. Controlled from the outside, Dr. Harshad’s
laser is waltzing through his patient’s prostrate; it’s called a laser prostatectomy. A miniature camera
and a beam of light from a fibre-optic wire, both at the end of a laparoscope, allow him to see ____ (10)
than with the naked eyes.

choose the correct option:

(a) body

(b) clinic

(c) world

(d) eye

(e) physique

Answer – (d)

Explanation –

Doctor Sanjay C definitely “remodels the eyes” as in the later part he talks about flattening or raising the
cornea. Hence option (d) is correct. The most fundamental step to answer such questions is to build a
strong vocabulary. Learn new phrases, synonyms, antonyms, and idioms every day. And last but not the
least advice is ‘practice more’.

Q.8)

The latest surgical tools are certainly fashionable, especially lasers and laparoscopes. But their role in
transforming surgery is ___ (6), as is the excitement they generate. Dr. Sanjay C from Delhi specialises in
remodelling the ____ (7) with what is called an excimer laser. "It's like etching on gold to design
jewellery", he says. "The laser manages to either flatten or raise the cornea, solving the problem of
myopia". Now watch Dr. Harshad P, in his clinic as he gazes not at this ____ (8) but at a television screen,
which his patient too is ____ (9) from the operating table. Controlled from the outside, Dr. Harshad’s
laser is waltzing through his patient’s prostrate; it’s called a laser prostatectomy. A miniature camera
and a beam of light from a fibre-optic wire, both at the end of a laparoscope, allow him to see ____ (10)
than with the naked eyes.

choose the correct option:

(a) family

(b) future

(c) patient

(d) awards

(e) reward
Answer – (c)

Explanation –

Doctor Harshad P. would be expected to look at his “patient” under normal circumstances to diagnose
the problem or solve it. Hence option ‘c’ is correct. You should try to understand the context of the
paragraph. Sometimes, the context of the paragraph also helps in identifying the correct option.

Q.9)

The latest surgical tools are certainly fashionable, especially lasers and laparoscopes. But their role in
transforming surgery is ___ (6), as is the excitement they generate. Dr. Sanjay C from Delhi specialises in
remodelling the ____ (7) with what is called an excimer laser. "It's like etching on gold to design
jewellery", he says. "The laser manages to either flatten or raise the cornea, solving the problem of
myopia". Now watch Dr. Harshad P, in his clinic as he gazes not at this ____ (8) but at a television screen,
which his patient too is ____ (9) from the operating table. Controlled from the outside, Dr. Harshad’s
laser is waltzing through his patient’s prostrate; it’s called a laser prostatectomy. A miniature camera
and a beam of light from a fibre-optic wire, both at the end of a laparoscope, allow him to see ____ (10)
than with the naked eyes.

choose the correct option:

(a) operating

(b) diagnosing

(c) watching

(d) criticising

(e) monitoring

Answer – (c)

Explanation –

The patient cannot be operating, diagnosing or criticising lying on the operating table. He can only be
“watching” the projection on the TV screen. Hence, option ‘c’ is correct. You should try to understand
the context of the paragraph. Many-a-time, the context of the paragraph also helps in identifying the
correct option.

Q.10)

The latest surgical tools are certainly fashionable, especially lasers and laparoscopes. But their role in
transforming surgery is ___ (6), as is the excitement they generate. Dr. Sanjay C from Delhi specialises in
remodelling the ____ (7) with what is called an excimer laser. "It's like etching on gold to design
jewellery", he says. "The laser manages to either flatten or raise the cornea, solving the problem of
myopia". Now watch Dr. Harshad P, in his clinic as he gazes not at this ____ (8) but at a television screen,
which his patient too is ____ (9) from the operating table. Controlled from the outside, Dr. Harshad’s
laser is waltzing through his patient’s prostrate; it’s called a laser prostatectomy. A miniature camera
and a beam of light from a fibre-optic wire, both at the end of a laparoscope, allow him to see ____ (10)
than with the naked eyes.

choose the correct option:

(a) tens of times better

(b) not much better

(c) restricted vision

(d) more colourfully

(e) much colored

Answer – (a)

Explanation –

Here only two options make sense, i.e., option(a) “tens of times better” and option (d) “more
colourfully” but ‘seeming more colourfully’ is not at all a desired advantage of the laser surgery. Option
‘a’, “tens of times better” is the required property of the laser. Hence option ‘a’ is correct.

Instruction for Q.11 to Q.15

Each question below has two blanks, each blank indicating that something has been omitted. Choose
the set of words for each blank that best fits the meaning of the sentence as a whole.

Q.11)

As sugar prices have (1)/ collapsed on, supply surges for four years in (2)/ a row, user industries (3)/ are
being repeated a bonanza. (4)/ No error (5).

(a) 1

(b) 2

(c) 3

(d) 4

(e) 5
Answer – (d)

Explanation –

Here, use of passive is improper. Hence, ‘are repeating a bonanza’ is the right usage. The very first thing
to consider in such type of questions is to strengthen your grammatical concepts. If you are not clear
with the basics, handling such questions would be a hard nut to crack for you.

Q.12)

Coffee production in India (1)/ is expected to touch a new peak this year, (2)/ as the crop prospects ae
encouraging (3)/ due to adequate rains. (4)/ No error (5).

(a) 1

(b) 2

(c) 3

(d) 4

(e) 5

Answer – (e)

Explanation –

No error. The very first thing to consider in such type of questions is to strengthen your grammatical
concepts. If you are not clear with the basics, handling such questions would be a hard nut to crack for
you. I am going to tell you some basic rules which you can remember while answering these questions.
However, you can follow some rules to make the process of identifying the right answers easy.
Firstly, check and identify the part of the speech (whether it is adjective, adverb, noun, pronoun etc.) in
the sentence. Second, identify the type of sentence (whether the sentence is an interrogative,
affirmative, or negative one). Third, check punctuation errors (and this is going to be the easiest one).
Fourth, identify the subject-verb agreement of the sentence.
If you follow these steps, it will become easier for you to answer these questions.

Q.13)

Country launched a naval operation (1)/ to stop human traffickers (2)/ by bringing migrants (3)/ across
the border. (4) No error. (5)

(a) 1

(b) 2
(c) 3

(d) 4

(e) 5

Answer – (c)

Explanation –

Here, it is a preposition-related error. Hence, ‘from bringing migrants’ is the right usage.

The very first thing to consider in such type of questions is to strengthen your grammatical concepts. I
am going to tell you some basic rules which you can remember while answering these questions.
However, you can follow some rules to make the process of identifying the right answers easy.
Firstly, check and identify the part of the speech (whether it is adjective, adverb, noun, pronoun etc.) in
the sentence. Second, identify the type of sentence (whether the sentence is an interrogative,
affirmative, or negative one). Third, check punctuation errors (and this is going to be the easiest one).
Fourth, identify the subject-verb agreement of the sentence.
If you follow these steps, it will become easier for you to answer these questions.

Q.14)

In the swimming pool area (1)/ the guards were busy (2)/ to stopping children (3)/ from falling into the
leg pool. (4) No error (5).

(a) 1

(b) 2

(c) 3

(d) 4

(e) 5

Answer – (c)

Explanation –

Use of ‘to’ is superfluous. Hence, ‘stopping children’ should be used. You should, firstly, check and
identify the part of the speech (whether it is adjective, adverb, noun, pronoun etc.) in the sentence.
Second, identify the type of sentence (whether the sentence is an interrogative, affirmative, or negative
one). Third, check punctuation errors (and this is going to be the easiest one). Fourth, identify the
subject-verb agreement of the sentence. If you follow these steps, it will become easier for you to
answer these questions.
Q.15)

If cooking is an expression of love and you want (1)/ your loved ones to know how much you love them
(2)/ it will be a pity if you could (3)/ did no much than order pizza. (4) No error (5).

(a) 1

(b) 2

(c) 3

(d) 4

(e) 5

Answer – (d)

Explanation –

The word should be used with the first form of verb. Hence, ‘do no more than order pizza’ is the right
usage. In comparative degree, ‘than’ is used. To solve the question of error spotting, you should follow
a systematic approach. Firstly, check and identify the part of the speech (whether it is adjective, adverb,
noun, pronoun etc.) in the sentence. Second, identify the type of sentence (whether the sentence is an
interrogative, affirmative, or negative one). Third, check punctuation errors (and this is going to be the
easiest one). Fourth, identify the subject-verb agreement of the sentence.

Instructions for Q.16 to Q.20

Each question below has two blanks, each blank indicating that something has been omitted. Choose
the set of words for each blank that best fits the meaning of the sentence as a whole.

Q.16)

Researchers _____ that eating less meat would help in _____ water resources in dry areas around the
world.

(a) opine; conserve

(b) said; procuring

(c) suggest; removing

(d) prove; store

(e) believe; preserving

Answer – (e)
Explanation –

‘opine’, ‘said’ and ‘prove’ are not suitable for the first blank as researchers always believe or suggest,
‘removing’ cannot be fit with the problem of water resources in dry areas. Only ‘preserving’ is suitable
for the second blank. Hence, ‘believe’ and ‘preserving’ are suitable fillers.

Q.17)

Harish____ with me some basic techniques that one can _____ at home without even owning a drum
set.

(a) made; shred

(b) imparted; try

(c) learned; balance

(d) fits; rehearses

(e) shared; practice

Answer – (a)

Explanation –

Use of ‘drum set’ invites the use of word ‘practice’ and hence the first blank will be filled with ‘shared’.
So, (e) is correct choice. You should build a strong vocabulary to clear the competitive exams’ English.
For this purpose, you should read editorials from the newspaper daily.

Q.18)

Forest department officials said that when the elephants were made to _____ from their trucks they
went straight to the spot where they had been _____ during the camp.

(a) jump; killed

(b) alight; tied

(c) enter; hurt

(d) step; played

(e) exit; enjoyed

Answer – (b)

Explanation –
As the elephants are transported. So, use of ‘alight’ is suitable for first blank. Also ‘tied’ gives the correct
context in the second blank. You should build a strong vocabulary to clear the competitive exams’
English. For this purpose, you should read editorials from the newspaper daily. It will not only help in
increasing the speed of reading but also aid in enhancing your vocabulary.

Q.19)

A collision between two houses ______ 6 people dead, _______ the driver of one of the buses.

(a) made; also

(b) left; including

(c) caused; combined

(d) resulted; except

(e) got; surpassing

Answer – (b)

Explanation –

As one of the drivers is injured and dead then, so use of ‘including’ is justified for the second blank.
Hence, (b) is the correct choice. You should build a strong vocabulary to clear the competitive exams’
English. Learn new phrases, synonyms, antonyms, and idioms every day. And last but not the least
advice is ‘practice more’.

Q.20)

The protests were ______ at ______ awareness among the elite and the educated.

(a) held; generating

(b) aimed; creating

(c) targeted; awakening

(d) focused; building

(e) concentrated; producing

Answer – (b)

Explanation –
Awareness is ‘created’ and hence second blank has the obvious usage. So, (b) is the correct choice. You
should build a strong vocabulary to clear the competitive exams’ English. Learn new phrases, synonyms,
antonyms, and idioms every day. And last but not the least advice is ‘practice more’.

Q.21)

In each of the following questions, a sentence split into four parts labelled 1, 2, 3, 4 is given. Rearrange
the parts to form the original sentence and select the correct order from among the five choices given
below and mark its number as your answer:

(1) acquires the nature of a corrective mechanism

(2) of different hues and degrees

(3) for a nation whose secular credentials are being nibbled at by various reactionary forces

(4) any reminder of the ideology professed by its first Prime Minister

(a) 4321

(b) 3241

(c) 1243

(d) 3124

(e) 1432

Answer – (b) 3241

Explanation –

For a nation whose secular credentials are being nibbled at by various reactionary forces of different
hues and degrees any reminder of the ideology professed by its first Prime Minister acquires the nature
of a corrective mechanism. You should try to understand the context of the sentence. Sometimes, the
context of the sentence also helps in identifying the correct option.

Q.22)

In each of the following questions, a sentence split into four parts labelled 1, 2, 3, 4 is given. Rearrange
the parts to form the original sentence and select the correct order from among the five choices given
below and mark its number as your answer:

(1) depends not on merit and ability but on quotas based on caste

(2) or have a good work ethic as one of its core values


(3) community and gender or any other criterion, can hardly be stable

(4) a society where access to education and jobs

(a) 3241

(b) 4231

(c) 3214

(d) 4123

(e) 4132

Answer – (e) 4132

Explanation –

A society where access to education and jobs depends not on merit and ability but on quotas based on
caste community and gender or any other criterion, can hardly be stable or have a good work ethic as
one of its core values. You should try to understand the context of the sentence. Sometimes, the context
of the sentence also helps in identifying the correct option.

Q.23)

In each of the following questions, a sentence split into four parts labelled 1, 2, 3, 4 is given. Rearrange
the parts to form the original sentence and select the correct order from among the five choices given
below and mark its number as your answer:

(1) to locate the right man for the right job

(2) with the dotcom frenzy refusing to suicide despite forecasts of a burnout

(3) the high exit volume and the paucity of time and avenues

(4) placement – fairs have moved in to rectify what software companies have been struggling with for
some time

(a) 4123

(b) 3124

(c) 2431

(d) 2341

(e) 3214
Answer – (b) 3124

Explanation –

The high exit volume and the paucity of time and avenues to locate the right man for the right job with
the dotcom frenzy refusing to suicide despite forecasts of a burnout placement – fairs have moved in to
rectify what software companies have been struggling with for some time.

Q.24)

In each of the following questions, a sentence split into four parts labelled 1, 2, 3, 4 is given. Rearrange
the parts to form the original sentence and select the correct order from among the five choices given
below and mark its number as your answer:

(1) the left democratic Front Government in Kerala

(2) a section of the police force and some political parties in the ruling alliance

(3) has shelved its plans to start the much- publicized people’s policing scheme

(4) faced with stiff opposition from the public

(a) 1342

(b) 2134

(c) 2341

(d) 4213

(e) 1423

Answer – (d)

Explanation –

Faced with stiff opposition from the public a section of the police force and some political parties in the
ruling alliance the left democratic Front Government in Kerala has shelved its plans to start the much-
publicized people’s policing scheme. You should try to understand the context of the sentence.
Sometimes, the context of the sentence also helps in identifying the correct option.

Q.25)
In each of the following questions, a sentence split into four parts labelled 1, 2, 3, 4 is given. Rearrange
the parts to form the original sentence and select the correct order from among the five choices given
below and mark its number as your answer:

(1) Psychiatrists, child workers and educationists are emphasizing the importance of bed time stories

(2) the practice of children watching television or playing computer games before going to bed

(3) and they are questioning

(4) since the kids are bidding adieu to fairy tales or the simple habit of reading at bed time for a variety
of reasons

(a) 2413

(b) 4231

(c) 2314

(d) 1234

(e) 1324

Answer – (e)

Explanation –

Psychiatrists, child workers and educationists are emphasizing the importance of bed time stories and
they are questioning the practice of children watching television or playing computer games before
going to bed since the kids are bidding adieu to fairy tales or the simple habit of reading at bed time for
a variety of reasons.

You should try to understand the context of the sentence. Sometimes, the context of the sentence also
helps in identifying the correct option.

Instruction for Q.26 to Q.30

Read the following passage carefully and answer the given questions. Certain words are given in bold to
help you locate them while answering some of the questions.

In a reversal of the norm elsewhere, in India policy-makers and economists have become optimists
while bosses do the worrying. The country's Central Bank has predicted that the country's economy is
likely to grow at a double-digit rate during the next 20-30 years. India has the capability with its vast
labour and lauded entrepreneurial spirit

But the private sector which is supposed to do the heavy lifting that turns India from the world's 10th
largest economy to its third largest by 2030 has become fed up. Business people often carp
about India's problems, but their irritation this time has a nervous edge. In the first quarter of 2011, GDP
grew at an annual rate 7.8%; in 2005-07 it managed 9-10%.

The economy may be slowing naturally as the low interest rates and public spending that got India
through the global crisis are belatedly withdrawn. At the same time, the surge in inflation caused by
exorbitant food prices has spread more widely, casting doubt over whether India can grow at 8-10% in
the medium term without overheating.

In India, as in many fast-growing nations, the confidence to invest depends on the conviction that
the long-term trajectory is intact, and it is that which is in doubt Big Indian firms too sometimes seem
happier to invest abroad than at home, in deals that are often hailed as symbols of the country's
growing clout, but sometimes speak of its weaknesses-purchases of natural resources that India has in
abundance, but struggles to get out of the ground.

In fact, a further dip in investment could be self-fulfilling if fewer roads, ports and factories are built,
this will hurt both short-term growth figures and reduce the economy’s long-term capacity. There is a
view that because a fair amount of growth is assured, the government need not try very hard. The
liberalisation reforms that began in 1991 freed markets for products and gave rise to vibrant
competition.

At the same time, what economists call factor markets, those for basic inputs

like land, power, labour etc. remain unreformed and largely under state control, etc. which creates
difficulties. Clearances today can take three to four years and many employers are keen to replace
workers with machines despite an abundance of labour force.

This can be attributed to labour laws which are inimical to employee creation and an education
system that means finding quality manpower a major problem. In fact, the planning Commission
concluded that even achieving 9% growth will need marked policy action in unreformed sectors. 20
years ago, it was said that the yardstick against which India should be measured was its potential and it
is clear that there remains much to do.

Q.26)

Which of the following can be said about the Indian economy at present?

(a) It can comfortably achieve double digit growth rate at present.

(b) High food prices have led to overheating of the economy.

(c) Citizens are affluent owing to laxity in regulation.

(d) Private sector confidence in India's growth potential is high.

(e) Unreformed sectors are a drag on economic growth.

Answer – (e)
Explanation –

Unreformed sectors are a drag on economic growth' can be said about the Indian Economy at
present. Reading questions first and then reading the paragraph saves the time of the aspirants. Read
the questions, write keywords of the questions in the rough sheet. This way, you can find the answers
from the reading comprehension very easily.

Q.27)

Why are employers reluctant to hire Indian labour force?

(1) India's labour force is overqualified for the employment opportunities available.

(2) High attrition rate among employees stemming from their entrepreneurial spirit.

(3) Labour laws are not conducive to generating employment.

(a) only 3

(b) 1 and 2

(c) 1 and 3

(d) all of these

(e) none of these

Answer – (a)

Explanation –

Employers are reluctant to hire Indian Labour Force because ‘Labour Laws are not conducive to
generating employment’. Reading questions first and then reading the paragraph saves the time of the
aspirants. Read the questions, write keywords of the questions in the rough sheet. This way, you can
find the answers from the reading comprehension very easily.

Q.28)

What is the state of India’s basic input sectors at present?

(a) These sectors attract Foreign Direct Investment because of their vast potential.

(b) These sectors are lagging as projects are usually awarded to foreign companies.

(c) These sectors are stagnating and badly in need of reforms.

(d) These sectors are well regulated as these are governed by state.
(e) none of the above.

Answer – (c)

Explanation –

The state of India's basic input sectors at present is that these sectors are stagnating and badly in need
of reforms.

Q.29)

What is the author’s main objective in writing the passage?

(a) Show casing the potential of India's growth potential to entice foreign investors.

(b) Exhorting India to implement measures to live up to its potential.

(c) Recommending India's model of development to other developing countries.

(d) Berating the private sector for not bidding for infrastructure development projects.

(e) Criticising the measures taken by India during the economic crisis.

Answer – (b)

Explanation –

The author's main objective is 'Exhorting India to implement measures to live up to its potential.’

Q.30)

What impact has the GDP growth of 7.8% had?

(1) Indian Industry is anxious about India's economic growth.

(2) India has achieved status as the world's third economy at present.

(3) Foreign investment in India has drastically in

(a) only 1

(b) 1 and 2

(c) 1 and 3

(d) all of these


(e) none of these

Answer – (e)

Explanation –

None of these is the right answer. Reading questions first and then reading the paragraph saves the time
of the aspirants. Read the questions, write keywords of the questions in the rough sheet. This way, you
can find the answers from the reading comprehension very easily.

Quantitative Aptitude

Q.1)
Marked price is 1.45 times of cost price. Selling price is Rs 1500. The profit is 25%. Find the
discount percentage.
(a) 14%
(b) 15%
(c) 16%
(d) 12%
(e) 13%
Answer - (a)
Explanation -
CP = 1200
SP = 1500
MP = 1740
Discount = 240
Discount% = 240/1740 x 100 = 13.79%

Instruction for Q.2 to Q.4

Identify the wrong term (X) and answer the questions that follow:
2, 3, 7, 22, 89, 445, 2677, 18740
Q.2)
Find the LCM of (X – 400) and 25.
(a) 125
(b) 250
(c) 225
(d) 275
(e) 450

Answer - (c)
Explanation -
First of all analyse the pattern:
2x1+1=3

3x2+1=7
7 x 3 + 1 = 22
22 x 4 + 1 = 89
89 x 5 + 1 = 446
446 x 6 + 1 = 2677

2677 x 7 + 1 = 18740
So, the wrong term (X) = 445
Now, X – 400 = 445 – 400 = 45
So, LCM of 45 and 25 = 225

Q.3)
20% of X is same as which number from left?
(a) First
(b) Second
(c) Third
(d) Fourth
(e) Fifth

Answer - (e)
Explanation -
X = 445
20% of 445 = 89

Q.4)
Which of the following is a prime number?
(a) X+2
(b) X+4
(c) X+5
(d) X+3
(e) X+6
Answer - (b)

Explanation -
X + 4 = 445 + 4 = 449 (it is a prime number)

Q.5)
In Jar A, there is 40 litres milk and 6 litres water. In Jar B, there is 78 litres mixture of milk and
water and the ratio is 10:3. A and B jars are emptied in a Jar C. In Jar C, there is already 6 litres
water. What is the percentage of water in Jar C?
(a) 77%
(b) 25%
(c) 23%
(d) 75%
(e) 21%
Answer - (c)

Explanation -

Milk Water Total Ratio


Jar A 40 6 46 20:3
Jar B 60 18 78 10:3
Jar C 100 24 + 6 = 30 124 + 6 = 130 10:3

Percentage of water in Jar C = 3/13 * 100 = 23%


Instruction for Q.6
Consider the given question and decide which of the following statements is sufficient to
answer the question.

Q.6)
What is the speed of train A?
Statement A: The length of train A is 180m and the length of train B is 220m.

Statement B: If train A crossed train B in 25 sec, both trains are running in the opposite
direction.

(a) If question can be answered using A alone but not using B alone
(b) If question can be answered using B alone but not using A alone
(c) If question can be answered using either of the statements alone
(d) If question can be answered using both statements together but not using either A or B
alone
(e) If question can’t be answered even using A and B together

Answer - (e)
Explanation -
In statement A, lengths of both the trains are given which is not sufficient to find the speed of
train A. Hence, A alone is not sufficient.

From statement B, we can find the relative speed as 400/25 = 16m/s. But again, statement B
alone is also not sufficient.
Even using both the statements together, we can only get the relative speed of train A with
respect to train B. But we cannot get the absolute value of speed.

Instruction for Q.7 and Q.8


Consider the below given two series (with two missing values P and Q) and answer the
questions thereafter:

15, 17, 11, 23, P, 33


400, Q, 300, 750, 2625, 11812.5
Q.7)
If R is a composite value such that 14 < R < 18, what can be the possible values of (P + R)?
1) 18
2) 19
3) 20

(a) 1 and 2
(b) 2 and 3
(c) 1 and 3
(d) 1, 2 and 3
(e) Only 1

Answer - (a)
Explanation -
The pattern of the first series is as follows:
15 + 2 = 17
17 – 6 = 11

11 + 12 = 23
23 – 20 = 3 (=P)
3 + 30 = 33
The pattern of the second series is as follows:

400 x 0.5 = 200 (=Q)


200 x 1.5 = 300
300 x 2.5 = 750
750 x 3.5 = 2625

2625 x 4.5 = 11812.5


Now, R can take two values, i.e., 15 and 16 (because R is a composite number)
So, P + R could also take two values, i.e., 18 and 19.

Q.8)
Find the value of (Q – 2) / P x 3
(a) 180
(b) 186
(c) 198
(d) 201
(e) 192
Answer - (c)

Explanation -
(Q – 2) / P x 3 = ((200 – 2) / 3) x 3 = 198

Instruction for Q.9 to Q.11

Consider the below two quadratic equations:


ax2 – 9x + 7 = 0
by2 – 8y + 4 = 0

• a and b are positive integers


• b>a
• 6/b is the bigger root of second equation
• The ratio of highest roots of equation 1 to 2 is 7:4.

Q.9)
What are the values of a and b respectively?
(a) 1, 2
(b) 2, 3
(c) 1, 3
(d) 3, 1
(e) 3, 2
Answer - (b)

Explanation -Roots of a quadratic equation = (-b ± √𝑏2 − 4𝑎𝑐 ) / 2a

Roots of first equation = 9 ± √81 − 28𝑎 / 2a

Bigger root = 9 + √81 − 28𝑎 / 2a

Smaller root = 9 - √81 − 28𝑎 / 2a


Roots of second equation = 8 ± √64 − 16𝑏 / 2Z

Bigger root = 8 + √64 − 16𝑏 / 2b

Smaller root = 8 - √64 − 16𝑏 / 2b


Bigger root = 6/b

8 + √64 − 16𝑏 / 2b = 6/b


b=3

Since a is a positive integer and a < b, a can either take the value of 1 or 2. By substituting the
values, you will get a = 2.

Q.10)

Find the roots of the first equation.


(a) 2; 2/3
(b) 1; 2
(c) 1; 7/2
(d) 2; 3
(e) 1; 5/4
Answer - (c)
Explanation -
Since a = 2, the first equation becomes 2x2 – 9x + 7 = 0

The roots are 1 and 7/2.

Q.11)
Find the roots of the second equation.

(a) 2; 3
(b) 1; 2
(c) 2/3; 2
(d) 4/3; 1
(e) 5/2; 3
Answer - (c)
Explanation -
Since b = 3, the second equation becomes 3y2 – 8y + 4 = 0
The roots are 2/3 and 2.

Q.12)
What is Amit’s present age?

1) Five years ago, Amit’s age was double that of his daughter’s age at that time.

2) Present ages of Amit and his daughter are in the ratio of 11:6 respectively.

3) Five years hence, the respective ratio of Amit’s age and his daughter’s age will become 12:7.

(a) Only 1 and 3 are sufficient to answer the question


(b) Only 1 and 2 are sufficient to answer the question
(c) Only 2 and 3 are sufficient to answer the question
(d) Any two of the three statements are sufficient to answer the question
(e) All 1, 2 and 3 are required to answer the question

Answer - (d)

Explanation -

From statement 2: let the present ages of Amit and his daughter be 11x and 6x years respectively.

From statement 1: 5 years ago, Amit’s age = 2 * His daughter’s age

From statement 3: 35 years hence, Amit’s age : Daughter’s age = 12 : 7

Any two of the above three will give Amit’s present age.

Q.13)
A invested 2000 more than B. A left the business after 8 months. If annual profit is 6800 and
share of A is 400F, then find the investment of B.

Note: 7F2 = 472 - √576


(a) 2400
(b) 4000
(c) 6000
(d) 7200
(e) 8000
Answer - (c)
Explanation -
In these types of questions, we multiply capital by time and then find the mutual profit ratio.
Capital invested by B = ‘p’
Then, capital invested by A = ‘2000 + p’

Now, A = (200 + p) x 8 months = 16,000 + 8p


B = p x 12 months = 12p
Profit sharing ratio between A and B = 16000 + 8p : 12p = 4000 + 2p : 3p

7F2 = 472 - √576


F=8

So, 400F = 3200


Share of A = 400F = ((4000 + 2p) / (4000 + 5p)) x 6800
3200/6800 = (4000 + 2p) / (4000 + 5p)
(4000 + 2p) / (4000 + 5p) = 8/17

p = investment of B = 6000

Q.14)
Rohan mixed two varieties of clay for selling the mixture in his shop. What is the quantity of
clay of the first variety in the mixture?
A) The price of the first variety of clay is Rs 36 per kg. Rohan earned a profit of 20% by
selling the mixture at Rs 57.60 per kg.
B) The price of second variety of clay is Rs 50 per kg. The difference between the quantity
of first variety and the second variety of clay in the mixture is 3 kg.

(a) If question can be answered using A alone but not using B alone
(b) If question can be answered using B alone but not using A alone
(c) If question can be answered using either of the statements alone
(d) If question can be answered using both statements together but not using either A or B
alone
(e) If question can’t be answered even using A and B together
Answer - (d)

Explanation -
From statement A, we get
Price of variety A = Rs 36
Mean price = 57.6 / 1.2 = Rs 48

But statement A alone is not sufficient.


From statement B, we get
Price of variety B = Rs 50
If we combine both A and B,

Ratio = 2 : 12 = 1 : 6
1 : 6 has a gap of 5
This gap of 5 represents 3 kg.

Now we can find out the answer.


Note: We don’t ned to find the exact value since this is a data sufficiency question.

Q.15)
What is the rate of the compound interest?

A) A sum of 1000, amounts to 1331 at the rate of compound interest.


B) The amount was invested for the period of three years.
C) The simple interest received on that amount in one year is equal to the compound
interest received on that amount in the first year.
(a) The data in statements A and C together are sufficient to answer the question, while the
data in statement B are not sufficient to answer the question.
(b) The data in statements A and B together are sufficient to answer the question, while the
data in statement C are not sufficient to answer the question.
(c) The data in statement A, B and C together are not sufficient to answer the question.
(d) The data in statement only A and B together or only statement C are sufficient.
(e) The data in all the statements A, B and C together are necessary to answer the question.

Answer - (e)
Explanation -
From the statement A: P = 1000 and A = 1331 so interest will become 1331-1000 = 331
From the statement B: Time = 3 years but we could not conclude that the rate of interest was
compounded annually or half-yearly.
From the statement C: we can conclude that the rate of interest was compounded annually
because the simple interest of one year will be equal to the compound interest of the first year
only if the rate of interest is compounded annually.
Now P = 1000, CI = 331 time =3 years and rate of interest is compounded annually so we can
easily find the rate of interest.
So, all the statements are needed to get our answer.

Q.16)

A boat with a speed of 21 km/hr in still water, travels from point A to B in the downstream
direction and returns to point A. Another boat with speed of 28 km/hr travels from B to A and
returns to point B. The difference between the time taken by them to cover the distance is 6.5
hrs. What is the distance between points A and B, if the speed of stream is 7 km/hr?
(a) 420 km
(b) 120 km
(c) 280 km
(d) 350 km
(e) 210 km
Answer - (e)
Explanation -
Speed of 1st boat, b1 = 21
Speed of 2nd boat, b2 = 28
Speed of stream, s = 7
1st boat downstream speed = b1 + s = 28
1st boat upstream speed = b1 – s = 14

2nd boat downstream speed = b2 + s = 35


2nd boat upstream speed = b2 – s = 21
Let the distance between A to B = ‘d’
d/28 + d/14 - d/35 - d/21 = 6.5

d = 210 km

Q.17)
A person can purchase three articles in Rs. 49. What is the price of costliest article?

A) The cost price of two articles each is Rs. 1 less than the cost price of costliest article.
B) The cost price of two articles is same.
C) The cost price of costliest article is 6.25% more than the cost price of cheapest article.

(a) Either statement A alone or statements B and C together are sufficient.


(b) Only statement C is sufficient.
(c) Only statement A and B together are sufficient.
(d) Only statement A and C together are sufficient.
(e) None of these

Answer - (a)
Explanation -
From the Statement A,
Let the CP of each of two cheapest articles = x and the CP of costliest article = x + 1

Then, x + x + x + 1 = 49,
x = 16
Therefore, the CP of costliest article = 16 + 1 = 17
From the Statement II, we can say that the cost price of two articles is same and from
Statement III, we can say that the cost price of costliest article is 6.25% more than the cost price
of cheapest article therefore by combining both the statement we can also get our answer.
Q.18)

ABC bank offers 12% interest per annum compounded half yearly on deposits. DEF Bank and GHI Bank
offer simple interest but the annual interest rate offered by GHI Bank is twice that of DEF Bank. Sameer
invests a certain amount in DEF Bank for a certain period and Seema invests Rs 20,000 in GHI Bank for
twice that period. The interest that would accrue to Sameer during that period is equal to the interest
that would have accrued had he invested the same amount in ABC Bank for one year. The interest
accrued to Seema is

(a) Rs 9992
(b) Rs 9888
(c) Rs 9910
(d) Rs 9844
(e) None of the above

Answer - (b)

Explanation -

ABC Bank has a rate of interest of 12% and compounds half yearly. This is the same as having a 6%
interest rate per half-year.

So, if Principal P is invested for a year in ABC bank, at the end of the year it becomes P(1.06)(1.06) =
P(1.1236)

Therefore, the interest rate when viewed as a Simple interest scheme is 12.36% per annum.

Seema invested in GHI Bank, which has twice the interest rate as DEF Bank and the quantum for which
the investment is made is also double, hence Seema effectively gets 4 times the interest that Sameer
gets for the same investment in ABC Bank.

Let’s say Sameer invested Rs 20,000 in DEF Bank.

Since this is the same as investing in ABC Bank for 1 year, his interest would be 12.36% of 20,000 = Rs
2472.

Now, for the same investment, Seema must earn 4 times that of Rs 2472.

So, Seema earns Rs 9888.

Q.19)

A double-decker bus travelled at 2/3rd of its normal speed and therefore reached the destination 45
minutes after the scheduled time. On its return journey, the bus initially travelled at its normal speed for
10 minutes but then stopped for 5 minutes for an emergency. The percentage by which the bus must
now increase its normal speed in order to reach the destination at the scheduled time, is nearest to

(a) 9%
(b) 6.67%
(c) 10%
(d) 13.33%
(e) 5%

Answer - (b)

Explanation -

Let’s assume speed of the bus to be x and the time taken be t.

Since speed is reduced to 2/3rd,

New speed = 2x/3

Since the speed is two-third, time taken = 3t/2

This 3t/2 is after the scheduled time, so extra t/2 = 45 minutes

t = 90 minutes

Bus travels at x km/h & takes 90 min.

And, bus travels at 2x/3 km/h takes 135 minutes.

So, the bus usually takes 90 minutes to cover the distance.

It travels 10 minutes at the usual speed. That is, it travels 1/9th of the time at the usual speed. So, it
covers 1/9th of the distance in 10 minutes.

To reach its destination on time, the bus has to travel the remaining 8/9th of the distance in 80 minutes.
Since the bus halts for 5 minutes, it should now cover 8/9th of the distance in 75 (80-5) minutes.

In other words, the bus has to cover the same distance in 75/80th of the usual time. In order to do so,
the speed must be 80/75th of the usual speed. Or the increased speed will be 5/75th or 1/15th of the
usual speed, which is an increase of 6.67%.

Instruction for Q.20 to Q.23

Study the following pie charts carefully and answer the questions given beside.

Total number of commodities being manufactured in a factory is 10,00,000 units. Total commodities
being sold are 2,80,000 units. There are 6 types of commodities namely A, B, C, D, E and F. Total number
of commodities manufactured of these six types in terms of percentage has been given in pie chart-1,
and pie chart-2 gives the number of commodities unsold in terms of degrees.
Q.20)

In pie chart-2, there is some discrepancy in one of the commodities in terms of number of units sold and
unsold. Identify that commodity.

(a) A
(b) B
(c) C
(d) D
(e) E

Answer - (e)

Explanation -

Commodity Manufactured Unsold Sold


A 1,80,000 40/360 x 7,20,000 = 80,000 1,00,000
B 1,20,000 60/360 x 7,20,000 = 1,20,000 0
C 1,50,000 55/360 x 7,20,000 = 1,10,000 40,000
D 2,00,000 45/360 x 7,20,000 = 90,000 1,10,000
E 1,30,000 70/360 x 7,20,000 = 1,40,000 -10,000
F 2,20,000 90/360 x 7,20,000 = 1,80,000 40,000
10,00,000 7,20,000 2,80,000

There is discrepancy in commodity E due to negative value in the sold column.

Q.21)

The average number of units unsold of commodities B, D and F together are what % of total
manufactured units of commodity B?

(a) 110.33%
(b) 108.66%
(c) 110.66%
(d) 108.33%
(e) None of the above

Answer - (d)

Explanation -

Average unsold of B, D and E = 3,90,000/3 = 1,30,000

Total manufactured of B = 1,20,000

Percentage = 1,30,000/1,20,000 x 100 = 108.33%

Q.22)

Half of units sold of commodity A, one-fourth units sold of commodity C and one-fifth units sold of
commodity D were sold to a particular customer. If the number of units sold to this particular customer
are represented on a pie chart, then what corresponding angle the number of units sold of C will make?
(approximately)

(a) 40°
(b) 42°
(c) 44°
(d) 46°
(e) 48°

Answer - (c)

Explanation -

Half of A sold = 50,000

One-fourth of C sold = 10,000

One-fifth of D sold = 22,000

Total = 82,000

Required answer = 10,000/82,000 x 360 = 44° (approx.)

Q.23)

It is known that 15%, 10% and 5% of units unsold of commodities A, C, and F respectively are defective,
while 10% and 20% of B and D respectively are defective. In B and D, how many more/less units are
defective with respect to A, C, and F?

(a) 5%
(b) 5.25%
(c) 6%
(d) 6.25%
(e) 7%

Answer - (d)

Explanation -

A = 15% x 80,000 = 12,000

C = 10% x 1,10,000 = 11,000

F = 5% x 1,80,000 = 9,000

B = 10% x 1,20,000 = 12,000

D = 20% x 90,000 = 18,000

A + C + F = 32,000

B + D = 30,000

Percentage = 32,000 – 30,000 / 32,000 x 100 = 6.25%


Instruction for Q.24 to Q.27

Study the table and answer the given question.

The below table contains data related to salary structure of individuals from different organizations in
March:

Individuals Basic salary (Rs) Total allowance Total deduction Net salary (Rs)
(Rs) (Rs)
P 21800 28600 - -
Q - - 4350 25850
R 10400 12400 2800 20000
S 11200 13800 - -
T - 21600 5700 -

Note:

I. Total Deduction = Provident Fund Deduction (which is 10% of the basic salary) + Other
deduction
II. Net salary = Basic salary + Total Allowance – Total Deduction
III. Few values are missing in the table. A candidate is expected to calculate the missing value, if it is
required to answer the given question, on the basis of the given data and information.

Q.24)

If Q’s total allowance was Rs.3,000 more than his basic salary, what was his total allowance?

(a) Rs. 18,000


(b) Rs. 16,600
(c) Rs. 15,500
(d) Rs. 19,000
(e) Rs. 22,000

Answer - (b)

Explanation -

Q’s net salary = Rs.25850


If the basic salary of Q be Rs. x, then
Total allowance = Rs. (x+3000)
Now,
x + x + 3000 - 4350 = 25850
=> x = 13600
Q’s total allowance= 13600 + 3000 = Rs. 16600
Q.25)

If the respective ratio of Provident Fund Deduction and other deduction of S was 7:13, what was S’s
other deduction?

(a) Rs. 3100


(b) Rs. 2080
(c) Rs. 3500
(d) Rs. 2150
(e) Rs. 3245

Answer - (b)

Explanation -

Provident Fund Deduction of S = (11200 x 10)/100 = Rs. 1120

Other deduction of S = (13/7) x 1120 = Rs. 2080

Q.26)
If other deduction of P was Rs. 4,720, what was his net salary?
(a) Rs. 35500
(b) Rs. 42510
(c) Rs. 51000
(d) Rs. 40000
(e) Rs. 43500
Answer - (e)

Explanation -
Total deduction for P = (21800*10)/100 + 4720 = Rs. 6900
P’s net salary = 21800 + 28600 – 6900 = Rs. 43500

Q.27)
Basic salary of S is what percent more than the basic salary of R?
(a) 1(1/12)%
(b) 3(4/13)%
(c) 5(1/12)%
(d) 7(9/13)%
(e) 4(3/8)%
Answer - (d)
Explanation -
Required % = (11200 - 10400)/10400 x 100 = 7(9/13)%

Instruction for Q.28 to Q.30

The given line graph shows the number of passengers travelled by 2 cab drivers and the table
shows the percentage of females who travelled in the cabs in different months.

Month Driver A Driver B


Percentage of females Percentage of females
May 30% 50%
June 50% 70%
July 20% 40%
August 60% 30%
September 40% 50%

Q.28)
Find the ratio of the average of the female in the month of June and July by Driver A to the
average of the males in the same month by Driver B.
(a) 55 : 42
(b) 42 : 55
(c) 11 : 17
(d) 17 : 11
(e) 29 : 27

Answer - (c)

Explanation -

Driver A Driver B
Total Female Male Total Female Male
May 150 45 105 200 100 100
June 250 125 125 350 245 105
July 200 40 160 250 100 150
August 350 210 140 150 45 105
September 300 120 180 400 200 200

Required ratio = 165/2 : 255/2 = 11 : 17

Q.29)

Driver A rejects 20% of the total passengers who booked for the cab in the month of May and
Driver B rejects 10% of the passengers who booked for the cab for the same month. Find how
much percent Driver A picks the passengers less than that of Driver B in the same month.
(a) 25%
(b) 50%
(c) 40%
(d) 20%
(e) 33.33%
Answer - (e)
Explanation -

Driver A Driver B
Total Female Male Total Female Male
May 150 45 105 200 100 100
June 250 125 125 350 245 105
July 200 40 160 250 100 150
August 350 210 140 150 45 105
September 300 120 180 400 200 200
In May, Driver A rejects = 20% x 150 = 30
In May, Driver B rejects = 10% x 200 = 20
So, driver A picks = 150 – 30 = 120

And driver B picks = 200 – 20 = 180


Required % = (180 – 120) / 180 x 100 = 33.33%

Q.30)

Find the ratio of number of female passengers who booked for Driver A in the month of July
and August to the number of male passengers who booked for Driver B in the month of May
and September.
(a) 2:3
(b) 1:4
(c) 3:7
(d) 5:6
(e) 2:5
Answer - (d)

Explanation -

Driver A Driver B
Total Female Male Total Female Male
May 150 45 105 200 100 100
June 250 125 125 350 245 105
July 200 40 160 250 100 150
August 350 210 140 150 45 105
September 300 120 180 400 200 200

Required ratio = 250 : 300 = 5 : 6

Reasoning

Q.1)
From which of the below given words maximum words can be formed by replacing the vowel
with other vowels without changing the position of any letter?
(a) SICK
(b) LIST
(c) TICK
(d) TEST
(e) FARM
Answer - (b)
Explanation -
The possible words are LIST, LAST, LEST, LOST, LUST.

Q.2)
In the word “REVOLUTION”, how many pairs of letters has as many vowels between them as in
the English Alphabet from left to right?
(a) two
(b) three
(c) four
(d) more than four
(e) None of the above
Answer - (d)
Explanation -
The pairs are RV, EO, OL, LT, UT, UI, IO, ON.

Q.3)
In which of the following pairs, at least one of the words cannot be formed into any other
word?
(a) Sink, tile
(b) deaf, bask
(c) face, dust
(d) clay, veil
(e) bean, arty
Answer - (b)
Explanation -
Sink can be formed into Inks
Tile can be formed into Lite
Deaf can be formed into Fade
Face can be formed into Café
Dust can be formed into Stud
Clay can be formed into Lacy
Veil can be formed into Evil
Bean can be formed into Bane
Arty can be formed into Tray
But Bask cannot be formed into any other word.

Q.4)
W≤A≤S≤P=T
Z>T>Y>E
J=H≥K≥P

(a) J<S
(b) J>S
(c) J≥S
(d) J≤S
(e) J = S or the relationship can’t be determined
Answer - (c)
Explanation -
J≥K≥P≥S

Instruction for Q.5 to Q.9


Study the following information and answer the questions given below.
Seven people P, Q, R, S, T, U, and V are from seven different countries India, Brazil, China, USA,
UAE, UK, and Russia not necessarily in the same order. Each one of them works in the same
company but on different designation i.e., Chief Executive Officer (CEO), Chairman, Manager,
Deputy Manager (DM), Assistant Manager (AM), Assistant and Clerk. These ranks are given in
descending order (ex. CEO is the senior and Clerk is the junior). P is senior to V but he is neither
CEO nor Manager. U is senior to R who is not from UAE. Two designations are between Q and
the one who is from Brazil. P is from China and immediate senior to the one who is from UAE. Q
is a junior to the AM and he is from Russia. More than four people are senior to S. The number
of people who are senior to the one who is from Brazil is the same as the number of people
junior to V. T is from the USA and is junior to one who is from India.

Q.5)
What is the designation of V?
(a) COO
(b) Manager
(c) DM
(d) Assistant
(e) AM
Answer - (e)
Explanation -
People Country Designation
U India CEO
T USA Chairman
R Brazil Manager
P China DM
V UAE AM
Q Russia Assistant
S UK Clerk

Q.6)
How many persons are senior to T?
(a) 1
(b) 2
(c) 3
(d) 4
(e) None
Answer - (a)
Explanation -
People Country Designation
U India CEO
T USA Chairman
R Brazil Manager
P China DM
V UAE AM
Q Russia Assistant
S UK Clerk

Q.7)
Who among the following is from UAE?
(a) P
(b) R
(c) S
(d) V
(e) U
Answer - (d)
Explanation -
People Country Designation
U India CEO
T USA Chairman
R Brazil Manager
P China DM
V UAE AM
Q Russia Assistant
S UK Clerk

Q.8)
Which of the following is the incorrect pair?
(a) P - China
(b) Assistant - S
(c) T - Chairman
(d) R - Brazil
(e) CEO - India
Answer - (b)
Explanation -
People Country Designation
U India CEO
T USA Chairman
R Brazil Manager
P China DM
V UAE AM
Q Russia Assistant
S UK Clerk

Q.9) How many designations are there between U and S?


(a) 1
(b) 2
(c) 3
(d) 4
(e) 5
Answer - (e)
Explanation -
People Country Designation
U India CEO
T USA Chairman
R Brazil Manager
P China DM
V UAE AM
Q Russia Assistant
S UK Clerk

Instruction for Q.10


In a family, T is the mother of J. J is the husband of C. C is the mother of K. K is married to L. Y is
the mother of L. Y is married to S. R is married to P. K is the brother of P. O is the daughter of K.

Q.10)
How is O related to R?
(a) Niece
(b) Nephew
(c) Son
(d) Can’t be determined
(e) Daughter
Answer - (a)
Explanation –

From the above figure, we can conclude that O is the niece of R.

Instruction for Q.11


The critical reasoning question is given below consists of a passage followed by three statements and a
question. You must read the passage, question, and statements carefully to choose the correct
alternative.
Passage: The sense that justice requires punishment for wrongs runs deep and is not the same as a mere
thirst for revenge or a desire to get even. On the contrary, punishment plays an important role in any
healthy moral ecosystem.
Statements: (A) Healthy moral ecosystem is desirable. (B) Forgiving is a bad practice. (C) Punishment at
times is to be given to the wrong people.

Q.11)
Which of the above three statements is an assumption implicit in the passage?
(a) Only A
(b) Both A and B
(c) Only B
(d) Both A and C
(e) Both B and C
Answer - (a)
Explanation -
A is an implicit assumption because the last line talks about the role of punishment in a healthy moral
ecosystem which implies that a healthy moral ecosystem is desirable.
Although the passage talks about the importance of punishment, it does not imply that forgiving is a bad
practice as nothing has been mentioned about it. So, B is not an implicit assumption.
C is also not an implicit assumption because it contains the word ‘at times’ which becomes conditional.
Also, punishment is mentioned for wrong doings and not wrong persons. The concept of wrong persons
has not been mentioned.

Instruction for Q.12


Read the following passage and find out the inferences stated through the passage.
Statement: A biofuel is produced through contemporary processes from biomass rather than
conventional fossil fuels. They are renewable energy resources and can play an important role in
minimizing CO2 emissions. Crop residues could be turned to biofuel but our farmers require proper
incentives and handholding.

Q.12)
Which of the following can be logically inferred from the statement above?
(a) Other countries are generating Bio fuels which are Eco-friendly.
(b) This is a good example of re-cycling where waste can be converted into something productive.
(c) Bio fuels have a high calorific value in comparison to the conventional fossil fuels.
(d) Government has not done much for the farmers in the past.
(e) None of these.
Answer - (b)
Explanation -
No other statement except option B can be inferred from the passage. Obtaining biofuel from biomass is
a good example of recycling.

Instruction for Q.13


In the question below is given a statement followed by two courses of action numbered I and II. You
have to assume everything in the statement to be true and on the basis of the information given in the
statement, decide which of the suggested course of action logically follow(s) for pursuing.

Q.13)
Statement: Urban India generates 62 million tonnes of waste annually, and it has been predicted that
this will reach 165 million tonnes in 2030.
Courses of Action:
I. Classification of the garbage into recyclable and non-recyclable material must be implemented
at every level in the country.
II. Regular waste audits should be conducted across the country so that smarter purchasing
decisions are made.

(a) Only I follows


(b) Only II follows
(c) Either I or II follows
(d) Neither I nor II follows
(e) Both I and II follow
Answer - (e)
Explanation -
The statement clearly shows the grim condition of waste being produced in urban India and its
projected growth. So, segregation of waste into recyclable and non-recyclable can help deal in waste
management. Also, waste audits will be equally useful.

Instruction for Q.14


In the question below are given a statement followed by two courses of action numbered I and II. A
course of action is a step or administrative decision to be taken for improvement, follow-up or further
action in regard to the problem, policy, etc. On the basis of the information given in the statement to be
true, then decide which of the suggested courses of action logically follow(s) for pursuing.

Q.14)
Statement: The management of XYZ Ltd has been at the receiving end of constant complaints from its
employees about the HR department. The management is concerned that poor HR management will
likely spread to impact all levels of the company causing issues such as employee conflict, poor team
building, personality conflicts among many others.
Which of the following statements is/are a viable course of action to tackle the problem?
I. Replacing the HR personnel with a HR management software.
II. Develop an effective training program for HR which includes guidance on the company's
interdepartmental communications improvement.
(a) Only I follows
(b) Only II follows
(c) Both I and II follow
(d) Neither I nor II follows
(e) Either I or II follows
Answer - (b)
Explanation -
‘I’ is not a valid course of action as replacing HR personnel with HR software will solve the problem. But
‘II’ is a valid course of action as effective training of HR department is required.

Instruction for Q.15 to Q.19


Study the following information carefully to answer the questions given below.
There are four floors in a given hall such that floor 2 is above floor 1 and floor 3 is above floor 2 and so
on. There are three flats in each floor such that flat 3 is in the east of Flat 2 and Flat 2 is in the east of
Flat 1. In each flat, a certain number of persons live and the same number of persons do not live in two
flats at the same time. There are six persons on an even number of flat and even number of floor. The
number of persons living in flat 3 of floor 2 is a multiple of 4 but less than 10. There are four persons on
either flat 1 or flat 2 of floor 4. The number of persons living in flat 1 of floor 2 is a multiple of 6 but less
than 18. There are 10 people living in flat 3 on floor 1. The total number of persons living on floor 2 is 26.
There are 11 persons in an even-numbered floor of flat 1. The total number of persons on floor 4 is 29.
The sum of the number of persons on flat 3 comprising all the floor is 37. One of the odd-numbered flats
of floor 3 contains 7 persons. The number of persons available on flat 1 of floor 1 is the difference in the
total number of persons available on flat 3 on floor 2 and flat 3 of floor 3. The number of people living
on flat 2 of floor 3 is a square of the number of persons available on flat 1 of floor 1. The total number of
persons on flat 2 comprising all floor is 32.

Q.15)
How many persons are in flat 3 in 2nd floor?
(a) 5
(b) 8
(c) 10
(d) Data inadequate
(e) None of these
Answer - (b)
Explanation -
Floors Flat 1 Flat 2 Flat 3
Floor 4 11 4 14
Floor 3 7 9 5
Floor 2 12 6 8
Floor 1 3 13 10

Q.16)
How many total persons are there in flat 1 in all floor?
(a) 32
(b) 29
(c) 33
(d) Data inadequate
(e) None of these
Answer - (c)
Explanation -
Floors Flat 1 Flat 2 Flat 3
Floor 4 11 4 14
Floor 3 7 9 5
Floor 2 12 6 8
Floor 1 3 13 10

Q.17)
12 persons are on which floor?
(a) floor 1
(b) floor 2
(c) floor 3
(d) floor 4
(e) Data inadequate
Answer - (b)
Explanation -
Floors Flat 1 Flat 2 Flat 3
Floor 4 11 4 14
Floor 3 7 9 5
Floor 2 12 6 8
Floor 1 3 13 10
Q.18)
Four of the following five are alike in a certain way, based on their position. Which of the following does
not belong to that group?
(a) 11
(b) 12
(c) 8
(d) 5
(e) 14
Answer - (d)
Explanation -
Floors Flat 1 Flat 2 Flat 3
Floor 4 11 4 14
Floor 3 7 9 5
Floor 2 12 6 8
Floor 1 3 13 10

Q.19)
Choose the correct option:
(a) 7 persons are in flat 2’s row 2.
(b) Total 20 persons are in row 3.
(c) Difference between total no. of persons between flat 1 and flat 3 is 4.
(d) 14 persons are in Row 2’s flat 3.
(e) None of these
Answer - (c)
Explanation -
Floors Flat 1 Flat 2 Flat 3
Floor 4 11 4 14
Floor 3 7 9 5
Floor 2 12 6 8
Floor 1 3 13 10

Q.20)
Statements: The COVID-19 pandemic is reaching a peak and the economy of the world is
suffering a lot because of it.
Assumptions:
i) Some pandemics are capable of destroying the world’s economy.
ii) The economy of the world can never be affected no matter how big a threat is.
(a) Only i) is implicit
(b) Only ii) is implicit
(c) Either i) or ii) is implicit
(d) None
(e) Both are implicit
Answer - (a)
Explanation -
Clearly, statement I is implicit because COVID-19 pandemic has destroyed the world economy
which implies that some pandemics can do so.
Statement II is not implicit because it is a contradictory statement.

Instruction for Q.21


In each of the following questions, a statement is given followed by two conclusions numbered
I and II. You have to consider the statement to be true, even if it seems to be at variance from
commonly known facts. You are to decide which of the given conclusions can be drawn
definitely from the given statement.
Q.21)
Statement: The banking system in India has four tiers: (a) scheduled commercial banks, (b)
regional rural banks, (c) cooperative banks, (d) payment banks and small finance banks.
Conclusions:
I. The scheduled commercial banks are the most significant category in India's banking system.
II. India is planning to expand its banking network.
(a) Only I follows
(b) Only II follows
(c) Both I and II follow
(d) Neither I nor II follows
(e) Either I or II follows
Answer - (d)
Explanation -
None of the two conclusions follow as nothing has been mentioned about them in the
statement. The statement just talks about the types of banks in India.

Instruction for Q.22 to Q.26


Read the given information and answer the below questions.
Ten Managers namely Raman, Ankit, Raja, Aman, Akshay, Ajay, Priyank, Rahul, Shubham and
Tarun from an MNC have meeting on five different days starting from Monday to Friday of the
same week but not necessarily in the same order. Managers have meeting at two different time
slots 08:45 AM and 12:45 PM. Shubham has a meeting on Tuesday at 08:45 AM. The number of
people who have meeting between Priyank and Aman is the same as the number of people
who have the meeting between Raja and Rahul. Ankit has the meeting on the day immediately
before Shubham. Shubham does not have meeting on any of the days before Priyank. There is
one manager who has meeting at 08:45 AM immediately before Tarun. Rahul does not have
meeting at 12:45 PM. Aman does not have meeting on any day after Akshay. Only three people
have meeting between Priyank and Akshay. Neither Priyank nor Akshay has any meeting on
Friday. Only 2 people have meeting between Ajay and Tarun. Ajay does not have any meeting
on any of the day after Rahul.

Q.22)
Which of the following is the group of people whose meeting slot is 8:45 AM?
(a) Priyank, Shubham, Akshay, Ajay, Rahul
(b) Priyank, Shubham, Raman, Ajay, Rahul
(c) Priyank, Shubham, Akshay, Ajay, Tarun
(d) Either A or B
(e) None of these
Answer - (a)
Explanation -
08:45 am 12:45 pm
Monday Priyank Ankit
Tuesday Shubham Aman
Wednesday Akshay Raja
Thursday Ajay Raman
Friday Rahul Tarun

Q.23)
What is time and day for meeting of Raman?
(a) Thursday – 8 : 45 AM
(b) Thursday – 12 : 45 PM
(c) Friday – 8 : 45 AM
(d) Friday – 12 : 45 PM
(e) None of these
Answer - (b)
Explanation -
08:45 am 12:45 pm
Monday Priyank Ankit
Tuesday Shubham Aman
Wednesday Akshay Raja
Thursday Ajay Raman
Friday Rahul Tarun
Q.24)
How many people have meeting between Raman and Shubham?
(a) 1
(b) 2
(c) 3
(d) Can’t be determined
(e) None of these
Answer - (e)
Explanation -
08:45 am 12:45 pm
Monday Priyank Ankit
Tuesday Shubham Aman
Wednesday Akshay Raja
Thursday Ajay Raman
Friday Rahul Tarun

Q.25)
Whose meeting is on Friday?
(a) Rahul and Tarun
(b) Raman and Ajay
(c) Rahul and Raman
(d) Either 2 or 3
(e) None of these
Answer - (a)
Explanation -
08:45 am 12:45 pm
Monday Priyank Ankit
Tuesday Shubham Aman
Wednesday Akshay Raja
Thursday Ajay Raman
Friday Rahul Tarun

Q.26)
Which of the following statement is definitely false?
(a) Shubham has meeting on Tuesday
(b) Aman has meeting on Tuesday
(c) Rahul has meeting on Friday
(d) Ajay has meeting on Monday
(e) All are true
Answer - (d)
Explanation -
08:45 am 12:45 pm
Monday Priyank Ankit
Tuesday Shubham Aman
Wednesday Akshay Raja
Thursday Ajay Raman
Friday Rahul Tarun

Instruction for Q.27


The following question consists of three statements numbered I, II and III. Decide if data given
in the statements are sufficient to answer the questions below.

Q.27)
Among five friends P, Q, R, S and T each studying in a different standard, viz 3rd, 4th, 5th, 6th
and 8th. In which standard does R study?
I. Q studies in a standard which is an odd number. R is Q’s senior but does not study in 8th
standard.
II. Only three students are senior to P. Q is senior to P but junior to R.
III. S is not junior to T.

(a) Inadequate data


(b) Only II
(c) Only I and II
(d) Only I and III
(e) All I, II and III
Answer - (c)
Explanation -
From I and II:
3rd 4th 5th 6th 8th
P Q R

Instruction for Q.28


The following question consists of three statements numbered I, II and III. Decide if the data
given in the statements are sufficient to answer the question. Read the statements carefully
and give the answer.

Q.28)
Which word would be coded as CHAPTER?
I. In the code language, ‘Chapter is complete’ is written as ‘DIBQUFS JT DPNMFUF’
II. In the same language, ‘This chapter is very easy’ is written as ‘UIJT DIBQUFS JT WFSZ FBTZ’
III. In the same language, ‘Manish is intelligent’ is written as ‘NBOJTI JT JOUFMMJHFOU’

(a) Only I and II are sufficient


(b) Only I and III are sufficient
(c) All the statements are needed to answer the question
(d) Only II and III are sufficient
(e) Question cannot be answered even with the information in all the statements
Answer - (c)
Explanation -
Using all the three statements together:
Is JT
Chapter DIBQUFS
Complete DPNMFUF

Instruction for Q.29


Given below is a question with two statements numbered I and II given below it. You have to
decide whether the data provided in the statements are sufficient to answer the question. Read
all the statements and give the answer.

Q.29)
Among A, B, C, D and E, who is in the middle while standing in a row?
Statements:
I. C, who is third to the left of D, is at the immediate right of A, and second to the left of E.
II. C is second to the left of E, who is not at any of the ends and who is third to the right of A.
(a) I alone is sufficient while II alone is not sufficient
(b) II alone is sufficient while I alone is not sufficient
(c) Either I or II is sufficient.
(d) Neither I nor II is sufficient
(e) Both I and II are required
Answer - (a)
Explanation -
Using I alone,
1st 2nd 3rd 4th 5th
A C B E D

Instruction for Q.30 to Q.34


Read the following information and answer the questions given below:
Eight friends – A, B, C, D, E, F, G and H – are seated in a row, but not necessarily in the same
order. Some of them are facing north and the others are facing south. Each of the friends likes a
different colour among white, green, blue, red, yellow, black, pink and purple.
I. E sits four places away from the person who likes pink colour. The person who likes pink faces
south.
II. D sits two places away from the person who likes pink.
III. More than two persons sit between D and E. D and E face the same direction.
IV. One of the immediate neighbours of the person who likes pink likes black. Either A or C likes
black.
V. B sits second to the right of the person who likes black but does not sit adjacent to D. Only
two persons sit between B and G.
VI. The person who likes yellow sits second to the left of B.
VII. The person who likes green sits to the immediate right of G and faces in the opposite
direction to that of the person who likes blue.
VIII. H sits to the immediate left of the person who likes blue. Only one person sits between F
and the person who likes blue.
IX. A sits to the immediate right of the person who likes purple. The person who likes red sits
second to the left of H.

Q.30)
Who among the following sits at an extreme end of the row?
(a) The person who likes yellow
(b) The person who likes blue
(c) The person who likes red
(d) The person who likes purple
(e) The person who likes black
Answer - (a)
Explanation –
D C G A F B E H
↓ ↑ ↓ ↑ ↓ ↓ ↓ ↑
white green pink black purple red blue Yellow

Q.31)
Four among the following five are alike in a certain way and hence form a group. Find the one
which does not belong to that group.
(a) B
(b) F
(c) A
(d) G
(e) D
Answer - (c)
Explanation -
D C G A F B E H
↓ ↑ ↓ ↑ ↓ ↓ ↓ ↑
white green pink black purple red blue Yellow

Q.32)
Which of the following is true about F?
(a) F faces north direction
(b) F sits third to the right of A
(c) F and H face the same direction.
(d) F likes purple
(e) F is the immediate neighbour of E
Answer - (d)
Explanation -
D C G A F B E H
↓ ↑ ↓ ↑ ↓ ↓ ↓ ↑
white green pink black purple red blue Yellow

Q.33)
What is the position of H with respect to the person who likes purple?
(a) Fourth to the left
(b) Third to the left
(c) Second to the right
(d) Third to the right
(e) Second to the left
Answer - (b)
Explanation -
D C G A F B E H
↓ ↑ ↓ ↑ ↓ ↓ ↓ ↑
white green pink black purple red blue Yellow

Q.34)
Find the odd one out.
(a) Red-Blue
(b) White-Green
(c) Pink-Black
(d) Green-Pink
(e) Black-Red
Answer - (e)
Explanation -
D C G A F B E H
↓ ↑ ↓ ↑ ↓ ↓ ↓ ↑
white green pink black purple red blue Yellow

Q.35)
Statements: L ≥ A ≥ C , K = Y ≤ C , H > D ≤ K , A > E < Y
Conclusions:
I. D < A
II. A = D
III. L > Y

(a) All the conclusions follow


(b) Either conclusion I or II follows
(c) Only conclusion III follows
(d) Only conclusion II and III follow
(e) None of the conclusions follows
Answer - (b)
Explanation -
For conclusion I: D < A
From statements I, II and III, we get:
D≤K=Y≤C≤A
Here, the common sign between D and A is '≤', hence D ≤ A.
Thus, conclusion I does not follow individually.
For conclusion II: A = D
From statements I, II and III, we get:
D≤K=Y≤C≤A
Here, the common sign between D and A is '≤', hence D ≤ A.
Thus, conclusion II does not follow individually.
On combining conclusion I and II we get D ≤ A.
Therefore, either conclusion I or II follows.
For conclusion III: L > Y
From statements I and II, we get:
Y≤C≤A≤L
Thus, the common sign between Y and L is '≤', Therefore Y ≤ L is the true relationship.
Hence conclusion III does not follow.

Q.36)
Statements: M > H = A , X ≥ G < H , Y < M < P , G > O > K

Conclusions:
I. P > X
II. G < P
III. Y < H

(a) All the conclusions follow


(b) Either conclusion I or II follows
(c) Only conclusion I and III follow
(d) Only conclusion II follows
(e) None of the conclusions follows
Answer - (d)
Explanation -
For conclusion I: P > X
From statements II and III, we get:
X≥G<H<M<P
Here, we can see the opposite sign between P and X, thus no relationship can be established
between them.
Thus, conclusion I does not follow.
For conclusion II: G < P
From statements I, II and III, we get:
G<H<M<P
Here, the common sign between G and P is ‘< ’. Hence G < P.
Hence conclusion II follows.
For conclusion III: Y < H
From statements I and III, we get:
Y<M>H
Here, we can see the opposite sign between Y and H, thus no relationship can be established
between them.
Thus, conclusion III does not follow.
Therefore, only conclusion II follows.

Instruction for Q.37 to Q.39


Given below are the codes for the digits/symbols. Study the conditions given below and answer
the questions accordingly:
digit/ 5 9 % 2 # & 0 @ 8 * 1 ! $ 3 4 ? 7
symbol
Code F C O G S B T E A L H M P J V Q D

Conditions:
i) If the right most element in the group is an odd digit and the left most element is an even
digit, then reverse the order of the code for the group.
ii) If the right most element is a symbol and the left most is an even digit, then the codes of
these two get interchanged.
iii) If both the right most and the left most elements are symbols, then the code for both will be
Z.
iv) If the right most element in the group is an even digit and the left most element is an odd
digit, then the codes for the first two elements get interchanged between them and the codes
for the last two elements get interchanged between them.
v) If the right most element is an odd digit and the left most element is a symbol, then the code
for both will be Y.

Q.37)
5@*8172
(a) EFLAHDG
(b) FELAHDG
(c) FELAHGD
(d) EFLAHGD
(e) None of the above
Answer - (d)
Explanation:
Condition (iv) will apply

Q.38)
40!371&
(a) VHDJMTB
(b) BTMJDHV
(c) TBMJDVH
(d) VTMJDHB
(e) ZTMJDHZ
Answer - (b)
Explanation -
Condition (ii) will apply

Q.39)
857&219
(a) AFDBGHC
(b) FADBGCH
(c) CHGBDFA
(d) CFDBGHA
(e) YFDBGHY
Answer - (c)
Explanation –
Condition (i) will apply

Instruction for Q.40


Study the below given information and answer the questions that follow:
In a certain code language,
‘board neck spiritual truck’ is written as ‘ga po ni fu’
‘spiritual bloom if preach’ is written as ‘dx ga bd pk’
‘truck for neck preach’ is written as ‘ni sr fu dx’
‘board bloom if bird’ is written as ‘bd po pk ke’

Q.40)
What is the code for ‘bloom’ as per the given code language?
(a) dx
(b) ga
(c) either ‘bd’ or ‘pk’
(d) either ‘ni’ or ‘po’
(e) po
Answer - (c)
Explanation -
ga – spiritual
dx – preach
po – board
sr – for
ke – bird
truck & neck (fu or ni)
bloom & if (bd or pk)

Instruction for Q.41


In the question below are given three statements followed by three conclusions I, II and III. You have to
take the given statements to be true even if they seem to be at variance from commonly known facts.
Read all the given conclusions and then decide which of the following conclusions logically follows from
the given statements disregarding commonly known facts.

Q.41)
Statements: Some Lipsticks are Gloss. Some Gloss are not Lip Balms. All Lip Balms are Lip Pencil.
Conclusions:
I. Some Lip Pencils being Gloss is a possibility.
II. All Lip Balms being Lipstick is a possibility.
III. Some Lip Pencil being Lip Balm is a possibility.
(a) Only Conclusion I follows
(b) Both Conclusion I and II follows
(c) Only Conclusion III follows
(d) Both Conclusion I and III follows
(e) None of these
Answer - (b)
Explanation –
The baisc diagram is as follows:

A possible diagram is as follows:

Statement III is not correct because it is not a possibility. Rather, it will always hold true.

Instruction for Q.42


Read the question in statement is given followed by two assumptions numbered I and II. Consider the
statement and decide which of the given assumptions are implicit.
Q.42)
Statement: “If you have obtained 75 per cent or more marks in XII Standard examination, your
admission to our coaching class for NEET is guaranteed”, a XYZ coaching institute advertisement.
Assumptions:
I. Bright students do not generally opt for attending coaching classes.
II. The coaching class has adequate capacity to accommodate all such students.

(a) Only assumption I is implicit


(b) Only assumption II is implicit
(c) Both assumption I and II is implicit
(d) Either assumption I or II is implicit
(e) Neither assumption I nor II is implicit
Answer - (b)
Explanation -
Students obtaining more than 75% marks would definitely include bright students. So, ‘I’ is not implicit
because then why would the institute make such advertisement.
‘II’ is an implicit assumption because a coaching institute would make such an advertisement only when
it has the required capacity.

Instruction for Q.43 and Q.44


Below in each question are given two statements (A) and (B). These statements may be either
independent causes or may be effects of independent causes or a common cause. One of these
statements may be the effect of the other statement. Read both the statements and decide which of the
following answer choices correctly depicts the relationship between these two statements.

Q.43)
(A) The meteorological department has issued a statement mentioning deficit rainfall during monsoon in
many parts of the country.
(B) The Government has lowered the revised estimated GDP growth from the level of earlier estimates.
(a) Statement (A) is the cause and statement (B) is the effect.
(b) Statement (B) is the cause and statement (A) is the effect.
(c) Both the statements (A) and (B) are independent causes.
(d) Both the statements (A) and (B) are effects of independent causes.
(e) Both the statements (A) and (B) are effects of some common cause.
Answer - (a)
Explanation -
When rainfall is less in monsoon, the agricultural output generally reduces due to lack of irrigation. So,
overall GDP will also reduce.
So, clearly, A is the cause and B is the effect.
Q.44)
(A) As per the statistics there has been a rise in the number of electric cars in country ‘X’ this year.
(B) The oil prices have fallen strongly in country ‘X’ this year and are estimated to stay low for at least
two years for now and not decrease further.
(a) Statement (A) is the cause and statement (B) is the effect.
(b) Statement (B) is the cause and statement (A) is the effect.
(c) Both the statements (A) and (B) are independent causes.
(d) Both the statements (A) and (B) are effects of independent causes.
(e) Both the statements (A) and (B) are effects of some common cause.
Answer - (a)
Explanation -
An increase in the number of electric cars means people may be shifting to electric cars from fuel cars.
So, accordingly, demand for fuel cars will reduce which will consequently reduce the demand of oil. As a
result, oil prices will reduce.

Instruction for Q.45


In making decisions about important questions, it is desirable to be able to distinguish between ‘strong’
and ‘weak’ arguments. ‘Strong’ arguments are those which are both important and directly related to
the question. ‘Weak’ arguments are those which are of minor importance and also may not be directly
related to the question or may be related to a trivial aspect of the question.

Q.45)
Statement: Should the institutes of higher learnings in India like IITs and IIMs be made totally free from
government control?
Arguments:
I. No, government needs to regulate functions of these institutes for national interest.
II. Yes, such institutes in the developed countries are run by non-government agencies.
III. No, these institutes are not capable to take policy decisions for smooth functioning.
(a) Only I is strong
(b) Only III is strong
(c) I and II are strong
(d) I and III are strong
(e) All the three are strong
Answer - (a)
Explanation -
2nd argument is based on example and hence it is a weak argument. 3rd argument is also weak.
Instruction for Q.46 to Q.50
Study the following information carefully to answer the following questions:
There are eight persons J, K, L, M, R, O, P and N are sitting around a rectangular table in such a way that
four of them sit at four corners of the Table while four sits in the middle of each of the four sides. The
one who sit at the four corners face outside the centre while those who sit in the middle of the sides
face inside. P sit second to the left of O. K sits second to the right of J. L sits third to the left of N. L faces
inside. R sits third to the left O. L is not the immediate neighbour of J.

Q.46)
Who among the following sits second to the right of K?
(a) M
(b) N
(c) L
(d) R
(e) None of these
Answer - (c)
Explanation -
From the given information, K sits second to the right of J. So, there will be two possible cases.

Further it is given that L is not the immediate neighbour of J. L faces inside. L sits third to the left of N.
So, from these there will be two possible places for L in Case 1(a) in which L face J and in Case 1(b), L
face K.
Now, it is given that R sits third to the left of O. Also, P sits second to the left of O. Hence, case 1(b) and
case 2 gets eliminated. So, only case 1(a) remains as follows:

Q.47)
Four of the following five belongs to a group. Find the one that does not belong to that group.
(a) L
(b) R
(c) K
(d) N
(e) J
Answer - (d)
Explanation –

Q.48)
Who among the following faces R?
(a) K
(b) L
(c) M
(d) N
(e) None of these
Answer - (a)
Explanation -
Q.49)
Who among the following sits third to the left of the one who sits second to the right of R?
(a) O
(b) P
(c) N
(d) L
(e) None of these
Answer - (b)
Explanation -

Q.50)
Who among the following sits exactly opposite to M?
(a) L
(b) R
(c) N
(d) P
(e) None of these
Answer - (e)
Explanation -

Instruction for Q.51 to Q.55


Study the following information carefully and answer the questions given below:
When a number and number arrangement machine is given an input line of words and numbers, it
arranges them following a particular rule. The following is an illustration of input and rearrangement.
Input: weight 34 93 and 14 density 84 concept to 49 74 be understood 56
Step 1: and weight 34 93 density 84 concept to 49 74 be understood 56 14
Step 2: 34 and weight 93 density 84 concept to 49 74 understood 56 14 be
Step 3: concept 34 and weight 93 density 84 to 74 understood 56 14 be 49
Step 4: 56 concept 34 and weight 93 84 to 74 understood 14 be 49 density
Step 5: to 56 concept 34 and weight 93 84 understood 14 be 49 density 74
Step 6: 84 to 56 concept 34 and weight 93 14 be 49 density 74 understood
Step 7: weight 84 to 56 concept 34 and 14 be 49 density 74 understood 93
Step 7 is the last step of the above arrangement as the intended arrangement is obtained.
As per the rules followed in the given steps, find out the appropriate steps for the given output.
Input: 36 24 maintain 87 safe distance 60 12 with other 98 cars

Q.51)
In which step of the given arrangement are the elements ‘24 cars 87’ found in the same order?
(a) Step 6
(b) Step 5
(c) Step 3
(d) Step 2
(e) Step 1
Answer - (c)
Explanation -
Input: 36 24 maintain 87 safe distance 60 12 with other 98 cars
Step 1: cars 36 24 maintain 87 safe distance 60 with other 98 12
Step 2: 24 cars 36 maintain 87 safe 60 with other 98 12 distance
Step 3: maintain 24 cars 87 safe 60 with other 98 12 distance 36
Step 4: 60 maintain 24 cars 87 safe with 98 12 distance 36 other
Step 5: safe 60 maintain 24 cars with 98 12 distance 36 other 87
Step 6: 98 safe 60 maintain 24 cars 12 distance 36 other 87 with

Q.52)
Which of the following is the second last step of the given arrangement?
(a) 98 safe 60 maintain 24 cars 12 distance 36 other 87 with
(b) 24 cars 36 maintain 87 safe 60 with other 98 12 distance
(c) 60 maintain 24 cars 87 safe with 98 12 distance 36 other
(d) maintain 24 cars 87 safe 60 with other 98 12 distance 36
(e) safe 60 maintain 24 cars with 98 12 distance 36 other 87
Answer - (e)
Explanation -
Input: 36 24 maintain 87 safe distance 60 12 with other 98 cars
Step 1: cars 36 24 maintain 87 safe distance 60 with other 98 12
Step 2: 24 cars 36 maintain 87 safe 60 with other 98 12 distance
Step 3: maintain 24 cars 87 safe 60 with other 98 12 distance 36
Step 4: 60 maintain 24 cars 87 safe with 98 12 distance 36 other
Step 5: safe 60 maintain 24 cars with 98 12 distance 36 other 87
Step 6: 98 safe 60 maintain 24 cars 12 distance 36 other 87 with

Q.53)
What is the position of ‘distance’ from the right end of the third last step of the given arrangement?
(a) Fifth
(b) Third
(c) Second
(d) Fourth
(e) Sixth
Answer - (b)
Explanation -
Step 4: 60 maintain 24 cars 87 safe with 98 12 distance 36 other

Q.54)
Which element is exactly between ’87’ and ’60’ in the third step of the given arrangement?
(a) distance
(b) with
(c) other
(d) safe
(e) cars
Answer - (d)
Explanation -
Step 3: maintain 24 cars 87 safe 60 with other 98 12 distance 36

Q.55)
Which element is sixth to the right of the element which is tenth from the right end of the last step of
the given arrangement?
(a) 36
(b) other
(c) distance
(d) 87
(e) 12
Answer - (a)
Explanation -
Step 6: 98 safe 60 maintain 24 cars 12 distance 36 other 87 with
’60’ is tenth from the right end.
The element which is sixth to the right of ‘60’ is ‘36’.

Instruction for Q.56 to Q.60


Read the given information carefully and answer the questions given beside:

Seven friends namely – Anand, Balaji, Chakor, Darpak, Easharjot, Falak and Girik are living in a building
of seven floors, ground floor numbered 1, first floor numbered 2 and so on till the top-most floor which
is numbered 7. Each person lives on a single floor. Total number of members in their family including
themselves is – one, six, one, two, four, three and two but not necessarily in the same order.

Girik’s family has 1 more person than Anand’s family. Chakor does not live on 6th floor. Balaji with his
family lives exactly below one of the couples but above the two people who live alone. There is only one
floor between the two people who live alone. There are three floors between Chakor and Girik. Chakor
lives above Girik. Easharjot lives on 3rd floor with his wife only. There are total 4 persons in Balaji’s
family including him.

Q.56)
Which one of the following lives on fifth floor?
(a) Chakor
(b) Darpak
(c) Falak
(d) Girik
(e) None of these
Answer - (a)
Explanation -
Floor Number Person Number of members
7 Anand 2
6 Balaji 4
5 Chakor 6
4 Darpak/Falak 1
3 Easharjot 2
2 Darpak/Falak 1
1 Girik 3

Q.57)
How many members are there in Anand’s family?
(a) 3
(b) 2
(c) 1
(d) 6
(e) None of these
Answer - (b)
Explanation -
Floor Number Person Number of members
7 Anand 2
6 Balaji 4
5 Chakor 6
4 Darpak/Falak 1
3 Easharjot 2
2 Darpak/Falak 1
1 Girik 3

Q.58)
Whose family consists of total three members?
(a) Anand
(b) Chakor
(c) Easharjot
(d) Girik
(e) None of these
Answer - (d)
Explanation -
Floor Number Person Number of members
7 Anand 2
6 Balaji 4
5 Chakor 6
4 Darpak/Falak 1
3 Easharjot 2
2 Darpak/Falak 1
1 Girik 3

Q.59)
How many person(s) live(s) between Easharjot and Girik?
(a) 3
(b) 2
(c) 1
(d) 4
(e) None of these
Answer - (c)
Explanation -
Floor Number Person Number of members
7 Anand 2
6 Balaji 4
5 Chakor 6
4 Darpak/Falak 1
3 Easharjot 2
2 Darpak/Falak 1
1 Girik 3
Q.60)
Which one of the following combinations of 'floor number - person - family members' is the correct
one?
(a) 2 - Girik - 1
(b) 3 - Easharjot – 5
(c) 6 - Chakor - 4
(d) 7 - Anand - 2
(e) None of these
Answer - (d)
Explanation -
Floor Number Person Number of members
7 Anand 2
6 Balaji 4
5 Chakor 6
4 Darpak/Falak 1
3 Easharjot 2
2 Darpak/Falak 1
1 Girik 3

You might also like